You are on page 1of 384

QUESTION BANK

PHYSICS
Gujarat Secondary and Higher
Secondary Education Board,
Gandhinagar
Price : ` 85.00
Published by :
Secretary
Gujarat Secondary and Higher Secondary Education Board,
Gandhinagar
I
Copyright of this book is reserved by Gujarat Secondary and Higher Secondary Education
Board, Gandhinagar. No reproduction of this book in whole or in part, or in any form is
permitted without written permission of the Secretary, Gujarat Secondary and Higher
Secondary Education Board, Gandhinagar.
Contribution
1 Dr. Hasmukh Adhiya (IAS) Principal Secretary , Education Department Gandhinagar
2 Shri R. R. Varsani (IAS) Chairman , G.S&H.S.E. Bord, Gandhinagar
3 Shri H. K. Patel (G.A.S) Dy. Chairman, G.S&H.S.E. Bord, Gandhinagar
4 Shri M. I. Joshi (G.E.S) Secretary , G.S&H.S.E. Bord, Gandhinagar
Coordination
1 Shri B. K. Patel O.S.D., G.S&H.S.E. Bord, Gandhinagar
2 Shri D. A.Vankar Assistant Secretary (Retd.), G.S&H.S.E. Bord, Gandhinagar
5 Shri G. M. Rupareliya Assistant Secretary, G.S&H.S.E. Bord, Gandhinagar
Expert Teachers
1. Shri J. M. Patel Shree J. M. Chaudhary Sarvajanik Vidhyalaya, Mehsana
2. Shri K. D. Patel J. N. Balika Vidhyalaya, Saraspur
3. Shri Mayur M. Raval P. J. Vakharia High School, Kalol
4. Shri S. G. Patel Sarkari Schook, Sector-12, Gandhinagar
5. Shri J. P. Joshi Diwan Ballubhai High School, Ahmedabad
6. Shri Vasudev B. Raval Vidhya Mandir High School, Palanpur
7. Shri Surendrabhai M. Rajkutir Convent of Jesus And Merry
8. Shri Sureshchandra H. Patel Alambic Vidhyalaya, Vadodara
9. Shri C. D. Patel Lalbahadur Shastri Vidhyalaya, Vadodara
10. Shri Mukesh N. Gandhi New English School, Nadiad
11. Shri Dineshbhai V. Suthar Retired Teacher
12. Shri S. S. Patel J. M. Chaudhary Sarvajanik Vidhyalaya, Mehsana
13. Shri Jayesh M. Purohit Ankur Vidhyalaya, Ahmedabad
14. Smt. Asha M. Patel Shree M.B. Vamdot Sarvajanik High School, Bardoli
15. Shri Maheshbhai Dhandhla Bhavnagar
16. Shri Mukesh M. Bhatt Bhavnagar
17. Shri Anilkumar Trivedi Anand
18. Shri Anand Thakkar Navchetan High School, Ahmedbad
19. Shri Sudhirkumar G. Patel Nutan High School, Visnagar
20. Smt. Anita Pillai Surat
II
PREFACE
Uptil now , the Students had to appear in various entrance examinations for
engineering and medical courses after std-12. The burden of examinations on the side of the
students was increasing day-by-day. For alleviating this difficulty faced by the students,
from the current year, the Ministry of Human Resource Development , Government of India,
has Introduced a system of examination covering whole country. For entrance to engineering
colleges, JEE(Main) and JEE(Advanced) examinations will be held by the CBSE. The
Government of Gujarat has except the new system and has decided to follow the examinations
to be held by the CBSE.
Necessary information pertaining to the proposed JEE (Main) and
JEE(Advanced) examination is available on CBSE website www.cbse.nic.in and it is requested
that the parents and students may visit this website and obtain latest information guidance
and prepare for the proposed examination accordingly. The detailed information about the
syllabus of the proposed examination, method of entrances in the examination /centers/
places/cities of the examinations etc. is available on the said website. You are requested to
go through the same carefully. The information booklet in Gujarati for JEE( Main) examination
booklet has been brought out by the Board for Students and the beneficieries and a copy of
this has been already sent to all the schools of the state. You are requested to take full
advantage of the same also However, it is very essential to visit the above CBSE website
from time to time for the latest information guidance . An humble effort has been made by
the Gujarat secondary and Higher Secondary Education Boards, Gandhinagar for JEE and
NEET examinations considering the demands of the students and parents , a question bank
has been prepared by the expert teachers of the science stream in the state. The MCQ type
Objective questions in this Question Bank will provide best guidance to the students and we
hope that it will be helpful for the JEE and NEET examinations.
It may please be noted that this Question Bank is only for the guidance of the
Students and it is not a necessary to believe that questions given in it will be asked in the
examinations. This Question Bank is only for the guidance and practice of the Students. We
hope that this Question Bank will be useful and guiding for the Students appearing in JEE and
NEET entrance examinations. We have taken all the care to make this Question Bank error
free, however, if any error or omission is found, you are requested to refer to the text
books.
M.I. Joshi R.R. Varsani (IAS)
Date: 02/ 01/ 2013 Secretary Chairman
III
I NDEX
IV
Unit Unit Name GSEB NEET JEE Page
No. No.
PART - I
1 Phisycs & Measurement Y Y Y 1
2 Kinematics Y Y Y 21
3 Laws of Motion Y Y Y 51
4 Work, Energy and Power Y Y Y 78
5 Rotationl Motion Y Y Y 107
6 Gravitation Y Y Y 147
7 Properties of Solids and Liquids Y Y Y 195
8 Thermodynamics Y Y Y 258
9 Kinetic Theory of Gases Y Y Y 288
10 Oscilations and Waves Y Y Y 321
1
Unit-1
Important Formula
2
SUMMARY
Measurement of large distance (Parallax Method)
equation D =
b
u
where D = distance of the planet from the earth.
where u = parallax angle.
b = distance between two place of observation.
Measurement of the size of a planet or a star.
equation
d

D
o = where D = distance of planet from the earth,
d = diameter of planet.
o = angular diameter of planet.
Measurement of mass
The gravitational force on an object, of mass m, is called the weight of the object.
1 amu = 1.66

10
27
kg = 1u
Estimation of Error
Absolute Error - Suppose the values obtained in several measurement of physical quantity
a are a
1
, a
2
, ............... a
n
If their arithmetic mean is a
then
1 2
1
.... 1

n
n
i
i
a a + a
a a
n n
=
+ +
= =

A
a
1
= a - a
1
,
A
a
2
=
a
- a
2
,
----------
A
a
n
=
a
- a
n
Aa
2
,

Aa
2 -------
Aa
n
are called absolute error
Average absolute error
1 2
1
... 1

=
A + A + +A
A = =

n
n
i
i
a a a
a a
n n
Fractional Error =
a
a
a
A
o
Percentage Error
Percentage error = oa

100 % =
a
100 %
a
A

Combination of errors
Addition Z = A + B Z A B A = A + A
Substraction Z = A B Z A B A = A + A
Division Z =
A
B

Z A B
Z A B
A A A
= +
Multiplication Z = A B
Z A B
Z A B
A A A
= +
Power Z =
n
A
Z A
n
Z A
A A
=
3
Rule for determining number of significant figures
All the non - zero digits are significant
All the zeros between two non zero digits are significant no matter where the decimal
point is it at all.
If the number is less then 1 then zeros on the right of decimal point but to the left of
the first non - zero digit are not significant.
In a number without decimal point the zeros on the right side of the last non zero digit
are not significant.
Dimensions and Dimensional formulas.
The expression of a physical quantity with appropriate powers of M, L, T, K, A etc
is called the dimensional formula of that physical quantity.
The power of exponents of M, L, T, K, A are called dimensions of that quantity.
Some important units of distance
15
1fermi (fm) 10 m =
o
10
1A 10 m =
11
1AU 1.496 10 m =
15
1light year 9.46 10 m =
16
1par sec 3.08 10 m =
4
MCQ Questions
For the answer of the following questions choose the correct alternative from among the
given ones.
Physics - scope and Excitement
- Physics, Technology and society.
- Fundamental sources of nature.
- Nature of Physical laws
1. Physics is one of the basic disciplines in the category of ............... sciences.
(A) Astro (B) Natural (C) Space (D) Genetic
2. Physics comes from a ............... word meaning nature
(A) Hindi (B) German (C) Greek (D) Sanskrit
3. Mechanics and newtons motion laws as ............... laws dependad.
(a) liner momentum (b) Energy conservation
(c) Gravitational (d) Charge conservation
4. What is the approximate value of the Radious of a nucleus ?
(a)
14
10 m
(b)
31
10 m
(c)
19
10 m
(d)
15
10 m
5. The scope for ratio of length is in order to ...............
(a)
40
10
(b)
40
10
(c)
20
10
(d)
30
10
6. The range of time scale is about ...............
(a)
10 26
10 sec to10 sec (c)
15 15
10 sec to10 sec
(b)
22 18
10 sec to10 sec (d)
20 25
10 sec to10 sec
7. Birth, evolution and death of stars etc. are studid in branch of physics known as ...............
(a) Thermodynamics (c) Astro physics
(b) Quantam physics (d) Electronics
8. ............... is a branch of physics in wich heat engine and refrigeratior efficiency is studied.
(a) optics (b) Thermodynamics (c) Mechanics (d) Quantom physics
9. What is full name of LHC
(a) Large hadron collider (c) Large heavy cullent
(b) Large hadron cullent (d) Light heavy cullent
10. The range of mass varies from ...............
(a)
15 26
10 kg to10 kg (b)
20 28
10 kg to10 kg (c)
30 55
10 kg to10 kg (d)
20 20
10 kg to10 kg
11. Length of Galaxies is in order of ...............
(a)
26
10 m (b)
36
10 m (c)
28
10 m (d)
14
10 m
12. The approximate value of charge of an electron is ...............
(a)
18
10 c (b)
15
10 c
+
(c)
38
10 c (d)
19
10 c
13. The universe is made up of ...............
(a) matter only (b) radiation only (c) vaccum (d) matter and radiation
5
14. Nucleus of molecule is made up of wich fundamental constituents ?
(a) only Electron (c) Electron and Proton
(b) Proton and neutron (d) Electron and neutron
15. In the development of nenotechnology and biotechnology ............... have played a vital role.
(a) ECG (b) ESR (c) NMR (d) AFM
16. What is full form of AFM ?
(a) Atomatic force mioroscope (c) Atomatic fire microscope
(b) Atomic force mirror (d) Atomic force microscope
17. What is full name of ECG ?
(a) Electron cardiograph (c) Electron colour gram
(b) Electro cardiograph (d) Electric colour graph
18. What is full name of ESR ?
(a) Electric space Radar (c) Electron spin Resonance
(b) Electron space Range (d) Electric spin Resonance
19. What is full name of NMR ?
(a) Nuclear magnetic Resonance (c) Nuclear mega Radar
(b) Neutron mega Resonance (d) Nuclear micro Radar
20. ............... deals with electric charge and magnatic phenomenna
(a) Dynamics (b) Electro dynamic (c) Themodynamic (d) Mechanis
21. At present state, there are .............. fundamental forces in nature.
(a) six (b) four (c) two (d) five
22. When charges are at rest the force is given by ............... law.
(a) coulombs (b) Newtons (c) Amperes (d) Faradays
23. The ................ force is the force of mutual attraction between any two objects by virtue of their
masses.
(a) Weak (b) Electromagnetic (c) Nuclear (d) Gravitational
24. The ............... force is the strongest of all fundamental forces.
(a) nuclear (b) Electromagnetic (c) Gravitational (d) Weak nuclear
25. Electromagnetic force is ...............
(a) attractive force only (c) repulsive force only
(b) attractive and repulsive force (d) a short range force
26. Which of the following force binds The particle in the nucleons ?
(a) Electromagnetic force (b) Strong force (c) Gravitational force (d) Weak force
27. Electromagnetic force is ............... range force
(a) Short (b) long (c) medium (d) very short
28. Quarks - Quarks force is produced between -
(a) Proton - neutron (b) proton - proton (c) neutron - neutron (d) (a),(b), (c) are true
6
29. Which partical are emitted during the | decay from the nucleus ?
(a) neutron and proton (c) electron and neutrino
(b) electron and neutron (d) electron and proton
30. ............... and ............... laws are called inverse square law
(a) Gravitation and weak (c) Coulombs and strong
(b) Gravitation and coulombs (d) Electromagnetic and coulombs
31. Which property of object is responsible for the electric force ?
(a) electric charge (b) pressure (c) volume (d) mass
32. Which property of object is responsible for the Gravitational force.
(a) electric charge (b) mass (c) pressure (d) volume
33. How much times is the strong nuclear force stronger then weak nuclear force ?
(a)
13
10
(b)
2
10
(c)
13
10
(d)
2
10
34. How much times is the strong nuclear force stronger then electro magnatic force ?
(a)
13
10
(b)
2
10
(c)
13
10
(d)
2
10
35. How much times is the electromagnatic force stronger then Gravitational force
(a)
13
10
(b)
13
10
(c)
36
10
(d)
36
10
36. Who has unified electromagnetism and optics ?
(a) Newton (b) Maxwell (c) Coulomb (d) Faraday
37. Who has unified terrestrial and celestial domains under a common law of Gravitational
(a) Newton (b) Maxwell (c) Coulomb (d) Farady
38. The weak nuclear force, Gravitational force and electromagnatic force are A, B and C Respectively
then ...............
(a) C > A > B (b) C > A < B (c) B > A > C (d) C < A < B
39. Range of weak nuclear force is ...............
(a)
15
10 km
(b)
14
10 km
(c)
18
10 km
(d)
20
10 km
40. Strong nuclear force close not exist on ...............
(a) Proton (b) nuclear (c) neutron (d) electron
41. The force acting between two point charges kept at a certain distance is F
1
Now magnitude
of charge are double and distance between them is double. The force acting between them is
F
2
find out the ratio of F
2
/F
1
= ...............
(a) 16 : 1 (b) 1: 16 (c) 1: 1 (d) 1: 8
42. If the resulting external force acting on system is zero then ............... of the system is constant
and if the resultant external torque acting on a system is zero then ............... of the system is
constart.
(a) total energy, angularmomentum (c) linermomentam, energy
(b) liner momentam, angularmomentum (d) angular and linear momentam
7
43. Space is homogeneous and isotropic so ............... law of servation is the result of this
(a) linear and angular momentum (c) energy and charge
(b) angular and linear momentum (d) charge and energy
44. Time is homogeneous so ............... law of conserbation is the result of this
(a) angular momentum (b) linear momentum (c) energy (d) charge
45. The basic reason behind existance of which conseration of law is still not known ?
(a) angular momentum (c) energy
(b) linear momentum (d) charge
46. The Gravitational force between any two body charges with distance as
n
F r o where n = ..........
(a) 1 (b) 2 (c) 3 (d) 2
47. Match the column
Column - I Column - II
(1) space is isotropic (P) conservation of linear momentum
(2) space is homogeneous (Q) conservation of energy
(3) Time is homogeneous (R) conservation of charge still not known
(4) Time is isotropic (S) conservation of angular momentam
(a) 1- (S), 2-(P), 3-(R), 4-(Q) (c) 1-(P), 2-(S), 3-(R), 4-(Q)
(b) 1-(S), 2-(P), 3-(Q), 4-(R) (d) 1-(R), 2-(Q), 3-(P), 4-(S)
Measurement and system of units
Units of physical quantities, system of units, SI system of units, fundamental or Base units. precision
in measurement. Error in measurement and significant figures.
Dimensions and Dimensional formula, Dimensional analysis and its uses.
48. Which of the following unit is not of length ?
(a) light year (b) fermi (c)
o
A
(d) becquerel
49. becquerel is a ............... unit and its symbol is ...............
(a) supplementary, Bq (b) fundamental, Bq (c) derived, Bq (d) derived, Bv
50. How many fundamental units are there in SI system ?
(a) 5 (b) 7 (c) 6 (d) 4
51. Which of the following physical quantity is fundamental ?
(a) viscosity (b) velocity (c) force (d) time
52. Poise is the unit of
(a) viscosity (b) velocity (c) force (d) time
53. Which unit of physical quantity remains same for all unit system ?
(a) meter (b) second (c) ampere (d) kilogram
8
54. Which of the following system of unit is not based on only units of mass length and time.
(a) SI (b) MKS (c) CGS (d) FPS
55. Which of the following symbol of unit does not follow practical norms for the use of SI system?
(a) Kg (b) kg. (c) k (d) A
56. Why derive luminous intensity simbol form of SI system ?
(a) cd (b) Cd (c) cd. (d) CD
57. What is the ratio of 10 micron to 1 nenometer ?
(a)
4
10 (b)
3
10 (c)
16
10 (d)
15
10
58.
1 femtometer
100 nenometer
= ...............
(a)
6
10 (b)
8
10 (c)
24
10 (d)
24
10
59. If value of gravitational constant in MKS is
2
11
2
Nm
6.67 10
kg

then value of G in
CGS = ...............
2
2
dyn cm
gm

(a)
9
6.67 10 (b)
7
6.67 10 (c)
8
6.67 10 (d)
5
6.67 10
60. A partical has an acceleration of 72 km/ min
2
find acceleration in SI system.
(a)
2
0.5 m/ s (b)
2
30 m/ s (c)
2
18 m/ s (d)
2
20 m/ s
61. 950 dyne = ............... newton
(a)
3
9.5 10 (b)
5
95 10 (c)
7
950 10 (d)
4
9.5 10
62. 100 picometer = ...............
(a)
8
10 cm (b)
7
10 m (c)
6
10 10 m (d)
8
10 10 m
63. 100 walt hour = ............... joule.
(a)
5
3.6 10 J (b)
6
3.6 10 J (c)
5
36 10 J (d)
6
36 10 J
64. If x meter is a unit of length then area of 1m
2
= ...............
(a) x (b) x
2
(c) x
2
(d) x
1
65. 1 Mev = ............... ev
(a)
7
10
(b)
4
10
(c)
5
10
(d)
6
10
66. Wave length of light radiation 0.000015 m = ...............
(a) 15 micron (b) 1.5 micron (c) 150 micron (d) 0.15 micron
67.
0
1
= ...............
(a)
600''
(b)
3600''
(c)
180''
(d)
3600'
68. 1 rad = ...............
(a)
0
180
(b)
0
3.14
(c)
0
180 | |
|
t
\ .
(d)
0
180
t | |
|
\ .
9
69. 1 g = ............... amu
(a)
23
6.02 10 (b)
23
6.02 10 (c)
27
1.66 10 (d)
27
1.66 10
70. 1 parsec = ...............
(a)
15
10 m
(b)
11
1.496 10 m (c)
15
1.496 10 m (d)
16
3.08 10 m
71. Which of the following unit does not represent the unit of power ?
(a) ampere/volt (c) (ampere)
2
ohm
(c) joule/second (d) ampere volt
72. Write the unit of angular acceleration in the SI system.
(a) N.Kg (b)
2
rad / (sec) (c) m/sec (d) N/kg
73. unit of universal gravitational constant is ...............
(a)
1
kg msec (b)
1
N m sec (c)
2 2
N m kg (d)
1
N mkg
74. The unit of stefen Boltzman constant ( o) is ...............
(a)
2 2 1
w m k (b)
2 3
w m k (c)
2 4
w m k (d)
2 4
w m k
75. Unit of momentum physical quantity ?
(a) newton - second (b) newton/second (c) Jule (d) Jule/second
76. Light year is a unit of ...............
(a) Mass (b) volume (c) density (d) Distance
77. Joule/seed is the unit of ...............
(a) Work (b) angular momentum (c) Pressure (d) Energy
78. The SI unit of momentum is ...............
(a) kg newton (b)
2 2
kg m s (c)
1
kg m (d)
1
kg ms
79. Volt/meter is the unit of ...............
(a) Work (b) viscosity (c) Electric fild intensity (d) velosity
80. The force F is represented by equation F =
1
P + Q

, where is the length. The unit of


P is same as that of ...............
(a) Surface tension (b) velocity (c) force (d) momentum
81. Write the unit of surface tension in SI system.
(a)
2
N
m
(b)
N
m
(c)
2
dyne
cm
(d)
dyne
cm
82. Which physical quantity has unit of pascal - secod ?
(a) Velocity (b) viscocity (c) energy (d) coefficient of viscocity
83. Which physical quantity has unit of joule - second ?
(a) velocity (b) planks constant (c) energy (d) vescocity
10
84. What is the least count of vernier callipers ?
(a)
4
10 m
(b)
5
10 m
(c)
2
10 m
(d)
3
10 m
85. What is the least count of screw gauge ?
(a)
4
10 m
(b)
5
10 m
(c)
2
10 m
(d)
6
10 m
86. For measurement of astronomical distance ............... is used.
(a) vernier callipers (b) spherometer (c) screwgauge (d) indirect method
87. Which mictoscope is used to measure the dimension of particle having dimension less than
0
4000 A ?
(a) electron microscope (b) simple microscope (c) optical microscope (d) none of above
88. In electron microscope electron behave like ...............
(a) charge (b) mass (c) particles (d) wave
89. Which wave length of light is used in an optical microscope ?
(a) radiowave (b) X - ray (c) infrared (d) visible
90. The intercepted area of the spherical surface about the center is 0.25m
2
having diameter 50
cm what will be solid angle ?
(a)
1
4 10 sr (b)
3
1 10 sr (c)
1
10 sr (d)
1
5 10 sr
91. One planet is observed from two diametrically opposite point A and B on the earth the angle
subtended at the planet by the two directions of observations is 1.8
o
. Given the diameter of
the earth to be about
7
1.276 10 m . What will be distance of the planet from the earth ?
(a)
8
40.06 10 m (b)
8
4.06 10 m (c)
13
400.6 10 m (d)
8
11 10 m
92. Find the distance at which 4 AU would subtend an angle of exactly
1"
of arc.
11 16
[1AU 1.496 10 m,1" 4.85 10 rad] = =
(a)
5
1.123 10 m (b)
5
11.23 10 m (c)
17
1.123 10 m (d)
17
11.23 10 m
93. The percentage error in the distance 100 5 cm is ...
(a) 5 % (b) 6% (c) 8 % (d) 20 %
94. In an experiment to determine the density of a cube the percentage error in the measurement
of mass is 0.25 % and the percentage error in the measurement of length is 0.50 % what will
be the percentage error in the determination of its density ?
(a) 2.75 % (b) 1.75 % (c) 0.75 % (d) 1.25 %
95. If
4
A b =
the fractional error in A is ...............
(a)
( )
4
b
b
A
(b)
b
b
A
(c)
b
4
b
A | |
|
\ .
(d) ( )
4
b A
96. If
2
3
A B
P
C
= where percentage error in A , B and C are respectively 2% 3% and 5% then
total percentage error in measurement of p
(a)18 % (b) 14 % (c) 21 % (d) 12 %
11
97. In the experiment of simple pendulum error in length of pendulum ( ) is 5 % and that of g
is 3 % then find percentage error in measurement of periodic time for pendulum
(a) 4.2 % (b) 1.2 % (c) 2 % (d) 4 %
98. Acceleration due to gravity is given by
2
GM
g
R
=
what is the equation of the fractional error g/ g A
in measurement of gravity g ? [G & M constant]
(a)
R

R
A
(b)
R
2
R
A
(c)
R
2
R
A
(d)
1 R
2 R
A
99. The period of oscillation of a simple pendulum is given by
T 2
g
= t

what is the equation of
the relative error
T
T
A
in measurement of period T ?
(a)
1
2
A

(b)
2
A

(c)
1
4
A

(d)
4
A

100. The length of a rod is (10.15 0.06) cm what is the length of two such rods ?
(a) (20.30 0.06) cm (b) (20.30 1.6) cm (c) (10.30 0.12) cm (d) (20.30 0.12) cm
101. For a sphere having volume is given by
3
4
V r
3
= t
What is the equation of the relative error
V
V
A
in measurement of the volume V ?
(a)
r
3
r
A
(b)
r
4
r
A
(c)
4 r
3 r
A
(d)
1 r
3 r
A
102. Kinetic energy K and linear momentum P are related as
2
p
K
2m
= . What is the equation of the
relative error
k
k
A
in measurement of the K ? (mass in constant)
(a)
p
p A
(b)
p
2
p
A
(c)
p
2 p A
(d)
p
4
p
A
103. Heat produced in a current carrying conducting wire is H = I
2
Rt it percentage error in I, R
and t is 2 % , 4 % and 2 % respectively then total percentage error in measurement of heat
energy ...............
(a) 8 % (b) 15 % (c) 5 % (d) 10 %
104. The resistance of two resistance wires are
1
R (100 5) = O and
2
R (200 7) = O are connected
in series. find the maximum absolute error in the equivalent resistance of the combination.
(a) 35O (b) 12O (c) 4O (d) 9O
105. The periodic time of simple pendulum is T 2
g
= t

relative error in the measurement of T and

are
a
and
b
respectively find relative error in the measurement of g
(a) a + b (b) 2b + a (c) 2a + b (d) a - b
12
106. A physical quantity x is given by x =
4
3
1
4
4
3
A B
C D
due to which physical quantgity produced the
maximum percentage error in x
(a) B (b) C (c) A (d) D
107. The resistance
V
R
I
=
where V 100 5 = volts and I 10 0.3 = anperes calculate the percentage
error in R.
(a) 8 % (b) 10 % (c) 12 % (d) 14 %
108. The number of significant figures in 0.000150 is ...............
(a) 3 (b) 5 (c) 2 (d) 4
109. Which of the following numerical value have significant figure 4 ?
(a) 1.011 (b) 0.010 (c) 0.001 (d) 0.100
110. What is the number of significant figures in
3
5.50 10 ?
(a) 2 (b) 7 (c) 3 (d) 4
111. The mass of substance is 75.5 gm and its volume is 25 cm
2
. Its density up to the correct
significant figure is ...............
(a)
3
3.02 gm/ cm (b)
3
3.200 gm/ cm (c)
3
3.02 gm/ cm (d)
3
3.1gm/ cm
112. The area of a rectangle of size 1.25 2.245 cm in significant figure is ...............
(a)
2
2.80625 cm (b)
2
2.81cm (c)
2
2.806 cm (d)
2
2.8062 cm
113. The significant figures in 500.5000 are ...............
(a) 5 (b) 3 (c) 7 (d) 6
114. Addition of measurement 15.225 cm, 7.21 cm and 3.0 cm in significant figure is ...............
(a) 25.43 cm (b) 25.4 cm (c) 25.435 cm (d) 25.4350 cm
115. Substract 0.2 J from 7.36 J and express the result with correct number of significant figures.
(a) 7.160 J (b) 7.016 J (c) 7.16 J (d) 7.2 J
116. After rounding of the number 9595 to 3 significant digits the value becomes ...............
(a) 9600 (b) 9000 (c) 9590 (d) 9500
117. How many significant numbers are there in
5
(2.30 4.70) 10 + ?
(a) 3 (b) 4 (c) 2 (d) 5
118. The radius of circle is 1.26 cm. According to the concept of significant figures area of it can
be represented as -
(a)
2
4.9850 cm (b)
2
4.985 cm (c)
2
4.98 cm (d)
2
9.98 cm
119. If A = 3.331 cm B = 3.3 cm then with regard to significant figure A + B = ...............
(a) 6.6 cm (b) 6.31 cm (c) 6.631 cm (d) 6 cm
13
120. If the length of rod A is (2.35 0.01) cm and that of B is (5.68 0.01) cm then the rod B is
longer than rod A by ...............
(a) (2.43 0.00) cm (b) (3.33 0.02) cm (c) (2.43 0.01) cm (d) (2.43 0.001) cm
121. In acceleration, The dimensions for mass ............... for length .. and for time
(a) 0,1,2 (b) 1,0,2 (c) 2,0,1 (d) 2,1,0
122. Dimensional formula for power is ...............
(a)
2 2 3
M L T
(b)
1 2 2
ML T
(c)
1 3 1
MLT
(d)
0 2 2
M L T
123. Dimensional formula for calories is ...............
(a)
1 1 2
MLT
(b)
2 1 2
M LT
(c)
1 2 2
ML T
(d)
2 2 2
M L T
124. Dimensional formula for thermal conductivity (k) is ..
(a)
2 1 2 1
M LT K
(b)
1 1 2 1
MLT K
(c)
1 0 3 1
ML T K
(d)
1 1 3 1
MLT K
125. Dimensional formula for Resistance (R) is ...............
(a)
1 1 3 1
MLT A
(b)
1 1 0 1
MLT A
(c)
1 2 3 2
ML T A
(d)
1 0 3 1
ML T A
126. Dimensional formula for conductance is ...............
(a)
1 2 3 2
M L T A
(b)
1 2 2 1
ML T A
(c)
1 2 3 2
ML T A
(d)
1 2 3 2
M L T A
127. Which physical quantity is represented by
1 3 3 2
M L T A ?
(a) Resistivily (b) Resistance (c) conductance (d) conductivity
128. Which physical quantity is represented by
1 3 3 2
M L T A
?
(a) Resistivity (b) Resistance (c) conductance (d) conductivity
129. Which physical quantity is represented by
1 1 3 1
MLT A
?
(a) Stress (b) Resistance (c) Electricfield (d) potential Difference
130. The dimensional formula of planks constant is ...............
(a)
3 2 1
M L T
(b)
1 2 1
ML T (c)
2 1 1
M LT (d)
1 2 3
ML T
131. Dimensional formula of latent heat is ...............
(a)
0 2 2
M L T
(b)
2 0 2
M L T
(c)
1 2 1
ML T
(d)
2 2 1
M L T
132. Dimensions of impulse are.
(a)
1 1 1
M L T
(b)
1 1 1
MLT
(c)
1 1 1
MLT
(d)
1 2 2
ML T
133. Write dimensional formula of coefficient of viscosity
(a)
1 2 1
ML T
(b)
1 1 1
M LT
(c)
1 1 1
ML T
(d)
1 1 1
MLT
134. Dimensional formula for torque is
(a)
2 2 3
M L T
(b)
2 1 2
M LT
(c)
1 1 2
MLT
(d)
1 2 2
ML T
135. Dimensional formula for capisitance (C)
(a)
1 2 4 2
M L T A
(b)
1 2 4 2
ML T A
(c)
1 2 3 1
M L T A
(d)
3 1 1 2
M LT A
14
136. Dimensional formula for Boltzmanns constant is ...............
(a)
1 1 2 1
MLT K
(b)
2 1 2 1
M LT K
(c)
1 2 2 1
ML T K
(d)
2 2 1 2
M L T K
137. Dimensional formula for electromotive force (emf)
(a)
2 1 1 3
M LT K
(b)
1 2 3 1
ML T K
(c)
1 1 3 1
MLT K
(d)
1 2 3 1
ML T K
138. Which physical quantity has dimensional formula as CR where C - capisitance and R - Resistance ?
(a) Frequency (b) current (c) Time period (d) acceleration
139. Write the dimensional formula of the ratio of linear momentum to angular momentum.
(a)
0 1 0
M L T
(b)
1 1 0
MLT
(c)
0 1 0
M LT
(d)
0 1 1
M LT
140. If L and R are respesented as the inductance and resistance respectively then the dimensional
formula of
R
L
will be ...............
(a)
2 1 2 1
M LT A
(b)
0 0 1 0
M L T A
(c)
1 1 0 1
ML T A
(d)
1 3 1 0
MLT A
141. Write the dimensional formula of r.m.s (root mean square) speed.
(a)
1 2 2
ML T
(b)
0 2 2
M L T
(c)
0 1 1
M LT
(d)
1 0 1
ML T
142. One physical quantity represented by an equation as
(p q)c
2
t
where p, q and c are length
then quantity is ..
(a) length (b) velocity (c) Area (d) volume
143. The dimensional formula of magnetic flux is ...............
(a)
1 2 2 1
ML T A
(b)
1 2 1 2
ML T A
(c)
1 2 2 2
ML T A
(d)
1 2 1 2
M L T A
144. Which physical quantity has unit of pascal - second ?
(a) Force (b) Energy (c) Coefficient of viscocity (d) velocity
145. Dimensional formula of CV ? where C - capacitance and V - potential different
(a)
1 2 4 2
ML T A
(b)
1 2 3 1
ML T A
(c)
0 0 1 1
M L T A
(d)
0 0 1 1
M L T A
146. The equation of a wave is given by Y A sin
x
k
v
(
= e
(

where e is the angular velocity and
v is the linear velocity. Write the dimensional formula of K
(a)
0 0 1
M L T
(b)
1 0 1
ML T
(c)
0 1 1
M LT
(d)
1 1 1
ML T
147. If P and q are diffrent physical quantities then which one of following is only possible dimensionally ?
(a) p + q (b)
p
q
(c) p q (d) p = q
148. From ( )
2
a
p v b
v
| |
+ =
|
\ .
constant equation is dimensionally correct find the dimensional formula
for b ? where P = preasure V = volume
(a)
0 3 0
M L T (b)
1 3 0
ML T (c)
0 1 3
M LT (d)
1 1 1
M LT
15
149. Pressure P = A cosBx + c sinDt where xin meter and t in time then find dimensional formula
of
D
B
(a)
1 1 1
MLT
(b)
0 1 1
M LT
(c)
1 1 0
MLT
(d)
1 0 1
M L T
150. Find the dimensional formula for energy per unit surface area per unit time
(a)
1 0 2
ML T
(b)
0 1 1
M LT
(c)
1 0 3
ML T
(d)
1 1 1
ML T
151. Equation of force
2
F at bt = + where F is force in Newton t is time in second, then write unit
of b.
(a)
1
Nm
(b)
2
Nm
(c)
Nm
(d)
2
Nm
152. Pressure
2
at
P
bx
= where x = distance, t= time find the dimensional formula for
a
b
(a)
1 0 4
ML T
(b)
1 1 1
MLT
(c)
1 0 2
ML T
(d)
1 0 2
M L T
+
153.
2
Bxt
0
F A (1 e ) =
where F is force and x is desplacement. write the dimension formula of B
(a)
2 1 1
M LT
(b)
0 1 2
M L T
(c)
1 0 2
ML T
(d)
1 2 1
ML T
154. Equation of physical quantity
2
at bt = + v where v = velocity t = time so write the dimensional
formula of a in this equation
(a)
0 1 1
M LT
(b)
1 1 1
MLT
(c)
0 1 2
M LT
(d)
1 2 0
ML T
155. Density of substance in CGS system is 3.125
3
gm/ cm what is its magnitude is SI system ?
(a) 0.3125 (b) 3.125 (c) 31.25 (d) 3125
156. The resistivity of resistive wire is
AR
L
=
where L = length of wire A = Area of wire and
R is resistance of wire find dimension formula of .
(a)
1 3 3 2
MLT A
(b)
1 2 3 2
ML T A
(c)
2 3 1 2
M LT A
(d)
2 3 3 2
M LT A
157. A cube has numerically equal volume and surface area calculate the volume of such a cube.
(a) 2000 Unit (b) 216 Unit (c) 2160 Unit (d) 1000 Unit
158. Which out of the following is dimensionally correct.
(a) p
2
= hg (b) p = h
2
g (c) p = hg (d) p = h
2
g
159. If energy
p q r
E G h c = where G is the universal gravitational constant. h is the planks constant
and c is the velocity of light, then the values of p, q and r are respectively
(a)
1 1 5
, ,
2 2 2
(b)
1 1 5
, ,
2 2 2
(c)
5 1 1
, ,
2 2 2
(d)
1 1 5
, ,
2 2 2
160. If the centripetel force is of the form m
a
v
b
r
c
find the values of a, b and c
(a) 1,2,1 (b) 1,2,1 (c) 1,3,2 (d) 1,3,1
16
161. equation of
0 2 1
[1 ( )] = + o
t
T T find out the dimensions of the coefficient of linear expansion
o suffix.
(a)
0 0 1 1
M L T K
(b)
0 1 1 1
M LT K
(c)
1 1 0 1
MLT K
(d)
0 0 0 1
M L T K
162. Test if the following equation are dimensionally correct (S = surface tension = density
P = pressure v = volume n = coefficient of viscocity r = redious)
(a)
2Scos
h
rg
u
=

(b)
p
v =

(c)
4
pr t
v
8n
t
=

(d) all correct


163. Match list - I with list - II
List - I List - II
(1) Joule (a) henry

ampere/sec
(2) Walt (b) coulomb

volt
(3) volt (c) metre

ohm
(4) Resistivity (d) (ampere)
2

ohm
(a) b,d,c,a (b) c,a,b,d (c) b,d,a,c (d) b,c,a,d
164. Match column - I with column - II
Column -I Column - II
(1) capacitance (a)
1 1 3 1
MLT A
(2) Electricfield (b)
1 2 1
ML T
(3) plancks constant (c)
1 2 4 2
M L T A
(4) Angular momentum (d)
1 2 1
ML T
(a) a,c,b,d (b) c,a,d,b (c) c,a,b,d (d) a,b,d,c
165. In the relation

P e ,
o
u
o
=
|
B
z
k
P is pressure, z is distance, k is boltz mann constant and u is
the temperature. The dimensional formula of B will be
(a)
0 2 0
M L T (b)
1 0 1
M L T (c)
1 1 1
M LT
(d)
1 1 0
MLT
17
1(B) 26(B) 51(D) 76(D) 101(A) 126(D) 151(D)
2(C) 27(B) 52(A) 77(B) 102(B) 127(A) 152(A)
3(C) 28(A) 53(B) 78(D) 103(D) 128(D) 153(B)
4(A) 29(C) 54(A) 79(C) 104(B) 129(C) 154(C)
5(B) 30(B) 55(B) 80(A) 105(C) 130(B) 155(D)
6(B) 31(A) 56(A) 81(B) 106(C) 131(A) 156(A)
7(C) 32(B) 57(A) 82(D) 107(A) 132(B) 157(B)
8(B) 33(A) 58(B) 83(B) 108(A) 133(C) 158(C)
9(A) 34(B) 59(C) 84(A) 109(A) 134(C) 159(A)
10(C) 35(C) 60(D) 85(B) 110(C) 135(D) 160(B)
11(A) 36(B) 61(A) 86(D) 111(D) 136(A) 161(D)
12(D) 37(A) 62(C) 87(A) 112(B) 137(B) 162(D)
13(D) 38(A) 63(A) 88(D) 113(C) 138(C) 163(C)
14(B) 39(C) 64(C) 89(D) 114(B) 139(A) 164(B)
15(D) 40(D) 65(D) 90(A) 115(D) 140(B) 165(B)
16(D) 41(C) 66(A) 91(B) 116(C) 141(C)
17(B) 42(B) 67(B) 92(C) 117(A) 142(C)
18(C) 43(A) 68(C) 93(A) 118(C) 143(A)
19(A) 44(C) 69(A) 94(B) 119(A) 144(C)
20(B) 45(D) 70(D) 95(C) 120(B) 145(D)
21(B) 46(D) 71(A) 96(C) 121(A) 146(A)
22(C) 47(B) 72(B) 97(D) 122(B) 147(B)
23(D) 48(D) 73(C) 98(B) 123(C) 148(A)
24(A) 49(A) 74(D) 99(A) 124(D) 149(B)
25(B) 50(B) 75(A) 100(D) 125(C) 150(C)
KEY NOTE
18
HINT
91
0
1.8 0.01 rad u = = t
7
b 1.27 10 m =
8
b
D 4.06 10 m = =
u
94
3
mass m
density
volume l

( )
( ) =
( )
percentage error in density
=
M l
[ + 3 ] 100
M l
A A | |

|
\ .
= 1.75 %
96
2
3
A B
P
C
=
P A B C
% [2 3 ]
P A B C
A A A A
= + +
= 21 %
97
l
T 2
g
= t
T 1 l 1 g
100 [ ] 100
T 2 l 2 g
A A A
=
= 4 %
100 length of two rods 2l =
= 2(10.15 0.06)
= (20.30 0.12) cm
103
2
heat energy H = I RT
H I R T
100 [2 ] 100
H I R T
A A A A
= + +
= 10 %
105
2
2
g 4
T
= t
l
g l T
2
g l T
A A A
= +
= b + 2q
33
Strong nuclear force
Electronmagnaticforce
=
2
1
10

= 10
2
34
Strong nuclear force
Weak nuclear force
=
-13
1
10
= 10
13
35
Electronmagnetic force
Gravational force
=
-2
-38
10
10
= 10
36
41
1 2
1 2
1
kq q
F
r
=

' '
1 2
2 2
2
kq q
F
r
=
49
6
4
9
10 10
10
10

=
58
15
8
9
10
10
100 10
=

60
2
72 1000
72 20
min 3600 (sec)
km m
= =
2
( )
64 Area =
2

A =
2 2
x m
2 2
2 2
A 1
1m x
x x
= = =
69 1 amu =
27
1.66 10 kg
=
24
1.66 10 gm
23
1gm 6.023 10 amu =
80
1 1
F p q
+
= +
|1
p F
+
=
F N(Neuton)
P surface tension
(meter)
= = =

90
2
A 0.025 m A =
2r 0.5m =
2
A
Solid angle 0.4Sr
r
A
= =
=
1
4 10 Sr
19
107
V
Resis tance R
I
=
R V I
R V I
A A A
= +
R 5 0.3
R 100 10
A
= +
R 8
R 100
A
=
R
% 8%
R
A
=
111
mass 75.5
density
volume 25
= =
= 3.02 =
3
3.1g/ cm
128
Resis tance Area
Resistivity
length

=
129
force
Electricfield
electric charge
=
130
2
mass (dis tance)
plank's cons tan t
time

=
131
heat energy
latent heat Q
mass
( ) =
133 2
Force time
coefficientof vis cos ity
(length)

=
141
2
Urms u root mean square speed = =
142 If p = q = c = L
then (p - q)c = L
2
= Area
144
d
If F = nA
dx
F
n = pascal second
d
dx

v
v
A
=
146
x
y Asin ( k) = e
v
x
k =
v
0 0 1
x
k M L T = =
v
148
2
a
P (v b) cons tant
v
| |
+ =
|
\ .
2
PV Pb constant
a ab
v v
+ =
PV Pb
0 3 0
V b M LT = =
149 cosBx dimensionl less =
0 0 0
Bx M L T =
0 0 0
0 1 0
M L T
B M L T
X
= =
Same as
0 0 1
D M L T =
0 1 1
D
M LT
B
=
151
2
F at bt = +
2
F bt at = =
2 2
F N
b
t m
= =
153
2
Bxt
F A (1 e ) =
2
Bxt dimensional less =
0 0 0
0 1 2
2
M L T
B M L T
xt
= =
155
3
gm
Density 3.125
cm
=
=
3
6 3
3.125 10 kg
10 m

= 3125
3
kg/ m
157
3
volume of cube V a =
2
total surface area of cube A = 6a
V A =
3 2
a 6a =
a 6 =
3
V (6) 216 unit = =
20
159
P q r
E G h c =
1 2 2
E ML T =
1 3 2
G M LT =
1 2 1
h M L T =
0 1 1
c M LT take it =
1 2 2 1 3 2 p 1 2 1 q
(ML T ) (M LT ) (M L T ) =
0 1 1 c
(M LT )
=
p q 3p 2q r 2pqr
M L T
+ + +
1 1 5
P , q , r
2 2 2
= = =
160
a b c
F m r o v
1 1 2
F M LT =
0 1 1
M LT = v
0 1 0
r M LT =
1 0 0
m M L T take it =
1 1 2 1 a 1 1 b 1 c
(M LT ) (M ) (LT ) (L ) =
=
a b c b
M L T
+
a 1, b 2, c 1 = = =
165
0 0 0
z
M L T
kB
o
=
u
kB
z
u
o =

and P =
o
|
u
kB
p pz
o u
| = =
0 2 0
M L T =
21
Unit - 2
Kinematics
22
SUMMARY
speed =
distance x
time t
Total distance
Average speed =
Total time

t 0
x
Instantaneous speed = lim
t
A
A
A

displacement r
Velocity =
time t
v
A
=
A

t 0
r r
Ins tan eous velocity lim
t dt
A
A A
= =
A

Average acceleration
Gave
t
Au
=
A

t
d
Instantaneous acceleration a lim
t dt
A
A
= =
A

Equation for Uniformally accelerated motion


(1)

0
+ at (3)
2
o
1
d = t + at
2
u
(2)
Vo V
s t
2
+ | |
=
|
\ .
(4) V
2
= Vo
2
+ 2ad

th
n o
a
Distance coveredin n SecondS V (2n 1)
2
= +
About Vectors
A

.
B

= AB cosq
A

= AB sinq
n
A

.
A

=
2
| |
A

= 0

i
.
i
=

j
.

j
=
k
.
i
= 1
i

i
=

j
=
k

k
= 0

i
.

j
=

j
.
k
=
k
.
k
= 0
i

j
= K

j

k
=
i

k

i
=

j
cosq =
.
B A
AB

A

=

A A A
B B B
i j k
x y z
x y z
23
A

^
B

then
A

.
B

= 0
A

^
B

then |
A

| = AB
A

||
B

then
A

.
B

= AB
A

||
B

then
A

= 0
|
A

| = |
B

| and
A

and
B

is Q the angle between


(1)

=0 then |
A

+
B

| = 2A
(2)

=180 then |
A

+
B

| = 0
(3)

=90 then |
A

+
B

| =
2
A
(4)

=60 then |
A

+
B

| =
3
A
(5)

=120 then |
A

+
B

| = A
For projectile
- Time to reach the highest point
o
sin
tm
g
u
=
- Maximum height H =
2
0
2
sin
2

g
- Range R =
2
0
sin2
g
- Maximum Range
2
0

R
g
=
-
o
2 sin
Flight time T
g
u
=
- Equation of trajectory
2
2 2
o
gx
y x tan
2 cos
= u
u u
- R 4Hcot = u
24
MCQ
For the answer of the following questions choose the correct alternative from among the given ones.
(1) A branch of physics dealing with motion without considering its causes is known as ....
(A) Kinematicas (B) dynamics
(C) Hydrodynemics (D) mechanics
(2) Mechanics is a branch of physics. This branch is ...
(A) Kinematics without dynamics (B) dynamics without Kinematics
(C) Kinematics and dynamics (D) Kinematics or dynamics
(3) To locate the position of the particle we need ...
(A) a frame of referance (B) direction of the particle
(C) size of the particle (D) mass of the particle
(4) Frame of reference is a ... and a ... from where an obeserver takes his observation,
(A) place, size (B) size, situation
(C) situation, size (D) place, situation
(5)
-2
B
-1
0
1 2 3
A
(m)
As shown in the figure a particle moves from 0 to A, and then A to B. Find pathlength and
displacement.
(A) 2m, 2m (B) 8m, 2m (C) 2m, 2m (D) 8m, 8m
(6) A particle moves from A to B and then it moves from B to C as shown in figure. Calculate
the ratio between path lenghth and displacement.
(A) 2 (B) 1 (C)
1
2
(D)
(7) A particle moves from A to P and then it moves from P to B as shown in the figure. Find
path length and dispalcement.
60
0
P
(A)
2
3
l
, l (B)
3
l
, l (C) 2l, l (D) l,
3
2l
25
(8) A car goes from one end to the other end of a semicircular path of diameter d. Find the
ratio between path legth and displacement.
(A)
3
2
t
(B) t (C) 2 (D)

2
(9) A particle goes from point A to B. Its displacement is X and pathlength is y. So
x
y
.....
(A) > 1 (B) < 1 (C) 1 > (D) 1 s
(10) As shown in the figure a partricle statrs its motion from 0 to A. And then it moves from
A to B.
AB
is an arc find the Path length
O
r
3
t
(A) 2r (B) r
3
t
+ (C)
| |
|
\ .

r 1 +
3
(D) ( ) r 1
3
t
+
(11) Here is a cube made from twelve wire each of length l. An ant goes from A to G through
path A-B-C-G. Calculate the displacement.

G
E
F
H
(A) 3l (B) 2l (C)
3l
(D)
l
3
(12) As shown in the figure particle P moves from A to B and particle Q moves from C to D.
Desplacements for P and Q are x and y respectivey then
5
4
3
2
1
1 2 3 4 5
O
(A) x > y (B) x < y (C) x = y (D) > x y
26
(13) Shape of the graph of position time given in the figure for a body shows that
t
x
o
(A) The body moves with constant acceleration
(B) The body moves with zero velocity
(C) The body returns back towards the origin
(D) nothing can be said
(14) The graph of position time shown in the figure for a particle is not possible because ...
t
x
o
(A) velocity can not have two values on one time
(B) Displacement can not have two values at one time
(C) Acceleration can not have two values at one time
(D) A, B and c are true
(15) An ant goes from P to Q on a circular path in 20 second Raidus OP = 10m. What is the
average speed and average velocity of it ?
1
2
0
0
p
O
R
(A)
1 1
ms , 3 ms
6
t
(B)
3
t
1 1
3
ms , ms
2
(C)
1 1
ms , 3 ms
3
t
(D)
1 1
ms , 6 ms t
(16) A particle is thrown in upward direction with initial velocity of 60 m/s. Find average speed and
average velocity after 10 seconds. [g = 10 ms
2
]
(A) 26ms
1
, 16ms
1
(B) 26ms
1
, 10ms
1
(C) 20ms
1
, 10ms
1
(D) 15ms
1
, 25ms
1
27
(17) The ratio of pathlength and the resepective time interval is
(A) Mean Velocity (B) Mean speed
(C) intantaneous velocity (D) intantaneous speed
(18) A car moving over a straight path covers a distance x with constant speed 10 ms
1
and then
the same distance with constant speed of V
2
. If average speed of the car is 16ms
1
, then V
2
= ....
(A) 30 ms
1
(B) 20 ms
1
(C) 40 ms
1
(D) 25 ms
1
(19) A bus travells between two points A ans B. V
1
and V
2
are it average speed and average
velocity then
(A) v
1
> v
2
(B) v
1
< v
2
(C) v
1
= v
2
(D) depends on situation
(20) A car covers one third part of its straight path with speed V
1
and the rest with speed V
2
. What
is its average speed ?
(A)
1 2
1 2
3
2
v v
v v +
(B)
1 2
1 2
2
3
v v
v v +
(C)
1 2
1 2
3
2
v v
v v +
(D)
1 2
1 2
3
2 2
v v
v v +
(21) Rohit completes a semicirular path of radius R in 10 seconds. Calculate average speed and average
velocity in ms
1
.
(A)
2 R 2R
,
10 10
t
(B)
R R
,
10 10
t
(C)
R 2R
,
10 10
(D)
2 R R
,
10 10
t
(22) A particle moves 4m in the south direction. Then it moves 3m in the west direction. The time
taken by the particle is 2 second. What is the ratio between average speed and average velocity
?
(A)
5
7
(B)
7
5
(C)
14
5
(D)
5
14
(23) A particle is projected vertically upwards with velocity 30ms
1
. Find the ratio of average speed
and instantaneous velocity after 6s. [g = 10ms
1
]
(A)
1
2
(B) 2 (C) 3 (D) 4
(24) The motion of a particle along a straight line is described by the function x = (3t 2)
2
. Calculate
the acceleration after 10s.
(A) 9ms
2
(B) 18mls (C) 36ms

(D) 6ms

(25) Given figure shows a graph at acceleration time for a rectilinear motion. Find average
acceleration in first 10 seconds.
o
t
15 10
5
10
2
m
a
S
| |
|
\ .
(A) 10ms
2
(B) 15ms
2
(C) 7.5ms
2
(D) 30ms
2
(26) A body starts its motion with zero velocity and its acceleration is 3m/s
2
. Find the distance travelled
by it in fifth second.
(A) 15.5m (B) 17.5m (C) 13.5m (D) 14.5m
28
(27) A body is moving in x direction with constant acceleration o. Find the difference of the
displacement covered by it in nth second and (n1)th second.
(A) (B)
2
o
(C)
3o
(D)
3
2
o
(28) What does the speedometer measure kept in motorbike ?
(A) Average Velocity (B) Average speed
(C) intantaneous speed (D) intantaneous Velocity
(29) The displacement of a particle in x direction is given by x = 9 5t + 4t
2
. Find the Velocity
at timt t = 0
(A) 8 ms
1
(B) 5 ms
1
(C) 3 ms
1
(D) 10 ms
1
(30) A freely falling particle covers a building of 45m height in one second. Find the height of the
point from where the particle was released. [g = 10ms
2
]
(A) 120m (B) 125m (C) 25m (D) 80m
(31) The distance travelled by a particle is given by s = 3 + 2t + 5t
2
The initial velocity of the particle
is ...
(A) 2 unit (B) 3 unit (C) 10 unit (D) 5 unit
(32) A particle is thrown in upward direction with Velocity V
0
. It passes through a point p of height
h at time t
1
and t
2
so t
1
+ t
2
= ....
(A)
0
g
v
(B)
v
0
2
g
(C)
2h
g
(D)
h
2g
(33) A particle is thrown in upward direction with initial velocity V
0
. It crosses point P at height h
at time t
1
and t
2
so t
1
t
2
= _______
(A)
2h
g
(B)
2
0
V
2g
(C)
2
0
2V
g
(D)
h
2g
(34) Ball A is thrown in upward from the top of a tower of height h. At the same time ball B starts
to fall from that point. When A comes to the top of the tower, B reaches the ground. Find the
the time to reach maximum height for A.
(A)
h
g
(B)
2h
g
(C)
h
2g
(D)
4h
g
(35) In the figure Velocity (V) position graph is given. Find the true equation.
x
o
X
V
o
V
(A)
0
0
0
x
x
v
v v = (B)
0
0
0
x
x
v
v v = +
(C)
0
0
0

x
x
v
v v = (D)
v
v v
0
0
0
= x +
x
29
(36) In the figure there is a graph of a x for a moving particle. Hence
da
=
dt
.... V
x
o
X
a
a
o
o
(A)
0
0
x
a
(B)
0
0
x
a
(C)
0
0
a
x
(D)
0
0
a
x
(37) A particle is moving in a straight line with intial velocity of 10 ms
1
. A graph of acceleration time of the
particle is given in the figure. Find velocity at t = 10 s.
o
10
5
a
(A) 25 ms
1
(B) 35 ms
1
(C) 45 ms
1
(D) 15 ms
1
(38) A graph of moving body with constant acceleration is given in the figure. What is the velocity after time t
?
o
t
A
V
D
B
C
E
t
1
(A)
DC
0A + 0E
BC
(B)
DC
0A + DE
BC

(C)
BC
AB+ 0E
DC

(D)
DC
0A + AD
BC

(39)
t
V
o
The graph given in the figure shows that the body is moving with .....
(A) increasing acceleration (B) decreasing acceleration
(C) constant velocity (D) increasing velocity
30
(40) Slope of the velocity-time graph gives of a moving body. .
(A) displacement (B) acceleration (C) initial velocity (D) final velocity
(41) The intercept of the velocity-time graph on the velocity axis gives.
(A) initial velocity (B) final velocity (C) average velocity(D) instanteneous velocity
(42) Here are the graphs of velocity time of two cars A and B, Find the ratio of the acceleration
after time t.
t
V
130
60
B
A
(A)
1
3
(B)
1
3
(C)
3
(D) 3
(43) Here is a velocity - time graph of a motorbike moving in one direction. Calculate the distance
covered by it in last two seconds.
o
V
t
1 2 3 4 5
C
10m
5
(A) 5 m (B) 20 m (C) 50 m (D) 25 m
(44)
a
t
In the above figure acceleration (a) time (t) graph is given. Hence V o .....
(A) a (B)
a
(C) a
2
(D) a
3
(45)
t
X
o
A
B
The graph of displacent (x) time (t) for an object is given in the figure. In which part of
the graph the acceleration of the particle is positive ?
(A) OA (B) AB
(C) 0 - A - B (D) acceleration is not positive at any part.
31
(46) In a uniformly accelerated motion the slope of velocity - time graph gives ....
(A) The instantaneous velocity (B) The acceleration
(C) The initial velocity (D) The final velocity
(47) The area covered by the curve of V t graph and time axis is equal to magnitude of ....
(A) change in velocity (B) change in acceleration
(C) displacement (D) final velocity
(48) An object moves in a straight line. It starts from the rest and its acceleration is 2ms
2
. After reaching a cer-
tain point it comes back to the original point. In this movement its acceleration is -3ms
-2
. till it comes to rest.
The total time taken for the movement is 5 second. Calculate the maximum velocity.
(A) 6 ms
1
(B) 5 ms
1
(C) 10 ms
1
(D) 4 ms
1
(49) The relation between time and displacement of a moving particle is given by
2
t 2 x = o where o is a
constant. The shape of the graph x y is ...
(A) parabola (B) hyperbola (C) ellips (D) circle
(50) Here are the graphs of x t of a moving body. Which of them is not suitable ?
(A)
t
x
o
(B)
t
x
o
(C)
t
x (D)
t
x
(51) Here are the graphs of v t of a moving body. Which of them is not suitable ?
(A)
t
V
(B)
t
V
(C)
t
V
(D)
t
V
Comprehension type questions
t
V
A
o
D
C
B
In the figure there is a graph of velocity time for a particle.
32
(52) Which area shows the displacement covered by the particle after time t
(A) closed fig AODCA (B) closed fig. ABCA
(C) closed fig. AODCBA (D) none of above
(53) Which part shows initial velocity of the particle ?
(A) OA (B) AB (C) AC (D) AOA
(54) How will you calculate the acceleration of the particle ?
(A) taking length of AB (B) taking magnitude of BC
(C) taking slope of AC (D) taking slope of AB
(55)
o
X
t
2
8
4
6
Given graph shows relation between position and time. Find correct graph of acceleration time
(A)
a
t
2
8
4
6
(B)
o
a
t
2
8
4
6
(C)
o
X
t
2
8
4
6
(D)
o
X
t
2 8 4
(56)
t
X
130
60
B
A
Here are displacement time graphs of particle A and B. If V V
A
and V
B
are velocities of the particles
respectively, then
A
B
V
V
= .....
(A)
1
3
(B) 3 (C)
1
3
(D)
3
33
(57)
X
t
2 8 4 6
Given graph shows relation between position (x) time (t) Find the correct graph of velocity
time.
(A)
o
V
t
2
8
4
6
(B)
V
t
2 8 4 6
(C)
V
t
2
8
4
6
(D)
V
t
2
8
4
6
(58) Particles A and B are released from the same height at an interval of 2 s. After some time t
the distance between A and B is 100m. Calculate time t.
(A) 8 s (B) 6 s (C) 3 s (D) 12 s
(59) As shown in the figure a particle is released from P. It reachet at point Q at time t
1
and reaches at point R at time t
2
so
1
2
t
t
= ....
(A)
1
3
(B)
1
2
(C)
2
1
(D)
4
1
(60) A particle moves in stright line. Its position is given by x = 2 + 5t 3t
2
.
Find the ratio of intial velocity and initial acceleration.
(A)
5
6
+ (B)
5

6
(C)
6
5
(D)
6

5
(61) A particle is moving in a circle of radius R with constant speed. It coveres an angle
u
in some
time interval. Find displacement in this interval of time.
(A)
2Rcos
2
u
(B)

2Rsin
2
(C) 2Rcosu (D) 2Rsinu
(62) A particle is moving in a straight line with initial velocity of 200 ms
1
acceleration of the particle
is given by a = 3t
2
2t. Find velocity of the particle at 10 second.
(A) 1100ms
1
(B) 300ms
1
(C) 900ms
1
(D) 100ms
1
34
(63) Angle of projection, maximum height and time to reach the maximum height of a particle are
u
, H and tm
respectivley. Find the true relation.
(A)
m
H
t
2g
=
(B)
m
2H
t =
g
(C)
m
4H
t =
g
(D)
m
H
t =
4g
(64) Particle A is projected vertically upward from a top of a tower. At the same time particle B is dropped
from the same point. The graph of distance (s) between the two particle varies with time is.
(A)
t
S
(B)
t
S
(C)
t
S
(D)
t
S
(65) A car is moving with speed 30m. Due to application of brakes it travells 30m before stopping.
Find its acceleration.
(A)
2
m
15
s
(B)
2
m
15
s
(C)
2
m
30
s
(D)
2
m
10
s
(66) A particle moves with a constant acceleration 2m/s
2
. Its intial velocity is 10m/s. Find velocity after t second.
(A) (10 + t) ms
1
(B) 5(2 + t)ms
1
(C) 2 (5 + t)ms
1
(D) (10 + t
2
) ms
1
(67) A particle moves in a straight lime with constant acceleration. At t = 10s velocity and displacement
of the particle are 16ms
1
and 39m respectively. What will be the velocity after 10 s ...
(A) 22 ms
1
(B) 18 ms
1
(C) 20 ms
1
(D) 28 ms
1
(68) A particle moves with constant acceleration 2m/s
2
in x direction. The distance travelled in fifth
second is 19 m. Calculate the distance travelled after 5 second.
(A) 50 m (B) 75 m (C) 80 m (D) 70 m
(69) Two bodies of masses m
1
and m
2
are dropped from heights H and 2H respectively. The ratio of time taken
by the bodies to touch the ground is ...
(A)
1
2
(B) 2 (C)
1
2
(D)
2
1
(70) A freely falling stone crashes through a horizontal glass plate at time t and losses half of its velocity. Af-
ter time
t
2
it falls on the ground. The glass plate is 60 m high from the ground. Find the total distance travelled
by the stone. [g = 10ms
2
]
(A) 120 m (B) 80 m (C) 100 m (D) 140 m
(71) A freely falling object travells distance H. Its velocity is V. Hence, in travelling further distance of 4H its
velocity will become ....
(A)
3V
(B)
5V
(C) 2V (D) 3V
h
3h
R
Q
P
35
(72) A ball is thrown vertically upward direction. Neglacting the air resistance velocity of the ball in
air will
(A) zero (B) decrease when it is going up
(C) decrease when it is coming down (D) remain constant
(73) Two particles P and Q get 5 m closer each second while travelling in opposite direction. They
get 1 m closer each second while travelling in same direction. The speeds of P and Q are
respectively ...
(A) 5 ms
1
, 1 ms
1
(B) 3 ms
1
, 4 ms
1
(C) 3 ms
1
, 2 ms
1
(D) 10 ms
1
, 5 ms
1
(74) Motion of a porticle is described by an equation
( )
1
2
= A + y
where v, y and A are velocity
distance and a constant respectively. Find the acceleratrion of the particle.
(A) 1 unit (B) 2 unit (C)
1
unit
2
(D) 3 unit
(75) The minumum distance in which a car can be stopped is x. The velocity of the car is V. If
the velocity is 2V then find the stopping distance.
(A) 2x (B) 4x (C) 3x (D)
1
x
2
(76) A particle moves in one direction with acceleration 2 ms
2
and initial velocity 3 ms
1
.After what
time its displacement will be 10 m ?
(A) 1 s (B) 2 s (C) 3 s (D) 4 s
(77) A goods train is moving with constant acceleration. when engine passes through a signal its speed
is U. Midpoint of the train passes the signal with speed V. What will be the speed of the last
wagon ?
(A)
2 2
V U
2
(B)
2 2
V U
2
+
(C)
2 2
2V U
2

(D)
2 2
2V U
(78) Displacement of a particle in y direction is given by y = t
2
5t + 5 where t is in second.
Calculate the time when its velocity is zero.
(A) 5 s (B) 2.5 s (C) 10 s (D) 3 s
(79) The area under acceleration versus time graph for any time interval represents...
(A) Intial velocity (B) final velocity
(C) change invelocity in the time interval
(D) Distance covered by the particle
(80) A ball is thrown vertically upward. What is the velocity and acceleration of the ball at the maximum
height ?
(A) gt ms
1
, 0 (B) 0, 9 ms
2
(C) g ms
1
, 0 (D) 0, gt ms
2
(81) The relation between velocity and position of a particle is V = Ax + B where A and B are constants.
Acceleration of the particle is 10 ms
2
when its velocity is V, How much is the acceleration when
its velocity is 2V.
(A) 20 ms
2
(B) 10 ms
1
(C) 5 ms
2
(D) 0
36
(82) A particle moves on a plane along the path y = Ax
3
+ B in such a way that
dx
c
dt
= . c, A,
B are constans. Calculate the acceleration of the particle.
(A)
^
2
3Axc jms (B)
^
2 2
5Axc jms
(C)
^
2 2
3Axc jms (D)
^ ^
2 2
c i 3Axc j ms
| |
+
|
\ .
(83) The relation between velocity and position of a particle is given by V x = o | . Its initial velocity
is zero. Find its velocity at time
1
t
B
=
(A) e ms
1
(B) 0 ms
1
(C)
1
1
ms
e
(D) e
2
ms
1
(84) An object moves in x - y plane. Equations for displacement in x and y direction are x = 3sin2t
and y = 3cos2t Speed of the particle is
(A) zero (B) constant and nonzero
(C) increasing with time t (D) decreasing with time t
(85) Motion of a particle is decribed by x = (t 2)
2
Find its velocity when it passes through origin.
(A) 0 (B) 2 ms
1
(C) 4 ms
1
(D) 8 ms
1
(86) To introduce a vector quantity ....
(A) it needs magnitude not direction (B) it needs direction not magnitude
(C) it need both magnitude and direction (D) nothing is needed
(87) Which pair of two vectors is antiparallel.
(A)
A
B
(B)
A
B
(C)
A
B
(D)
A
B
(88) In the above figure
P

and
Q

are two vectors. What from followings is true


P
Q
(A)
P

and
Q

are equal (B)


P

and
Q

are perpendicular
(C)
P

and
Q

are antiparallel (D)


P

and
Q

are in same direction


(89) Which from the following is a scalar ?
(A) Electric current (B) Velocity (C) acceleration (D) Electric field
(90)
P

and
Q

are equal vectors what from the followings is true.


(A)
P

and
Q

are antiparallel (B)


P

and
Q

are parallel
(C)
P

and
Q

may be perpendicular (D)


P

and
Q

may be free vectors


37
(91)
P

=
Q

is true, if ...
(A) their magnitudes are equal
(B) they are in same direction
(C) their magnitudes are equal and they are in same direction
(D) their magnitudes are not equal and they are not in same direction
(92)
A

and
B

are in opposite direction so they are


(A) parallel vectors (B) anti parallel vector
(C) equal vector (D) perpendicular vector
(93)
^ ^ ^
A 1 2 j 2k = + +

. Calculate the angle between


A

and Y axis.
(A)
1
5
sin
3
(B)
1
1
sin
3
(C)
1
10
sin
3
(D)
1
5
cos
3
(94) A

and
B

are nonzero vectors. Which from the followings is true ?


(A)
( )
2 2
2 2
A + B A B = 2 A + B

(B)
( )
2 2
2 2
A B A B 2 A B + =

(C)
2 2
2 2
A B A B A B + = +

(D)
2 2
2 2
A B A B A B + =

(95)
C A B = +

and A = B = C Find the angle between
A

and
B

(A)
3
t
(B)
6
t
(C)
2
3
t
(D) 0
(96) The resultant of two vectors is maximum when they.
(A) are at right angles to each other (B) act in oppsite direction
(C) act in same direction (D) are act 120
0
to each other
(97) The resultant of two veetors
A

and
B

(A) can be smaller than A B in magnitude


(B) can be greater than A + B in magnitude
(C) cant be greater than A + B or smaller than A B in magnitude
(D) none of above is true
(98) The resultant of two forces of magnitude 2N and 3N can never be.
(A) 4N (B) 1N (C) 2.5N (D)
1
N
2
(99) The sum of
P

and
Q

is at right agnles to their difference then


(A) A = B (B) A = 2B (C) B = 2A (D) A = B = A B
(100) Magnitudes of A, B and C

are 41, 40 and 9 respectively,
A B C = +

Find the angle between
A and B

(A)
1
9
sin
40
(B)
1
9
sin
41
(C)
1
9
tan
41
(D)
1
41
tan
40
(101) If

A 3i 4j 9k = + +


is multiplied by 3, then the component of the new vector along z direction is ...
(A) 3 (B) + 3 (C) 27 (D) + 27
38
(102)
A B +

is perpendicular to
A

and B 2 A B = +

What is the angle between
A and B

(A)
6
t
(B)
5
6
(C)
2
3
(D)

3
(103) Out of the following pairs of forces, the resultant of which can not be 18N
(A) 11 N, 7 N (B) 11 N, 8 N (C) 11 N, 29 N (D) 11 N, 5 N
(104)

A 3i 2j 5k = +


and

B i j 2k = + +


Find
A 2B +

(A)
40
(B)
42
(C)
39
(D) 2
(105) What is the angle between
Q

and the resultant of


P Q +

and
Q P

(A) 90
0
(B) 60
0
(C) 0 (D) 45
0
(106)

A 2i 2j k = +


and

B 2i j 2k =


Find
3A 2B

(A)

2i 7j k + +

(B)

2i 8j k +

(C)

2i + 8j + k

(D)

i + 7j + k

(107) Linear momentajm of a particle is

( )
3i + 2j k

kgms
1
. Find its magnitude.
(A)
14
(B)
12
(C)
15
(D)
11
(108)
A B = C, Then C

is perpendicular to
(A)
A

only (B)
B

only
(C)
A

and
B

both when the angle between them is ...


(D)
A

and
B

both whatever to be the angle between them


(109) If A B 0 =

then
(A) A

must be zero (B) B

must be zero
(C) either
A 0, B 0 = =

or 0 u = (D) either
A = 0, B = 0

or

=
2
(110) y component of
A B

is ....

A Axi + Ayj Azk = +

B Bxi + Byj Bzk = +


(A) AxBy AyBx (B) AzBx AxBz (C) AxBz AzBx (D) AzBy AyBz
(111) If A B 0 =

then
A B

= ...
(A) AB (B)
A
B
(C)
1
AB
2
(D) 0
(112) If

A 4i + 3j 2k =


and

B 8i + 6j 4k =


the angle between
A

and
B

is
(A) 45
0
(B) 0 (C) 60 (D) 90
39
(113)

A 2i 3j k = +


and

B 8i + 6j 4k =


then A B

= ....
(A) 28 (B) 14 (C) 0 (D) 7
(114) If

A 2i + 5j k =


and

B 3i 2j 4k =


the angle between
A

and
B

is ...
(A) 0 (B)

2
(C)

4
(D)

6
(115) Which statement is true ?
(A)
A B B A =

(B)
A B = B A

(C)
A B B A =

(D)
A B AB =

(116) Which from the following is true ?
(A)
A B
cos
AB

u =

(B)
A B
sin
AB

u =

(C)
A B
tan =
A - B


(D)
AB
cot
A B
u =


(117) ( ) ( )
A B A B + =

....
(A) 0 (B) A
2
+ B
2
(C)
2 2
A B +
(D) A
2
B
2
(118) The angle between
i + j

and z axis is ....
(A) 0 (B) 45 (C) 90 (D) 180
(119)

A 2i j 2k = +


and

B i 2j 2k = +


what is the angle between
A

and
B

(A) cos
1
0.8888 (B) cos
1
0.4444 (C) sin
1
0.4444 (D) sin
1
0.8888
(120)

A Pi 2Pj k =


and

B 3i + 2j 14k = +


are perependcular to each other. Then p = ...
(A) 3 (B) 4 (C) 2 (D) 1
(121) In the above triangle AC = 5, BC = 8 and B =
6
t
Find the value of angle A.
8
5
6
t
(A) sin
1
0.6 (B) sin
1
0.8 (C) sin
1
0.12 (D) sin
1
0.4
(122)

A l + j 2k =

and

B 2i j k = +


Find the unit vectior in direction of
A B

(A)

( )
1
i 5j 2k
23

(B)

( )
1
i + 5j 3k
35

(C)

( )
1
i 5j 3k
29

(D)

( )
1
i 5j 3k
35

40
(123) What is unit vector along
i + j

?
(A)
i + j
2

(B)
i + j
2

(C)
i + j
3

(D)
i j
2

(124) Unit vector of A B

is

k
. Unit vector of
A

is i

Then what is the unit vector of


B

(A)
j

(B)
j

(C) any unit vector in xy plane (D) any unit vector in xz plane
(125) Find a unit vector in direction of

i + 2j 3k

(A)

( )
1
i + 2j 3k
7

(B)

( )
1
i + 2j 3k
2

(C)

( )
1
i + 2j 3k
14

(D)

( )
1
i + 2j 3k
5

(126) Find a unit vector perpenduicular to both
A

and
B

(A)
A B
AB


(B)
A B
ABsin

(C)
A B
ABcos

(D)
A B
ABsin


(127) If resultant of

A 2i + j k =

B i 2j + 3k =


and
C

is unit vector in y direction, then


C

is
(A)
j

(B)

3i 2j + 2k

(C)
j

(D)

2i + 3k

(128)
A

and
B

are two vectors



A B U = U
Now find the true option.
(A)
A

and
B

equal vectors (B)


A

and
B

are in opposite direction


(C) A

and
B

are in same direction (D)


A B

(129) Unit vector in direction of
A

is
(A)
A

(B)
A
A

(C) A A

(D)
A
A

(130)

2 2
i + j + pk
3 3

is a unit vector so p = .....
(A)
2
3
(B)
1

3
(C) 1 (D)
1
9
(131) Find a unit vector from the followings.
(A)
i + j

(B)
i j

(C)
1 1
i + j
2 2

(D)
1 1
i j
2 2

(132) Train A is 56 m long and train B 54 m long. They are travelling in opposite direction with velocity
m
15
s
and
m
5
s
respectively. The time of crossing is.
(A) 12 s (B) 6 s (C) 3 s (D) 18 s
41
(133) Graphs of velocity time for two cars A and B moving in a straight line are given in the fig.
The area covered by PQRS gives.
t
A
V
P
B
R
t
2
S
Q
t
1
(A) distance from A to B at time t
2
(B) distance from A to B at time t
1
(C) distance from A to B in time interval t
2

t
1
(D) change in distance from A to B in time interval t
2
t
1
(134)
o
A
V B
t
Graphs velocity time is given for cars A and B moving in a straight line in same direction.
At time t = 0 they are moving in the direction from A to B, then.
(A) They will meet once (B) They will never meet
(C) They will meet twice (D) none of above is true
(135) Velocity of particle A with respect to particle B is
m
4
s
while they are moving in same direction.
And it is
m
10
s
while they are in opposite direction. What are the velocities of the particles with
respect to the stationary frame of reference.
(A) 7 ms
1
, 3 ms
1
(B) 4 ms
1
, 5 ms
1
(C) 7 ms
1
, 4 ms
1
(D) 10 ms
1
, 4 ms
1
(136) Stone A is thrown in horizontal direction with velocity of 10 ms
1
at the same time stone B freely
falls vertically in downword direction. Calculate the velocity of B with respect to A after 10 second.
(A) 10 ms
1
(B)
1
101 ms
(C)
1
10 101ms
(D) 0
(137) A car moves horizontally with a speed of 3 ms
1
. A glass wind screen is kept on the front side
of the car. Rain drops strike the screen vertically. With the Velocity of 5 ms
-1
Calculate the velocity
of rain drops with respect to a ground.
(A) 6 ms
1
(B) 4 ms
1
(C) 3 ms
1
(D) 1 ms
1
42
(138) A man crosses a river through shortest distance D as given in the figure.
R V

is velocity of
water and
m V

is velocity of man in still river water. If


mR V

is relative velocity of man w.r.t.


river, then find the angle made by swimming man with the shortest distance AB
A
B
D
(A)
1 R
m R
V
tan
V V
(B)
2 2
1 m
R m
V
tan
V V
(C)
2 2
1 R
m R
V
tan
V V
(D)
2 2
1 R
R m
V
tan
V V
(139) A particle has initial velocity
( )
1
2i + 3j ms

and has acceleration
( )
2
i + j ms

. Find the velocity
of the particle after 2 second.
(A)
( )
1
3i + 5j ms

(B)
( )
1
4i + 5j ms

(C)
( )
1
3i + 2j ms

(D)
( )
1
5i + 4j ms

(140) Motion of a body in x - y plane is described by
( )
2
r 2ti 5t 6t j m
(
= + +


Then find velocity
of the body at t = 1 second.
(A)
1
144ms
(B)
1
148ms
(C)
1
150ms
(D)
1
260ms
(141) A particle moves in x - y plane. The position vector of the particle is given by
( )
2
r 3ti 2t j m =


Find
the rate of change of
u
at t = 1 second. Where
u
is the anghe betheen direction of motion and x
(A)
16
25
(B)
12
25
(C)
12

25
(D)
16
9
(142) x and y co-ordinates of a particle moving in x-y plane at some instant are
2 2
3
x 2t and y t
2
= =
Calculate y co-ordinate when its x coordinate is 8m.
(A) 3 m (B) 6 m (C) 8 m (D) 9 m
(143) A particle in xy plane is governed by x = A cos

t, y = A (1 sin

t). A and

are constants.
What is the speed of the particle.
(A) A

t (B) A

2
t (C) A

cos

t (D)
2
t
A sin
2
e
e
43
(144) A particle is moving in a xy plane with y = 2x and Vx = 2 t. Find Vy at time t = 3 second.
(A) 2 ms
1
(B) 3 ms
1
(C) +3 ms
1
(D) 2 ms
1
(145) t
1
and t
2
are two values of time of a projectile at the same height t
1
+ t
2
= ...
(A) Time to reach maximum height (B) fight time for the projectile
(C)
3
4
time of the fight time. (D)
3
2
time of the fight time.
(146) Eequetion of a projectile is given by y = Ax Bx
2
. Find the range for the particle.
(A)
A
B
(B)
A
4B
(C)
A
2B
(D)
2A
B
(147) Angle of projection of a projectile with horizonal line is
u
at time t = 0, After what time the
angle will be again u ?
(A)
Vcos
g
u
(B)
Vsin
g
u
(C)
0
V sin
2g
u
(D)
0
2Vsin
g
(148) A particle is projected with initial speed of V
0
and angle of
u
. Find the horizontal displacement
when its velocity is perpendicular to initial velocity.
(A)
2
0
V
gtan
(B)
2
0
V
gsin
(C)
0
V sin
g
u
(D)
2
0
V
tanu
(149) Intial anlge of a projectile is u and its initial velocity is V
0
. Find the angle of velocity with horizontal
line at time t.
(A)
1
0
g
sin 1 t
V cos
(
(
u

(B)
1
0
g
tan 1 t
V cos
(
(
u

(C)
(
(

1
0
g
tan tan t
Vcos
(D)
1
0
g
sin tan t
Vcos
(
(

(150) A stone is projected with an angle

and velocity V V
0
from point P. It strikes the ground at point
Q. If the both P and Q are on same horizontal line, then find average velocity.
(A)
0
V cos
(B)
0
V sin
(C)
0

V cos
2
(D)
0

Vsin
2
(151) An object is projected with initial velocity of 100 ms
1
and angle of 60. Find the verticle velocity
when its horizontal displacement is 500 m. (g = 10 ms
1
)
(A) 93.35 ms
1
(B) 93.35 ms
1
(C) 8.65 ms
1
(D) 98 ms
1
(152) Angle of projection of a projectile is changed, keeping initial velocity constant. Find the rate of
change of maximum height. Range of the projectile is R.
(A)
R
4
(B)
R
3
(C)
R
2
(D) R
(153) A cartasian equation of a projectile is given by y = 2x 5x
2
. Calculate its initial velocity.
(A)
1
10ms
(B)
1
5ms
(C)
1
2ms
(D) 4 ms
1
44
(154) A body travelling in a circle at constant speed.
(A) has a constant velocity (B) is not accelerated
(C) has an outward radial acceleration (D) has an inward radial acceleration
(155) The speed of a particle moving in a circle of radius r = 4 meter is 10 ms
1
. What is its radial acceleration ?
(A) 25 ms
2
(B) 20 ms
2
(C) 10 ms
2
(D) 15 ms
2
(156) If f is the frequency of a body moving in a circular path with constant speed. a is its centrifugal accelera-
tion, so.
(A) a f o (B)
2
a f (C)
3
a f (D)
1
a
f
Following question is Assertion - Reason type question. choose
(A) If both Assertion - Reason are true, reason is correct explanation of Asserton.
(B) If both Assertion - Reason are true but reason is not correct explanation of Asserton. (C) If
Asserton is true but Reason is false.
(D) If Reason is true but Asserton is false.
(157) Asserton :
At the highest point of projectile motion the velocity is not zero.
Reason : Only the verticle component of velocity is zero. Where as horizontal component still exists.
(A) a (B) b (C) c (D) d
Match coloum type question.
(158) A balloon rise up with constant net acceleration of 10 ms
2
. After 2 second a particle drops from the
balloon. After further 2 s match the following (take g = 10 ms
2
)
Table - 1 Table 2
(A) Hight of the particle from ground (P) zero
(B) Speed of particle (Q) 10 SI unit
(C) Displacement of particle (R) 40 SI unit
(D) Accelereation of the particle (S) 20 SI unit
(A) A - P, B - Q, C - R, D - S
(B) A - Q, B - R, C - S, D - P
(C) A - R, B - P, C - S, D - Q
(D) A - R, B - S, C - P, D - Q
Comprehensions type questions.
A particle is moving in a circle of radius R with constant speed. The time period of the particle is T Now
after time t =
T
6
.
(159) Average speed of the particle is
(A)
R
6T
t
(B)
2 R
3T
t
(C)
2 R
T
t
(D)
R
T
(160) Average velocity of the particle is
(A)
3R
T
(B)
6R
T
(C)
2R
T
(D)
4R
T
(161) Range of a projectile is R and maximum height is H. Find the area covered by the path of the projectile
and horizontal line.
(A)
2
RH
3
(B)
5
RH
3
(C)
3
RH
5
(D)
6
RH
5
45
KEY NOTE
1 A 32 B 63 B 94 A 125 C
2 C 33 A 64 A 95 C 126 B
3 A 34 C 65 B 96 C 127 B
4 D 35 D 66 B 97 C 128 C
5 B 36 C 67 C 98 D 129 B
6 A 37 C 68 B 99 A 130 B
7 A 38 A 69 C 100 B 131 C
8 D 39 B 70 D 101 C 132 B
9 C 40 B 71 B 102 B 133 C
10 C 41 A 72 B 103 D 134 A
11 C 42 B 73 C 104 B 135 A
12 A 43 C 74 C 105 C 136 C
13 C 44 C 75 B 106 C 137 B
14 D 45 D 76 B 107 A 138 C
15 B 46 B 77 D 108 D 139 B
16 B 47 C 78 B 109 D 140 D
17 B 48 A 79 C 110 B 141 C
18 C 49 B 80 B 111 A 142 B
19 D 50 C 81 A 112 B 143 C
20 A 51 D 82 B 113 C 144 D
21 C 52 C 83 C 114 B 145 B
22 B 53 A 84 B 115 B 146 A
23 A 54 D 85 A 116 C 147 D
24 B 55 D 86 C 117 A 148 A
25 C 56 A 87 D 118 C 149 C
26 C 57 A 88 B 119 B 150 A
27 A 58 B 89 A 120 C 151 B
28 C 59 B 90 B 121 B 152 C
29 B 60 B 91 C 122 D 153 B
30 B 61 B 92 B 123 B 154 D
31 A 62 A 93 A 124 C 155 A
156 B
157 A
158 C
159 C
160 B
161 A
46

HINT
(9) Pathlenth is always greater or equal to displacement
(18)
1 2
1 2
2VV
Average speed =
V + V
(23)
2 2
0 0
V 2V
h = speed =
2g 2g 6
(24)
dx dv
V = and a =
dt dt
(25)
change in velocity
Find
time
change in velocity is the area covered by the graph.
(30)
2 2
2 1
1 1
h h = g (t +1) gt = 45
2 2
(31)
ds
V = = 2 +10t
dt
(32,33)
2
0
1
h = V t gt
2
2
0
1
gt V t + h = 0
2

2 0
2V 2h
t + + = 0
g g

There are two real values of t.


(35) slope is
0
0
V
x
(37) Aera covered by a t graph gives change in velocity. .
(42) acceleration = slope =
tan
(44)
da
= constant
dt
da dv
= k
dv dt

da k
=
dv a

ada = kdv
} }
2
a
= kv
2

47
(45)
2
2
d x
a =
dt
(48) If maximum velocity is V.
V = a
1
t
1
and V = a
2
t
2
1 2
1 2
v v
T = t + t = +
a a
1 2
1 2
a a T
V =
a + a
(59)
2 2
1 2
1 1
h = gt and (h + 3h) = gt
2 2
(61)
Q
R
R
S
S
2 2 2
R R 2R cos +
2 2 2
= 2R -2R Cos 2R (1-Cos)

= 2Rsin
2
s =
(65)
2
0
V
stopping distance ds =
2g
(66) V = V
0
+ at
2
0
1
d = V t + at
2
(69)
2 2
1
H = gt H t
2
(70) At time t velocity V = gt
(70)
2
0
1
y = V t + gt
2
2
t 1 t
y = gt g
2 2 2
| |
|
\ .
(73) V
1
+ V
2
= 5
V
1
V
2
= 1 So 2V
1
= 6
V
1
= 3m/s
(74)
1
2
V = (A + y)
48
dv dv dy
a =
dt dy dt
=
(75)
2
v
s =
2a
(81) V = Ax + B
a = Av
a V
(82)
2
2
dx dx
= c and = 0
dt dt
y = Ax
3
+ B
2
dy dx
= 3Ax
dt dt

2
2
d y dx dx
and = 6Ax
dt dt dt
2
2
2
d y
= 6Axc
dt

2 2
^ ^
2 2
d x d y
Now a = i + j
at dt

(95)
C A B = +

2 2 2
C = A + B + 2ABcos
(116)
A B = ABcos

A B = ABsin

(127)
^
A + B+ C = j

(130)
2
4 4
+ + p =1
9 9
(131) Velocity of A wrt ground
^
AG
V = 10 l

Velocity of B w.r.t. ground


^
BG
V = gt j

BA BG GA
V = V V +

49
(138)
B
V

M V
MB V
MW WB MB V + V = V

MW V = Velocity of man w.r.t. water

WB V = Velocity of water w.r.t. Bank

MB V = Velocity of man w.r.t. Bank

WB
MB
V
tan =
V
(141)
( )
2
r = 3ti 2t j


dr
V = = 3i 4t j
dt


4
tan = t
3
(142) x = 2t
2
= 8
t = 2s
2 2
3 3
y = t = (2) = 6m
2 2
(143) x = Acoswt y = A (i sinwt)
dx
V = = Asinwt
dv
Vy = A

coswt
2 2
V = Vx + Vy
(145)
o
A
B
C
t
1
= t (0A) = t (BC)
t
2
= t (0B)
t
1
+ t
2
= t (0A) + t (0B)
t
1
+ t
2
= t (BC) + t (0B)
50
(146) y = Ax Bx
2
dy
= A 2Bx
dx

At maximum height A 2Bx = 0


A
x =
2B

(147)
Q
(148)
0 0 0
V = V cosi + V sinj


( )
0 0
V = V cosi + V sm gt j


0
V V = 0

0
V
t = Now find x
gsin

(149) At time t V
x
= V
0
=
0
V cos
y
V = Visin gt
0
0
V sin gt Vy
tan = =
Vx V cos
(150) Range
2
0
V sin2
R =
g
0
f
2Vsin
Flight time t =
g
f
R
Average velocity =
t
(156) Cenrifugal acceleration
2
2
2 2
r
v 2r 1
a = = = 4 rf
r T r
| |
|
\ .
(161)
R
0
A = ydx
}
2
0
gx
take y = xtan
2V xcos
51
Unit - 3
Laws of Motion
52
SUMMARY
Important Points
The law of inertia given by Galileo was represented by Newton as the first law of motion :" If no external
force acts on a body, the body at rest remains at rest and a body in motion continues to move with the
same velocity."
This law gives us definition of force.
The momentum of a body

= v m p is a vector quantity.It gives more information than the velocity.It's
unit is
1
kgms

or Ns and dimensional formula M


1
L
1
T
-1
.
Newton's second law of motion : The time-rate of change in momentum of a body is equal to the
resultant external force applied on the body and is in the direction of the exterrnal force.

F =
dP
dt
ma =

is the vector relationship


The SI unit of force is newton (=N) . 1 N = 1 kg ms
2
. This law gives the value of force.It is consistent
with the first law.(

F =0 indicates that
a

=0). In this equation the acceleration of the body a

is that
which it has when the force is acting on it.(Not of the Past!) .

F
is only the resultant external force.
The impulse of force is the product of force and the time for which it acts. when a large force acts for
a very small time, it is difficult to measure

F and t A but the change in momentum can be measured,


which is equal to the impulse of force (

F

t A
)
Newton's third law of motion: " To every action there is always an equal and opposite reaction."
Forces always act in pairs, and,

F
AB
=

F
BA
. The action and the reaction act simultaneously.They act on
different bodies,hence they cannot be cancelled by adding. But the resultant of the forces between
different parts of the same body becomes zero.
The law of conservation of momentum is obtained from Newton's second law and the third law.It is
written as-"The total momentum of an isolated system remains constant."
The concurrent forces are those forces of which the lines of action pass through the same point. For
equilibrium of the body, under the effect of such forces, E

F must be = 0 Moreover, the sum of the
corresponding components also should be zero. ( E F
x
= 0, E F
y
=o, E F
z
= 0)
- Friction is produced due to the contact force between the surfaces in contact.It opposes the impending or
the real relative motion.
Static frictional force f
s
sf
s (max) =

s
N and the kinetic friction is f
k
=
k
N
53
S
= coefficient of static friction
k
=

coefficient of kinetic friction and
k s
<
* The reference frame , in which Newton's first law of motion is obeyed is called the inertial frame of
reference and the one in which it is not obeyed is called non-inertial frame of reference. The frame of
reference with constant velocity is an inertial frame of reference and one which has acceleration is non-
inertial frame of reference.
* On a body performing uniform circular motion a force equal to mv
2
/ r acts towards the centre of the
circular path. This is called the centripetal force.
The maximum safe speed on level curved road is v
max
= rg
s
The maximum safe speed on a banked curved road is v
max
=
|
|
.
|

\
|

+
tan 1
tan
rg
s
s
* Motion of a body on a friction less inclined plane:
As shown in figure body of mass m is placed on a smooth inclined plane making an angle

with the
horizontal.
Here, ( ) N cos mg 1 = u ( ) ma sin mg 2 = u
acceleration of body

= gsin

.
* Motion of a body on a Rough inclined plane:
(1) Body moving down : If body moving down
with acceleration a then,
mg sin

ma N
s
= and N = mg cos

( ) ma cos mg sin mg
s
=

a ( ) cos sin g
s
=
(2) Body moves up : If body is moving upwards with acceleration a then ( )
s
g sin cos . a = +
Pseudo Force : In non-inertial frame of reference due to acceleration one more additional force acting on
a body in the opposite direction of acceleration of frame of reference is called pseudo force (F
P
) .
when a man of weight m climbs on the rope with acceleration a then tension in the rope is T = m(g + a).
When man sliding down with acceleration a then tension in the rope is T = m(g a).
54
* When masses in contact:
In figure masses m
1 ,
m
2
and m
3
are placed in contact on a surface.
acceleration
1 2 3
F
a
m m m
=
+ +
Force on m
1
: F
1
= F
Force on m
2
: F
2
= (m
2
+ m
3
)
( )
2 3
1 2 3
m m F
a
m m m
+
=
+ +
Force on m
3
:
( )
3
3 3
1 2 3
m F
F m a
m m m
= =
+ +
* Masses connected by strings:
Consider two masses
1
m and
2
m placed on a frictionless horizontal surface connected by a light
inextensibule string.
If
1
m is pulled by a force F then a tension T is developed,
in the string.
For
1
m : For
1 2 2
F T = m a , m : T = m a

Tension in string
2
1 2
,
m F
T
m m
=
+
acceleration
1 2
.
F
a
m m
=
+
a
2
m
1
m
T T F
F
a
1
m
2
m
3
m
55
MCQ
For the answer of the following questions choose the correct alternative from among the given ones.
1. The velocity of a body of mass 20 kg decreases from 20 ms
1
to 5 ms
1
in a distance of 100 m. Force
on the body is
(A) -27.5 N (B) -47.5 N (C) -37.5 N (D) -67.5 N
2. A ball of mass 0.2 kg is thrown vertically upwards by applying a force by hand. If the hand moves
0.2 m while applying the force and the ball goes upto 2 m height further, find the magnitude of the
force. (Consider g = 10 ms
2
)
(A) 16 N (B) 20 N (C) 22 N (D) 4 N
3. Formula for true force is
(A)
F ma =
(C)
( ) d mv
F
dt
= (B)

dv
F m
dt
=
(D)
2
2
d x
F = m
dt
4. A particle moves in the XY plane under the influence of a force such that its linear momentum is
^ ^
P(t) A[ i cos (kt) j sin(kt)]

=
where A and k are constants. The angle between the force and
momentum is
(A) 0
0 `
(B) 30
0
(C) 45
0
(D) 90
0
5. Force of 5 N acts on a body of weight 9.8 N. what is the acceleration produced in ms
-2
(A)49.00 (B) 5.00 (C) 1.46 (D) 0.51
6. A lift is going up. The total mass of the lift and the passenger is 1000 kg The variation in the speed of
the lift is as given in the graph. The tension in the rope pulling the lift at t= 10.5 sec will be
(A) 8000 N
(B) Zero
(C) 12000 N
(D) 17400 N
7. Same force acts on two bodies of different masses 2 kg and 4 kg initially at rest. The ratio of times
required to acquire same final velocity is
(A) 2:1 (B) 1:2 (C) 1:1 (D) 4:16
8. Which of the following quantities measured from different inertial reference frames are same
(A) Force (B) Velocity (C) Displacement (D) Kinetic Energy
2 10 12
3.6
1
t (sec)
56
9. 10,000 small balls, each weighing 1 g strike one square cm of area per second with a velocity
100 ms
1
in a normal direction and rebound with the same velocity. The value of pressure on the surface
will be
(A)
2 3
Nm 10 2

(B)
2 5
Nm 10 2

(C)
2 7
Nm 10

(D)
2 7
Nm 10 2

10. When the speed of a moving body is doubled


(A) Its acceleration is doubled
(B) Its momentum is doubled
(C) Its kinetic energy is doubled
(D) Its potential energy is doubled
11. A particle moves in the XY Plane under the action of a force F such that the components of its linear
momentum P at any time t are Px = 2 cost, Py = 2 sint. The angle between F and P at time t is
(A) 90
0
(B) 0
0
(C) 180
0
(D) 30
0
12. A player caught a cricket ball of mass 150 g moving at the rate of 20 ms
-1
. If the catching process be
completed in 0.1 s the force of the blow exerted by the ball on the hands of player is
(A) 0.3 N (B) 30 N C) 300 N (D) 3000 N
13. A body of mass 5 kg starts from the origin with an initial velocity
^ ^
-1
u=30 i +40 j ms

. If a constant
Force

^ ^
F= i +5 j N
| |
|
\ .

acts on the body, the time in which the y-component of the velocity becomes
zero is
(A) 5 s (B) 20 s (C) 40 s (D) 80 s
Third law of motion
14. Swimming is possible on account of
(A) First law of motion (B) second law of motion
(C) Third law of motion (D) Newton's law of gravitation
15. A cold soft drink is kept on the balance.When the cap is open, then the weight
(A) Increases (B) Decreases (C) First increase then decreases (D) Remains same
57
Conservation of linear momentum and impulse
16. A wagon weighing 1000 kg is moving with a velocity 50 km h
-1
on smooth horizontal rails. A mass
of 250 kg is dropped into it. The velocity with which it moves now is
(A)
1
h km 5 . 2

(B)
1
h km 20

(C)
1
h km 40

(D)
1
h km 50

17. The Figure shows the Position-time (x-t) graph of one dimensional motion of a body of mass 0.4 kg .
The magnitude of each impluse is
(A) 0.2 Ns (B) 0.4 Ns
(C) 0.8 Ns (D) 1.6 Ns
Equllibrium of Forces
18. Three Forces F
1
, F
2
and F
3
together keep a body in equilibrium. If F
1
= 3 N along the positive X- axis,
F
2
= 4N along the positive Y-axis ,then the third force F
3
is
(A) 5 N -making an angle

= tan
1
( )
4
3
with negative y-axis
(B) 5 N - making an angle

= tan
1
( )
3
4
with negative y-axis
(C) 7 N - making an angle

= tan
1
( )
4
3
with negative y-axis
(D) 7 N - making an angle

= tan
1
( )
3
4
with negative y-axis
19. A solid sphere of mass 2 kg is resting inside a cube as shown in the figure. The cube is moving with
a velocity

^ ^
1
v = 5t i+2t j ms
| |
|
\ .

Here t is the time in second. All surfaces are smooth. The sphere
is at rest with respect to the cube. what is the total force exerted by the sphere on the cube
(g = 10 ms
2
)
(A)
N 29
(B) 29 N
(C) 26 N (D)
N 89
y
0
x
A B
D C
x (m)
0 2 4 6 8 10 12 14 16
t (s)
58
20. A particle of mass 2 kg is initially at rest. A force acts on it whose magnitude changes with time. The
force time graph is shown below.
The velocity of the particle after 10s is
(A) 10 ms
-1
(B) 20 ms
-1
(C) 75 ms
-1
(D) 50 ms
-1
21. A block of mass 4 kg is placed on a rough horizontal plane. A time dependent force F = Kt
2
acts on
a block , where k = 2 2
s
N
, co-efficient of friction 0.8 = .Force of friction between the block and the
plane at t = 2 S is.....
(A) 32 N (B) 4 N (C) 2 N (D) 8 N
22. A 7 kg object is subjected to two forces (in newton)
^ ^
1
F =20i+30 j

and
^ ^
2
F = 8i-5 j

The magnitude of
resulting acceleration in ms
2
will be
(A) 5 (B) 4 (C) 3 (D) 2
23. A car travelling at a speed of 30 km/h is brought to a halt in 8 metres by applying brakes. If the same
car is travelling at 60 km/h it can be brought to a halt with the same breaking power in
(A) 8 m (B) 16 m (C) 24 m (D) 32 m
24. A given object takes n times more time to slide down 45
0
rough inclined plane as it takes to slide down
a perfactly smooth 45
0
incline. The coefficient of kinetic friction between the object and the incline is
(A)
2
1
2 n
(B) 2
1
1
n

(C) 2
1
1
n

(D)
2
1
1 - n
25. Two bodies of equal masses revolve in circular orbits of radii R
1
and R
2
with the same period Their
centripetal forces are in the ratio.
(A)
2
1
2
R
R
|
|
.
|

\
|
(B)
2
1
R
R
(C)
2
2
1
R
R
|
.
|

\
|
(D)
1 2
R R
26. Two masses M and
2
M
are joined together by means of light inextensible string passed over a
frictionless pulley as shown in fig. When the bigger mass is released, the small one will ascend with an
acceleration
(A)
3
g
(B)
3
2
g
(C) g (D)
2
g
0
10
20
F (N)
t (s)
2
M
M
59
27. A 0.5 kg ball moving with a speed of 12 ms
-1
strikes a hard wall at an angle of 30
0
with the wall.
It is reflected with the same speed and at the same angle. If the ball is in contact with the wall for
0.025 S the average force acting on the wall is
(A) 96 N (B) 48 N (C) 24 N (D) 12 N
28 A shell of mass 200g is ejected from a gun of mass 4 kg by an explosion that generates 1.05 KJ of
energy. The initial velocity of the shell is
(A) 100 ms
-1
(B) 80 ms
-1
(C) 40 ms
-1
(D) 120 ms
-1
29. A gramophone record is revolving with an anguler velocity e. A coin is placed at a distance r from the
centre of the record. The coefficient of static friction is . The coin will revolve with the record if
(A) r = ge
2
(B)
2
r
g

<
(C)
2
g
r

<
(D)
2
g
r

>
30. A stone of mass 2 k g is tied to a string of length 0.5 m It the breaking tension of the string is 900N,
then the maximum angular velocity the stone can have in uniform circular motion is
(A) 30 rad/s (B) 20 rad/s (C) 10 rad/s (D) 25 rad/s
31. A body of mass 6 kg is hanging from another body of mass 10 kg as shown in fig. This conbination is
being pulled up by a string with an acceleration of 2 ms
2
. the tension T
1
is (g = 10 ms
2
)
(A) 240 N
(B) 150 N
(C) 220 N
(D) 192 N
32. A sparrow flying in air sits on a stretched telegraph wire. If the weight of the sparrow is W ,which of the
following is true about the tension T produced in the wire?
(A) T = W (B) T < W (C) T = 0 (D) T >W
33. Fig. shows the displacement of a particle going along X-axis as a function of time. The force acting on
the particle is zero in the region
(A) AB
(B) BC
(C) CD
(D) None of these
T
1
10 kg
6 kg
a
T
2
X
A
D
i
s
p
l
a
c
e
m
e
n
t
B C
D
Time
Y
60
34. A Force (F) varies with time (t) as shown in fig. Average force over a complete cycle is-
(A) Zero (B)
2
F
0
(C)
t
0
F
(D)
0
F 2
35. A body of mass 0.05 kg is falling with acceleration 9.4 ms
2
. The force exerted by air opposite to
motion is N (g=9.8 ms
2
)
(A) 0.02 (B) 0.20 (C) 0.030 (D) Zero
36. The average force necessary to stop a hammer with 25 NS momentun in 0.04 sec is ___________N
(A) 625 (B) 125 (C) 50 (D) 25
37. Newton's third law of motion leads to the law of consrevation of
(A) Angular momentum (B) Energy (C) mass (D) momentum
38. A ball falls on surface from 10 m height and rebounds to 2.5 m. If duration of contact with floor is
0.01 sec. then average aceleration during contact is _______________ms
-2
(A) 2100 (B) 1400 (C) 700 (D) 400
39. A vehicle of 100 kg is moving with a velocity of 5
s
m
. To stop it in
10
1
sec, the required force
in opposite direction is _______________N
(A) 50 (B) 500 (C) 5000 (D) 1000
40. The linear momentum P of a particle varies with the time as follows.
2
bt a P + =
Where a and b a r e
constants. The net force acting on the particle is _____________
(A) Proportional to t (B) Proportional to t
2
(C) Zero (D) constant
41. A vessel containing water is given a constant acceleration a towards the right, along a straight horizontal
path. which of the following diagram represents the surface of the liquid ?
(A) (B) (C) (D)
42. A body of 2 kg has an initial speed 5 m/s. A force act on it for some time in the directine of motion.
The force
( ) F

---------time (t) graph is shown in figure. The final speed of the body is _________
(A) 9.25 ms
-1
(B) 5 ms
-1
(C) 14.25 ms
-1
(D) 4.25 ms
-1
2 4 4.5 6.5
F (N)
4
2.5
t (s)
F
t
0
F0
a a a a
61
43. which of the following statement is correct?
(A) A body has a constant velocily but a varying speed.
(B) A body has a constant speed but a varying value of acceleration.
(C) A body has a constant speed and zero accelaration.
(D) A body has a constant speed but velocity is zero.
44. A force of 8 N acts on an object of mass 5kg in X- direction and another force of 6 N acts on it in Y -
direction. Hence, the magnitude of acceleration of object will be
(A) 1.5 ms
2
(B) 2.0 ms
2
(C) 2.5 ms
2
(D) 3.5 ms
2
45. Three forces are acting simultaneously on a particle
moving with velocity

V
.These forces are represented
in magnitude and direction by the three sides of a
triangle ABC. The particle will now move with velocily
__________
(A) Less than
v

(B) greater than


v

(C) v

in the direction of the largest force BC (D)


v

remaining unchanged.
46. A plate of mass M is placed on a horizontal frictionless surface and a body of mass m is placed on this
plate, The coefficient of dynamic friction between this body and the plate is . If a force 2mg. is
applied to the body of mass m along the horizonal direction the acceleration of the plate will be
________________
(A)
g
M
m
(B)
( )
g
m M
m
+
(C)
g
M
m 2
(D)
( )
g
m M
m 2
+

47. On the horizontal surface of a truck ( ) 6 . 0 = , a block of mass 1 kg is placed. If the truck is
accelerating at the rale of
2
s / m 5 then frictional force on the block will be_____________N
(A) 5 (B) 6 (C) 5.88 (D) 8
48. Two blocks of nass 8 kg and 4 kg are connected a heavy string Placed on rough horizontal Plane, The
4 kg block is Pulled with a constant force F.The co-efficient of friction between the blocks and the
ground is 0.5, what is the value of F, So that the tension in the spring is constant throughout during
the motion of the blocks ? (g=10 ms
2
)
(A) 40 N (B) 60 N
(C) 50 N (D) 30 N
C
A B
M
2mg
m
8 kg
4 kg F
62
49. Seven blocks, each of mass 1 kg are arranged one above the other as
shown in figure. what are the values of the contact forces exerted on
the third block by the forth and the second block respectively ?
( g = 10 ms
-2
)
(A) 40 N, 50 N (B) 50 N, 40 N
(C) 40 N, 20 N (D) 50 N, 30 N
50. A man is standing on a spring balance. Reading of spring balance is 60 kgf. If man jumps outside
balance, then reading of spring balance____________
(A) First increase than decreases to zero (B) Decreses
(C) Increases (D) Remains same
51. A car turns a corner on a slippery road at a constant speed of 10 m/s. If the coefficient of friction is
0.5, the minimum radius of the arc at which the car turns is_____________meter.
(A) 20 (B) 10 (C) 5 (D) 4
52. A person standing on the floor of a lift drops a coin. The coin reaches the floor of the lift in time to if
the lift is stationary and the time t
2
if it is accelerated in upward direction. Than
(A)
2 1
t t = (C)
2 1
t t <
(B) t
1
> t
2
(D) Cannot say anything
53. A lift of mass 1000 kg is moving with an acceleration of 1 ms
2
in upward direction Tension developed
in the rope of lift is _________________N (g = 9.8 ms
-2
)
(A) 9800 (B) 10,000 (C) 10,800 (D) 11,000
54. Three blocks of masses m
1
,m
2
and m
3
are connected by massless strings as shown in figure, on a
frictionless table. They are Pulled with a force N 40 T
3
= . If
1
10 m kg = , kg 6 m
2
= and
kg 4 m
3
= the tension T
2
will be =________________N
(A) 20 (B) 40
(C) 10 (D) 32
55. If the surfaccs shown in figure are frictionless, the ratio of
1
T and
2
T is __________
(A)
2 : 3
(B)
3 : 1
(C)
5 : 1
(D)
1 : 5
7
6
4
2
m
1
m
2
m
3
T
1
T
2
T
3
= 40 N
3kg
15kg
T
2
T
1
12kg
30
0
F
63
56. Three masses 1 kg, 6 kg and 3 kg are connected to each
other with threads and are placed on a table as shown in
figure. The alcceleration with which the system is moving
is ______ ms
-2
(g=10ms
-2
)
(A) zero (B) 1
(C) 2 (D) 3
57. A rope which can withstand a maximum tension of 400 N hangs from a tree. If a monkey of mass 30 kg
climbs on the rope in which of the following cases-will the rope break?
(take g =10 ms
-2
and neglect the mass of rope)
(A) When the monkey climbs with constant speed of 5 ms
1
(B) When the monkey climbs with constant acceleration of 2 ms
2
(C) When the monkey climbs with constant acceleration of 5 ms
2
(D) When the monkey climbs with the constant speed of 12 ms
1
58. An object of mass 3 kg is moving with a velocity of 5 m/s along a straight path. If a force of 12 N is
applied for 3 sec on the object in a perpendicuiar to its direction of motion.The magnitude of velocity
of the particle at the end of 3 sec is____________m/s.
(A) 5 (B) 12 (C) 13 (D) 4
59. Same forces act on two bodies of different mass 2 kg and 5 kg initialy at rest. The ratio of times
required to acquire same final velocity is ____________
(A) 5:3 (B) 25:4 (C) 4:25 (D) 2:5
60. A body of mass 5 kg starts motion form the origine with an initial veiocily
^ ^
0 =30 i +40 j m/s v

If a
constant force
^ ^
F = ( i +5 j)N

acts on the body, than the time in which the Y-component of the
velocity becomes zero is __________________
(A) 5 s (B) 20 s (C) 40 s (D) 80 s
61. A Block of mass 300 kg is set into motion on a frictionless
horizontal surface with the help of frictionless pulley and
a rope system as shown in figure. What horizontal force
F should be applied to produce in the block an aeceleration
of 1 ms
-2
?
(A) 150 N (B) 100 N
(C) 300 N (D) 50 N
6 kg
T
2
T
1
T
1
T
2
1 kg 3 kg
F
a=1 ms
-2
64
62. A body of mass m rests on horizontal surface. The coefficient of friction between the body and the
surface is . If the body is Pulled by a force P as shown in figure, the limiting friction between body
and surface will be__________
A) mg (B)
(

+
2
P
mg
(C)
(


2
P
mg
(D)
(

2
P 3
mg
63. Three blocks A , B, and C of equal mass m are placed one over
the other, one on a smooth horizontal ground as shown in
figure. Coefficient of friction between any two blocks of A, B
and C is 0.5. What would be the maximum value of mass of
block D so that the blocks A, B and C move without slipping
over each other ?
(A) 3 m (B) 5 m
(C) 6 m (D) 4 m
64. A train is moving along a horizontal track. A pendulum suspended from the roof makes an angle of 4
0
with the vertical, The acceleration of the train is ___________ms
-2
(g = 10 ms
-2
)
(A) 0.6 (B) 0.7 (C) 0.5 (D) 0.2
65. A bag of sand of mass m is suspended by rope. a bullet of mass
30
m
is fired at it with a velocity
and gets emmbedded into it. The velocity of the bag finally is _________
(A)
30
31
(B)
31
30
(C)
31

(D)
30

66. Three blocks having equal mass of 2 kg are hanging on a string passing over a
pulley as shown in figure. what will be the tension produced in a string connecting
the blocks B and C
(A) zero (B) 13.1 N
(C) 3.3 N (D) 19.6 N
67. A partly hanging uniform chain of length L is resting on a rough horizontal table.

is the maximum
possible length that can hang in equilibrium The coefficient of friction between the chain and table is
____________
(A)
L
L+
l
l
(B)
L
l
(C)
L
l
(D)
L
l
l
T
2
T
1
C
B
A
A
B
C
D
65
68. As shown in figure,the block of 2 kg at one end and the other of 3 kg at the other end of a light string
are connected. It the system remains stationary find the magnitude and direction of the frictional force
(g = 10 ms
-2
)
(A) 20 N, downward on slope (B) 20 N, upward on slope
(C) 10 N Downward on slope (D) 10 N upward on slope
69. A particle is resting over a smooth horizontal floor, At t = 0, a horizontal force starts acting on it.
Magnitude of the force increses with time according to law F =
t o
, where o = is constant Match the
column after seeing the figure.
Column-1 Column-2
(a) curve (i) shows. (p) velocity against time
(b) curve (ii) shows (q) velocity against acceleration
(r) acceleration against time
(A) (i)-p (ii)-q (B) (i)-q, (ii)-r (C) (i)-r, (ii)-p (D) (i)-q, (ii)-p
70. A car of mass 1000 kg travelling at 32 m/s clashes into a rear of a truck of mass 8000 kg moving in the
same direction with a velocity of 4 m/s. After the collision the car bounces with a velocity of
8 ms
1
. The velocity of truck after the impact is ____________m/s
(A) 8 (B) 4 (C) 6 (D) 9
71. A Block of mass m = 2 kg is resting on a rough inclined plane of inclination 30
0
as shown in fignre.
The coefficient of friction between the block and the plane is =0.5 . What minimum force F shuld
be applied perpendicular to the plane of block so that block does not slip on the plane ? (g=10ms
2
)
(A) zero (B) 6.24N
(C) 2.68 N (D) 4.3 N
72. The upper half of an inclined plane of inclination

is perfectly smooth while the lower half is rough A


body starting from the rest at top come back to rest at the bottom, then the coefficient of friction for the
lower half is given by____________
(A)
s
sin = (B)
s
cot = (C)
s
2cos = (D)
s
2tan =
73. A Block of mass m = 4 kg is placed over a rough inclined plane as shown in figure, The coefficient of
friction between the block and plane is
s
= 0.6. A force F = 10 N is applied on the block of an
angle at 30
0.
The contact force between the block and the plane is ___________
(A) 27.15 N (B) 16.32 N
(C) 10.65 N (D) 32.16 N
2

k
g
3 kg
30
0
y
x
(ii)
(I)
45
0
30
0 F
30
0
N = F + mg cos 30
0
66
74. The motion of a particle of a mass m is describe by
2
gt
2
1
ut y + =
. Find the force acting on the
particle.
(A) F = ma (B) F = mg (C) F = 0 (D) None of these
75. A balloon has a mass of 10 g in air, The air escapes from the balloon at a uniform rate with a velocity
of 5
s
cm
and the balloon shrinks completely in 2.5 sec. calculate the average force acting on the
balloon.
(A) 20 dyne (B) 5 dyne (C) 0 dyne (D) 10 dyne
76. Two bodies A and B each of mass m are fixed together by a massless spring A force F acts on the mass
B as shown in figure. At the instant shown, a body A has an acceleration a. what is the accelaration of
B?
(A)
|
.
|

\
|
a
m
F
(B) FT
(C)
|
.
|

\
|

m
F
a
(D) a
77. With what acceleration (a) should a box descend so that a block of mass M placed in it exerts a force
4
Mg
on the floor of the box?
(A)
3
g 4
(B)
4
g 3
(C)
4
g
(D) 3g
78. A mass of 6 kg is suspended by a rope of length 2 m form the ceiling.
A force of 50 N in the horizontal dircction is applied at the mid Point
P of the rope- as shown in figure. what is the angle the rope makes
with the vertical in equilibrium ? (g = 10 ms
-2
) Neglect mass of the
rope.
(A) 40
0
(B) 30
0
(C) 35
0
(D) 45
0
79. The minimum force required to start pushing a body up a rough (cofficient of ) inclined plane is F
1
.
While the minimum force needed to prevent it from sliding down is F
2
. If the inclined plane makes an
angle
u
from the horizontal. such that tan 2 u = than the ratio
1
2
F
F
is
(A) 4 (B) 1 (C) 2 (D) 3
m m
F
A B
T1
P
1m
w
60 N
T2
50 N
67
80. When forces F
1
, F
2
, F
3
are acting on a particle of mass m such that F
2
and F
3
are mutually perpendicular,
then the particle remains stationary. If the force F
1
is now removed than the acceleration of the
particle is
(A)
0
Fb
m
(B)
1 2
FF
m
(C)
m
F - F
3 2
(D)
2
F
m
Asseration and reason type question
Asseration and reason are given in following question. Each question have four options. One of them is
correct select it.
(a) Asseration is true. Reason is true and reason is correct explanatin for Assertion.
(b) Asseration is true. Reason is ture but reason is not the correct explanatin of assertion.
(c) Asseration is ture. Reason is false.
(d) Asseration is false. Reason is true.
81. Asseration : Frictional forces are conservative forces.
Reason : Potential energy can be associated with frictional forces.
(A) a (B) b (C) c (D) d
82. Asseration : A body of mass 1 kg is moving with an accelaration of 1ms
-1
.
The rate of change of its
momentum is 1 N.
Reason : The rate of change of momentum of body = force applied on the body.
(A) a (B) b (C) c (D) d
83. Asseration : A body falling freely under gravity becomes weightless.
Reason : R = m(g a) = m(g g) = 0
(A) a (B) b (C) c (D) d
84. Asseration : It is difficult to move bike with its breaks on.
Reason : Rolling friction is converted into sliding friction, which is comparatively larger.
(A) a (B) b (C) c (D) d
Comprehension :-
According to newton's second low of mation, F= ma, where F is the force required to produce an
accelaration a in a body of mass m. If a = 0 than F = 0. If a force acts on a body for t seconds, the effect of
the force is given by impulse = F

t = change in linear momentum of the body.


68
with the help of the passage given above, choose the most appropriate alternative for each of the
following questions.
85 A cricket ball of mass 150 g. is moving with a velocity of 12 m/s and is hit by a bat so that the ball
is turned back with a velocity of 20 m/s. If duration of contact between the ball and the bat is
0.01 sec. The impulse of the force is
(A) 7.4 NS (B) 4.8 NS (C) 1.2 NS (D) 4.7 NS
86. Average force exerted by the bat is
(A) 480 N (B) 120 N (C) 1200 N (D) 840 N
87. The force acting on a body whose linear momentum changes by 20 kgms
-1
in 10 sec is
(A) 2 N (B) 20 N (C) 200 N (D) 0.2 N
88. An impulsive force of 100 N acts on a body for 1 sec What is the change in its linear momentum ?
(A) 10 N-S (B) 100 N-S (C) 1000 N-S (D) 1 N-S
89. Match the column
Column - I Column - II
(a) Body lying on a horizontal surface (p) is a self adjusting force
(b) Static friction (q) is a maximum value of static friction
(c) Limiting friction (r) is less than limiting friction
(d) Dynamic friction (s) force of friction = 0
(A) a-s, b-p, c-q, d-r (B) a-p, b-q, c-r, d-s
(C) a-s, b-r, c-q, d-p (D) a-r, b-q, c-p, d-s
90. A block B, placed on a horizontal surface is pulled with initial velocity V. If the coefficient of kinetic
friction between surface and block is , than after how much time, block will come to rest ?
(A)
g
v
(B)
g
v
(C)
g
v
(D)
v
g
91. As shown in figure a block of mass m is attached with a cart. If the coefficient of static friction between
the surface of cart and block is than what would be acceleration o of cart to prevent the falling of
block ?
(A)
mg

>
(B)
m
g
>
(C)

g
>
(D)
g

<
69
92. A particle of mass m is at rest at t = 0. The force exerting on it in x-direction is F(t) = F
o
e
-bt
. Which one
of the following graph is of speed ( ) V t t
(A) (B)
(C) (D)
KEY NOTE
1 C 31 D 61 A 91 C
2 C 32 D 62 C 92 B
3 C 33 A 63 A
4 D 34 A 64 B
5 B 35 A 65 C
6 A 36 A 66 B
7 B 37 D 67 D
8 A 38 A 68 A
9 D 39 C 69 C
10 B 40 A 70 D
11 A 41 C 71 C
12 B 42 C 72 D
13 C 43 C 73 A
14 C 44 B 74 B
15 C 45 D 75 A
16 C 46 A 76 A
17 C 47 A 77 B
18 C 48 B 78 A
19 C 49 B 79 D
20 D 50 A 80 A
21 D 51 A 81 D
22 A 52 C 82 A
23 D 53 C 83 A
24 B 54 D 84 A
25 B 55 D 85 B
26 A 56 C 86 A
27 C 57 C 87 A
28 A 58 C 88 B
29 C 59 D 89 A
30 A 60 C 90 A
0
F b
m
vt
t
0
F b
m
vt
t
0
F b
m
vt
t
0
F b
m
vt
t
70
HINT
1. F = ma = m
2 2
2S
v u | |
|
\ .
2.
g 2
u
H
2
max
=
max
H g 2 u =
This velocity is supplied to the ball by hand and initially the hand was at rest. It acquires this velocity
in distance of 0.2 meter.
2
2
u
a
S
=
So upword force F = m (g+a)
3. According to newton's second law force = rate of change of linear momentum.
4.
( )
^ ^
P t A i cos kt j sin kt

| |
=
|
\ .
|
.
|

\
|
=

) t ( P
dt
d
F
= Ak
^ ^
i sin kt j cos kt
| |

|
\ .
( )
2
F.P A k cos kt.sin kt sin kt.cos kt = +

0 P . F =

5. As weight W = mg = 9.8 N

m = 1 Kg
F
a
m
=
6. At 10.5 sec the lift is moving upword with acceleration
2
ms 8 . 1
2
6 . 3 0
a

=

=
Tension in rope T = m (g-a)
= 1000 (9.8 - 1.8)
= 8000 N
71
7.
a
1
t
a
V
t o =
(V is constant)
2
1
4
2
m
m
a
a
t
t
2
1
1
2
2
1
= = = =
[
m
1
a o
as F is constant]
9.
( ) ( )
n mv - -mv
F
P= =
A
A


2nmv
=
A
11.
^ ^
P Pxi Py j

= +
,
dp
F
dt

=
Now
o
. 0 90 F P

= =
12. Force exerted by ball,
dv
F m
dt
| |
=
|
\ .
13. v
y
= 40 ms
1
Fy = 5 N m = 5 kg
So a
y
=
1
1
y
F
ms
m

=
as
y y
v u at = +
0 = 40 1t

t = 40s
14. Swimming is a result of pushing water in the opposite direction of the motion
15. Gas will cane out with sufficient speed in forward direction, So recetion of this forward force wil change
the reading of the spring balance.
16. According to the principle of conservation of linear momentum

km
100050 = 1250v \ v = 40
h
17. impulse = ( )
m
f i
P v v A =
19. As
^ ^
5t i 2t j

= +
^ ^
a 5i 2 j = +

( )
^ ^
ma
x y
F i m g a i

= + +
( )
2
2
1 1
y
F m ax g a

= + +
= 26 N
ay
ax
max
m(g+ ) ay
72
20. Velocity after 10 sec.
2
1
=
(Area enclosed between F - t graph) = 50 ms
-1
21. Fmax = mg = 0.8

10 = 32 N
applied force at t = 2S is
F = kt
2
= 2(2)
2
= 8 N
The body fails to move. Hence the force of friction at t = 2s = applied force = 8 N
22.
2 1 F F F

+ =
F
a
m
=
23. Here distance
2
dou
as is doubled, d becomes 4 times.
24.
2
1
S ut at
2
= +
as u = 0
2S
t
a
=
for smooth plane a = gsin

For rough plane


1
a g (sin- cos) =
( )
1
2
sin- cos
S
t
g
=

1
2
sin
S
t nt n
g
= =
( )
2
n g sin- cos g sin =
When

= 45
o
sin

= cos

=
2
1

2
n
1
- 1 =
25.
( )
2
2
2 2
2
2 4
m R
mv mR
F mR
R R T T
e
t t | |
= = = =
|
\ .
As m and T are same for two bodies
F oR
26.
( )
1 2
1 2
m m g
a
m m

=
+
M
M
2
g
M
M
2
| |

|
=
|
+
\ .

3
g
=
73
27. As impulse = change in linear momentun
Ft =
0
2 sin30 mv

2m sin30
F
t
0
=
28.
1 1 2 2
m v +m v =0
1 1
2 1
2
m v
v = v = -
m 20
Now
2 2
1 1 2 2
1 1
E = m v + m v
2 2
By solving weight
1
v = 100 ms
-1
29. The coin will revolve with the record if centrifugal force mrw
2
< f

2
mr <R<mg e
2
g
r <
e
30.
2
30
F
F mr
mr
= e e = =
rad
s
31. Total mass pulled up m = 10 + 6 = 6 kg
( ) a g m T
1
+ =
33. d-t graph AB is linier

Velocity is uniforce

a = 0

F = 0
35. ( ) a g m Fair =
36.
N 625
04 . 0
25
t
P
F = =
A
A
=
38.
0 1
2 v gh = and
1
2 v gh =
( )
2 1
madt m v v = +
2 1
v v
a
dt
+
=
40.
t F bt 2 0
dt
dp
F o + = =
42. ( )
2 1
. F t m v v A = and
F. t Area of lower part of graph F t A =
74
44.
2
2
2
1
F F F + =
and
M
F
a =
46. Resultant force F = 2 mg- mg = mg
accleration in plate
m
F
a =
48. 0 a = g 8 T =

g 4 T F + =
51.
2

g
v
v rg r = =
52.
2
1
1
d gt
2
=
and ( )
2
2
1
d g a t
2
= +
By comparing both eqn.
( )
2 2
1 2
1 1
gt g a t
2 2
= +
2 1
t t >
53. Tensile force T = m(g+a)
54. Tensile force
( )
3 2 1
2 1 3
2
m m m
m m T
T
+ +
+
=
55. acceleration
3 2 1
m m m
F
a
+ +
=
a m T
1 2
=
( ) a m m T
2 1 1
+ =
56. acceleration
10
6 3 1
1 3
a
|
.
|

\
|
+ +

=
57. Calculate F = m (g+a)
58.
P F t

A = A
m (v
y
v
0y
) =
F t A
12
y
v j =
and resultant velocity of t = 3 Sec
2 2
x y
v v v =
59. ( )
1 1 0 1 1
- Ft m v v m = =
( )
2 2 0 2
- Ft m v v m v = =
2
1
2
1
m
m
t
t
=
75
60. Fy = m
|
|
.
|

\
|
t
V V
0 y
61. Tension T = 2 F

ma = 2 F

F =
2
ma
62. N + P sin

= mg

N = mg - P sin

= mg -
2
P
limiting friction F
s
= N
s
63. g M (max) F
A s s
=
max
max
A
F
g
a
2
m
= =
and
D
max D
D
m g
a M 3m
3m m
= =
+
OR W T T T
3 2 1
= + +
g . M mg 3 g m 2 mg
D
= + +
m 3 m
2
1
6 m 6 M
D
= = =
64. ma = mg sin

a = 10 sin 4
o
a = 0.7 ms
-2
65. According to the low of conservation of momentum
m
m
30 30
m
v v
| |
= +
|
\ . 31
v
v =
66. for Block A T
2
-2g = 2a ....(1)
for block B T
1
+2g-T
2
= 2a ....(2)
for block C 2g - T
1
= 2a ....(3)
Adding (1) & (2) T
1
= 4a ....(4)
from (3) & (4) 2g-4a = 2a
m
m
30 30
m
v v
| |
= +
|
\ .
3
g
a =

N 1 . 13
3
g
4 a 4 T
1
= = =
67.
L
M
=
Weight of chain of lenght

= g
Weight of a chain of lenght (
L
) = ( ) g - L
s s
R (L- ) g
S
f = = and g
S
f =
( )
s
g . L- g =

=
L
s
76
68. for 3 kg body m
1
a = m
1
g-T
1 1
T m g m a......(1) =
for 2 kg body T - m
2
gsin

= m
2
a
2
2
m g
T m a ......(2)
2
=
from (1) and (2) a = g = 10 ms
-2
S 2
= m a=20 N f
69.
dp
F t t P tdt
dt
= o = o = o }
2
t

2
mv
o
=
2
t v Which shows graph of
2
t v is parabola so (ii) - (P)
now
1
(2 )
2
dv m
t
dt
=
o
a t Which shows graph of a - t is linier so (i)....R
70. according to eqn
1 1
1 1 2 2 1 1 2 2
m v m v m v m v

+ = +


1
2
9
m
v
s
=
71. N F mgcos30 = + and N 30 sin mg
s
=
72. For upper half
2 2
0
v - v = 2 ad
( )
2
2
sin g 2 0 = u
2
sin v gl = u
For lower half
2 2
1 0
v v 2a ' d =
| |
s
0 gl sin 2g sin cas .
2
u= u u

tan 2
2
=
73. 45 cos mg 30 sin F N = +
( ) N max f
s s
=
( )
2 2
c s
f N +f max =
75.
( ) .
dp d dm
F mv v
dt dt dt
| |
= = =
|
\ .

dm 10
4gram
sec
dt 2.5
= =
and
cm
5
s
v =
76.
T F ' ma = ma F ' ma =
a
m
F
' a =
77. As the box is descending
( ) a g m R =
mg 3g
mg ma a
4 4
= =
77
78. N 50 sin T
1
= and N 60 cos T
1
=
5

4
tan =


0
40 =
79. ( ) cos sin mg F
1
+ =
( ) cos sin mg F
2
=

3
- 2
2
- tan
tan
F
F
2
1
=
+
=
+
=
80. Here
3 2 1
F F F + =
a
F
a
1
=
90.
0 0
0 v v at as v and v v = + = =
t
V
a =

ma f =

ma mg =
V V
g t
t g
= =
91. pseudo force exerted on block F = m
force of friction f = m N =
when tall not will bolck , W f >
mg m >
m
g
>
92. F(t) = F
0
e
bt

ma = F
0
e
bt

a =
bt 0
e
m
F


dv
dt

=

bt o
e
m
F

dt
bt o
F
dv e dt
m

=
Integrating
0
t
bt
Fo
v e dt
m

=
}

0
t
bt
o
F e
v
m b

(
=
(



( )
1
bt o
F
v e
mb

=
which is exponential function So graph (b) is true.
78
Unit - 4
Work Energy
And Power
79
SUMMARY

The product of the magnitude of the displacement during the action of a force and
the magnitude of the component of the force in the direction of displacement is
known as work. Its unit in joule and its dimensional formula is M
1
L
2
T
-2
.

If angle between force and displacement is u


(i) for u = 0 W = Fd (ii) for u =
t
2
w = 0 (iii) for u = t w = -Fd

If
u
is an acute, work is positive and work is done by the force on the body. If u
is an obtuse angle, work is negative and work is done by the body aganist the force.

Work on a body lying on rough horizontal


surface acted by force. As there is no
displacement along Y-axis
N + Fsin
u
= Mg

N = Mg - F sinu
Responsible resultant force for displacement
along X-direction
= F cosu - N = F cosu - (Mg - Fsinu)
Work W = [F cosu - (Mg - Fsinu]d = [F(cosu - sin u) - Mg] d

The work done by a variable force is given by W =


F . d
f
i
l
}

If a variable force and displacement due to it are in the same direction the area
below the F - x graph gives value of work.

The ability of a body to do work by virtue of its motion is known as its Kinetic
energy. If the velocity of a body of mass `m' is `v' its kinetic energy is
K =
m 2 / p mv
2
1
2 2
=
. It is scalar quantity..

Work Energy Theorem : Work done by a resultant force on a body is equal to


the change in its kinetic energy. W
2 2
0 0
1
mv mv K
2

1
= = =
2

When a body has the ability to do work due to its position in a force field or its
configuration. It is known as potential energy. It is scalar quantity.

If the gravitational potential energy, due to the gravitational field of Earth, is


randomly taken to be zero on its surface the potential energy of a body of mass
m, at height h is mgh, where g is the gravitational acceleration. The value of `h' is
negligible compared to the radius of the earth.

The sum of the potential energy (U) and the kinetic energy (K) of a substance is
called the mechanical energy. E = K + U.
80

Considering potential energy of a spring as zero in its normal state, if its langth is changed
by x, the potential energy of the spring is U =
2
kx
2
1
. Here k is the spring constant. Unit
of k is N/m and dimensional formula is M
1
L
0
T
-2

The forces for which work done is independent of the path of motion of the body but
depends only on initial and final positions, are called conservative force. The force of
gravitation or the restoring force developed in a spring due to its compression or extension
are conservative forces.

The relation between the conservative force and the potential energy is F =
dU
dx
.

The time-rate of doing work is called power. Its unit is watt (J/s). Its dimensional formula
is M
1
L
2
T
-3
.
The power P = W / t or P = F . v

1 horse power 746 watt
Unit of electric energy for domestic use is 1 unit = 1 kWh = 3.6 10
6
J

If during collision of two bodies the kinetic energy is conserved the collision said to be
elestic.

A body of mass m
1
moving with velocity v
1
undergoes elastic collision with a body of mass m
2
moving with velocity v
2
in the direction of v
1
If after collision their velocities are
'
1
v and
'
2
v .
'
1
v = 2
2 1
2
1
2 1
2 1
v
m m
m 2
v
m m
m m
+
+
+


and
'
2
v =
1 2 1
1 2
1 2 1 2
2m m m
v v
m m m m

+
+ +

In case of complete inelastic collision bodies colliding move together after collision with a
common velocity v. In this case
2 1
2 2 1 1
m m
v m v m
v
+
+
=

A body of mass m
1
moving with velocity v
1
collides with a stationary body of mass m
2
elastically. They move with velocities
'
1
v and
'
2
v making angles u
1
and u
2
with direction of v
1
then
2 2 2 1 1 1 1 1
cos ' v m cos ' v m v m u + u =
2 1 2 1 1 1
sin ' v m sin ' v m 0 u u = and
2
2 2
2
1 1
2
1 1
' v m ' v m v m + =

As Newton's low of impact, the co-efficient of restitution is,


e =
2 1
1 2
' '
, ' '
v v
e
v v

depends on the types of materials of bodies colliding.


For perfectly elastic collision e = 1 and for perfectly inelastic collision e = 0.
Equations of velocities after collision of two bodies can be written as :
2
2 1
2
1
2 1
2 1
1
v
m m
m ) e 1 (
v
m m
) e m m (
' v
+
+
+
+

=
and
1 1 2
2 1 2
1 2 1 2
(1 e)m (m e m )
v' v v
m m m + m
+
=
+
81
MCQ
For the answer of the following questions choose the correct alternative from among the given ones.
1. How much is the work done in pulling up a block of wood weighing 2KN for a
length of 10m on a smooth plane inclined at an angle of 30
o
with the horizontal ?
(A) 1.732 KJ (B) 17.32 KJ (C) 10 KJ (D) 100 KJ
2. A force of 7N, making an angle
u
with the horizantal, acting on an object displaces
it by 0.5m along the horizontal direction. If the object gains K.E. of 2J, what is the
horizontal component of the force ?
(A) 2N (B) 4 N (C) 1 N (D) 14 N
3. A 60 kg JATAN with 10 kg load on his head climbs 25 steps of 0.20m height each.
what is the work done in climbing ? (g = 10 m/s
2
)
(A) 5 J (B) 350 J (C) 100J (D) 3500J
4. A ball of mass 5 kg is stiding on a plane with intial velocity of 10 m/s. If co-
efficient of friction between surface and ball is
2
1
, then before stopping it will
describe...... (g = 10 m/s
2
)
(A) 12.5 m (B) 5 m (C) 7.5 m (D) 10 m
5. The relationship between force and
position is shown in the figure given (in
one dimensional case) calculate the
work done by the force in displacing a
body from x = 0 cm to x = 5 cm
(A) 30 ergs (B) 70 ergs
(C) 20 ergs (D) 60 ergs
6. The force constant of a wire is K and that of the another wire is 3k when both the
wires are stretched through same distance, if work done are W
1
and W
2
, then...
(A)
2
2 1
w 3w = (B) W
2
= 0.33W
1
(C) W
2
= W
1
(D) W
2
= 3W
1
7. A ball is released from the top of a tower. what is the ratio of work done by force
of gravity in first, second and third second of the motion of the ball ?
[h
n
(2n 1)]
(A) 1 : 2 : 3 (B) 1 : 4 : 9 (C) 1 : 3 : 5 (D) 1: 5 : 3

6
x (cm)
20
10
0
10
20
F
o
r
c
e

(
D
y
n
e
)
82
8. A spring of spring constatnt 10
3
N/m is stretched initially 4cm from the
unstretched position. How much the work required to stretched it further by another
5 cm ?
(A) 6.5 NM (B) 2.5 NM (C) 3.25 NM (D) 6.75 NM
9. The mass of a car is 1000 kg. How much work is required to be done on it to make
it move with a speed of 36 km/h ?
(A) 2.5

10
4
J (B) 5

10
3
J (C) 500

J (D) 5

10
4
J
10. A body of mass 6 kg is under a force, which causes a displacement in it given by
S =
3
t 2
3
(in m). Find the work done by the force in first one seconds.
(A) 2 J (B) 3.8 J (C) 5.2 J (D) 24 J
11. A 8 kg mass moves along x - axis. Its
accelerations as a function of its
position is shown in the figure. What is
the total work done on the mass by the
force as the mass moves from x = 0 to
x = 6cm ?
(A) 48

10
-3
J (B) 98

10
-3
J
(C) 4.8

10
-3
J (D) 9.8

10
-3
J
12. The work done by a force acting
on a body is as shown in the graph.
what is the total work done in
covering an intial distance of
15m ?
(A) 50 J (B) 75 J
(C) 100 J (D) 25 J
13. A spring gun of spring constant
2
90 10 N/ M
is compressed 4cm by a ball of mass
16g. If the trigger is pulled, calculate the velocity of the ball.
(A) 60 m/s (B) 3 m/s (C) 90 m/s (D) 30 m/s
14. A uniform chain of length 2m is kept on a table such that a length of 50cm hangs
freely from the edge of the table. The total mass of the chain is 5kg. What is the
work done in pulling the entire chain on the table. (g = 10 m\s
2
)
(A) 7.2 J (B) 3 J (C) 4.6 J (D)

120 J
F
o
r
c
e

(
N
)
20
15
10
5
0
0
0

2 4 6

a
(
c
m
/
s
e
c
)
2

x (m)
83
15. A uniform chain of length L ans mass M is lying on a smooth table and one third
of its is hanging vertically down over the edge of the table. If g is acceleration due
to gravity, the work required to pull the hanging part on to the table is
(A) MgL (B) MgL / 3 (C) MgL/9 (D) MgL / 18
16. A cord is used to lower vertically a block of mass M by a distance d with constant
down-word acceleration
9
2
.work done by the cord on the block is
(A) - Mgd/2 (B) Mgd/4 (C) -3Mgd/4 (D) Mgd
17. A block of mass 5 kg is resting on a smooth surface. At what angle a force of 20N
be acted on the body so that it will acquired a kinetic energy of 40J after moving
4m
(A) 30
0
(B) 45
0
(C) 60
0
(D) 120
0
18. Natural length of a spring is 60 cm, and its spring constant is 2000N/m. A mass
of 20kg is hung from it. The extension produced in the spring is..... (g = 9.8 m/s
2
)
(A) 4.9 cm (B) 0.49 cm (C) 9.8 cm (D) 0.98 cm
19. The potential energy of a body is given by U = A - Bx
2
(where x is displacement).
The magnitude of force acting on the particle is
(A) constant (B) proportional to x
(C) proportional to x
2
(D) Inversely proportional to x
20. A uniform chain of length L and mass M is lying on a smooth table and (
4
1
)
th
of
its length is hanging vertically down over the edge of the table. If g is acceleration
due to gravity, the work required to pull the hanging part on to the table is
(A) MgL (B) MgL/9 (C) MgL/18 (D) MgL/ 32
21. If Wa, Wb, and Wc

represent the work done in
moving a particle from X to Y along three different
path a, b, and c respectively (as shown) in the
gravitational field of a point mass m, find the
correct relation between Wa, Wb and Wc
(A) Wb > Wa > Wc (B) Wa< Wb < Wc
(C) Wa > Wb > Wc (D) Wa = Wb = Wc
22. An open knife edge of mass m is dropped from a height h on a wooden floor. If
the blade penetrates upto the depth d into the wood, the average resistance offered
by the wood to the knife edge is,
(A) mg (B) mg (1+
d
h
) (C)
2
h
mg(1 )
d
+ (D) mg (1 -
d
h
)
84
23. A toy car of mass 4 kg moves up a ramp
under the influence of force F plotted
against displacement x. The maximum
height attained is given by
(Take g = 10 m/s
2
)
(A) ymax = 5m
(B) ymax = 10m
(C) ymax = 15m
(D) ymax = 20m
24. A particle of mass 0.5kg travels in a straight line with velocity v =
2
3
ax
,
Where a = 5
1
s m
2
1

. The work done by the net force during its displacement from
x = 0 to x = 2m is
(A) 50 J (B) 45 J (C) 25 J (D) None of these
25. Velocity time graph of a particle of mass 2kg
moving in a straight line is as shown in figure.
Wrok done by all forces on the particle is
(A) 400 J
(B) -400 J
(C) -200 J
(D) 200 J
26. A mass of M kg is suspended by a weight-less string, the horizontal force that is
required to displace it until the string makes an angle of 60
o
with the intial varticale
direction is
(A)
Mg / 3
(B) Mg . 2 (C)
Mg / 2
(D) Mg . 3
27. Given below is a graph between a
variable force (F) (along y-axis)
and the displaement (X) (along x-
axis) of a particle in one
dimension. The work done by the
force in the displacement interval
between 0m and 30m is
(A) 275 J (B) 325 J
(C) 400 J (D) 300 J
85
28. Force F on a particle moving in a
straight line varies with distance d
as shown in the figure. The work
done on the particle during its
displacement of 12m.
(A) 27 J (B) 24 J
(C) 36 J (D)26 J
29. A force F = Ay
2
+ By + C acts on a body in the y-direction. The work done by this
force during a displacement from y = -a to y = a is
(A)
3
Aa 2
3
(B) ca 2
3
Aa 2
3
+
(C) ca
2
Ba
3
Aa 2
2 3
+ + (D) None of these.
(ENERGY AND CONSERVATVE, NON
CONSERVATIVE FORCE)
30. A spring with spring constant K when streched through 2cm the potential energy
is U. If it is streched by 6cm. The potential energy will be......
(A) 6U (B) 3U (C) 9U (D) 18U
31. If linear momentum of body is increased by 1.5%, its kinetic energy increases
by.......%
(A) 0% (B) 10% (C) 2.25% (D) 3%
32. With what velocity should a student of mass 40 kg run so that his kinetic energy
becomes 160 J ?
(A) 4 m/s (B)
8
m/s (C) 16 m/s (D) 8 m/s
33. A body of mass 1 kg is thrown upwords with a velocity 20 m/s. It momentarily
comes to rest after a height 18m. How much energy is lost due to air friction.
(g = 10 m/s
2
)
(A) 20 J (B) 30 J (C) 40 J (D) 10 J
34. Two bodies of masses m
1
and m
2
have equal kinetic energies. If P
1
and P
2
are
their respective momentum, what is ratio of P
2
: P
1
?
(A) m
1
: m
2
(B)
1 2
m : m (C)
2 1
m : m (D) m
1
2
: m
2
2
86
35. A body having a mass of 0.5 kg slips along the
wall of a semispherical smooth surface of radius
20 cm shown in figure.What is the velocity of
body at the bottom of the surface ? (g = 10 m/s
2
)
(A) s / m 2 (B) 2 m/s
(C) 2 s / m 2 (D)

4 m/s
36. Two bodies of masses m and 3m have same momentum. their respective kinetic
energies E
1
and E
2
are in the ratio.....
(A) 1 : 3 (B) 3 : 1 (C) 1 : 3 (D) 1 : 6
37. What is the velocity of the bob of
a simple pendulum at its mean
position, if it is able to rise to
vertical height of 18 cm
(Take g = 10 m/s
2
)
(A) 0.4 m/s (B) 4 m/s
(C) 6 m/s (D) 0.6 m/s
38. A particle is placed at the origin and a force F = kx is acting on it (Where K is
positive constant) If Ucos = 0, which one of the following graph of U(x) versus
x. (where U is the potential energy function)
(A) (B)
(C) (D)
R = 20cm
0.5 Kg
18 cm
U( ) x
Y
X
U( ) x
Y
X
Y
X
U( ) x
Y
X
U( ) x
87
39. A force time graph for a linear
motion is shown in figure where the
segments are circular. what is linear
momentum gained between zero and
8 second ?
(A) -2 t N S (B) 0 NS
(C) 4 t NS (D) -6 t NS
40. A particle is dropped from a height-h. A constant horizontal velocity is given to the
particle. Taking g to be constant everywhere kinetic energy E of the particle with
respect to time t is correctly shown in....
(A) (B)
(C) (D)
41. Which of the following graph is correct between kinetic energy (E), potential
energy (U) and height (h) from the ground of the particle.
(A) (B)
(C) (D)

2 4 6 8
t(s)
F
o
r
c
e

(
N
)

+2
0
2
E
t
E
t
E
t
E
E
n
e
r
g
y
U
Height
E
E
n
e
r
g
y
U
Height
E
E
n
e
r
g
y
Height
U
E
n
e
r
g
y
U
Height
E
E
t
88
42. An engine pump is used to pump a liquid of density continuosly through a
pipe of cross-sectional area A. If the speed of flow of the liquied in the pipe
is v, then the rate at which kinetic energy is being imparted to the liquid is
(A)
2
1
A V
3
(B)
2
1
A V
2
(C)
2
1
A V (D) A V
43. The velocity of a body of mass 400 gm is ( )

3i 4j
m/s. So its kinetic energy
is ......
(A) 5 J (B) 10 J (C) 8 J (D) 16 J
44. A particle is moving under the influence of a force given by F = kx, where k
is a constant and x is the distance moved. What energy (in joule) gained by the
particle in moving from x = 1m to x = 3m ?
(A) 2 k (B) 3 k (C) 4 k (D) 9 k
45. A spring is compressed by 1 cm by a force of 4 N. Find the potential energy
of the spring when it is compressed by 10 cm.
(A) 2 J (B) 0.2J (C) 20 J (D) 200 J
46. when 2kg mass hangs to a spring of length 50 cm, the spring stretches by 2 cm.
The mass is pulled down until the length of the spring becomes 60 cm. What is the
amount of elastic energy stored in the spring in this condition, if g = 10 m/s
2
(A) 10 J (B) 2 J (C) 2.5 J (D) 5 J
47. The potential energy of a projectile at its highest point is
( )
th
1
2
the value of its
initial kinetic energy. Therefore its angle of projection is ......
(A) 30
0
(B) 45
0
(C) 60
0
(D) 75
0
48. Two bodies P and Q have masses 5 kg and 20 kg respectively. Each one is acted
upon by a force of 4 N. If they acquire the same kinetic energy in times t
P
and t
Q
then

the ratio
tq
tp
= ..........
(A)
2
1
(B) 2 (C)
5
2
(D)
6
5
49. A particle of mass 0.1kg is subjected
to a force which varies with distance
as shown in figure. If it starts its
jounery from rest at x = 0. What is
the particle's velocity squre at x = 6
cm ?
(A) 0 (m/s)
2
(B) 240
2
(m/s)
2
(C) 240
3
(m/s)
2
(D) 480 (m/s)
2
|
F(N)
x(m)
2 4 6
89
50. The potential energy of 2kg particle, free to move along x axis is given by
J
2
x
4
x
) X ( U
2 4
|
|
.
|

\
|
=
. If its mechanical energy is 2 J, its maximum speed is.... m/s
(A)
2
3
(B)
2
(C)
2
1
(D) 2
51. If the K.E. of a body is increased by 44%, its momentum will increase by.......
(A) 20 % (B) 22 % (C) 2 % (D) 120 %
52. A bullet of mass 0.10 kg moving with a speed of 100 m/s enters a wooden block
and is stopped after a distance of 0.20m. what is the average resistive force exerted
by the block on the bullet ?
(A) 2.5 x 10
2
N (B) 25 N (C) 25 x 10
2
N (D) 2.5 x 10
4
N
53. A simple pendulum is released from A
as shown in figure. If 10 g and 100 cm
represent the mass of the bob and length
of the pendulum. what is the gain in K.E.
at B ? ( g = 10 m/s
2
)
(A) 0.5 J (B) 5

10
-2
J
(C) 5 J (D) 0.5

10
-3
J
54. A rifle bullet loes
( )
th
1
10
of its velocity in passing through a plank. The least
number of such planks required just to stop the bullent is
(A) 5 (B) 10 (C) 11 (D) 20
55. A sphere of mass m moving the velocity v enters a hanging bag of sand and stops.
If the mass of the bag is M and it is raised by height h, then the velocity of the
sphere was
(A)
h 29
m
m m+
(B)
h 29
m
M
(C)
h 29
m M
m
+
(D)
h 29
M
m
56. A particle is acted upon by a force F
which varies with position x as shown
in figure. If the particle at x = 0 has
kinetic energy of 20 J. Then the
calculate the kinetic energy of the
particle at x = 16 cm.
(A) 45 J (B) 30 J
(C) 70 J (D) 135 J
60
0
B
A
2 4 6 8 10 12 14 16
10
5
0
-5
-10




|
F(N)
x(m)
90
57. A frictionless track 12345 ends
in a circular loop of radius R. A
body slids down the track from
point 1 Which is 6 cm. Maximum
value of R for the body to
successfully complete the loop is
(A) 6 cm (B)
4
15
cm
(C)
12
5
cm (D)
5
12
cm
58. If the water falls from a dam into a turbine wheel 19.6m below, then the velocity
of water at the turbine is ...... ( g = 9.8 m/s
2
)
(A) 9.8 m/s (B) 19.6 m/s (C) 39.2 m/s (D) 98.0 m/s
59. A bomb of 12 kg divedes in two parts whose ratio of masses is 1:4. If kinetic
energy of smaller part is 288 J, then momentum of bigger part in kgm/sec will be
(A) 48 (B) 72 (C) 108 (D) Data is incomplete
60. An ice-cream has a marked value of 700kcal. How many kilo-watt-hour of
energy will it deliver to the body as it is digested (J = 4.2 J/cal)
(A) 0.81 kwh (B) 0.90 kwh (C) 1.11 kwh (D) 0.71 kwh
61. A spherical ball of mass 15 kg
stationary at the top of a hill of height
82m. It slides down a smooth surface
to the ground, then climbs up another
hill of height 32m and finally slides
down to horizontal base at a height of
10 m above the ground. The velocity
attained by the ball is
(A) 30
10
m/s (B) 10
30
m/s (C) 12
10
m/s (D) 10
12
m/s
62. A bomb of mass 10 kg explodes into 2 pieces of mass 4 kg and 6 kg. The
velocity of mass 4 kg is 1.5 m/s, the K.E. of mass 6 kg is .......
(A) 3.84 J (B) 9.6 J (C) 3.00 J (D) 2.5 J
63. A bomb of mass 3.0 kg explodes in air into two pieces of masses 2.0 kg and 1.0
kg. The smaller mass goes at a speed of 80m/s. The total energy imparted to the
two fragments is
(A) 1.07 KJ (B) 2.14 KJ (C) 2.4 KJ (D) 4.8 KJ
64. The bob of simple pendulum (mass m and length l) dropped from a horizontal
position strike a block of the same mass elastically placed on a horizontal
frictionless table. The K.E. of the block will be
(A) 2 mgl (B) mgl /2 (C) mgl (D) zero
91
65. A gun fires a bullet of mass 40 g with a velocity of 50 m/s. Because of this the
gun is pushed back with a velocity of 1 m/s. The mass of the gun is
(A) 1.5 kg (B) 3 kg (C) 2 kg (D) 2.5 kg
66. The decreases in the potential energy of a ball of mass 25 kg which falls from
a height of 40 cm is
(A) 968 J (B) 100 J (C) 1980 J (D) 200 J
67. If a man increase his speed by 2 m/s, his K.E. is doubled, the original speed of
the man is
(A) (2+2
2
) m/s (B) (2 +
2
) m/s (C) 4 m/s (D) (1+2
2
) m/s
68. The potential energy of a conservative system is given by U(X) = (x
2
-5x) J.
Then the equilibrium position is at....... (where x in m)
(A) x = 1.5 m (B) x = 2m (C) x = 2.5 m (D) x = 5 m
69. The potential energy of a particle
varies with distance x as shown in the
graph. The force acting on the particle
is zero at
(A) C (B) B
(C) B and C (D) A and D
70. A nucleus at rest splits into two nuclear parts having same density and radii in
the ratio 1:2. Their velocites are in the ratio
(A) 2:1 (B) 4:1 (C) 6:1 (D) 8:1
71. A body of mass 1.5 kg slide down a curved track
which is quadrant of a circle of radias 0.75 meter.
All the surfaces are frictionless. If the body starts
from rest, its speed at the bottom of the track
is ..... ( g = 10 m/s
2
)
(A) 3. 87 m/s (B) 2 m/s
(C) 1.5 m/s (D) 0.387 m/s
72. A single conservative force F(x) acts on a 2.5 kg particle that moves along the x-
axis. The potential energy U(x) is given by U(x) = (10 + (x - 4)
2
)

where x is in
meter. At x = 6.0m the particle has kinetic energy of 20J. what is the mechanical
energy of the system ?
(A) 34 J (B) 45 J (C) 48 J (D) 49 J
A
B
C
D
x
|
W( ) x
0.75m
0
.
7
5
m
92
POWER
73. A body initially at rest undergoes one dimensonal motion with constant
acceleration. The power delivered to it at time t is proportional to.....
(A)
2
1
t
(B) t (C)
2
3
t
(D) t
2
74. An electric motor develops 5KW of power. How much time will it take to lift
a water of mass 100 kg to a height of 20 m ? ( g = 10 m/s
2
)
(A) 4 sec (B) 5 sec (C) 8 sec (D) 10 sec
75. Bansi does a given amount of work in 30 sec. Jaimeen does the same amount
of work... in 15 sec. The ratio of the output power of Bansi to the Jaimeen is....
(A) 1:1 (B) 1:2 (C) 2:1 (D) 5:3
76. A rope-way trolly of mass 1200kg uniformly from rest to a velocity of 72 km/
h in 6s. What is the average power of the engine during this period in watt ?
(Neglect friction)
(A) 400 W (B) 40,000 W (C) 24000 W (D) 4000 W
77. A body of mass m is accelarated uniformly from rest to a speed v in time T. The
instantaneous power delivered to the body in terms of time is given by.....
(A) t .
T
mv
2
2
(B)
2
2
2
t .
T
mv
(C)
t .
T 2
mv
2
(D)
2
2
t .
T 2
mv
2
78. 1 kg apple gives 25 KJ energy to a monkey. How much height he can climb by
using this energy if his efficiency is 40%. (mass of monkey = 25 kg and g = 10
m/s
2
)
(A) 20m (B) 4m (C) 30m (D) 40m
79. A force of ( )

2i 3j k +
N acts on a body for 5 second, produces a displacement
of ( )

3i 5j k + +
. What was the power used ?
(A) 4W (B) 20 W (C) 21 W (D) 4.2 W
80. If the force F is applied on a body and it moves with a velocity v, the power will
be
(A) Fv (B)
V
F
(C)
2
V
F
(D)
2
Fv
81. From an automatic gun a man fires 240 bullet per minute with a speed of 360 km/
h. If each weighs 20 g, the power of the gun is
(A) 400 W (B) 300 W (C) 150 W (D) 600 W
93
82. A body of mass M is moving with a uniform speed of 10 m/s on frictionless
surface under the influence of two forces F
1
and F
2
. The net power of the system
is
1 2
F M F
(A) M F F 10
2 1
(B) M ) F F ( 10
2 1
+ (C) M ) F F (
2 1
+ (D) Zero
83. A coolie 2.0m tall raises a load of 75 kg in 25 from the ground to his head and
then walks a distance to 40m in another 25. The power developled by the coolie
is ( g = 10 m/s
2
)
(A)0.25 kw (B) 0.50 kw (C) 0.75 kw (D) 1.00 kw
84. A body is moved along a straight line by a machine delivering a constant
power. The velocity gained by the body in time t is proportional to.....
(A)
4
3
t
(B)
2
3
t
(C)
4
1
t
(D)
2
1
t
ELASTIC - INELASTIC (COLLISION)
85. The coefficient of restitution e for a perfectly elastic collision is
(A) 1 (B) 0 (C)

(D) -1
86. Two balls at same temperature collide. What is conserved
(A) Temperature (B) velocity (C) kinetic energy (D) momentum
87. A particle of mass m moving with horizontal speed 6 m/s as shown in figure.
If m<<M then for one dimensional elastic collision, the speed of lighter particle
after collision will be
(A) 1 m/s in original direction.
(B) 2 m/s opposite to the original direction.
(C) 1 m/s opposite to the original direction.
(D) 2 m/s in original direction.
88. A rubber ball is dropped from a height of 5 m on a planet where the acceleration
due to gravity is not known. On bouncing, it rises to 1.8 m. The ball losses its
velocity on bouncing by a factor of
(A)
25
16
(B)
25
9
(C)
5
3
(D)
5
2
89. Three objects A, B and C are kept in a straight line on a frictionless horizontal
surface. These have masses m, 2m and m respectivelly. The object A moves
towards B with a speed 9 m/s and makes an elastic collision with it. Thereafter,
B makes compelety inelastic collision with C. All motion occur on the same
straight line. Find final speed of the object C
94
(A) 3 m/s
(B) 4 m/s
(C) 5 m/s (D) 1 m/s
90. Two solid rubber balls P and Q having masses 200 g and 400 g respectively are
moving in opposite directions with velocity of P equal to 0.3 m/s. After
collision the two balls come to rest, then the velocity of Q is
(A) 0.15 m/s (B) 1.5 m/s (C) -0.15 m/s (D) Zero
91. A sphere collides with another sphere of identical mass. After collision, the two
sphere move. The collision is inelastic. Then the angle between the directions
of the two spheres is
(A) Different from
90


(B)
90

(C)
O

(D)
45

92. A ball is allowed to fall from a height 20m . If there is 30% loss of energy due
to impact, then after one impact ball will go up to
(A) 18 m (B) 16 m (C) 12 m (D) 14 m
93. If a skater of weight 4 kg has intial speed 40 m/s and 2
nd
one of weight 6 kg
has 6 m/s. After collision, they have speed (couple) 6 m/s. Then the loss in K.E.
is.....
(A) 48 J (B) zero (C) 96 J (D) None of
these
94. A metal ball of mass 2 kg moving with a velocity of 36 km/h has a head on
collision with a stationary ball of mass 3 kg. If after the collision, the two balls
move togather, the loss in kinetic energy due to collision is
(A) 40 J (B) 60 J (C) 100 J (D) 140 J
95. A neutron having mass of 1.67

10
-27
kg and moving at 10
8
m/s collides with a
deutron at rest and sticks to it. If the mass of the deutron is 3.34

10
-27
kg then
the speed of the combination is
(A) 3.33

10
7
m/s (B) 3

10
5
m/s (C) 33.3

10
7
m/s (D) 2.98

10
5
m/s
96. A mass of 100g strikes the wall speed
5 m/s at an angle as shown in figure
and is rebounds with the same speed.
If the contact time is
3
5 10

sec, what
is the force applied on the mass by the
wall
(A) 3 100 N to right
(B) 100 N to right
(C) 3 100 N to left
(D) 100 N to left
95
97. Three identical sperical balls A, B and C are placed on a table as shown in the
figure along a straight line. B and C are at rest initially. The ball A hits B head on
with a speed of 10 m/s. Then after all collision A and B are brought to rest and C
takes off with velocity of...... (elastic collision)
(A) 20 m/s (B) 2.5 m/s (C) 10 m/s (D) 7.5 m/s
98. A ball dropped from a height of 4m rebounds to a height of 2.4m after hitting
the ground. Then the percentage of energy lost is
(A) 40 (B) 50 (C) 30 (D) 600
99. A billiard ball moving with a speed of 8 m/s collides with an identical ball
originally at rest. If the first ball stops after collision, then the second ball will
move forward with a speed of..... (elastic collision)
(A) 8 m/s (B) 4 m/s (C) 16 m/s (D) 1.0 m/s
100. A bullet of mass m moving with velocity v strikes a block of mass M at rest and
gets embedded into it. The kinetic energy of the composite block will be
(A)
) m m (
M
mv
2
1
2
+

(B)
M
) m m (
mv
2
1
2
+

(C)
) M m (
m
MV
2
1
2
+

(D)
) M m (
m
mv
2
1
2
+

101. A body of mass m


1
is moving with a velocity v. It collides with another
stationary body of mass m
1.
They get embeded. At the point of collision, the
velocity of the system
(A) Increases
(B) Decreases but does not become zero
(C) Remains same
(D) Become zero
102. Two small particles of equal masses start
moving in opposite directions from a point. A
in a horizontal circular orbit. Their tangential
velocities are v and 2v, repectively, as shown in
the figure. Between collisions, the particles
move with constant speeds. After making how
many elastic collisions, other than that at A,
these two particles will again reach the point
A
(A) 1 (B) 2
(C) 3 (D) 4
96
103. Four identical balls are lined in a straight grove made on a horizontal
frictionless surface as shown. Two similar balls each moving with a velocity v
collide elastically with the row of 4 balls from left. What will happen
(A) One ball from the right rolls out with a speed 2v and the remaining balls will
remain at rest.
(B) Two balls from the right roll out speed v each and the remaining balls will
remain statinary.
(C) All the four balls in the row will roll out with speed
v
4
each and the two
colliding balls will come to rest.
(D) The colliding balls will come to rest and no ball rolls out from right.
(ASSERTION & REASON)
* Assertion and Reason are given in following questions. Each question have
four option. One of them is correct it.
(1) If both assertion and reason and the reason is the correct explanation of the
Assertion.
(2) If bothe assertion and reason are true but reason is not the correct
explanation of the assertion.
(3) If the assertion is true but reason is false.
(4) If the assertion and reason both are fare.
104. Assertion : In the elastic collision between two bodies, the relative speed of the
bodies after collision is equal to the relative speed before the collision.
Reason : In the elastic collision the linear momentum of the system is
conserved.
(A)1 (B) 2 (C) 3 (D) 4
105. Assertion : When a gas is allowed to expand, work done by gas is positive.
Reason : Force due to gaseous pressure and displacement (of position) on in
the same direction.
(A) 1 (B) 2 (C) 3 (D) 4
97
106. Assertion : A light body and heavy body have same momemtum. Then they also
have same kinetic energy.
Reason : Kinetic energy does not depend on mass of the body.
(A) 1 (B) 2 (C) 3 (D) 4
107. Assertion : Mountain roads rarely go straight up the slope.
Reason : slope of mountains are large, therefore, more chances of vehicle to
slip from roads.
(A) 1 (B) 2 (C) 3 (D) 4
108. Assertion : The change in kinetic energy of a particle is equal to the work done
on it by the net force.
Reason : Change in kinetic energy of particle is equal to the work done only
in case of a system of one particle.
(A) 1 (B) 2 (C) 3 (D) 4
109. Assertion : Work done in moving a body over a closed loop is zero for every
force in nature.
Reason : Work done does not depend on nature of force.
(A) 1 (B) 2 (C) 3 (D) 4
110. Assertion : A weight lifter does no work in holding the weight up.
Reason : Work done is zero because distance moved is zero.
(A) 1 (B) 2 (C) 3 (D) 4
111. Assertion : stopping distance =
Kineticenergy
Stoppingforce
Reason : Work done in stopping a body is equal to K.E. of the body.
(A) 1 (B) 2 (C) 3 (D) 4
112. Assertion : The mass equivalent of 1000 kwh energy is 40 microgram.
Reason : This follows from E = mc
2
where
8
C 3 10 m s =
(A) 1 (B) 2 (C) 3 (D) 4
113. Assertion : Work done by centripetal force is zero.
Reason : This is because entripetal force is always along the tangent.
(A)1 (B) 2 (C) 3 (D) 4
114. Assertion : Two bodies of different masses have same momentum. Their kinetic
energy are in the inverse ratio of their masses.
Reason : K.E. =
2
mv
2
1
(A) 1 (B) 2 (C) 3 (D) 4
98
115. Assertion : Linear momentum is conserved in both, elastic and inelastic
collisions.
Reason : Total energy is conserved in all such collisins.
(A) 1 (B) 2 (C) 3 (D) 4
116. Assertion : Both, a stretched spring and a compressed spring have potential
energy.
Reason : Work is done against the restoring force in each case.
(A) 1 (B) 2 (C) 3 (D) 4
(COLUMN)
117. A force F = kx (where k is positive constant) is acting on a particle. Match
column-I and column-II, regarding work done in displacing the particle.
Column - I Column - ii
(a) From x = -4 to x = -2 (P) Positive
(b) From x = -2 to x = -4 (Q) zero
(c) From x = -2 to x = +2 (R) negative
(A) a - R, b - P, c - Q (B) a - P, b - Q, c - R
(C) a - R, b - Q, c - P (D) a - Q, b - P, c - R
118. A body falls freely under the action of gravity from a height h above the
ground.
Column - i Column - ii
(a) P.E. = 2(K.E.) (P) constant at every point
(b) P.E. = K.E. (Q) at height
3
h
(c) P.E. =
2
1
(K.E.) (R) at height
3
h 2
(d) P.E.+ K.E. (S) at height
2
h
(A) a - P, b - Q, c - R, d - S (B) a - Q, b - P, c - S, d - R
(C) a - S, b - R, c - Q, d - P (D) a - R, b - S, c - Q, d - P
119. Two vehicles moving on a horizontal road are stopped by same retarding force.
Column - I Column - ii
(a) When they have same K.E. (P) faster body stop in larger distance
(b) When they have different masses (Q) larger body stops in larger distance.
but same velocity
(c) When both have same momentum (R) heavier body stops in larger distance.
(d) When both have same mass but (S) stopped in same distance.
different velocities
(A) a - P, b - Q, c - R, d - S (B) a - Q, b - P, c - S, d - R
(C) a - S, b - R, c - Q, d - p (D) a - R, b - S, c - Q, d - P
99
KEY NOTE
1 B 26 A 51 A 76 B 101 B
2 B 27 B 52 C 77 A 102 B
3 D 28 A 53 B 78 D 103 B
4 D 29 B 54 A 79 A 104 D
5 A 30 C 55 A 80 A 105 A
6 D 31 D 56 B 81 A 106 D
7 C 32 B 57 D 82 D 107 A
8 C 33 A 58 B 83 C 108 B
9 D 34 B 59 A 84 D 109 D
10 D 35 B 60 A 85 A 110 A
11 A 36 B 61 C 86 D 111 A
12 B 37 C 62 C 87 A 112 A
13 D 38 C 63 D 88 D 113 C
14 B 39 B 64 C 89 B 114 B
15 D 40 A 65 C 90 C 115 B
16 A 41 A 66 B 91 A 116 A
17 C 42 A 67 A 92 D 117 A
18 C 43 A 68 C 93 D 118 D
19 B 44 C 69 C 94 B 119 C
20 D 45 A 70 D 95 A
21 D 46 D 71 A 96 C
22 B 47 A 72 A 97 C
23 B 48 A 73 B 98 A
24 A 49 D 74 A 99 A
25 B 50 A 75 B 100 D
100
HINT
1. W Fdcos = u
2. u = A cos Fd K = w
3. ; ) d ( ) Mg ( d F W = = where M = m
1
+ m
2
4. a g = and
2 2
0
v v 2 gd =
( )
2
vo
d v 0
2 g
= =

5. W = Area under curve of F x graph.


6. According
2
1
W kx
2
=
7. (2n 1)
n
h o
8. ( )
2 2
2 1
1
W k x x
2
=
9. According to work energy theorm W k =A
10. =
3
t 2
s
3
acceleration
2
2
d s
a 4t
dt
= = , work
0 0
F ds mads
l l
W = =
} }
11. W = F x = max = (m) (Area covered by curve of a x graph)
12. work W = Area covered by curve of Fx graph
13. Loss in P.E. of spring = gain in K.E. of ball
14. W =
2
mgl
2n
, where n is, fraction of length of the chain hanging from the table.
15. W =
2
2
mgl
n
( n = 3 Given)
16. Tension in the cord, T = M (g-
2
g
) =
2
mg
. Work done by cord = -Td = -
2
mgd
17. Fd cos

=
2
1
mv
2
-
2
1
mu
2
18.
F mg
x
K k
= =
19.
du
F
dx
=
101
20.
2
2
mgl
w
n
=
( n = 4 given)
21. Gravitational force is a conservative force and work done against it is a point function
i.e. does not depends on the path.
22.
2 2
0
v v 2ad, (v 0) = = and gh 2 v
0
=
( )
2
o 2gh 2(g a)d a = +
23. Work done = gain in potential energy Area under curve = mgh
24. Intial velocity at x = 0; v
1
= 0
Final velocity at x = 2; v
2
=
3
2
5 2
Work done =
2 2
2 1
1
m( v v )
2

25. Intial velocity of particle v
1
= 20 m/s
Final velocity of particle v
2
= 0
2 2
2 1 2 1
1
W k k k m(v v )
2
=A = =
26.
0
W Fl sin
U mgl(1 cos )
W U; ( 60 )
= u
= u
= u =
27. Work = Area under (F - d) graph
28. Work = Area under (F - d) graph
29. W Fdy =
}
=
( )
2
2
a
Ay By C dy
+

+ +
}
3 2
Ay By
cy
3 2
a
a
+

(
= + +
(

3
2Aa
2Ca
3
= +
30. Energy U o x
2
( k constant)
102
31.
2
2
P K
m 2
P
K o =
( m constant)
dk dp
2
k p
=
32.
2
mv
2
1
k =
m
k 2
v =
33. Energy lost due to air friction =
2
1
mv
2
- mgh
34.
2
1
2
1
m
m
p
p
m P = o
35. mgR =
2
mv
2
1
36.
m
1
E
m 2
p
E
2
o =
37. gh 2 V =
38.
2
kx
U
2
=
39. F.
t A
= momentum, The resulatant fore is zero, therefore F.
t A
= 0
40. Kinetic Energy (K.E)
2
v o and
2 2
t v o ....
41. P.E (U) h and E = K + U = constant
42.
2 2
2
1 mv 1 A V 1
A
2 t 2 t 2
l
v

= =
43. Find the value of v

and than, K.E =


2
1
mv
2
44. . dx kx U
3
1
} =
45. Spring constant
2
kx
2
1
U ,
x
F
K = =
46. Spring constant
2
kx
2
1
U ,
x
F
K = =
103
47. Hmax =
2 2 2 2
o 0 0 0
v Sin mv sin 1
, U mg Hmax Ko
2g 2 2
u u
= = =
48. According to equation
2
Q Q
2
p p
v m
2
1
v m
2
1
=

p
Q
V
V
,
Using impulse momentum,
p p
P
Q Q Q
m v
Ft
=
FT m v
49.
=
2
mv
2
1
area covered by curve of Fx , graph
50. K.E. is maximum than P.E. minimum.
so = = 0
dx
du
0 OR
For x 4 =
. min U
4
1
) x ( U = =
51. E E =
1
and E 44 . 1 E
2
= . Than
E Po
52. F d =
2
1
m( )
2 2
2 1
v v .
53. Loss of potential energy = Kinetic energy gained.
54. Let the thickness of one planks be s and n planks are arranged just to stop bullet.
2 2
0 0
9
v v 2as , put v v
10
= =
2
0
v 100
Now
2as 19
=
2
0
v
n
2as
=
55. By conservation of linear momentum

mv = (m+M) V
sys
By convervation of low of energy.
2
1
(m+M) v
2
sys = (m+M) gh
Find Vsys and put it sin egn.(1)
56. Work done = Area under FX graph with proper sign.
57. Condition for up direction loop (Parabola) h =
2
5
R
58. P.E. of water = K.E. at turbine. mgh =
2
mv
2
1
59. The bomb into two masses
1 2
m +m =12kg and
1
2
m 1
m 4
=
, K.E.
energy of smaller part =
2
1 1
1
m v
2
104
60. 1 k cal = 10
3
calorie = 4200J =
6
10 3.6
4200

kwh
6
10 3.6
4200 700
cal K 700

=
kwh
61. From the conservation of energy,
mg
2
mv
2
1
) h h (
2 1
=
62.
1 1 2 2
m v +m v =0 and than K.E. =
2
2 2
1
m v
2
63. , v m
2 1
= , v m
2 2
Total energy of system =
2 2
1 1 2 2
1 1
m v m v
2 2
+
64. The collision between bob and block is elastic P.E. K.E. =
65.
B B G G
v m v m =
66. mgh U= A
67. Initial kinetic energy, E =
2
mv
2
1
Final kinetic energy, 2E =
2
) 2 v ( m
2
1
+
68. For a conservation field. Force,
dx
du
F =
,
At equilibrium position, F = 0
69.
dx
du
F =
it is clear that slope of U - x curve is zero at point B and C.
70. Given
1 2
=

2
1
M
M
=
3
2
1
R
R
|
|
.
|

\
|
According to low conservation of linear momentum,
3
2
1
1
2
V
V
|
.
|

\
|
=
71. Conservation of energy, mgh =
2
mv
2
1
72. P.E. at X = 6m is U = 10 + (6 - 4)
2
= 14 J
Mechanical energy = K.E. + P.E. = 20 + 14 = 34 J
105
73. Power, P = Fv = mav
74. Power, P =
t
mgh
75. Power, P
t
1
o
76.
m 2
P
2
E=
77.
P 2mE ; P E = o
78.
P 2mE ; P E = o
79.
t
W
P=
t
d . F
P

=
80. .............
81. Power of gun =
Total K.E.of fired bullet
time

2 1
mv
n
t
2
=
82. Work done by forces = 0 ;
p
t
W=
83.
KW 75 . 0
2
) 2 )( 10 )( 75 (
t
mgh
P = = =
84.
2
1
mv kt (K, Cons tan t)
2
=
,
1
2
2k
v t
m
=
,
1
2
2k
dx t dt.
m
=
Take integration.
85. ............
86. ............
87.
|
|
.
|

\
|
+
+
|
|
.
|

\
|
+

=
2 1
2 2
1
2 1
2 1
1
m m
u m 2
u
m m
m m
v
where m
1
= m and m
2
= M; m << M.
m
1
= 0, v
1
= -u
1
+ 2u
2
88.
;
h
h
v
v
1
2
1
2
=
i.e. fractional loss in velocity
1
2
v
v
1 =
89.
1 1
2
1 2
2m u
v 6m/ s
m m
= =
+
i.e. After elastic collision B strike to C with velocity of 6 m/s, Now for B and C
sys
2m m 3m V
6m/s
Rest

(2m) (6) + 0 = (3m) (V sys)


90.
p p Q Q
m v m v 0 + =
106
91. Angle will be 90
o if
collision is perfectly elastic.
92. mgh ' = 70% of mgh
93. Loss in K.E. = (initial K.E. - Final K.E.) of system
2 2 2
1 1 2 2 1 2
1 1 1
m u m u (m m )v
2 2 2
= + +
94. By low of conservation of momentum
2

10 = (2+3) v v 4m s =
Loss in K.E. =
2
1
(2)(10)
2
-
2
1
(5)(4)
2
95. Momentum of neutron = Momentum of combination
96. Force = Rate of change of momentum.
Intial momentum
1

p mvsin j mvcos j = u + u

Final momentum
2

p mvsin j mvcos j = u + u

t
P
F
A
A
=

97. In elastic head on collision velocity gets interchanged.


98. U
1
= mgh
1
and U
2
= mgh
2
% energy lost =
100
U
U U
1
2 1

99. In elastic head on collision velocites gets interchanged.


100. By conservation of momentum, mv + M(o) = (m+M) v
Velocity of composite block V =
v
M m
m
|
.
|

\
|
+
K.E. of composite block =
2
v ) m M (
2
1
+
101. By conservation of momentum, v ) m m ( ) 0 ( m V m
2 1 2 1
+ = +
102. Let initially particle x is moving in anticlock-wise direction and y in clockwise
direction.
As the ratio of velocities of x and y
particles are
Vx 1
Vy 2
=
, therefore ratio of
their distance covered will be in the ratio
of 2:1. It means they collide at B.
so, after two collision these two particles
will again reach the point A.
103. Momentum and kinetic energy is conserved only in this case.
107
Unit - 5
Rotational Motion
108
SUMMARY
* Important Formula, Facts and Terms
1. Centre of mass of system of particles

=
=
+ + +
+ +
=
n
1 n n
n
n
n 2 1
n
n
2
1
1
1
cm
m
r m
m m m
r m r m r m
R
for rigid body

= n
n
cm r m R . M
for Two body System

2
2
1
1
r m r m =
OR
1 2 1 2 2 1
1 1 2 2
r m r +r m +m
+1= +1 OR =
m m r r


1 2 1 2
2 1
1 1 2 1 2 2
r m +m rm rm
= r = OR r =
m m +m m +m r

2.
1 2 n
1 2 n
cm
m v + m v + ......... + m v
V =
M

1 2
.........
cm
n
P Mv p p p = = + + +

similarly
M
a m a m a m
a
n
n
2
2
1
1
cm
+ + +
=
n 2 1 cm F F F a M F + + + = =
3. Torque =
T
=
F r
=
o I
sin rF =

= product of force and perpendicular distance between point of rotation


and line of action.
Angular momentum = L r P I = = e

|
L
| = rpsin
u
= product of linear momentum and perpendicular distance between point of
rotation and line of action.
Moment of inertia = I =
2
n n
2
2 2
2
1 1
r m r m r m + + +
=
=
n
1 n
2
n n
r m
109
4. Low of conservation of angular momentum
As L I = e

p
d L d
I I
dt dt
e
= = o =



dL
dt
t =

when t

= then L remains constant


Its geometrical representation in planetary motion
Let
dt
dA
is an areal velocity
Then m
dA L dA L
= OR =
dt 2 dt 2m
5. Radius of gyration {K}
As I =
2
n n
2
2 2
2
1 1
r m r m r m + + +
If all particles have same mass then
= m
( )
2
n
2
2
2
1
r r r + + +
( )
n
r r r
nm I
2
n
2
2
2
1
+ + +
= Here (nm = M)
2
mk =
n
) r r r (
k
2
n
2
2
2
1
+ + +
=
6. Some relations between linear and rotational motion.
v = rw
v w r =

Here w =
d
dt
u
2
2
d w d
dt dt
u
o = =

r T a a a = +

where
T
wv r
r
a a = = o
2 2 2 2 2 2
r t
a a a v r = + = e + o


2 2 2 2 2 4 2
r r r = e e + o = e + o
7. Equilibrium of a rigid body.
When
0 F F F F n 2
1
= + + + =
it is in linear equilibrium
When
2
1
0 n P P P P = + + + =

it is in rotational equilibrium
8. Two theorm for moment of inertia
Y X Z
I I I + = Theorm of perpendicular axis.
2
cm
Md I I + = Theorm of Parallel axis.
110
9. Rolling down of body on an inclined plane.
V
2 2
2 2
2 2 sin
,
1 1
gh
a
K K
R R
(
u
(
= =
(
(
+ +
(
(

Condition for rolling without sliding

| |
,
K
R
1
tan
2
2 s
+
u
>
For ring
u > tan
2
1
s
(K = R)
disc
|
.
|

\
|
= u >
2
R
K tan
3
1
s
solid sphere
|
|
.
|

\
|
= u > R
5
2
K tan
7
2
s
10. Rotational Kinetic Energy.
R.K.E =
2
2 2 2
2
1 1
&
2 2
V V
I MK I MK
R R
| |
e = = e=
|
\ .

I 2
L
2
=
Total kinetic Energy = Rotational K.E. + Linear K.E.

2 2
2 2 2
2 2
1 1 1
1
2 2 2
V K
MK MV MV
R R
(
= + = +
(

Now (1) 2
2
R
K
E . LinearK
E . K . R
=
(2)
2
2
2
2 2 2
2
.
.
1
K
R
K
R
Rotational K E K
Total K E R K
= =
+
+
Percentage rotational K.E. =
% 100
1 2
2
2
2
R
K
R
K

|
|
.
|

\
|
+
111
(3)
2
2
R
K
2 2
2
1
1
K R
R
E . K Total
E . K nal Translatio
+
=
+
=
Comparison between physical quantities of linear motion and rotational motion
Translational motion Rotational motion
Linear displacement,
d
Angular displacement,
u
Linear velocity,
V
Angular velocity,
w
Linear acceleration,
dt
v d
a = Angular acceleration,
dt
w d
= o
Mass, m Moment of inertia, I
Linear Momentum,
v m P =
Angular momentum,
w I L =
Force,
a m F =
Torque,
dt
L d
= t
Newton's Second Law of Motion, A result similar to newtown's Second Law,
dt
P d
F =
dt
L d
= t
Translational kinetic energy K =
2
mv
2
1
Rotational kinetic energy K =
2
1
I
2
e
m 2
p
2
=
I 2
L
2
=
Work, W =
d F
Work, W =
tu
Power, P = Fv Power, P = w t
Equations of linear motion taking place Equations of rotational motion taking place
with constant linear acceleration with constant angular acceleration :
v = V
O
+ at w = t W
O
o +
d =
2
O
1
v t + at
2
2
0
at
2
1
t W + = u
2ad =
2 2
0
v -v 2au =
2
0
2
W W
Law of conservation of linear momentum Law of conservation of angular momentum
when
0 F =
then
P
is constant Impulse when 0 P =

then L is constant Impulse


linear
1 2 P P t F = =
rotational 2 1 t L L = t =

Value of V, a, t for some Rolling Bodies
Shape of Body velocity velocity acceleration Time 2
2
R
K
Ring/Hollow cylinder gh
( )
2
1
sin gl u
u sin g
2
1
u sin g
2

1
Disc/Solid cylinder
gh
3
4
2
1
sin gl
3
4
|
.
|

\
|
u
u sin g
3
2
u sin g
3
2
1
Solid Sphere
gh
7
10
2
1
sin gl
7
10
|
.
|

\
|
u
u sin g
7
5
u sin g
14
5
2
Shell/Hollow spher
gh
5
6
2
1
sin gl
5
6
|
.
|

\
|
u
u sin g
5
3
u sin g
10
3
2
Moment of inertia an radius of gyration for some symmetric bodies
Body Axis Figure I K For rolling
body 2
2
R
K
Thin rod of Passing through its
2
ML
12
1
3 2
L
-
Length L centre and per pendicular
to its length
Ring of Any diameter
2
MR
2
1
2
R
-
radius R
Ring of Passing through its
2
MR
R 1
radius R centre and perpen-
dicular to its plane
Circular disc Passing through its
2
MR
2
1
2
R
2
1
radius R centre and perpen-
dicular to its plane of
Circular disc Any diameter
2
MR
4
1
2
R
-
of radius R
113
Body Axis Figure I K For rolling
body 2
2
R
K
Hollow Geometrical
2
MR
R 1
cylinder of axis of the
radius R cylinder
Solid cylinder Geometrical axis
2
MR
2
1
2
R
2
1
of radius R of the cylinder
Solid sphere Any diameter
2
MR
5
2
R
5
2
5
2
of radius R
Hollow Any diameter
2
MR
3
2
R
3
2 2
3
sphere
of radius R
MCQ
For the answer of the following questions choose the correct alternative from among the given ones.
1. The centre of mass of a systems of two particles is
(A) on the line joining them and midway between them
(B) on the line joining them at a point whose distance from each particle is proportional
to the square of the mass of that particle.
(C) on the line joining them at a point whose distance from each particle inversely
propotional to the mass of that particle.
(D) On the line joining them at a point whose distance from each particle is proportional
to the mass of that particle.
2. Particles of 1 gm, 1 gm, 2 gm, 2 gm are placed at the corners A, B, C, D, respectively
of a square of side 6 cm as shown in figure. Find the distance of centre of mass of the
system from geometrical centre of square.
{A} 1 cm
{B} 2 cm
{C} 3 cm
{D} 4 cm
3. Three particles of the same mass lie in the (X, Y) plane, The (X, Y) coordinates of their
positions are (1, 1), (2, 2) and (3, 3) respectively. The (X,Y) coordinates of the centre of
mass are
{A} (1, 2) {B} (2, 2) {C} (1.5, 2) {D} (2, 1.5)
114
4. Consider a two-particle system with the particles having masses
1
M , and
2
M . If the first
particle is pushed towards the centre of mass through a distance d, by what distance should
the second particle be moved so as to keep the centre of mass at the same position?
{A}
2 1
1
M M
d M
+
{B}
2 1
2
M M
d M
+
{C}
2
1
M
d M
{D}
1
2
M
d M
5. Four particles A, B, C and D of masses m, 2m, 3m and 5m
respectively are placed at corners of a square of side x as
shown in figure find the coordinate of centre of mass take A
at origine of x-y plane.
{A}
|
.
|

\
|
10
x 7
, x 2
{B}
|
.
|

\
|
7
x 10
, x 2
{C}
|
.
|

\
|
7
x 10
,
2
x
{D}
|
.
|

\
|
10
x 7
,
2
x
6. From a uniform circular disc of radius R, a circular disc of radius
6
R
and having centre at a
distance +
2
R
from the centre of the disc is removed. Determine the centre of mass of remaining
portion of the disc.
{A}
70
R
{B}
70
R +
{C}
7
R
{D}
7
R
+
7. A circular plate of uniform thickness has a diameter of 56 cm. A circular portion of diameter
42 cm. is removed from +ve x edge of the plate. Find the position of centre of mass of the
remaining portion with respect to centre of mass of whole plate.
{A} - 7 cm {B} + 9 cm {C} - 9 cm {D} + 7 cm
8. Two blocks of masses 10 kg an 4 kg are connected by a spring of negligible mass and placed
on a frictionless horizontal surface. An impulse gives velocity of 14 m/s to the heavier block in
the direction of the lighter block. The velocity of the centre of mass is :
{A} 30 m/s {B} 20 m/s {C} 10 m./s {D} 5 m/s
9. A particle performing uniform circular motion has angular momentum L., its angular frequency is
doubled and its K.E. halved, then the new angular momentum is :
{A} {B} {C} 2L {D} 4L
10. A circular disc of radius R is removed from a bigger disc of radius 2R. such that the circumferences
of the disc coincide. The centre of mass of the remaining portion is R from the centre of mass
of the bigger disc. The value of o is.
{A} {B} 1/6 {C} {D} 1/3
11. Three point masses M1, M2 and M3 are located at the vertices of an equilateral triangle of
side 'a'. what is the moment of inertia of the system about an axis along the attitude of the triangle
passing through M1, ?
{A} ( )
4
a
M M
2
2 1
+ {B} ( )
4
a
M M
2
3 2
+ {C} ( )
4
a
M M
2
3 1
+ {D} ( )
4
a
M M M
2
3 2 1
+ +
115
12. A body of mass m is tied to one end of spring and whirled round in a horizontal plane
with a anstant angular velocity. The elongation in the spring is one centimeter. If the angular
velocity is doubted, the elongation in the spring is 5 cm. The original length of spring is
{A} 16 cm {B} 15 cm {C} 14 cm {D} 13 cm
13. A cylinder of mass 5 kg and radius 30 cm, and free to rotate about its axis, receives an angular
impulse of 3 kg M
2
S
-1
initially followed by a similar impulse after every 4 sec. what is the angular
speed of the cylinder 30 sec after initial imulse ? The cylinder is at rest initially.
{A}
1
S rad 7 . 106

{B}
1
S rad 7 . 206

{C} 107.6 rad S
-1
{D} 207.6 rad S
-1
14. Two circular loop A & B of radi r
a
and r
b
respectively are made from a uniform wire. The ratio
of their moment of inertia about axes passing through their centres and perpendicular to their planes
is
8
I
I
A
B
=
then
ra
rb
Ra is equal to
{A} 2 {B} 4 {C} 6 {D} 8
15. If the earth were to suddenly contract so that its radius become half of it present radius, without
any change in its mass, the duration of the new day will be
{A} 6 hr {B} 12 hr {C} 18 hr {D} 30 hr
16. In HC1 molecule the separation between the nuclei of the two atoms is about ( ) m 10 A 1 A 27 . 1
10
=

.
The approximate location of the centre of mass of the molecule is
i


with respect of Hydrogen
atom ( mass of CL is 35.5 times of mass of Hydrogen)
{A} 1 {B} 2.5 {C} 1.24 {D} 1.5
17. Two bodies of mass 1kg and 3 kg have position vector
( ) k

2 i

+ +
and (-3i-2j+k) respectively
the center of mass of this system has a position vector
{A} k

2 i

2 + {B}
k

2 +
{C}
k

2
{D}
k

+ +
18. Identify the correct statement for the rotational motion of a rigid body
{A} Individual particles of the body do not undergo accelerated motion
{B} The center of mass of the body remains unchanged.
{C} The center of mass of the body moves uniformly in a circular path
{D} Individual particle and centre of mass of the body undergo an accelerated motion.
19. A car is moving at a speed of 72 km/hr the radius of its wheel is 0.25m. If the wheels are
stopped in 20 rotations after applying breaks then angular retardation produced by the breaks
is
{A} -25.5 2
s
rad
{B} -29.5 2
s
rad
{C} -33.5 2
s
rad
{D} -45.5 2
s
rad
20. A wheel rotates with a constant acceleration of 2.0
2
sec rad If the wheel start from rest. The
number of revolution it makes in the first ten seconds will be approximately.
{A} 8 {B} 16 {C} 24 {D} 32
21. Two discs of the same material and thickness have radii 0.2 m and 0.6 m their moment of inertia
about their axes will be in the ratio
{A} 1 : 81 {B} 1 : 27 {C} 1 : 9 {D} 1 : 3
116
22. A wheel of mass 10 kg has a moment of inertia of 160 kg
2
m
about its own axis. The
radius of gyration will be _______ m.
{A} 10 {B} 8 {C} 6 {D} 4
23. One circular rig and one circular disc both are having the same mass and radius. The ratio of
their moment of inertia about the axes passing through their centres and perpendicular to their
planes, will be
{A} 1 : 1 {B} 2 : 1 {C} 1 : 2 {D} 4 : 1
24. One solid sphere A and another hollow sphere B are of the same mass and same outer radii.
The moment of inertia about their diameters are respectively
A
I and
B
I such that
{A}
B A
I I = {B}
B A
I I > {C}
B A
I I < {D}
dB
dA
I
I
B
A
=
(radio of their densities)
25. A ring of mass M and radius r is melted and then molded in to a sphere then the moment of
inertia of the sphere will be..
{A} more than that of the ring {B} Less than that of the ring
{C} Equal to that of the ring {D} None of these
26. A circular disc of radius R and thickness R/6 has moment of inertia I about an axis passing through
its centre and perpendicular to its plane. It is melted and recasted in to a solid sphere. The moment
of inertia of the sphere about its diameter as axis of rotation is
{A} I {B}
8
I 2
{C}
5
I
{D}
10
I
27. One quater sector is cut from a uniform circular disc of radius R. This
sector has mass M. It is made to rotate about a line perpendicular to
its plane and passing through the centre of the original disc. Its moment
of inertia about the axis of rotation is
{A}
2
MR
2
1
{B}
2
MR
4
1
{C}
2
MR
8
1
{D}
2
MR 2
28. A thin wire of length L and uniform linear mass density is bent
in to a circular loop with centre at O as shown in figure. The moment
of inertia of the loop about the axis xx' is .
{A}
2
2
L
8

{B}
3
2
L
16

{C}
3
2
5 L
16

{D}
3
2
3 L
8

29. Two disc of same thickness but of different radii are made of two different materials such that
their masses are same. The densities of the materials are in the ratio 1:3. The moment of inertia
of these disc about the respective axes passing through their centres and perpendicular to their
planes will be in the ratio.
{A} 1 : 3 {B} 3 : 1 {C} 1 : 9 {D} 9 : 1
117
30. Let I be the moment of inertia of a uniform square plate about an axis AB that passes
through its centre and is parallel to two of its sides CD is a line in the plane of the plate
that passes through the centre of the plate and makes an angle of Q with AB. The moment
of inertia of the plate about the axis CD is then equal to.
{A} I {B} u
2
sin I {C} u
2
cos I {D}
2
cos I
2
u
31. A small disc of radius 2 cm is cut from a disc of radius 6 cm. If the distance between their
centres is 3.2 cm, what is the shift in the centre of mass of the disc
{A} -0.4 cm {B} -2.4 cm {C} -1.8 cm {D} 1.2 cm
32. A straight rod of length L has one of its ends at the origin and the other end at x=L If the mass
per unit length of rod is given by Ax where A is constant where is its center of mass.
{A} L/3 {B} L/2 {C} 2L / 3 {D} 3L / 4
33. A uniform rod of length 2L is placed with one end in contact with horizontal and is then inclined
at an angle o to the horizontal and allowed to fall without slipping at contact point. When it becomes
horizontal, its angular velocity will be..
{A} w =
L 2
sin g 3 o
{B} w =
o sin g 3
L 2
{C} w =
L
sin g 6 o
{D} w =
o sin g
L
34. A cubical block of side a is moving with velocity V on a
horizontal smooth plane as shown in figure. It hits a ridge at
point O. The angular speed of the block after it hits O is .
{A}
3
4
v
a
{B}
3
2
v
a
{C}
3
2
v
a
{D} zero
35. Consider a body as shown in figure, consisting of two identical
bulls, each of mass M connected by a light rigid rod. If an
impulse J = MV is imparted to the body at one of its ends,
what would be its angular velocity. What is V ?
{A} V / L {B} 2V / L {C} V / 3L {D} V / 4L
36. A thin circular ring of mass M and radius r is rotating about its axis with a constant angular velocity
w. Two objects each of mass m are attached gently to the opposite ends of a diameter of the
ring. The ring will now rotate with an angular velocity.
{A}
( 2 )
2
M m
M m
e
+
{B}
( ) 2
M
M m
e
+
{C}
M
M m
e
+
{D}
( ) 2 M m
M
e +
37. A smooth sphere A is moving on a frictionless horizontal plane with angular speed e and centre
of mass velocity v. It collides elastically and head on with an identical sphere B at rest. Neglect
friction everywhere. After the collision, their angular speeds are e
A
and e
B
respectively, Then
{A} e
A
< e
B
{B} e
A
= e
B
{C} e
A
= e {D} e = e
B
38. Two point masses of 0.3 kg and 0.7 kg are fixed at the ends of a rod of length 1.4 m and
of negligible mass. The rod is set rotating about an axis perpendicular to its length with a uniform
angular speed. The point on the rod through which the axis should pass in order that the work
required for rotation of the rod is minimum, is located at a distance of ..
{A} 0.4 m from mass of 0.3 kg {B} 0.98 m from mass of 0.3 kg
{C} 0.7 m from mass of 0.7 kg {D} 0.98 m from mass of 0.7 kg
118
39. In a bicycle the radius of rear wheel is twice the radius of front wheel. If
F
r and
r
r are
the radius, v
F
and v
r
are speed of top most points of wheel respectively then...
{A} v
r
= 2v
F
{B} v
F
= 2v
r
{C} v
F
= v
r
{D} v
F
> v
r
40. From a circular disc of radius R and mass 9M, a small disc of radius
R/3 is removed from the disc. The moment of inertia of the remaining
portion about an axis perpendicular to the plane of the disc and passing
through O is.
{A}
2
MR 4 {B}
2
MR
9
40
{C}
2
MR 10 {D}
2
MR
9
37
41. A child is standing with folded hands at the centre of a platform rotating about its central axis
the kinetic energy of the system is K. The child now stretches his arms so that the moment of
inertia of the system doubles. The kinetic energy of the system now is
{A} 2 K {B} K/2 {C} K/4 {D} 4K
42. If the earth is treated as a sphere of radius R and mass M. Its angular momentum about the
axis of rotation with period T is..
{A}
3
MR
T
t
{B}
T
MR
2
t
{C}
T 5
MR 2
2
t
{D}
T 5
MR 4
2
t
43. If the angular momentum of any rotating body increases by 200%, then the increase in its kinetic
energy will be..
{A} 400% {B} 800% {C} 200% {D} 100%
44. The M.I. of a body about the given axis is 1.2 kgm2 initially the body is at rest. In order to
produce a rotational kinetic energy of 1500 J. an angular acceleration of
2
sec rad 25 must be
applied about that axis for duration of .
{A} 4 sec {B} 2 sec {C} 8 sec {D} 10 sec
45. An automobile engine develops 100kw when rotating at a speed of 1800 r.p.m. what torque does
it deliver ?
{A} 350 Nm {B} 440 Nm {C} 531 Nm {D} 628 Nm
46. The moment of inertia of two rotating bodies A and B are A
I and
B
I . ( )
B A
I I > and their angular
momentum are equal. If their K.E. be
A
K and
B
K respectively then.
{A} K
A
, K
B
{B} 1
KA
KB
> {C} 1
KB
KA
= {D}
2
1
KA
KB
=
47. The centre of mass of the disc undergoes S.H.M. with angular frequency e equal to..
{A}
m
k
{B}
m
k 2
{C}
m 3
k 2
{D}
m 3
k 4
48. Three rings, each of mass P and radius u are arranged as shown in the
figure the moment of inertia of the arrangement about YY' axis will be.
{A}
2
P
2
7
u {B}
2
P
7
2
u {C}
2
P
5
2
u {D}
2
P
2
5
u
119
49. If distance of the earth becomes three times that of the present distance from the sun then
number of days in one year will be .
{A} 365

3 {B} 365

27 {C}
3 3 365
{D}
3 3
365
50. A solid sphere and a solid cylinder having same mass and radius roll down the same incline the
ratio of their acceleration will be.
{A} 15 : 14 {B} 14 : 15 {C} 5 : 3 {D} 3 : 5
51. The ratio of angular momentum of the electron in the first allowed orbit to that in the second
allowed orbit of hydrogen atom is
{A}
2
{B}
2
1
{C} {D} 2
52. A player caught a cricket ball of mass 150 gm moving at a rate of 20 m/s If the catching process
is Comlitad in 0.1 sec the force of the flow exerted by the ball on the hand of the
player .. N
{A} 3 {B} 30 {C} 150 {D} 300
53. Two disc one of the density 7.2 gm/cc and other of density 8.9 gm/cc are of the same mass
and thickness their moment of inertia are in the ratio of
{A}
9 . 8
2 . 7
{B}
2 . 7 9 . 8
1

{C}
2 . 7
9 . 8
{D}
2 . 7 9 . 8
54. Two identical hollow spheres of mass M and radius R are joined
together and the combination is rotated about an axis tangential to
one sphere and perpendicular to the line connecting their centers.
The moment of inertia of the combination is ________.
{A} 10
2
MR
{B}
2
MR
3
4
{C}
2
MR
3
32
{D}
2
MR
3
34
55. A rod of length L rotate about an axis passing through its centre and normal to its length with
an angular velocity e. If A is the cross-section and D is the density of material of rod. Find
its rotational K.E.
{A}
3
1
2
AL D
2
e {B}
3
1
6
AL D
2
e {C}
3
1
24
AL D
2
e {D}
3
1
12
AL D
2
e
56. Initial angular velocity of a circular disc of mass M is w
1
Then two spheres of mass m are attached
gently two diametrically oppsite points on the edge of the disc what is the final angular velocity
of the disc?
{A} 1
w
M
m M
|
.
|

\
| +
{B} 1
w
M
m 4 M
|
.
|

\
| +
{C} 1
w
m 4 M
M
|
.
|

\
|
+
{D} 1
w
m 2 M
M
|
.
|

\
|
+
57. A circular disc x of radius R is made from an iron plate of thickness t. and another disc Y of
radius 4R is made from an iron plate of thickness t/4 then the rotation between the moment of
inertia
X
I and
y
I is ___________
{A}
x y
I 64 I = {B}
x y
I 32 I = {C}
x y
I 16 I = {D}
x y
I I =
58. A Pulley of radius 2 m is rotated about its axis by a force ( )N t 5 t 20 F
2
= where t is in sec
applied tangentially. If the moment of inertia of the Pulley about its axis of rotation is 10 , KgM
2
120
the number of rotations made by the pulley before its direction of motion is reversed is :
{A} more than 3 but less then 6 {B}more than 6 but less then 9
{C} more than 9 {D} Less then 3
59. Two spheres each of mass M and radius R/2 are connected with
a mass less rod of length 2R as shown in figure. What will be
moment of inertia of the system about an axis passing through
centre of one of the spheres and perpendicular to the rod?
{A}
2
MR
5
21
{B}
2
MR
5
2
{C}
2
MR
2
5
{D}
2
MR
21
5
60. Four particles each of mass 'm' are lying symmetrically on the
rim of the disc of mass M and radius R moment of inertia
of this system about an axis passing through one of the particles
and perpendicular to plane of disc is _____
{A}
2
MR 16 {B}( )
2
R
M 16 M 3
2
+
{C}( )
2
R
M 12 M 3
2
+ {D} Zero
61. A mass M is supported by a mass less string wound around
a uniform cylinder of mass M and radius R as in figure. With
what acceleration will the mass fall on release?
{A} 2/3g {B} g/2
{C} g {D} 4g/3
62. Calculate rotational K.E. of earth due to its rotation about its own axis.
24
e
M =610 kg R
e
=6400 Km
{A}
29
10 2 . 6
Joule {B}
29
10 6 . 2
Joule {C}
29
10 62
Joule {D}
29
10 26
Joule
63. A cord is wound round the circumference of wheel of radius r. the axis of the wheel is horizontal
and moment of inertia about it is I A weight mg is attached to the end of the cord and falls
from the rest. After falling through the distance h. the angular velocity of the wheel will be.
{A}
mr I
gh 2
+
{B}
2
1
2
mr I
mgh 2
(

+
{C}
2
1
2
mr 2 I
mgh 2
(

+
{D} gh 2
64. If rotational K.E. is 50% of translational K.E. then the body is ..
{A} Ring {B} solid cylinder {C} Hollow sphere {D} Solid sphere
65. A meter stick of mass 400 gm is pivoted at one end and displaced through an angle 60
0
the
increase in its P.E. is ______ J.
{A} 2 {B} 3 {C} Zero {D} 1
66. Tow uniform rod of equal length but different masses are rigidly
joined to form L shaped body which is then pivoted as shown.
If in equilibrium the body is in the shown configuration ratio
M/m will be.
{A} 2 {B} 3
{C}
2
{D}
3
121
67. A light rod carries three equal masses A, B and C as shown in
the figure the velocity of B in vertical position of rod if it is released
from horizontal position as shown in the figure is .
{A} g 2 {B}
7
g 18
{C}
3
g 4
{D}
8g
3

68. A gramophone record of mass M and radius R is rotating with angular speed W. If two pieces
of wax each of mass M are kept on it at a distance of R/2 from the centre on opposite side
then the new angular velocity will be..
{A}
2

{B}
m
M m

+
{C}
M
M m

+
{D}
M m
M
+
69. A solid cylinder rolls down a smooth inclined plane 4.8m high without slipping what is its
linear speed at the bottom of the plane, if it starts rolling from the top of the plane?
(take g = 10 m/S
2
)
{A} 4 m/S {B} 2 m/S {C} 10 m/S {D} 8 m/S
70. The M.I of a disc of mass M and radius R about an axis passing
through the centre O and perpendicular to the plane of disc is
2
MR
2
.
If one quarter of the disc is removed the new moment of inertia
of disc will be..
{A}
3
MR
2
{B}
4
MR
2
{C}
2
MR
8
3
{D}
2
MR
2
3
71. The moment of inertia of a uniform rod about a perpendicular axis passing through one of its
ends is I
1
. The same rod is bent in to a ring and its moment of inertia about a diameter is
I
2
, Then
2
1
I
I
is.
{A}
3
2
t
{B}
3
4
2
t
{C}
3
8
2
t
{D}
3
16
2
t
72. A molecule consist of two atoms each of mass 'm' and separated by a distance of 'd' If 'K' is
the average rotational K.E. of the molecule at particular temperature then its angular frequency is.
{A}
m
k
d
2
{B}
m
k
2
d
{C}
k
m
d 2 {D}
k
m
4
d
73. A car is moving with a constant speed the wheels of the car make 120 rotations per minute
the breaks are applied and the car comes to rest in 8 sec how many rotation are completed
by the wheels before the car is brought to rest.
{A} 4 {B} 6 {C} 8 {D} 10
74. The angular momentum of a wheel changes from 2L to 5L in 3 seconds what is the magnitudes
of torque acting on it?
{A} L {B} L/2 {C} L/3 {D} L/5
122
75. A uniform disc of mass 500kg and radius 2 m is rotating at the rate of 600 r.p.m. what is
the torque required to rotate the disc in the opposite direction with the same angular speed in
a time of 100 sec ?
{A} Nm 600t {B} Nm 500t {C} Nm 400t {D} Nm 300t
76. The moment of inertia of a meter scale of mass 0.6kg about an axis perpendicular to the scale
and passing through 30 cm position on the scale is given by (Breath of scale is negligible). ________
{A} 0.104
2
m kg {B} 0.208
2
m kg {C} 0.074
2
m kg {D} 0.148
2
m kg
77. How much constant force should be applied tangential to equator of the earth to stop its rotation
in one day ?
{A} N 10 3 . 1
22
{B} N 10 26 . 8
28
{C}
23
1.3 10 N {D} None of these
78. A constant torque of 1500 Nm turns a wheel of moment of inertia 300 kg
2
m about an axis
passing through its centre the angular velocity of the wheel after 3 sec will be.... rad/sec
{A} 5 {B} 10 {C} 15 {D} 20
79. A disc of mass M and radius R is rolling with angular
speed w on a horizontal plane, as shown in figure. The
magnitude of angular momentum of the disc about the
origin O is ______
{A}
2
1
MR
2
e {B}
2
MR e
{C}
2
3
MR
2
e {D}
2
2 MR e
80. A mass m is moving with a constant velocity along the line parallel to the x-axis, away from the
origin. Its angular momentum with respect to the origin
{A} Zero {B} remains constant {C} goes on increasing {D} goes on decreasing
81. A body is rolling down an incline plane. If the rotational K.E. of the body is 40% of its translational
K.E. then the body is .
{A} ring {B} Cylinder {C} solid sphere {D} hollow sphere
82. A spherical ball rolls on a table without slipping, then the fraction of its total energy associated
with rotation is .
{A} 2/5 {B} 3/5 {C} 2/7 {D} 3/7
83. A binary star consist of two stars A (2.2 Ms) and B(mass 11Ms) where Ms is the mass of sun.
They are separated by distance d and are rotating about their centre of mass, which is stationary.
The ratio of the total angular momentum of the binary star to the angular momentum of star B.
about the centre of mass is _____.
{A} 6 {B} {C} 12 {D}
84. A small object of uniform density rolls up a curved surface with initial
velocity 'u'. It reaches up to maximum height of
2
3
4
v
g
with respect
to initial position then the object is ____.
{A} ring {B} solid sphere {C} disc {D} hollow sphere
85. A particle of mass m slides down on inclined plane and reaches the bottom with linear
velocity V. If the same mass is in the form of ring and rolls without slipping down the same inclined
plane. Its velocity will be______.
{A} V {B}
2V
{C}
2
V
{D} 2V
123
GRAPHICAL QUESTIONS.
86. Moment of inertia of a sphere of mass M and radius R is I. keeping mass constant if
graph is plotted between I and R then its form would be.
{A} {B} {C} {D}
87. According to the theorem of parallel axis
2
cm
md I I + = the graph between
d I
will be
{A} {B} {C} {D}
88. The graphs between angular momentum L and angular velocity w will be.
{A} {B} {C} {D}
89. The graphs between loge L and loge P is ____ where L is angular momentum and P is
linear momentum
{A} {B} {C} {D}
90. Let Er is the rotational kinetic energy and L is angular momentum then the graph between
L Er
e log and Loge can be
{A} {B} {C} {D}
124
MACHING COLUMN TYPE
91. Match list I with list II and select the correct answer.
List - I List - II
System Moment of inertia
(x) A ring about it axis
2
MR
) 1 (
2
(y) A uniform circular disc about it axis
2
MR
5
2
) 2 (
(z) A solid sphere about any diameter
2
MR
5
7
) 3 (
(w) A solid sphere about any tangent
2
MR ) 4 ( s
2
MR
5
9
) 5 (
Select correct option
Option? X Y Z W
{A} 2 1 3 4
{B} 4 3 2 5
{C} 1 5 4 3
{D} 4 1 2 3
92. Match the shape of graph with given pair of physical quantities.
Physical Quantities
{X} Moment of inertiadistance {Z}Angular momentum(L)
letargen parallel axis angular geloaty (w)
{Y}
r e
E log L log
e
{W} L log
e
P log
e
Shape of graph
{P} {Q} {R} {S}
Select Correct Option
Option? X Y Z W
{A} R S P Q
{B} Q P S R
{C} S Q R P
{D} P R Q S
125
ASSERTION - REASONING TYPE
92. In the following questions statement - 1 (Assertion) is followed by statement - 2 (Reason). Each
question has the following four choices out of which only one choice is correct.
{A} Statement - 1 is correct (true), Statement - 2 is true and Statement- 2 is correct explanation
for Statement - 1
{B} Statement -1 is true, statement -2 is true but statement-2 is not the correct explanation four
statement -1.
{C} Statement - 1 is true, statement-2 is false
{D} Statement-2 is false, statement -2 is true
93. Statement -1 The angular momentum of a particle moving in a circular orbit with a constant
speed remains conserved about any point on the circumference of the circle.
Statement -2 If no net torque outs, the angular momentum of a system is conserved.
94. Statement -1 A sphere and a cylinder slide without rolling from rest from the top of an inclined
plane. They will reach the bottom with the same speed.
Statement -2 Bodies of all shapes, masses and sides slide down a plane with the same
acceleration.
95. Statement -1 Friction is necessary for a body to roll on surface
Statement -2 Friction provides the necessary tangential force and torque.
96. Statement -1 A body tied to a string is moved in a circle with a uniform speed. If the string
suddenly breaks the angular momentum of the body becomes zero.
Statement -2 The torque on the body equals to the rate of change of angular momentum.
97. Statement -1 If there is no external torque on a body about its centre of mass, then the velocity
of the centre of mass remains constant.
Statement -2 The Linear momentum of an isolated system remains constant.
98. Statement -1 Two cylinder one hollow and other solid (wood) with the same mass and identical
dimensions are simultaneously allowed to roll without slipping down an inclined plane from the
same height. The hollow will reach the bottom of inclined plane first.
Statement -2 By the principle of conservation of energy, the total kinetic energies of both the
cylinders are identical when they reach the bottom of the incline.
99. Statement -1 A thin uniform rod AB of mass M and length L is hinged at one end A to the
horizontal floor initially it stands vertically. It is allowed to fall freely on the floor in the vertical
plane, The angular velocity of the rod when its ends B strikes the floor
L
g 3
Statement -2 The angular momentum of the rod about the hinge remains constant through out
its fall to the floor.
100 .Statement -1 If the cylinder rolling with angular speed- w. suddenly breaks up in to two equal
halves of the same radius. The angular speed of each piece becomes 2w.
Statement -2If no external torque outs, the angular momentum of the system is conserved.
126
PASSAGE BASED QUESTIONS
Passage - I
A Solid sphere of mass M and radius R is released from rest at the top of a frictionless inclined plane
of length 'd' and inclination ?.
In case (a) it rolls down the plane without slipping and in case (b) it slides down the plane
101. The ratio of the acceleration of the sphere in case (a) to that in case (b) is
[A] 1[B] 2/3 [C] 5/7 [D] 7/9
102. The ratio of the velocity of the spheres when it reaches the bottom of the plane in case (a) to
that in case (b) is
[A]
2
[B]
3
2
[C]
7
5
[D]
3
7
103. The ratio of time taken by the sphere to reach the bottom in case (a) to that in case (b) as
[A] 1 [B]
2
3
[C]
2
[D]
5
7
Passage - II
A uniform disc of mass M and radius R rolls without slipping down a plane inclined at an angle
u with the horizontal.
104. The acceleration of the centre of mass of the disc is
[A] u sin g [B]
3
sin g 2 u
[C]
3
sin g u
[D]
3
cos g 2 u
105. The frictional force on the disc is
[A]
3
sin Mg u
[B]
3
sin Mg 2 u
[C] u sin Mg [D] None
106. The magnitude of torque acting on the disc is
[A] MgR [B] u sin MgR [C]
3
sin MgR 2 u
[D]
3
sin MgR u
107. If the disc is replaced by a ring of the same mass M and the same radius R, the ratio of the
frictional force on the ring to that on the disc will be
[A] 3/2 [B] 2 [C]
2
[D] 1
Passage - III
A solid cylinder of mass M and R is mounted on a frictionless horizontal
axle so that it can freely rotate about this axis. A string of negligible mass
is wrapped round the cylinder and a body of mass m is hung from the
string as shown in figure the mass is released from rest then___
108. The acceleration with which the mass falls is
[A] g [B]
M
mg
[C]
|
.
|

\
|
+ m M
mg
[D]
( ) m 2 M
Mng 2
+
127
109. The tension in string is
[A] mg [B]
( ) m M
Mmg
+
[C]
( ) m 2 M
Mmg 2
+
[D]
( ) 2
Mg
M m +
110. The angular speed of cylinder is proportional to h
n
, where h is the height through which mass
falls, Then the value of n is___
[A] zero [B] 1[C] [D] 2
111. The moment of inertia of a uniform circular disc of mass M and radius R about any of its diameter
is
2
MR
, what is the moment of inertia of the disc about an axis passing through its centre
and normal to the disc?
[A]
2
MR
[B]
2
MR
2
1
[C]
2
MR
2
3
[D]
2
MR 2
112. A solid cylinder of mass M and radius R rolls down an inclined plane of height h. The angular
velocity of the cylinder when it reaches the bottom of the plane will be.
[A]
gh
R
2
[B]
2
gh
R
2
[C]
3
gh
R
2
[D]
gh
R 2
1
113. A cylinder of mass m and radius r is rotating about its axis with constant speed v Its kinetic
energy is _____
[A] 2mv
2
[B] mv
2
[C]
1
2
mv
2
[D] mv
2
114. A circular disc of mass m and radius r is rolling on a smooth horizontal surface with a constant
speed v. Its kinetic energy is _____
[A]
1
4
mv
2
[B]
1
2
mv
2
[C]
3
4
mv
2
[D] mv
2
115. A solid sphere is rotating about a diameter at an angular velocity w. if it cools so that its radius
reduces to 1/n of its original value. Its angular velocity becomes_____
[A]
n
e
[B] 2
n
e
[C] ne [D] n
2
e
116. In above question (115)
If the original rotational K.E. of the sphere is K, Its new value will be_____
[A] 2
n
K
[B] 4
n
K
[C]
K n
2
[D]
K n
4
117. A solid sphere is rotating about a diameter due to increase in room temperature, its volume increases
by 5%, If no external torque acts. The angular speed of the sphere will.
[A] increase by nearly 1/3 % [B] decrease by nearly 1/3 %
[C] increase by nearly % [D] decrease nearly by %
118. A cylinder of mass M has length L that is
3
times its radius what is the ratio of its moment
of inertia about its own axis and that about an axis passing through its centre and perpendicular
to its axis?
[A] 1 [B]
3
1
[C]
3
[D]
2
3
128
119. A uniform rod of length L is suspended from one end such that it is free to rotate about an
axis passing through that end and perpendicular to the length, what maximum speed must be imparted
to the lower end so that the rod completes one full revolution?
[A] gL 2 [B] gL 2 [C] gL 6 [D] gL 2 2
120. The height of a solid cylinder is four times that of its radius. It is kept vertically at time t=o on
a belt which is moving in the horizontal direction with a velocity v =
2
t 45 . 2
where v in m/s
and t is in second. If the cylinder does not slip, it will topple over a time t = ____
[A] 1 second [B] 2 sec. [C] 3 sec. [D] 4 sec.
121. The moment of inertia of a thin rod of mass M and length L about an axis passing through the
point at a distance L/4 from one of its ends and perpendicular to the rod is _____
[A]
48
ML 7
2
[B]
12
ML
2
[C]
9
ML
2
[D]
3
ML
2
122. A thin uniform rod AB of mass M and length L is hinged at one end A to the horizontal floor.
Initially it stands vertically. It is allowed to fall freely on the floor in the vertical plane. The angular
velocity of the rod when its end B strikes the floor is ____
[A]
L
g
[B]
L
g 2
[C]
L
g 3
[D]
L
g
2
123. A circular disc of radius R is free to oscillate about an axis passing through a point on its rim
and perpendicular to its plane. The disc is turned through an angle of 60? and released. Its angular
velocity when it reaches the equilibrium position will be__
[A]
R 3
g
[B]
R 3
g 2
[C]
R
g 2
[D]
R
g 2
2
124. The moment of inertia of a hollow sphere of mass M and inner and outer radii R and 2R about
the axis passing through its centre and perpendicular to its plane is
[A]
2
MR
2
3
[B]
2
MR
32
13
[C]
2
MR
35
31
[D]
2
MR
35
62
125. If a is aerial velocity of a planet of mass M its angular momentum is
[A] M [B] 2 MA [C]
M A
2
[D]
2
AM
126. A wheel having moment of inertia 2 kg
2
M
about its vertical axis, rotates at the rate of 60 rpm
about this axis. The torque which can stop the wheels rotation in one minute will be..
[A]
Nm
15
t
[B]
Nm
18
t
[C]
m N
15
2

t
[D]
Nm
12
t
127. A wheel is rotating at 900 rpm about its axis. When power is cut off it comes to rest in 1 minute,
the angular retardation in rad / sec is ___
[A]
2
t
[B]
4
t
[C]
6
t
[D]
8
t
128. What is the moment of inertia of a solid sphere of density and radius R about its diameter?
[A]
5
105
176
R
[B]
5
176
105
R
[C]
2
105
176
R
[D]
2
176
105
R
129
129. A wheel is subjected to uniform angular acceleration about its axis. Initially its angular velocity
is zero. In the first two second it rotate through an angle u
1
, in the next 2 sec. it rotates through
an angle u
2
, find the ratio
2
1
u
u
= ____
[A] 1 [B] 2 [C] 3 [D] 4
130. A gramophone record of mass M and radius R is rotating at an angular velocity w. A win of
mass M is gently placed on the record at a distance R/2. from its centre. The new angular velocity
of the system is
[A]
( ) m M 2
wM 2
+
[B]
( ) m 2 M
wM 2
+
[C] e [D]
M
wm
KEYNOTE
Q.No. Ans Q.No. Ans Q.No. Ans Q.No. Ans Q.No. Ans
1 C 31 A 61 A 91 D 121 A
2 A 32 C 62 B 92 B 122 C
3 B 33 A 63 B 93 D 123 B
4 C 34 64 B 94 A 124 D
5 D 35 A 65 D 95 A 125 B
6 A 36 B 66 D 96 D 126 A
7 C 37 C 67 D 97 D 127 A
8 C 38 B 68 C 98 D 128 B
9 B 39 C 69 D 99 C 129 C
10 D 40 A 70 C 100 A 130 A
11 B 41 B 71 C 101 C
12 B 42 D 72 A 102 C
13 A 43 B 73 C 103 D
14 A 44 B 74 A 104 B
15 A 45 C 75 C 105 A
16 C 46 B 76 C 106 D
17 B 47 D 77 A 107 A
18 B 48 A 78 C 108 D
19 A 49 C 79 C 109 D
20 B 50 A 80 110 C
21 A 51 C 81 C 111 B
22 D 52 B 82 C 112 C
23 B 53 C 83 A 113 D
24 C 54 D 84 C 114 C
25 B 55 C 85 C 115 D
26 C 56 D 86 D 116 C
27 A 57 A 87 C 117 B
28 D 58 A 88 A 118 A
29 B 59 A 89 B 119 C
30 A 60 B 90 B 120 A
B
A
130
Hints
1. Let R
cm
is at origin
2 2 1 1
cm r m r m R M + =
1 1 2 2
O = m r + m r

1 1 2 2
m r m r =

1 2
2 1
r m
=
r m
ve sign ignore as distance
2. Here
^ ^
A
r = 0i + 6J

^ ^
B
r = 6i + 6J

^ ^
C
r = 6i + 0J

^ ^
D
r = 0i + 0J

M = 1 + 1 + 2 + 2 = 6 gm
D D C C B B A A
cm r m r m r m r m R M + + + =
3. The x and y co.ordinates of centre of mass are
1 1 2 2 3 3
1 2 3
m x + m x + m x
x =
m + m + m
as m
1
= m
2
= m
3
1 2 3
1
(x +x +x) =2
3
=
similtaraly for y = 2
(x, y) = (2, 2)
4. m
1
x
1
= m
2
x
2
and m
1
(x
1
d) = m
2
(x
2
d)
1 2
md = m d'
1
2
m d
d' =
m

5. Here at origin A (0, 0), B (x, 0), C (x, x), D (0, x)


(m 0) + (2mx) + (3mx) + (4m 0)
=
m + 2m + 3m + 4m
cm
x

cm
(m 0) + (2m 0) + (3mx) + (4m )
y =
m + 2m + 3m + 4m
x
r
1
r
2
m
1 m
2
131
6. Let mass per unit area of disc = m
Mass of disc = M =
2
R m t
Mass of removed disc =
2
2
R R m
M' = m =
6 36
t | |
t
|
\ .
from figrue
R
00' =
2
R
M 0 = M' + (M M')x
2

R
M' x = M' + Mx
2
M' R
x =
M M' 2
| |

|
\ .
7. Let mass per unit area of Plote = m
Mass of whole Plote =
2
56
M = m
2
| |
t
|
\ .
Mass of removed part =
2
1
42
M = m
2
| |
t
|
\ .
Mass of remaining Portion M
2
= M M
1
C.M of whole disc R = O at origin
C.M of removed Plote = r
1
= 28 21 = 7cm
C.M of remaining Portion r
2
= ?
i i 2 2
M O = Mr + M r
8. The Velocity of C.M. is given by
1 1 2 2
1 2
m + m
=
m + m
cm
v v
V
9.
2
1 1 1
E = = =
2 2 2
I I L e e e e
2E 2E'
L = L' =
'

e e
10. Let m is the mass of unit area then mass of big disc =
2
(2R) m = M t
Let m is the mass of unit area then mass of small disc = tR
2
m =
1
M
M =
4
Mass of remaining Portion = M
2
= M M
1
2
3M
M =
4
132
Let G be the C.M of remaining Portion
M
2
(OG) = M
1
(OO)
3M M
( R) = R
4 4
o
1
=
3
o
11. The moment of inertia about AD = ?
I = m
1
(Perpendicular distance of m
1
from AD)
2
+ m
2
(Perpendicular distance of m
2
from AD)
2
+ m
3
(Perpendicular distance of m
3
from AD)
2
2 2
2 3
9 9
0 + m + m
2 2
| | | |
=
| |
\ . \ .
2
2 3
a
(m + m )
4
=
12. Let L is original length & K spring anstant then
m (L + x
1
) w
1
2
= kx
1
& M (L + x
2
) w
2
2
= kx
2
Taking ratio
2
1 1 1
2
2 2
L +
=
L +
x x
x x
| | | | e

| |
e
\ . \ .
given x
1
= 1cm, x
2
= 5cm and e
2
= e
1
13. Inital angular momentum = L
i
= Ie
i
= I x 0 = 0
angular mumentum after initial inpulse = 3kgm
2
s
1
angular mumentum after initial 4 sec = 3 + 3 = 6kgm
2
s
1
angular mumentum after initial 8 sec = 6 + 3 = 9kgm
2
s
1
angular mumentum after initial 28 sec = 24 kgm
2
s
1
angular mumentum after initial 30 sec = 24 kgm
2
s
1
Ie = 24 here
2
2 2
MR 1
I = = 5 (0.3) = 0.225kgm
2 2

24 24
= = = 106.7
I 0.225
e rad s
1
14. I
A
= m
a
r
a
2
, I
B
= m
b
r
b
2
2
b b B
A a a
m r I
=
I m r
| |

|
\ .
Let K is the mass of unit length of the wire then
a a
m = (2 r )k t and
b b
m = (2 r )k t
b b
a a
m r
=
m r

2 3
B b b b
A a a a
I m r r
= 8 = =
I m r r
| | | | | |

| | |
\ . \ . \ .
b
a
r
= 2
r

133
15. Let M be the mass and R
1
the initial radius of the earth e
1
is the angular veloalty of the rotation
of the earth, the duration T
1
of the day is
1 2
1 2
2 2
T = and T =
t t
e e
According to law of conservation of angular momentum
I
1
e
1
= I
1
e
2
16. m
1
= 1 m
2
= 35.5 r
1
= 0
^
2
r = 1.27i
1 1 2 2
cm
1 2
m r + m r
r =
m + m

17.
1 1 2 2
cm
1 2
m r + m r
r =
m + m

18. Theory [B] The centre of mass of lucky remains uncharged.


19.
o
G 72 1000/3600
= = 80rad/sec
0.25
o
r

e
e = O, = 2 n = 2 20 = 40 rad u t t t As
2 2
o
2 = w w ou
20.
2
1
= wot + t = 100 rad
2
u o u
21.
2 2 2
1 1
I = MR = ( R t )R
2 2
t
As t are same
4
I R o
4
1 1
2 2
I R
=
I R
| |

|
\ .
22. I = MK
2
= 160
2
160 160
K = = = 16
m 10
K = 4
23.
2
2
I ring MR 2
= =
1
I Disc 1
MR
2
2 : 1
24.
2 2
A Solid
2
I = I = MR = 0.4MR
5
and
2 2
B hollow
2
I = I = MR = 0.66MR
3
A B
I < I
25. I
ring
= MR
1
2
As Volume and Mass remain same
2
Solid 2
2
I = MR
5
R
2
<<< R
1
134
26. Volume of disc =
3
2 1
1 1
R
V = R t =
6
t
t
3
3 1
2
R 4
= R
6 3

Volume of Sphere =
3
2 2
4
V = R
3
t
3 3
1 2
R = 8R
1 2
R = 2R
I
1
= M.I of disc =
2
1
1
I = MR
2
I
2
= M.I of sphere =
2
2 1
2
2 MR I
MR = =
5 5 2 5
27. Mass of the ontire disc would be AM and its moment of inertia about the given axis would
be
2
1
2
(AM)R . For the given section the moment of inertia about the since axis be one qvarter
of this is
2
1
2
(AM)R .
28. Mass per unit length of the wire =
Mass of L length M = L
When it is lent in form of circularring
2 r = L t
L
R =
2

t
Moment of inertia of ring about given axis =
2
3
MR
2
29. M.I of disc =
2
2
1 1 M 1 M
MR = M =
2 2 t 2 t
| |
|
t t
\ .
2
2
M M
= R =
R t t


`
t t
)

As their mass & thickness are some


1
I o

30. Let I
Z
be the M.I of square plote about the axis passing through the centre and perpendicular
to the plane of square, hence according to Perfendicular axis theorm.
I
Z
= I
AB
+ I
A'B'
Also I
Z
= I
CD
+ I
C'D'
As axis are symmetric I
AB
= I
A'B'
=
Z
I
2
And I
CD
= I
C'D'
=
Z
I
2
So we can say that I
AB
= I
A'B'
= I
CD
= I
C'D'
= I
31. Let the radius of complete disc is a & that of small disc is b After small disc is cut from complete
disc let the C.M. shift to O
2
at distance x
2
flem original centre O.
The Position of new C.M. is givenly let 6 is mass percunitarea.
2
cm 2 2
6 b 1
X =
6 a 6 b
t
t t
135
32. Let the mass of an element of length dx of the rod located at a distance X away from left
and is
M
dx
L
. the x cordinate of the C.M. is given by. .
Total mass of rod =
1 2
0
AL
Ax dx =
2

}
1
cm
2
0
1 1
x = xdm = x (Ax dx)
M AL
2
| |
|
\ .
} }
33. By Conservation of Energy P.E. of rod = Rotational K.E. Mg
1
2
sind =
1
2
Ie
2
=
1
2

2
2
mL
3
e
3g sin
=
L
o
e
As here l = 2L
3g sin
=
2L
o
e
34. Angular momentum of Block w.r.t. O before collision with
O = MV
a
2
on collision the block will rotate about the side
passing through O. Now its angular momentum = Iw
Acc. to law of conservation of angular momentum
MV
a
2
| |
|
\ .
= Ie
2 2
M M
+
6 2
a a | |
= e
|
\ .

3
=
4
v
v
e
here I is the M.I of block about the axis perpendicular to the plane passing through O.
35. Given system of two purtides will rotate about its centre of mass.
initial angular momentum =
L
M
2
v
| |
|
\ .
Final angular momentum
2
L
2I = 2M
2
| |
= e e
|
\ .
By law of conservation of angular momentum
L
MV
2
| |
|
\ .
=
2
L
2M
2
| |
e
|
\ .

V
=
L
e
36. Initial angular momentum of ring L = Ie = MR
2
e
Final angular momentum of ring and particles = (MR
2
+ 2mR
2
) e'
As No external forque so According to Law of conservation of angular momentum.
MR
2
e = (MR
2
+ 2mR
2
) e'
wM
' =
(M + 2m)
e
136
37. As it is head-on elastic collision between two idential balls there fore they will exchange their
linear vecocity is A comes to rest and B starts moving with linear velocity V.
As there is no friction any where, forque on both the spheres about their centre of mass is
zero and their angular velocities remains unchanged.
Therefore e
A
= e and e
B
= 0
38. I = 0.3x
2
+ 0.7 (1.4 x)
2
For minimum work moment of inertia of the system should be minimum is
dI
= 0 = 03 2x 0.7 2 (1.4 x) = 0
dx

x = 0.98 m
[B] 0.98 m from mass of 0.3 kg
39. Angular speed for both wheels are different but lincar speed for both same so V
F
= V
r
40. M.I of complete disc about O point
I Total =
2
1
2
(gM) R
Radius of removed disc =
R
3
As
2
M = R t i.e.
2
M R o
Mass of removed portion =
gM
M
g
=
M.I of removed disc about it own axis =
2
2
1 R MR
M =
2 3 18
| |
|
\ .
M.I of removed disc about O. I removed =
2
2 2
2
cm
MR 2R MR
I + Mx = + M =
18 3 2
| |
|
\ .
I
total
= I removed disc + I remaining part
41.
2
L
E = = K
2I
given
1
K
I
o
If L is constant when child stretches his arms the moment of inertia
of system get doubled so kinetic energy will becomes half i.e.
K
2
42. Moment of inertia of sphere
2
2
I = MR
5
about its axis
L = Ie
43.
2
L
E =
2I
2
E L o
44. Rotational K.E. =
2
1
I = 1500
2
e
According to w = wo + t o
137
45. e = 1800 rpm
2 1800
= 60 rad/sec
60
t
= t
P = te
46. As
2
I
K =
2I
2
A
A
J
K =
2I
and
2
B
B
J
K =
2I
47. As M is the mass of disc, the force is producing angular accoration in the disc, therefore
2
4 4k
= m( ) =
3 3m
kx
x e e
48. For Ring 1 and 2 I
1
and I
2
about
' 2
3
YY = MR
2
and for Ring 3. I
3
about
' 2
1
YY = MR
2
M.I. of system
1 2 3
I = I + I + I
49.
2 3
2 2
2 3
1 1
T r
=
T r
50. 2 2
g sin
a =
(1 + k / R )
u
1
5g sin
a =
7
u

and
2
2g sin
a =
3
u
51.
nh
L =
2t
52. Impulse = Change in momentum = F t
L
F =
t
A

A
53.
2
2
1 1
1 1
2
2
2 2
2 2
1
M R
I R
2
= =
1
I R
M R
2
54. Apply theorm of parallel axis
55. Rotational
2
1
K.E. = I
2
e
56. If t = 0 I
1
w
1
= I
2
w
2
57. Mass = Volume = M = tR
2
t
M.I of x is
2
x 1 1
1
I = m R
2
58. Here direction of Motion will be reversed when force F = 0 or 20 t 5r
2
= 0 or t = 4sec.
If o is angular accellaration then forque = I = F.r t o OR
2
10 = (20t 5t ) 2 o OR
2
= 4t t o
and
d
w =
dt
u
Also
d
= t
dt
u
o
138
2 2 3
d = .t dt = (4t t ) t dt = (4t t ) u o Taking intigration
3 4
4t t
=
3 4
u If n rotations are completed in As then Putting t = 4
4 64 64
= 2 n = 64 =
3 3

u t
n = 3.4
which is 3 < 3.4 < 6
59.
2 2
2
2 R 2 R
I = M + M(2R) + M
5 2 5 2
| | | |
| |
\ . \ .

2
21
MR
5
=
60. According to the Parallel axis theorm M.I of disc about an axis passing through particle (3)
and perpendicular to plane of disc is
2 2 2
1 3
MR + MR = MR
2 2
=
61. In falling through a height h.
P.E. = K.E
2 2 2 2 2
1 1 1 1 1
= + = +
2 2 2 2 2
mgh mv I mv mr
| |
e e
|
\ .
According to V
2
v
2
= 2ad taking v = 0 & d = h, V
2
= 2ah so
3
mgh = m 2ah
4

62.
2 24 6 2 37 2
2 2
I = MR = 6 10 (6.4 10 ) = 9.83 10 kgm
5 5

Angularvelocity of earth
2

T
t
=
K.E. of rotation of the earth
2
1
I
2
= e
63. We know
2
2
k
r
2gh
=
1 +
v
2 2 2 2
V 2gh 2mgh
= =
2 r + k mr + mk
e
64. kr = 50% k
T
65. Centre of mass of stick is at midpoint when it is displaced by 60
0
Its c.m. rises up to height
h from figure = cos = (1 cos )
2 2 2
l l l
h u u
so increase in P.E. = mgh =
(1 cos )
2
l
mg u
o
1
0.4 10 (1 cos60 )
2
= = 1 Joule
139
66. Net forque about O should be zero
Hence
o o
1 1
Mg sin 60 = Mg sin30
2 2
(
`
(
)
67. Loss in PE = gain in agular kinetic energy
2
2
+ mg + mg =
3 3 2
l l l
Mg l Iw
| |
|
\ .
Here I = Moment of inertia about fixed point
2 2
2 2
2 14
M + m m = m
3 3 9
l l
l l
| | | |
= +
| |
\ . \ .
From fig.
2 2
1 14 36g 2 36 89
2 = = As V r =
2 9 14l 3 14 7
l
g l
mgl ml
l
| |
e e = e=
|
\ .
68. Ie = I'e'
2 2 2
MR MR R
= + 2m '
2 2 4
( | |
e e
( |
\ .
M = (M + m) ' e e
69.
2
2
2gh
=
k
1 +
V
r
70. New Mass
3
M
4
M.I of disc =
2
MR
2
New MI =
' 2 2
1 3 3
= =
2 4 8
I M R MR
(
(

[C]
71. For a rod of mass M and length L., the MI about a perpendicular axis passing through one
and is
2
1
ML
I =
3
when it is bent to form a ring, then L = 2 R t
L
R =
2
t
The M.I of the ring about its diameter is
2 2 2
2 2 2
MR ML ML
I = = =
2 4 2 8 t t
72. The M.I of the molecule =
2 2
d d
M + m
2 2
| | | |
| |
\ . \ .
2 2
d md
I = 2m =
4 2
| |
|
\ .
The Rotational K.E. of the moldule
2
1
( ) =
2
K Ie
2K
=
I
e
140
73.
2 120
= = 4 rad/sec
60
o
t
e t
Now e = e
0
+ ot
Total angle descrited in 8 second is
2
o
1
= w t + t
2
u o
74.
dL 5L 2L 3L
= = = = L
dt 3 3
t
75. For disc
2
MR 500 4
I = =
2 2

angular speed =
2 600
=
60
t
e
so angular momentum L = Ie
final angular momentum in opposite direction =
2
kgm
1000 20
sec
t
So change in angular momentum =
2
L = 2 1000 20 kgm / sec A t
dL 2 1000 20
= = = 400 N.m.
dt 100
t
t t
76. By Parallel axis therom solved problem.
77.
1
= 2 rad/day e t and e
2
= 0, t = 1 day
2 1
w w
=
t
o
Forque required to stop the earth = T = Io = F.R.
I.
F =
R
o
78.
d
= I = I
dt
e
t o
79. The disc have two types of motion translational and rotational so there will be two types of
angularmotion there fore total angular momentum should be total of both
T R
L = L + L
80. L = Momentum x perpenlicular distance between
point of rotation and line of action
= m.V.y all remain constant
L = remaing constant
81.
40
R.K.E. = T.K.E
100
2 2 2
1 4 1 1
= =
2 10 2 5
I mv mv e
2
2 2
2
1 V 2
mk =
2 5
mv
r

141
82. Total energy
2 2 2 2 2 2
1 1 1 2 1
= + = +
2 2 2 5 2
E I mv mr mr e e e
83. Here C.M. wrt A.
5
5 5
11M 5d
d =
11M + 2.2M 6
| |
=
|
\ .
2
2
5 5
5d d
= 2.2 M w = 55M
6 36
A
L w
| |
|
\ .
2
2
5 5
d d
= 11 M = 11M
6 36
B
L
| |
e e
|
\ .
84. At the highest point the whole energy is conserted to P.E. of the object.
P.E. = mgh,
2
3v
h =
4g
2 2
3v 3m
PE = mg =
4 4
v
g

Total K.E. of body =


2
2 2 2
2
1 1 1 1
+ = +
2 2 2 2
IV
mv Iw mv
r
Now, K.E. = P.E.
85. Here in first case
2
1
=
2
mg mvh = 2gh v
In second case
1
2
2
2
K
R
2gh
' =
1 +
v
(
(
(

As for the ring K = R ' = gh v
86.
2
2
I = MR
5
2
I R o
This relation shows that the graph between I and R will be paracola symatric to I axis
87. Graph should be paracola symmetric to I - axis, but it should not pass from origin because
there is a constant value I
cm
plesent for d = 0
88. L = Iw L o w If I constant so graph between L and w will be straight line with constant
stop and passing from origin.
89. = L r P

e e e
log L = log P + log r
If graph is drawn between log
e
L and log
e
P then it will be straight line which will not pass
through origin at log
e
P = 0 log
e
L = log
e
r
90.
2
r
L
E =
2I
so Log E
r
= 2 log L log (2I)
So the graph
r
log E log L will be straightline with constant stop and when Log L = 0.
Log E
r
= log (2I)
142
93. The correct choice is [D] since the centrifetal force is radial. Forque is zero so L = constant.
94. The correct choice is [A] If a body slides down an indined plane its acaleration is
a = g (sin cos ) u u which depends only on g,
u
and .
98.
cm
2
g sin
a =
I
1 +
mR
u
I
cm
of hollow cylinder is loss, so it will have more accderation and will take less time to reach
bottom.
99. Loss in P.E. is equal to gain in rotational K.E. As the centre of mass of the rod falls through
the distance
L
2
2
2 2
L ML 3g
Mg I =
2 2 3 L
| | 1 1
= e = e e
|
2
\ .
100. If I is the M.I. of the complete cylinder, The M.I of each Riece becomes
I
2
since L = Ie,
the angular speed of each Piece becomes 2e.
101. In case (a) accelaration down the plane is
1
2
g sin 5
= = g sin
I 7
1 +
MR
a
u
u
| |
|
\ .
So
1
2
a 5
=
a 7
In ase (b)
2
a = g sinu
102. using V
2
= 2ad we can find.
2
1 1 1
2
2 2 2
v v 5
= =
v 7 v
a
a

103. From
2
1
d = at
2
, we find that
1
2
d 7
=
d 5
104. Mg sin f = Ma u
t = I . o As
2
1
I = MR
2
,
a
=
R
o
and t = fR
Hence
2
1 a 1
fR = MR = MRa
2 R 2

105.
1
f = Ma
2
Mg sin
f =
3
u

106.
MgR sin
fR =
3
u
t =
143
107. For a ring I = MR
2
Correct choice is
108. Mg T = Ma, T is Tension in string forque on cylinder t = TR = Io
2
2
I 1 a 1
= = MR = Ma
R 2 R 2
o | |
t
|
\ .
2mg
a =
(M + 2m)
[D]
109.
MMg
T =
M + 2M
[D]
110. From conservation of energy we have
2 2
1 1
= +
2 2
Mgh MV Ie

2 2 2 2 2 2
1 1 1 1
+ (2 + M)
2 2 2 4
MR MR m R
| |
= e e = e
|
\ .
1
2
2
4mgh
=
(M + 2m) R
(
e
(

As
1
2
h e o
111. According to perpendicular axis theorm
2
x y
1
I = I = MR
4
(Considering two perpendicular diameter in x & y directions)
So
2 2 2
x y c
1 1 1
I I = I = MR + MR = MR
4 4 2

112. For solid cylinder


2
1
I = MR
2
As
2 2
1 1
= +
2 2
Mgh MV Ie
113. K.E. in Rotation
2
1

2
Iw =
Here
2
1
I mr
2
= and
V
=
r
e
therefore
2
2 2
1 1 1
K.E = MR =
2 2 r 4
v
mr
| |

|
\ .
114. Rotational
2
1
K.E = I
2
e & Translational
2
1
K.E =
2
mv
2 2
1 1
K.E = +
2 2
Mv Ie
144
115. A solid sphere
2
2
M.I =
5
mr
As
2
2
1 1 1 1
2 2
= I = =
5 5
r
Iw mr m
n
| |
e e e
|
\ .
2 1
1 2
= = n
n
e
e e e
116.
2
1
K =
2
Ie and
2
1 1 1
1
=
2
K I e
2
2 2 2 2
1 1 2
1 2 r 1 2 1
K = m = mr (n )
2 5 n 2 5 n
| |
e e
|
\ .
2 2 2
1
1
K = I kn
2
n e =
117.
3
4
V = r
3
t
4
logV = log + 3 log r
3
t | |
|
\ .
differentiating we have
dv dr
= 3
v r
dr 1 dv 1 1
= = 0.5% = %
r 3 v 3 6

No external forque so Ie = constant
2
2
mr
5
= e = constant
2
r = constant e 2 log r + log = log c e
dr d
2 + = 0
r
e

e
d dr 1 1
= 2 = 2 % = %
dt 6 3
e

e
-ve sign for decrensing e
118.
2 2
2
1 2
1 L
= , = M +
2 4 12
R
I MR I
| |
|
\ .
119. In one full revolution the incrase in P.E = MgL, where M is the mass of rod.
2
2 2
1 1
MgL = =
2 2 3
ML
I
| |
e e
|
\ .
120. The cylinder will topple when the forque mgr equals the forque
h
ma
2
=
so
h
ma = mgr
2
2gr g
a = =
h 2

Now V = 2.45t
2
and
dV
=
dt
a
2
d
a = [2.45(t) ] = 4.gt
dt
145
121. Using I = I
cm
+ Md
2
Parallel axis theorm
L L L
d = =
2 4 4
and
2
cm
ML
I =
12
2 2 2
ML ML 7ML
I = + =
12 16 48

122. Loss in P.E = gain in Rotational K.E. centre of Mass of rod is at


L
2
.
MgL
P.E =
2
Gain in
2
2 2
1 1 1
R.E = =
2 2 2 3
ML
I
| |
e e
|
\ .
2 2
MgL ML 3g
= =
2 6 L
e
e
123. P.E. at 60
0
= Mgh (1 cos ) u
Eqvillibrium at
2
1
K.E =
2
I
| |
e
|
\ .
For I According to Parallel axis theorm (as d = R)
2 2 2 2
cm
1 3
I = I + MR = MR + MR = MR
2 2
Here
2 o
1
= Mgh (1 cos 60 )
2
Ie
124. We can obtain hollow sphere as it solid sphere of radius R is removed from a solid sphere
of 2R mass of hollow sphere M = M
1
M
2
It is density
3
1
4
M = (2 )
3
R t and
3
2
4
M =
3
R t
2
28
=
3
M R t
Moment of inertia of hollow sphere
2 2
1 2
2 2
I = M (2R) M R
5 5
By substituting the values of M
1
and M
2
125. Areal Velocity
2
R
A =
T
t
and
2
T =
t
e
2 2
2
R R 2A
A = = =
2 2 R
t e e
e
t
= I L e
146
126. Here I = 2k
g
m
2
n
1
= 60rpm = 1rps
T = ? n
2
= 0 t = 1mn = 60 sec
2 1
I ( )
T = I. =
t
e e
o
127. Angular retardation
2 1

=
t
e e
o
128. For sphere
2 3 2
2 2 4
= =
5 5 3
I MR R R
| |
t
|
\ .
129. From
2
1
= wt + t
2
u o
2
1
1
= 0 + (2) = 2
2
u o o
2
1 2
1
+ = 0 + (4) = 8
2
u u o o
130. Angular momentum of Recoul L = Ie Where
2
1
I = MR
2
Let w' be the angular velocity after putting coin of mass m at distance
R
2
from centre the angular
momentum of system L' ( I + mr
2
) e' since T = 0 so L' = L
2
(I + mr ) ' I e = e
2
2 2 2 2
2
1
2
1
2
I
' = = =
I + mr + mr 2mr
1 +
MR
MR
MR
e
e e
e
| |
|
\ .
but
R
r =
2
2 M
' =
2M + m
e | |
e
|
\ .
147
Unit - 6
Gravitational
148
SUMMARY
1. Gravitational force between two point masses is
1 2
2
Gm m
F
r
=
2. Acceleration due to Gravity
(I) on the surface of earth =
2
2
9.81
GM
ms
R

=
(II) At a height h from surface of earth
1
2
2
(1 )
1
= =
| |
+
|
\ .
g h
g g
R
h
R
if h << R
(III) At a depth d form the surfce of earth
1
/(1 )
d
g g
R
=

g
1
= g if d = R i. e. on the surface of earth
(IV) Effect of rotation of earth at latitude
g
1
= g Re
2
cos
2

- at the equator = 0
g
1
= g Re
2
= minimum value
- At the pole = 90
0
g
1
= g Re
2
= maximum value
- At the equator effect of rotation of earth is maximum and value of g is minimum.
- At the pole effect of rotation of earth is zero and value of g is maximum.
3. Field Strength
Gravitational field strength at a point in gravirtational field is defined as,
F
E

=
m
= gravitational force per unit mass
Due to point mass
2
GM
E
r
= (towards the mass)
2
r
1
E
Due to solid sphere
inside points
3
=
i
GM
E r
R
At r = 0, E = 0 at the center
At r = R,
2
GM
E
R
= i.e. on the surface
out side points E
o
=
2
GM
or
r
2 o
r
1
E
149
At r

, E 0
Due to a sphericell shell -
inside points E=0
outside points E
0
=
2
GM
r
just outside surface
2
GM
E
R
=
on the surface E r graph is discontinuous
on the axis of a ring
3
2 2
2
( )
=
+
r
GMr
E
R r
At r = 0, E = 0 i.e. at the center
If r >> R,
2
=
GM
E
r
, i.e. ring behaves as a points mass
As 0 r E
4. Gravitational potential :-
(i) Gravitational potential at a point in a gravitational field is defined as the negative of work
done in moving a unit mass from infinity to that point per unit mass, thus
p
p
w w
V
m


=
(ii) Due to point mass

Gm
V
rm
=
0 0 = v as r and v as r
(iii) Due to solid sphere
inside points
2 2
(1.5 0.5 )
GM
Vi R r
R
=
as r = R =
GM
V
R
i.e. on the surface
V - r graph is parabola for inside points
out side points
GM
v
r
=
(iv) Due to sphercal shell
inside points
GM
Vi
R
=
outside points =
GM
Vi
R
150
(v) on the axis of a ring
2 2
GM
Vr
R r
=
+
at r = 0,
=
GM
V
R
i.e. at center
5. Gravitational potential Energy
(i) This is the work done by gravitational forces in arranging the system from infinite sepration
in the present position
(ii) Gravitational potential energy of two point massess is
1 2
Gm m
U
r
=
(iii) To find the gravitational points energy of more than two points masses we have to make
pairs of masses. Neighter of the pair should be repeated. For example in case of four point
masses.
4 3 4 2 4 1 3 2 3 1 2 1
43 23 41 32 31 21
m m m m m m m m m m m m
U G
r r r r r r
(
= + + + + + +
(

for n point masses total number of pairs will be
( 1)
2
n n
(iv) If a point mass m is plaled on the surface of earth the potential energy here is Uo
o
GMm
U
R
=
and potential energy at height h is
h
GMm
U
(R h)
=
+
the difference in potential energy would be
AU = Uh Uo =
mgh
1 + h/r
If h << R, AU = mgh
6. Relation between field strength

E
and potential V
(i) if V is a function of only one variable (Say r) then
dV
E
dr
= =
slope of U r graph
(ii) If V is funtion at three coordinates variable; e x, y, and z then
v v v

E i j k
x y z

| | c c c
= + +
|
c c c
\ .
151
7. Escape velocity
(i) From the surface of earth
2
2
2
GM GM
Ve gR as g
R R
= = =
=11.2 km/ sec
(ii) Escape velocity does not depend upon the angle which particle is projected form the surface
and the mass of body
8. Motion of satellites
(i) orbital speed
GM
Vo
r
=
(ii) time period
3/ 2 2 3
2
T r T r
GM

=
(iii) Kinetic energy
2
=
GMm
K
r
(iv) Potential energy
GMm
U
r
= = ...
(v) Total Mechanical energy.
GMm
E
r
=
9. Keplers laws
- First law :
Each planet moves in an elliptical orbit with the sun at one focus of ellipse
- Second law :
The radius vectors drawn form the sun to a planet, sweeps out equal area in equal time interval
i.e. areal Velocity is constant.
this law is derived from the law of conservation of angular momentum
2
dA L
dt m
=
= constant here L is the angular momentum and m is mass of planet
- Third law
2 3
T r
where r is semi-major axis of elliptical path
- The gravitational force acting between two bodies is always attractive. It is independent of medium
between bodies. It holds good over a wide range of distance. It is an action and reaction pair.
It is conservative force. It is a central force and obey inverse square law as
2
1/ F r
- The value of G is never zero any where but the value of g is zero at the center of earth.
- the acceleration due to gravity is independent of mass, shape, size etc of falling body.
- the rate of decrease of the acceteration due to gravity with height is twice as compared to that
with depth.
- It the rate of rotation of earth increases the value of acceleration due to gravity decreases at
all points on the surface of earth except at poles.
152
- If the radius of planet decreasees by n% keeping its mas unchanged, the accelreotion due to
gravity on its surface increases by 2n%.
- If the mass of a planet increases by n% keeping its radius unchanged the acceleration due to
gravity on its surface increases by n%.
- The value of g at a location gives the value of intensity of gravitational field at the location.
- The orbital velocity of a satellite is independent of mass of the satellite but depends upon the
mass and radius of planet around which the rotation is taking place
- The value of orbital velocity for a satellite near the surface of earth is 7.92 kms
1
.
- The direction of orbital velocity of satellite at an instant is along the tangent to the orbital path
at that instant.
- The work done by a satellite in a complete orbit is zero.
- For a satellite orbiting close to the surface of earth (h << R), the time period of revolution
is 2 / 84.6 R g = minutes and angular velocity.
2
/ 0.001237 / sec g R rad
T

= = =
- A geostationary satellite revolves around the earth from west to east. Its period of revoluton
is one day i.e. 24 hours. The orbital velocity of geostationary satellite is 3.08 kms
1
. Its height
above the surface of earth is about 36000 km. The relative angular velocity of geostationary
satellite w.r.t earth is zero.
- When a satelilte is orbiting in its orbit, no energy is required to keep it in its orbit.
- When the total energy of a satellite is negative, it will be moving in either a circular or an elliptical
orbit.
- When the total energy of a satellite is zero, it will escape away from its orbit and its path becomes
parabolic.
- When the height of satellites is increased its potential energy will increase and K.E. will decrease.
- When the velocity of satellite is increased, its total energy will increase and it will start orbiting
in a circular path of larger radius.
- For a satellite orbiting in a circular orbit, the value of potenial energy is always greater than
its K.E.
- If the velocity of a satellite orbiting the earth is increased by 41.4% or its K.E. is doubled,
then it will escape away from the gravitational field of earth
- Escape velocity =.
2
orbital velocity
- If the gravitational force is inversely proportional to the nth power of distance r, then the orbital
velocity of a satellite
/ 2
0
n
V r and time period is
( 2)
2
n
T r
+

- When a body is projected horizontally with velocity v, from any height from the surface of earth,
then the following possibilities are there.
(i) If v < v
0,
the body fails to revolve around the earth and finally falls to the surface of earth.
(ii) If v = v
0,
the body will revolve around the earth in circular orbit.
(iii) If v < v
e
the body will revolve around the earth in elliptical orbit.
(iv) If v = v
e
, the body will escape from the gravitational field of earth.
(v) If v > v
e
the body will escape, following a hyperbolic path.
153
- If earth suddently contracts to
1
th
n
of its present size without any change in mass, the duration
of the day will be nearly 24/n
2
hours.
- Force function F(r) is related with potential energy function U(r) by a relaion
=
dU
F
dr
- A given planet will have atmosphere if the root mean square velocity of molecules in its atmosphere
(i.e. V
rms
=
3RT
M
) is smaller than escape velocity for that planet.
- In the weightlessness state, the bodies donot have weight but they do possess inertia on account
of their mass. the bodies floating inside the space craft may collide with each other and crash.
- If a body is released from a height aqual to n times the radius of earth, then its striking velocity
on the surface of earth is
2
1
ngR
n
=
+
- If polar ice caps melt then moment of inertia, Angular velocity will decrease and period of rotaiton
of earth increase.
- The, line joining the places on earth having same value of g are called isogams.
- Gravity meter and Etvos gravity balance are used to measure changes in accelaration due to
gravity.
154
Newtons Law of Gravitation
For the answer of the following questions choose the correct alternative from among the
given ones.
1. Two identical solid copper spheres of radius R are placed in contact with each other. The
gravitational force between them is proportional to
(A) R
2
(B) R
-2
(C) R
-4
(D) R
4
2. The gravitational force Fg between two objects does not depend on
(A) sum of the masses (B) product of masses
(C) Gravitational constant (D) Distance between the masses
3. The atmosphere is held to the earth by
(A) clouds (B) Gravity (C) Winds (D) None of the above
4. Two sphere of mass m
1
and m
2
are situated in air and the gravitational force between them
is F. The space around the masses is now filled with liquid of specific gravity 3. The gravitational
force will now be
(A) F (B) 3F (C) F/3 (D) F/9.
5. A satellite of the earth is revolveing in a circular orbit with a uniform speed v. If the gravitational
force suddennly disappears, the satellite will
(A) Contineue to move with velocity v along the original orbit.
(B) Move with a Velocity v, tangentially to the original orbit.
(C) Fall down with increasing velocity.
(D) Ultimately come to rest somewhere on the original orbit.
6. Correct form of gravitational law is
(A)
1 2
2
Gm m
= F
r
(B)
1 2
2
Gm m
= F
r

(C)
1 2
3
Gm m
= F r
r

(D)
1 2
3
Gm m
= F r
r


7. Mass M is divided into two parts xM and (1 - x) M. For a given separation, the value of
x for which the gravitational force between the two pieces becomes maximum is
(A) 1 (B) 2 (D) 1/2 (D) 4/5
8. The earth (mass = 6 x 10
24
kg) revolves around the sun with angular velocity 2 10
7
rad/
sec in a circular orbit of radius 1.5 x 10
8
km. The force exerted by the sun on the earth is
= ................... N
(A) 18 10
25
(b) zero (C) 27 10
39
(D) 36 10
21
9. Two particle of equal mass go round a circle of radius r. Under the action of their mutual
gravitational force.
The speed of each particle is =..................
(A)
1 1
2r Gm
=
(B)
2
Gm
r
=
(C)
1
2
Gm
r
=
(D)
4Gm
r
=
10. The distance of the moon and earth is D the mass of earth is 81 times the mass of moon.
At what distance from the center of the earth, the gravitational force will be zero
(A) D/2 (B) 12D/3 (C) 4D/3 (D) 9D/10
11. One can easily Weight the earth by calculating the mass of earth using the formula
(in usual notation)
(A)
Re
g
G
(B) g/G Re
2
(C)
2
Re
G
g
(D)
3
Re
G
g
155
12. Three equal masses of m kg each are plced the vertices of an equilateral triangle PQR and
a mass of 2m kg is placed at the centroid 0 of the triangle which is at a distance of
2 m
from each of vertices of triangle. The force in newton. acting on the mass 2m is = ............
(A) 2 (B) 1 (C)
2
(D) zero
13. Which of the follwing statement about the gravitational constant is true
(A) It is a force
(B) It has no unit
(C) It has same value in all system of unit
(D) It depends on the value of the masses.
14. Two point masses A and B having masses in the ratio 4 : 3 are seprated by a distance of
lm. When another point mass c of mass M is placed in between A and B the forces A and
C is 1/3rd of the force between band C, Then the distance C form A is = ............... m
(A) 23 (B) 1/3 (C) 1/4 (D) 2/7
15. The gravitational force between two point masses m
1
ans m
2
at separation r is given by
1 2
2
=
mm
F G
r
The constant k ...............
(A) Depends on system of units only.
(B) Depends on medium between masses only.
(C) Depends on both (a) and (b)
(D) is independent of both (a) and (b)
Acceleration Due to Gravity
16. As we go from the equator to the poles, the value of g ...............
(A) Remains constant (B) Decreases (C) I ncreases (D) Decreases upro latitude of 45
o
17. If R is the radius of the earth and g the acceleration due to gravity on the earths surface,
the mean density of the earth is = ...............
(A) 4 3 / G gR (B) 3 4 / R gG (C) 3 4 / g RG (D) 12 / RG g
18. The radius of the earth is 6400 km and g=10ms
-2
. In order that a body of 5 kg weights zero
at the equator, the angular speed of the earth is = ............... rad/sec
(A) 1/80 (B) 1/400 (C) 1/800 (D) 1/600
19. The time period of a simple pendulum on a freely moving artificial satellite is ............... sec
(A) 0 (B) 2 (C) 3 (D) Infinite
20. A spherical planet far out in space has mas Mo and diameter Do. A particle of m falling near
the surface of this planet will experience an acceleration due to gravity which is equal to
(A)
2
0
/ GM Do (B)
2
4 / mGMo Do (C)
2
4 / GMo Do (D)
2
/ GmMo Do
21. A body weights 700 g wt on the surface of earth How much it weight on the surface of planet
whose mass is 1/7 and radius is half that of the earth
(A) 200g wt (B) 1400g wt (C) 50 g wt (D) 300g wt.
22. The value of g on he earth surface is 980 cm/sec
2
. Its value at a height of 64 km from the
earth surface is ............... cm5
2
(a) 960.40 (B) 984.90 (C) 982.45 (D) 977.55
156
23. If earth rotates faster than its present speed the weight of an object will.
(A) increases at the equator but remain unchanged of the poles.
(B) Decreases at the equator but remain unchanged at poles.
(C) Remain unchanged at the equator but decreases at poles.
(D) Remain unchanged at the equator but increases at the poles.
24. The moons radius is 1/4 that of earth and its mass is 1/80 times that of the earth. If g represents
the acceleration due to gravity on the surface of earth, that on the surface of the moon is
..............
(A) g/4 (B) g/5 (c) g/6 (D) g/8
25. The depth of at which the value of acceleration due to gravity becomes 1/n the time the value
of at the surface is (R = radius of earth)
(A) R/n (B)
( 1) n
n
R

(C) R/n
2
(D)
1
n
R
n
| |
|
+
\ .
26. If the density of smalll planet is that of the same as that of the earth while the radius of the
planet is 0.2 times that of lthe earth, the gravitational acceleration on the surface of the planet
is ...............
(A) 0.2g (B) 0.4g (c) 2g (D) 4g
27. If mass of a body is M on the earth surface, than the mass of the same body on the moon
surfae is
(A) M/6 (B) 56 (C) M (D) None of these
28. An object weights 72 N on the earth. Its weight at a height R/2 from earth is = .............. N
(A) 32 (B) 56 (C) 72 (D) zero
29. If the radius of earth is R then height h at which value of g becomes one - fourth is
(A) R/4 (B) 3R/4 (C) R (D) R/8
30. If the mass of earth is 80 times of that of a planet and diameter is double that of planet and
g on the earth is 9.8 ms
-2 ,
then the value of

g on that planet is = ............... ms
-2
(A) 4.9 (B) 0.98 (c) 0.49 (D) 49
31. Assuming earth to be a sphere of a uniform density, what is value of gravitational acceleration
in mine 100 km below the earth surface = ............... ms
-2
(A) 9.66 (B) 7.64 (C) 5.00 (D) 3.1
32. Let g be the acceleration due to gravity at earths surface and k be the rotational K.E. of earth
suppose the earths radius decreases by 2% keeping alt other quantities same then
(A) g decreases by 2% and K decreases by 4 %
(B) g decreases by 4% and K increases by 2%
(C) g increases by 4% and K increases by 4%
(D) g decreases by 4% and K increases by 4%
33. A body weight 500 N on the surface of the earth. How much would it weight half way below
the surface of earth
(A) 125N (B) 1250N (C) 500N (D) 1000N
34. The radii of two planets are respectively R
1
and R
2
and their densities are respectively
1
and

2
the ratio of the accelerations due to gravity at their surface is ...............
(A)
1 2
1 2 2 2
1 2
: . = g g
R R

(B)
1 2 1 2 1 2
: : = g g R R
(C)
1 2 1 2 2 1
: : = g g R R (D)
1 2 1 1 2 2
: : = g g R R
157
35. At what height over the earths pole, the free fall acceleration decreases by one percent
= .................. km (Re = 6400 km).
(A) 32 (B) 80 (C) 1.253 (D) 64
36. Weight of a body is maximum at
(A) moon (B) poles of earth (C) Equator of earth (D) Center of earth
37. At what distance from the center of earth, the value of aceeleration due to gravity g will be
half that of the surfaces (R = Radius of earth)
(A) 2R (B) R (C) 1.414 R (D) o.414 R
38. The acceleration due to gravity near the surface of a planet of radius R and density d is proportional
to
(A) d/R
2
(B) dR
2
(C) dR (D) d/R
39. The acceleration due to gravity is g at a point distance r from the center of earth R. if r < R
then
(A) g o r (B) g o r
2
(c) g o r
-2
(D) g o r
-1
40. The density of a newly discoverd planet is twice that of earth. The acceleration due to gravity
at the surface of the planet is equal to that at the surface of earth. If the radius of the earth
is R, the radius of planet would be .....................
(A) 2R (B) 4R (C) 1/4 R (D) .......
41. Density of the earth is doubled keeping its radius constant then acceleration, due to gravity will
be .................. ms
-2
(g = 9.8 ms
2
)
(A) 19.6 (B) 9.8 (C) 4.9 (D) 2.45
42. Weight of body of mass m decreases by 1% when it is raised to height h above the earths
surface. If the body is taken to a depth h in a mine. change in its weight is
(A) 2% decreases (B) 0.5% decreases
(C) 1% increases (D) 0.5% increases
43. If density of earth increased 4 times and its radius becomes half of then out weight will be...
(A) Four times it present calue (B) doubled
(C) Remain same (D) halved
44. A man can jump to a height of 1.5 m on a planet A what is the height ne may be able to
jump on another planet whose density and radius are respectively one- quater and one- third
that of planet A
(A) 1.5 m (B) 15 m (C) 18 m (D) 28 m
45. If the value of g acceleration due to gravity, at earth surface fis 10ms
2
. its value in ms
2
at the center of earth, which is assumed to be a sphere of Radius R meter and uniform density is
(A) 5 (B) 10/R (C) 10/2R (D) zero
46. A research satellite of mass 200 kg. circles the earth in an orbit of avrage radius 3R/2 where
R is radius of earth. Assuming the gravitational pull 10 N, the pull on the satellite will be =..........N
(A) 880 (B) 889 (C) 890 (D) 892
47. Acceleration due to gravity on moon is 1/6 of the acceleration due to gravity on earth. If the
ratio of densities of earth
e
and moon
m
is
5
3
=
e
m

then radius of moon Rm in terms


of Re will be ...............
(A)
5
Re
18
(B)
1
Re
6
(C)
3
Re
16
(D)
1
Re
2 3
158
48. The acceleration of a body due to the attraction of the earth (radius R) at a distance 2R from
the surface of the earth is = ...............
(g = acceleration due to gravity at the surface of earth)
(A) g/9 (B) g/3 (C) g/4 (D) 9
49. The height at which the weight of a body becomes 1/16 th its weight on the surface of
(radius R) is
(A) 3R (B) 4R (C) 5R (D) 15R
50. A spherical planet has a mass Mp and diameter Dp A particle of mass m falling freely near
the surface of this planet will experience an acceleration due to gravity, equal to
(A) 2
4 GMp
Dp
(B) 2
GMpm
Dp
(C) 2
GMp
Dp
(D) 2
4GMpm
Dp
51. Assuming the earth to have a constant density, point out which of following curves show the
variation acceleration due to gravity from center of earth to points far away from the surface
of earth ...............
(D) None of these
Gravitational potential, Energy and Escape Velocity
52. In a gravitational field, at a point where the gravitational potential is zero
(A) The gravitational field is necessarily zero
(B) The gravitational field is not necessarily zero
(C) Nothing can be said definetely, about the gravitational field
(D) None of these
53. The mass of the earth is 6.00

10
24
kg and that of the moon is 7.40

10
22
kg. The constant
of gravitation G = 6.67

10
-11
Nm
2
kg
2
. The potential energy of the system is -7.79

10
28
Joules. the mean distance between the earth and moon is = ............. meter.
(A) 3.80

10
8
(B) 3.37

10
8
(C) 7.60

10
8
(D) 1.90

10
2
54. The masses and radii of earth and moon are M
1
,

R
1
and M
2
,

R
2
respectively. Their centres
are d distance of apart. The minimum velocity with which a particle of mass m should be projected
from a point midway between their centres so that it esacapes to infinity is...............
(A)
1 2
2 ( )
G
M M
d
+
(B)
1 2
2
2 ( )
G
M M
d
+
(C)
1 2
2 ( )
Gm
M M
d
+
(D)
1 2
1 2
( )
2
( )
Gm M M
d R R
+
+
159
55. A rocket is launched with velocity 10 kms
-1
. If radius of earth is R then maximum height attained
by it will be = ..............
(A) 2R (B) 3R (C) 4R (D) 5R
56. What is the intensity of gravitational field at the center of spherical shell
(A)
2
Gm
r
(B) g (C) zero (D) None of these
57. Escape velocity of a body of 1 kg. on a planet is 100 ms
-1
. Gravitational potential energy of
the body at the planet is =............... J
(A) -5000 (B) -1000 (C) -2400 (D) 5000
58. A body of mass m kg starts falling from a point 2R above the earths surface. Its K.E. when
it has fallen to a point R above the Earths surface = .....................
[R - Radius of Earth, M-mass of Earth G-Gravitational constant]
(A)
1
2
GMm
R
(B)
1
6
GMm
R
(C)
2
3
GMm
R
(D)
1
3
GMm
R
59. The Gravitational P.E. of a body of mas m at the earths surface is -mgRe. Its gravitational
potential energy at a height Re from the earths surface will be = ............ here
(Re is the radius of the earth)
(A) 2 mgRe (B) 2 mgRe (C)
1
Re
2
mg
(D)
1
Re
2
mg
60. A body is projected vertically upwards from the surface of a planet of radius R with a velocity
equal to half the escape velocity for that planet. The maximum height attained by the body is
..........
(A) R/3 (B) R/2 (C) R/4 (D) R/5
61. Energy required to move a body of mass m from an orbit of radius 2R to 3R is ................
(A)
2
GMm

12R
(B)
2
3
G m
R
M
(C)
8
G m
R
M
(D)
6
G m
R
M
62. Radius of orbit of satellite of earth is R. Its K.E. is proportional to
(A)
1
R
(B)
1
R
(C) R (D)
3/2
1
R
63. A particle falls towards earth from infinity. Its velocity reaching the earth would be.............
(A) infinity (B) 2gR (C) 2 gR (D) zero
64. The escape velocity of a sphere of mass m from earth having mass M and Radius R is given by
(A)
2GM
R
(B)
2
GM
R
(C)
2GMm
R
(D)
GM
R
65. The escape velocity for a rocket from earth is 11.2 kms
1
value on a planet where acceleration
due to gravity is double that on earth and diameter of the planet is twice that of earth will be
= ........... kms
1
(A) 11.2 (B) 22.4 (C) 5.6 (D) 53.6
66. The escape velocity from the earth is about 11 kms
1
. The escape velocity from a planet having
twice the radius and the same mean density as the earth is =............ kms
1
.
(A) 22 (B) 11 (C) 5.5 (C)15.5
160
67. If g is the acceleration due to gravity at the earths surface and r is the radius of the earth,
the escape velocity for the body to escape out of earths gravitational field is.................
(A) gr (B) 2gr (C) g/r (D) r/g
68. The escape velocity of a projectile from the earth is approximately
(A) 11.2 kms
1
(B) 112 kms
1
(C) 11.2 ms
1
(D) 1120 kms
1
69. The escape velocity of a particle of mass m varies as...................
(A) m
2
(B) m (C) m
0
(D) m
-1
70. The escape velocity of an object from the earth depends upon the mass of earth (M), its mean
density (), its radius (R) and gravitational constant (G), thus the formula for escape veloctiy is
(A)
8
3
= R G

(B)
8
3
= M GR

(C) 2GMR = (D)


2
2
=
GM
R

71. Two small and heavy sphere, each of mass M, are placed distance r apart on a horizontal surface
the gravitational potential at a mid point on the line joining the center of spheres is
(A) zero (B)
GM
r

(C)
2GM
r

(D)
4

GM
r
72. The escape velocity of a body from earths surface is Ve. The escape velocity of the same
body from a height equal to 7 R from earths surface will be
(A)
2
Ve
(B)
2
Ve
(C)
2 2
Ve
(D)
4
Ve
73. An artificial satellite is revolving round the earth in a circular orbit. its velocity is half the escape
velocity. Its height from the earth surface is = ............... km
(A) 6400 (B) 12800 (C) 3200 (D) 1600
74. The escape velocity of a planet having mass 6 times and radius 2 times as that of earth is..........
(A)
3 Ve
(B) 3 Ve (C)
2 Ve
(D) 2Ve
75. There are two planets, the ratio of radius of two planets is k but the acceleration due to gravity
of both planets are g what will be the ratio of their escape velocity.
(A)
( )
1/ 2
kg (B)
( )
1/ 2
kg (C)
( )
2
kg (D)
( )
2
kg
76. The escape velocity of a body on the surface of the earth is 11.2 km/sec. If the mass of the
earth is increases to twice its present value and the radius of the earth becomes half, the escape
velocity becomes = ............... kms
1
(A) 5.6 (B) 11.2 (C) 22.4 (D) 494.8
77. Given mass of the moon is
1
81
of the mass of the earth and corresponding radius is
1
4
of
the earth, If escape velocity on the earth surface is 11.2 kms
1
the value of same on the surface
of moon is = ....... kms
1
.
(A) 0.14 (B) 0.5 (C) 12.5 (D) 5
161
78. 3 particle each of mass m are kept at vertices of an equilateral triangle of side L. The gravitational
field at center due to these particies is ................
(A) zero (B)
2
3GM
L
(C)
2
9Gm
L
(D) 2
12
3
Gm
L
79. Escape velocity on the surface of earth is 11.2 kms
1
Escape velocity from a planet whose masses
the same as that of earth and radius 1/4 that of earth is = ....... kms
1
(A) 2.8 (B) 15.6 (C) 22.4 (D) 44.8
80. The velocity with which a projectile must be fired so that it escapes earths gravitational does
not depend on ..................
(A) mass of earth
(B) Mass of the projectile
(C) Radius of the projectiless orbit
(D) Gravitational constant
81. The escape velocity for a body projected vertically upwards from the surface of earth is 11
kms
-1
. If the body is projected at an angle of 45
0
with the vertical, the escape velocity will
be ............kms
-1.
(A)
11
2
(B)
11 2
(C) 22 (D) 11
82. The acceleration due to gravity on a planet is same as that on earth and its radius is four times
that of earth. What will be the value of escape velocity on that planet if it is Ve on the earth
(A) Ve (B) 2 Ve (C) 4 Ve (D) Ve/2
83. A particle of mass 10g is kept on the surface of a uniform sphere of mass 100 kg and radiius
10 cm. Find the work to be done aginst the gravitational force between them to take the particle
is away from the sphere (G = 6.67

10
-11
SI unit)
(A) 6.67

10
-9
J (B) 6.67

10
-10
J (C) 13.34

10
-10
J (D) 3.33

10
-10
J
84. A particle of mass M is situated at the center of a spherical shell of same mass and radius
a the magnitude of gravitational potential at a point situated at a / 2 distance from the center
will be
(A)
4GM
a
(B)
GM
a
(C)
2GM
a
(D)
3GM
a
85. The mass and radius of the sun are 1.99 x 10
30
kg and R = 6.96 x 10
8
m. The escape
velocity of rocket from the sun is =.......... km/sec
(A) 11.2 (B) 12.38 (C) 59.5 (D) 618
86. The mass of a space ship is 1000 kg. It is to be lauched from earths surface out into free
space the value of g and R (radius of earth) are 10ms
-2
and 6400 km respectively the required
energy for this work will be = ................. J
(A) 6.4

10
11
(B) 6.4

10
8
(C) 6.4

10
9
(D) 6.4

10
10
162
87. The diagram showing th variation of gravitational potential of earth with distance from the center
of earth is
(A) (B)
(C) (D)
88. A shpere of mass M and Radius R2 has a concentric cavity of Radius R
1
as shown in figure.
The force F exerted by the shpere on a particle of mass m located at a distance r from the
center of shhere varies as ) r (O s s
(A) (B)
(C) (D)
163
89. Which one of the following graphs represents correctly the variation of the gravitational field
with the distance (r) from the center of spherical shell of mass M and radius a
(A) (B)
(C) (D)
90. Which of the following graphs represents the motion of a planet moving about the sun.
(A) (B)
(C) (D)
91. The curves for P.E. (U) K.E. (Ek) of two particle system ae shown in figure . At what points
systemwill be bound.
(A) Only at point D
(B) Only at point A
(C) At point D and A
(D) At points A, B and C
E
k
164
92. The correct graph representing the variation of total energy (E) kinetic energy (K) and potential
energy (U) of a satellite with its distance from the centre of earth is...........
(A) (B)
(C) (D)
93. A shell of mass M and radius R has a point mass m placed at a distance r from its center.
The gravitational potential energy U(r) v will be
(A) (B)
(C) (D)
Motion of satellite
94. If Ve and Vo are represent the escape velocity and orbital velocity of satelllite correspoinding
to a circular orbit of radius r, then
(A) Ve = Vo (B)
2 Vo Ve =
(C)
/ 2 Ve Vo =
(D)ve and vo are not related
E
n
e
r
g
y
E
n
e
r
g
y
E
n
e
r
g
y
E
n
e
r
g
y
165
95. If r represents the radius of the orbit of a saltellite of mass m moving around a planet of mas
M, the velocity of the satellite is given by
(A)
2
gM
r
= (B)
2
GMm
r
= (C)
GM
r
= (D)
2
GM
r
=
96. Two satellites of mass m
1
and m
2
(m
1
> m
2
) are revolving round the earth in cirular orbits
of r
1
and r
2
( r
1
> r
2
) respectively. Which of the following statement is true regarding their
speeds V
1
and

V
2
(A) V
1
= V
2
(B) V
1
< V
2
(C) V
1
> V
2
(D)

1 2
1 2
V V
r r
=
97. A satellite which is geostationary in a particular orbit is taken to another orbit. Its distance from
the centere of earth in new orbit is two times of the earlier orbit. The time period in second
orbit is .........hours.
(a) 4.8 (B)
48 2
(C) 24 (D)
24 2
98. As astronaut orbiting the earth in a circular orbit 120 km above the surface of earth,gently drops
a spoon out of space-ship. The spoon will
(A) Fall vertically down to the earth
(B) move towards the moon
(C) Will move along with sapace - ship
(D) Will move in an irregualr way then fall down to earth
99. The period of a satellite in cirular orbit around a planet is independent of
(A) the mass of the planet (B) the radius of the planet
(C) mass of the satellite (D) all the three parameters (A), (B) and (C)
100. Two satellites A and B go round a planet p in circular orbits having radii 4 R and R respectively
if the speed of the satellite A is 3V, the speed if satellite B will be
(A) 12V (B) 6V (C) 4/3 V (D) 3/2 V
101 A small satellite is revolveing near earths surface. Its orbital velocity will be nearly
= .......... kms
1.
(A) 8 (B) 4 (C) 6 (D) 11.2
102 A satellite revolves around the earth in an elliptical orbit. Its speed
(A) is the same at all points in the orbit
(B) is greatest when it is closest to the earth
(C) is greatest when it is farthest to the earth
(D) goes on increasing or decresing continuously
depending upon the mass of the satellite
103. If the height of a satellite from the earth is negligible in comparison of the radius of the earth
R, the orbital velocity of the stellite is ............
(A) gR (B) gR/2 (C) / g R (D) gR
166
104 A satellite is moving around the earth with speed v in a circular orbit of radius r. If the orbit
radius is decreasd by 1% its speed will
(A) increase by 1% (B) increase by 0.5%
(C) decrrease by 1% (C) Decrrease by 0.5%
105. orbital velocity of an artifiicial satellite does not depend upon
(A) mass of earth (B) mass of satellite
(C) radius of earth (D) acceleration due to gravity
106. orbital velocity of eaths satellite near the surface is 7 kms
1
. when the radius of orbit is 4 times
that of earths radius, then orbital velocity in that orbit is =........kms
-1
(A) 3.5 (B) 17 (C) 14 (D) 35
107. Two identical satellites are at R and 7R away from each surface, the wrong statement is
(R - Radius of earth)
(A) ratio of total energy will be 4
(B) ratio of kinetic energes will be 4
(C) ratio of potential energies will be 4
(D) ratio of total energy will be 4 but ratio of potential and kinetic energies will be 2
108. Which one of follwoing sttements regarding artificial satellite of earth is incorrect
(A) The orbital velocity depends on the mass of the satellite
(B) A minimum velocity of 8kms
-1
is required by a satellite to orbit quite close to the earth.
(C) The period of revolution is large if the radius of its orbit is large
(D) The height of geostationary satellite is about 36000 km from earth
109. The weight of an astronuat, in an artificial satellite revolving around the earth is
(A) zero
(B) Equal to that on the earth
(C) more than that on earth
(D) less than that on the earth
110. The distance of a geo-stationary satellite from the center of the earth (Radius R= 6400km)
is nearest to
(A) 5R (B) 7R (C) 10R (D) 18R
111. A geo-stationary satellite is orbiting the earth of a height of 6R above the surface of earth, R
being the radius of earth. The time period of another satellite at a height of 2.5 R from the
surface fo earth is = ...............hr
(A) 6 (B)
6 2
(C) 10 (D)
6/ 2
112. If the gravitational force between two objects were proportional to
1
R
(and not as
2
1
R
)where
R is separation between them, then a particle in circular orbit under such a force would have
its orbital speed v proportional to
(A)
2
1
R
(B) R
0
(C) R
1
(D)
1
R
113. A satellite moves around the earth in a circular orbit of radius r with speed v, If mass of the
satellite is M , its total energy is
(A)
2
1
2
MV
(B)
2
1
2
MV
(C)
2
3
2
MV
(D) MV
2
167
114. A satellite with K.E. E
k
is revolving round the earth in a circular orbit. How much more K.E.
should be given to it so that it may just escape into outerspace ?
(A) E
k
(B) 2 E
k
(C)
1
E
2
k
(D) 3 E
k
115. Potential energy of a satellite having mass m and rotating at a height of 6.4 x 10
6
m from
the surface of earth
(A) -0.5 mg Re (B) -mg Re (C) -2mgRe (D) 4 mgRe
116. When a satellite going round the earth in a circular obrit of radius r and speed v loses some
of its energy, then r and v changes as
(A) r and v bothe will increase
(B) r and v both will decease
(C) r will decrease and v will increase
(D) r will increase and v will decrease
117. The time period of a satellite of earth is 5 hours If the sepration between the earth and the
satellite is increased to four times the previous value, the new time period will become ...... hours
(A) 10 (B) 120 (C) 40 (D) 80
118. A person sitting in a chair in a satellite feels weightless because
(A) the earth does not attract the objects in a satellite
(B) the normal force by the chair on the person balances the earths attraction
(C) the normal force is zero
(D) the person in satellite is not accelerated
119. Two satellites A and B go round a planet in cirular orbits having radii 4R and R respectively
If the speed of satellite A is 3v, then speed of satellite B is
(A) 3v/2 (B) 4v/2 (C) 6v (D) 12v
120. A satellite moves in a circle around the earth, the radius of this circlr is equal to one half of
the radius of the moons orbit the satellite completes one revolution in .........lunar month
(A) 1/2 (B) 2/3 (C) 2
3/2
(D) 12
3/2
121. The additional K.E. to be provided to a satellite of mass m revolving around a planet of mass
M, to transfer it from a circular orbit of radius R
1
to another radius R
2
(R
2
> R
1
) is
(A)
2 2
1 2
1 1
| |

|
\ .
GMm
R R
(B)
1 2
1 1 | |

|
\ .
GMm
R R
(C)
1 2
1 1
2
| |

|
\ .
GMm
R R
(D)
1 2
1 1 1
2
| |

|
\ .
GMm
R R
122. Rockets are launched in eastward direction to take advantage of
(A) the clear sky on eastem side
(B) the thiner atmosphere on this side
(C) earths rotation
(D) earths tilt
168
123. A satellite of mass m is orbiting close to the surface of the earth (Radius R = 6400 km) has
a K.E. K. The corresponding K.E. of satellite to escape from the earths gravitational field is
(A) k (B) 2 k (C) mg R (D) m k
124. A planet moving along an elliptical orbit is closest to the sun at a distance r
1
and farthest away
at a distance of r
2
. If v
1
and v
2
are the liner velocities at these points respectively, then the
ratio
1
2
v
v
is ............
(A)
1
2
r
r
(B)
2
1
2
r
r
| |
|
\ .
(C)
2
1
r
r
(D)
2
1
2
r
r
| |
|
\ .
125. The time period T of the moon of planet Mars(Mm) is related to its orbital radius R as
(G = Gravitational constant)
(A)
2
2
4
=
R
T
GMm

(B)
2 2
2
4 GR
T
Mm
=
(C)
2
2
2
=
R G
T
Mm

(d) T
2
= 4t Mm GR
2
126. A geostationary satellite is orbiting the earth at a height of 5 R above that of surface of the
earth. R being the radius of the earth. The time period of another satellite in hours at a height
of 2R from the surface of earth is ............ hr
(A) 5 (B) 10 (C) 6 2 (D) 6/ 2
127. The figure shows ellipticall orbit of a planet m about the sun s. the shaded area SCD is twice
the saded area SAB. If t
1
is the time for the planet to move from C and D and t
2
is the time
to move from A to B then
(A) t
1
> t
2
(B) t
1
> 4t
2
(C) t
1
= 2t
2
(D) t
1
= t
2
128. The period of a satellite in a circular orbit of radius R is T. the period of another satellite
in a circular orbit of radius 4R is
(A) 4T (B) T/4 (C) 8T (D) T/8
129. If the earth is at one- fourth of its present distance from the sun the duration of year will be
(A) half the pesent Year
(B) one-eight the present year
(C) one-fourth the present year
(D) one-sixth the present year
130. The orbital speed of jupiter is
(A) greater than the orbital speed of earth (B) less than the orbital speed of earth
(C) equal to the orbital speed of earth (D) zero
169
131. Keplers second law regarding constancy of aerial velocity of a palnet is consequence of the
law of conservation of
(A) energy (B) angular Momentum
(C) linear momentum (D) None of these
132. The largest and shortest distance of the earth from the sun are r
1
and r
2
its distance from
the sun when it is at the perpendicular to the major axis of the orbit drawn from the sun
(A)
1 2
4
r r +
(B)
1 2
1 2
r r
r r +
(C)
1 2
1 2
2r r
r r +
(D)
1 2
3
r r +
133. According to keplar, the period of revolution of a planet (T) and its mean distance from the
sun (r) are related by the equation
(A)
3 3
tan T r cons t = (B)
2 3
tan T r cons t

=
(C)
3
tan Tr cons t = (D)
2
tan T r cons t =
134. A satellite of mass m is circulating around the earth with constant angular velocity. If radius of
the orbit is Ro and mass of earth M , the angular momentum about the center of earth is
(A)
m GMRo
(B)
M GMRo
(C)
GM
m
Ro
(D)
GM
M
Ro
135. The earth E moves in an elliptical orbit with the sun s at one of the foci as shown in figure.
Its speed of motion will be maximum at a point ..........
(A) C (B) A (C) B (D) D
136. The period of revolution of planet A around the sun is 8 times that of B. The distance of A
from the sun is how many times greater than that of B from the sun.
(A) 2 (B) 3 (C) 4 (D) 5
137. The earth revolves round the sun in one year. If distace between then becomes double the new
period will be .......... years.
(A) 0.5 (B) 2 2 (C) 4 (D) 8
138. The maximum and minimum distance of a comet from the sun are 8 x 10
12
m and 1.6 x 10
12
m. If its velocity when nearest to the sun is 60 ms
-1
, What will be its velocity in ms
-1
when
it is farthest ?
(A) 6 (B) 12 (C) 60 (D) 112
139 The period of moons rotation around th earth is nearly 29 days. If moons mass were 2 fold
its present value and all other things remained unchanged the period of moonss rotation would
be nearly .....days
(A)
29 2
(B) 29/ 2 (C) 29 2 (D) 29
170
140. If the velocity of planet is given by
a b c
G M R = then
(A) a =1/3 b = 1/3 c = -1/3
(B) a =1/2 b = 1/2 c = -1/2
(C) a =1/2 b = -1/2 c = 1/2
(D) a =1/2 b = -1/2 c = -1/2
141. The radius of orbit of a planet is two times that of earth. The time period of planet is.........years.
(A) 4.2 (B) 2.8 (C) 5.6 (D) 8.4
142. If r denotes the distance between the sun and the earth, then the angular momentum of the
earth around the sun is proportional to
(A) r
3/2
(B) r (C) r
1/1
(D) r
2
143. What does not change in the field of central force
(A) potential energy (B) Kinetic energy
(C) linear momentum (D) Angular momentum
144. A thin uniform annular disc (See figure) of mass M has outer
radius 4R and inner radius 3R. The work required to take a
unit mass from point P on its axix to infinity is..............
(A)
2
(4 2 5)
7
GM
R

(B)
2
(4 2 5)
7
GM
R

(C)
4
GM
R
(D)
1) 2 (
5R
2GM

145. Suppose the gravitational force varies inversely as the n


th
power of distance the time period
of planet in circular orbit of radius R around the sun will be proportional to
(A)
1
2
n
R
+ | |
|
\ .
(B)
1
2
| |
|
\ .
n
R (C) R
n
(D)
2
2
| |
|
\ .
n
R
146. If the radius of the earth were to shrink by 1% its mass remaing the same, the accelration due
to gravity on the earths surface would
(A) decrease by 2% (B) remain Unchanged
(C) increase by 2% (D) increases by 1%
147. A body of mass m is taken from earth surface to the height h equal to radius of earth, the
increase in potential energy will be
(A) mg R (B)
1
2
mgR
(C) 2 mgR (D)
1
4
mgR
148. An artificial satellite moving in a circular orbit around earth has a total (kinetic + potential lenergy)
Eo, its potential energy is
(A) -Eo (B) 1.5 Eo (C) 2 Eo (D) Eo
171
149. Two bodies of masses m
1

and m
2
are initially at rest at infinite distance apart. They are then
allowed to move towards each other under mutual agravitational attraction Their relative velocity
of apporach at sepration distance r between them is
(A)
1
2
1 2
2 ( )

(
(

G m m
r
(B)
1
2
1 2
2 ( ) + (
(

G m m
r
(C)
1
2
1 2
2
(
(

r
Gm m
(D)
1
2
1 2
2 (
(

r
Gm m
150. A geostationary satellite orbits around the earth in a circular orbit of radus 3600 km the time
period of a satellite orbiting a few hundred kilometers above the earths surface (Rearth = 6400
km) will approximately be = ...... h
(A) 1/2 (B) 1 (C) 2 (D) 4
KEY NOTE
1 . C 25. B 49. C 73. A 97. B 121 D 145. A
2 . A 26. A 50. A 74. A 98. C 122. C 146. C
3. B 27. C 51. C 75. A 99. C 123 B 147. B
4. A 28. A 52. A 76. C 100. B 124. C 148. C
5. B 29. C 53. A 77. C 101. A 125. A 149. B
6. C 30. C 54. A 78. A 102. B 126. C 150. C
7. A 31. A 55. C 79. C 103. D 127. C
8. D 32. C 56. C 80. B 104. B 128. C
9. C 33. B 57. A 81. D 105. B 129. B
10. D 34. D 58. B 82. B 106. A 130. B
11. B 35. A 59. D 83. B 107. D 131. B
12. D 26. B 60. A 84. D 108. A 132. C
13. A 27. C 61. D 85. D 109. A 133. B
14. A 38. C 62. A 86. D 110. B 134. A
15. A 39. A 63. B 87. C 111. D 135. B
16. C 40. D 64. A 88. B 112. B 136. C
17. C 41. A 65.B 89. D 113. A 137. B
18. C 42. B 66. A 90. C 114. A 138. A
19. D 43. B 67. B 91. D 115. A 139. D
20. C 44. C 68. C 92. C 116. C 140. B
21. B 45. D 69. C 93. C 117. D 141. C
22. A 46. B 70. A 94. B 118. C 142. C
23. B 47. A 71. D 95. D 119. C 143. D
24. B 48. A 72. C 96. B 120. C 144.A
1 . C 25. B 49. C 73. A 97. B 121 D 145. A
2 . A 26. A 50. A 74. A 98. C 122. C 146. C
3. B 27. C 51. C 75. A 99. C 123 B 147. B
4. A 28. A 52. A 76. C 100. B 124. C 148. C
5. B 29. C 53. A 77. C 101. A 125. A 149. B
6. C 30. C 54. A 78. A 102. B 126. C 150. C
7. A 31. A 55. C 79. C 103. D 127. C
8. D 32. C 56. C 80. B 104. B 128. C
9. C 33. B 57. A 81. D 105. B 129. B
10. D 34. D 58. B 82. B 106. A 130. B
11. B 35. A 59. D 83. B 107. D 131. B
12. D 26. B 60. A 84. D 108. A 132. C
13. A 27. C 61. D 85. D 109. A 133. B
14. A 38. C 62. A 86. D 110. B 134. A
15. A 39. A 63. B 87. C 111. D 135. B
16. C 40. D 64. A 88. B 112. B 136. C
17. C 41. A 65.B 89. D 113. A 137. B
18. C 42. B 66. A 90. C 114. A 138. A
19. D 43. B 67. B 91. D 115. A 139. D
20. C 44. C 68. C 92. C 116. C 140. B
21. B 45. D 69. C 93. C 117. D 141. C
22. A 46. B 70. A 94. B 118. C 142. C
23. B 47. A 71. D 95. D 119. C 143. D
24. B 48. A 72. C 96. B 120. C 144.A
172
HINT
(1) (c)
3. 2
2 2 4
2 2
4
( )
( )( ) 4
3
(2 ) 4 3
= = =
G R
G m m
F R
R R


(4) (A) Gravitational force does not depend upon the medium
(5) (B) Due to inertia to direction
(6) (D) the force exerted by sun on the earth
F = me
2
R

24 7 2 11
(6 10 ) (2 10 ) (1.5 10 )

=

21
36 10 = N
(7)
2
( 1) ( 1) F xm x m m x x =
for maximum force
0
dF
dx
=
2 2
2 0 1/ 2 = = =
dF
m m x x
dx
(9) (C) cenripetal force provided by the gravitational force of attraction between two particites
2
2
( )( )
(2 )
=
m G m m
r r

1
2
Gm
r
=
(10) (D)
For will be zero at the point of zero intensity
1
1 2
81 9
10 81
= = =
+ +
m m
x D D
m m m m
11. B
2
Re
=
e
GM m
mg
where Me and Re is the mass and radius of the earth respectivel
2
Re =
e
g
M
G
12. (D) Here F
OA
= F
OB
= F
OC
=
2
( ) (2 ) G m m
r
OA OB OC
F F F F = + +

(D - X)
173
=
= 0
14. (A) let a point mass C is placed at a distance of x m from the point mass A as show in figure
4
3
=
a
b
m
here
m
2
( )( )
.........( ) =
G m mA
FAC i
x
2
( )( )
.........( )
(1 )
=

G m mB
FBC ii
x
According to given problem
1
4
FAC FBC =
with the help of equ (i) and (ii)
2 2
( )( ) 1 ( )( )
3 (1 )
= =

G m mA G m mB
x x
2 2 2
2 2 2
4
4
3(1 ) 3 3(1 ) (1 )
= = =

A
B
m x x x
m x x x
2 2 2
1
x
x x
x
= = =

2
3 2
3
x x m = = =
17. (c)
3
2
4
3
= =
GM
g and M R
R

3
2
4 3
.
3 4
= =
G g
g R
R RG

18. (c) for condition of weight lessness at equator


3
10 1
sec
6400 10 800
g
R
rad
=
= =

19. (D) Time peripd of simple pendylym 2 ' = T l g


In artificial satellite g = 0 = T
mA m
mB
(1x)
C
x
174
20. (C) 2 2 2
4
( 2)
GM GMo GMo
g
R Do Do
= = =
21. (B) we know that
2
GM
g
R
=
on the planet g
p
2
7
4 7
4
= =
GM
g
R
Hence the weight on the planet = 700
4
7
= 400 gm et
22.
2 2
2
' 6400
' 960.400
( ) 6400 64

| | | |
= = =
| |
+ +
\ . \ .
g R
g ms
g R h
23.
1 2 2
= g g R Cos
Rotation of the earth results in the decreased weight apparently.this decrease in weight is not
felt at the poles as the angle of latitude is 90
24. (B) using 2
GM
g
R
=
we get g
m
= g/5
25. (B)
1
d ( 1)
g g (1 )
R
n
d R
n

= =
26. (A)
4
g GR and g' 4/3GR'
3
= =
g' R'
0.2 g' 0.2g
g R
= = =
27. (C) mass does not vary from place to place
28. (a)
2 2
2
4
'
9
| | | |
= = =
| |
+ +
\ . \ .
R
R R
g g g g
R h R
4
W' 4/9 w (72) 32 N
9
= = =
29. (c)
R h solving by
h R
R
g g = = |
.
|

\
|
+
= 4 / 9 '
2
30. (c)
2
2 2
p e
Mp Re 1 9.8
g g 9.8 (2) 0.49ms
Me Rp 80 20

| |
| | | |
= = = =
| | |
\ . \ .
\ .
31. (A)
2
d 100
g' g 1 9.8 1 9.66 ms
R 6400

| | | |
= = =
| |
\ . \ .
175
32. (C)
2
R
GM
g =
and k =
2
L
2I
If mass of the earth and its angular momentum ramains constant then
2
1
g
R

and
2
1
k
R

i.e. if radius of earth decreases by 2% then g and k both increase by 4%


33. (B) Weight on surface of earth, mg = 500N and weight below the surface of earth at
d =
R
2
, mg
1
= ( )
d mg
mg 1 mg 1 1/2 250N
R 2
| |
= = =
|
\ .
34. (D)
1
1 1
2 2 2
g
4 R
g GR
3 g R
= =

35. (A)
2
r
GM
g

g
1
r or
r
1
g
2

If g decreases by one percent then r should be increase by 1/2 % i.e.


1
6400 32
2 100
R km = =

37. (C)
R h R
h R
R
h R
R
g g 2
2
1
'
2
= +
+
= |
.
|

\
|
+
=

( )
h 2 1 R 0.414 R = =
Hence distance form center = R + 0.414R
= 1.414R
38. (C)
4
g GR g d R ( d given in the problem)
3
= =
39. (A) insided the earth

3
4
g' =
Gr r g '
40. (D)
4
g GR
3
=

1
(1)
Re 2
| |
| |
| |
= =
|
| |
|
\ .
\ .
\ .
p
e
e p
g
Rp
g


Re
2
2
R
Rp = =
41. (A) g o
42. (B) For height
100
4
24
100%
g
g
=
%
For depth
0.5 1/2
R
h
R
d
100%
g
g
= = = =
43. (B) g o R
176
44. (C)
2
B A
A B
V 1 H g
H H
2g g H g
= =
Now
A
B
g
g
12
=
as g o R
B A
B A
A B
H g
12 H 12 H 12 1.5 18 m
H g
= = = = =
46. (B)
2
2
R R
g' g g 4/9.g(g 10 ms )
R
R h
3
2

| |
| |
|
= = = =
|
| +
\ .
\ .
47. (A)
e e
m m
g 4 Re
g GR g R .
3 g Rm

= =


Rm
Re
.
3
5
1
6
=

Re
18
5
Rm =
48. (A)
' / 9
2 2
| | | |
= = =
| |
+ +
\ . \ .
R R
g g g g
R R R R
49. (C) 2
) / 1 (
'
R h
g
g
+
=
2
) / 1 ( 16 R h
g g
+
=
16 1
2
= |
.
|

\
|
+
R
h
4 1 = +
R
h
3 =
R
h
R h 3 =
50. (A) Gravitational attraction fore on particle B
2
) 2 / (Dp
GMpm
Fg =
Acceleration of paritcle due to particle due to gravity 2
4
Dp
GMp
m
Fg
a = =
177
51. (C)
2
1
g r it(r r) and g (it r R)
r
< >
52. (A)
dx
dv
I =
53. (A)
r
GMm
U =
r
24 22 11
28
10 6 10 7 10 67 . 6
10 79 . 7

=

m 10 3.8 r
8
=
54. (A)
2 / 2 /
2 1
d
GM
d
GM
V + =
1 2
2GM
Now P.E mv (m m )
d
= = +
(m mass of particie)
K.E = P.E
2
1 2
1 2
( )
2
= +
GM
m m m
d

1 2
( )
2
G M M
d
+
=
55. (C) If the body is projected with velocity Ve) ( < then height up to where it rises
2 2
2
4 ( )
11.2
1
10
R R
h R approx
Ve
V
= = =
| |

|
\ .
57. (A)
2GM GMm
Ve P.E. U 5000 J
R R
= = =
R
GM
= 100
5000 =
R
GM
58. (B) P.E. = =
+
GMm GMm
U
r R h
U
initial
=
3
GMm
R
and U
final
=
2
GMm
R
loss of P.E. = gain in K.E =
2
GMm
R

3
GMm
R
=
6
GMm
R
178
59. (D)
2 1
mgh mgRe mgRe
U U U
Re
1 h/Re 2
1
Re
= = = =
+
+
2
mg Re
U ( mg Re)
2
= mgRe
2
1
U
2
=
60. (A) If body is projected with velocity Ve) ( < then height up to which it will rise
R/3
1 4
R
1
Ve/2
Ve
R
h ve/2 u but
2
=

|
.
|

\
|
= =
61. (D) change in potential energy in displacing a body from r
1
and r
2
is given by
1 2
1 1 1 1
2 3 6
GMm
U GMm GMm
r r R R R
(
(
A = = =
(
(


62. (A) K.E =
2
GMm
R R
1
K.E.
63. (B) this should be equal to escape velocity i.e. = 2gR
64. (A) A Escape velocity does not depend on the mass of the projectile
65 (B)
p
e
g
Vp Rp
. 2 2 2
Ve g Re
= = =
1
22.4Kms 11.2 2 2Ve Ve

= = =
66. (A)
8
3
2GM
Ve = = R G
R

Ve R if constant
since the planet is having double radius incomparision to earth therefore the escape velocity
becomes twice i.e. 22 kms
-1
69. (C) Because it does not depend on the mass of projetile
71. (D)
Gravitational potential of A at 0
r
GM
r
GM 2
2 /
= =
of B at 0
r
GM
r
GM 2
2 /
= =
Total potential at 0
4GM
r
=
179
72. (C)
r
1
Ve
where r is the position of body from the surface
1 2 1
2
2 1
V r R 7R V
2 2 V
V r R 2 2
+
= = = =
73. (A)
R
2GM
2
1
h R
GM
V =
+
=
h) 2(R 4R + =
h R 6400km = =
74. (A)
3
2
1
6
Rp
Re
Me
Mp
Ve
Vp
= = =
Ve 3 Ve =
75. (A)
1/2
(kg) gk
R2
R1
.
g2
g1
V2
V1
2gR = = = =
76. (C)
R
M
Ve
R
2GM
Ve =
If M becomes double and R becomes half then escape velocity; becomes two times
77. (C) on earth Ve =
1
2GM
11.2 kms
R

=
on moon
1
2GM 4 2 2GM 2 11.2
Vm . 2.5 kms
R R 9 R 9

= = = =
78. (A) Due to three particles net intensity at the center
A B C
I I I I 0 = + + =

because out of these three intensities ARE equal in magnitude
and between each other is 120
79. (C)
R
1
Ve if R becomes
4
1
then Ve will be 2 times
80. (D) Escape velocity does not depends upon the angle of projection
82. (B)
p
e
g
Vp Rp
2gR . 1 4 2
Ve g Re
u = = = =
Ve = 2Ve
83. (B) potential energy of system of two mass
J 10 6.67
10 10
10 10 100 10 6.67
R
GMm
U
10
2
3 11


= =
so, the amount of work done to take the particle up to infinte will be 6.67

10
-10
J
180
84. (D) Vp = Vsphere + Vpartical
a
GM
a
GM
a
GM 3
2 /
= + =
85. (D) Ve =
R
GM 2
=
11 30
1
2 6.67 10 1.99 10
618 kms
6.98 108

86. (D)
2
GMm GMm m
W 0- - gR mgR
R R R
| |
= = = =
|
\ .
3
10 6400 10 1000 =
9
10 64 =
J 10 6.4
10
=
87. (C)
GM GM GM
Vin Vsurface Vout
R R r
= = =
88. (B)
1
F 0 When 0 r R = < <
beccause intensity is zero inside Caviy
Fincrease When
2 1
R r R < <
2
R r when
R
1
F >
89. (D) Intensity will zero inside the spherical shell
I = upto r = a and Io
2
1
r
when r > a
90. (C) keplers law T
2
r
3
91. (D) System will be bound at points where total energy is negative. In the given curve at point
A, B, and C the P.E. is more than K.E.
92. (C)
2r
GMm
E and
2r
GMm
K
r
GMm
- U = = =
For satellite U, K, and E varies with r and also U and E remains negativw where K reamin
always positive
93. (C) Gravitational P.E. = m x gravitational potential U = mv so the graph of U will be same
as that of V for a spherical shell
94. (B)
0 0
Ve 2gR and V gR Ve 2 V = = =
96. (B)
2 1 2 1
V V then r r it
r
GM
< > =
181
97. (B)
3
2
T r if r becomes double then time period will become (2)
3/2
times so new time period
will be 24 x 2 2 hr i.e. = 48 2
98. (C) The velocity of the spoon will be equal to the orbital velocity when dropped out of the
space ship
100. (B)
A B
B A
GM V R R 1

R V R 4R 2
= = = =
A
B
B
V 3V
V 6V
V VA
= =
104. (B)
r
1

% increase in speed =
1
%
2
decrease in radius
=
1
2
(1%) = 0.5 %
105. (B)
r
GM
=
106. (A)
r
1
=
it orbital radius becomes 4 times then orbital velocity will becomes halt
107. (D) orbital rudius of satellites r
1
= R+R = 2R
r
2
= R+7R = 8R
2
2
1
1
r
GMm
U and
r
GMm
U = =
1 2
1 2
GMm GMm
K and K
2r 2r
= =
4
E
E
K
K
U
U
2
1
2
1
2
1
= = =
110. (B) 6R from the surface of earth and 7R from the center
111. (D) Distance of satellite from the center are 7R and 3.5R respectively
2
3/2
3/2
2 2
1 1
T r 3.5R
T 24 6 24 hr
T r 7R
| |
| |
= = =
| |
\ .
\ .
112. (B Gravitational force provides the required centripetal force for orbiting the satellite
R
K
R
m
2
= becuse
R
1
F
o
R
182
113. (A) Total energy = -K.E. =
2
1

2
m
114. (A) B.E. = -K.E.
And it this amount of energy(Ek) given to satellite it will escape into outer space
115. (A) P.E. =
GMm
r
=
Re +
GMm
h
=
2 e
GMm
R
=
2
Re
2 Re
g m
=
1
2
mg Re = 0.5 mg Re
116. (C) B. E. =
GMm
r
if B.E. decreases the r also decreases and V increases as
1
r

117. (D)
3/2
3/2 2
2 1 1 1
1
R
T T T (4) 8T 40 hr
R
| |
= = = =
|
\ .
118. (C)
4
2 = = =
B A
A B
V r R
V r R
120. time period of revolution of moon around the earth = 1 lunar month
3/2
3/2
2/3 e e
e
m m
T r 1
T 2
T r 2

| |
| |
= = =
| |
\ .
\ .
lunar month
121 (D)
2 1
2R
GMm
K.E.
2R
GMm
= +
|
|
.
|

\
|
=
2 1
1 1
2
.
R R
GMm
E K
122. Because Earth rotation from west to east direction
123. (B)
2
GMm
k
R
=
124. (C) v
1
r
1
=

v
2
r
2
(angular momentum is constant)
125. (A) Time period
2 / 3
2 2
GM
R
R
GMm
R
T

= =
GMm
R
T
3 2
2
4
=
126. (C)
2 3 3
1 1
2 3 3
2 2
T R (6R)
8
(3R) T R
= = =
2
2
24 24
72
8
T

= =
2
6 2 T =
183
127. (C)
A L
A t
t 2m
=
SCD 1
1 2
SAS 2
A t 2A
t 2t
A t A
= = =
128. (C)
3
3
2
2
1 1
2 2
T R R
T R 4 R
| |
| |
= =
| |
\ .
\ .

2 1
8 T T =
129. (B)
3 2
r T since
T
8
1
T
4
1
T
T
1
3
2
1
= |
.
|

\
|
=
|
|
.
|

\
|

130. (B) orbital radius of Jupiter > orbital radius of Earth


J e
e j
V r
V r
=
As r
j
< r
e
there fore Vj < Ve e
131. (B)
2
L dA
dt m
=
= constant
132. (C) The earth moves around the sun in elliptical path, so by using the properties of ellipse
e) (1 r
1
+ = a and r
2
= (1-e) r
2
, a =
1 2
2
+ r r
2 2
2 1
) 1 ( a e r r =
where a= semi major axis
b= semi minor axis
e= eccentircity
Now required distance = sem latysrectum = b
2
/9
2 1
2 1
2 1
2 1
2
2
r r
r 2r
)/2 r (r
r r
a
) e (1
a
+
=
+
=

=
133. (B) T
2
/ r
3
= constant T
2
r
-3
= constant
134. (A) Angular momentum = Mass

orbital velocity x Radius


0
0
R
R
GM
m =
0
m GMR =
135. (B) speed at the earth will be maximum when its distance from the sun is minimum because
m r = constant
136 (C) ( )
3 3
2 2
3
A A A 2
A B B
B B B
T r 4
8 r 4 8 4r
T r 4
| | | |
= = = =
| |
\ . \ .
137. (B)
years 2 2 T 2 2 (2)
r
r
T
T
2
2
3
2
3
1
2
1
2
= = =
|
|
.
|

\
|
=
184
138. (A) By conservation of angular momentum (mvr) = constant
Vmin X r
max
= Vmax X r
min
12
1
12
60 1.6 10 60
Vmin 12 ms
8 10 5


= = =

139. (D) Time period does not depends upon the mass of satellite
140. (B)
1 1 1
2 2 2
GM
G M R
R

= =
141. (B)
( ) yar 2.8 2 1
R
R
T T
2
3
2
3
1
2
1 2
= =
|
|
.
|

\
|
=
142. (C) Angular momentum of earth around the sun
L = M
E
Vo r
2
S
E E S
GM
M .r M .GM .r
r
= =
M
E
= mass of earth
M
S
= mass of Sun
r = Distance between sun and the earth
r L
143. (D) For central force toraqe is zero
dL
0 L Constant
dt
= = =
144. (A) Wext
F
U U

Gdm
0 (1)
x
(
=
(

}
2
2 2
M 2 r dr
G
7R
16R r
=

+
}
2
2 2
2 GM rdr
7R
16 R r
=
+
}
2 2
GM d2 GM
(2)
7R a 7R
= = }
| |
4R
3R
2 2
2
r 16R
R 7
GM 2
+ =
| | 5R R 2 4
7R
GM 2
=
| | 5 2 4
7R
GM 2
=
185
145. (A)
2
2 2 n 1
n 2 n
1 4 1
m R m R T R
R T R
+
| |

|
\ .

n+1
2
T R
146. (C)
2
R
GM
g =
is mass ramains constant then
2
1
R

147. (B) mgR
2
1
R
h
1
mgh
U =
+
=
148. (C) P.E. = 2. total energy = 2E
0
Because we know U =
GMM
r
and Eo =
2
GMM
r
149. (B) let velocities of these masses at r distance from each other be v
1
and v
2
respectively
By conservation of momentum
( )
1 1 2 2 1 1 2 2
m V m V 0 m V m V ____________ 1 = =
By conservation of energy
change in P.E. = change in K.E.
2
2 2
2
1 1
2 1
V m
2
1
V m
2
1
r
m Gm
+ =
( ) ii _______
r
m Gm 2
m
V m
m
V m
2 1
2
2
2 2
1
2
1 1
= +
on solving (i) and (ii)
( )
2 1
2
1
2
2
m m r
Gm
+
=
and
( )
2 1
2
2
m m r
2Gm

1
+
=
( )
1 2
1 2
2G m m
Vapp V V
r
+
= + =
150. (C)
3
3
2
2
2 2
2
1 1
T r 6400
T 24 2 hours
T r 3600
| |
| |
= = =
| |
\ .
\ .
186
Assertion - Reason Type Questions
Direction (Read the following uestions and choose)
(A) If both Assertion and Reason are true and the Reason is correct explanation of assertion
(B) If both Assertion and Reason are true, but reason is not correcte explanation of the Assertion
(C) If Assertion is true, but the Reason is false
(D) If Assertion is faluse, but the Reason is true
1. Assertion : The value of acc. due to gravity (g) does not depend upon mass of the body
Reason : This follows from
2
GM
g
R
=
, where M is mass of planet (earth) and R is radius of
planet (earth)
(a) A (b) B (c) C (d) D
2. Assertion : Unit of gravitational field intensity is N/kg or ms
-2
Reason : Gravitational field intensity
2
2
ms
kg
kg.m/sec
kg
N
mass
Force

= = =
(a) A (b) B (c) C (d) D
3. Assertion : The time period of a geostationary satellite is 24 hours
Reason : Such a satellite must have the same time period as the time taken by the earth to
complete one revolution about its axis
(a) A (b) B (c) C (d) D
4. Assertion : Even when orbit of a satellite is elliptical, its plane of rotaiion passes through the
center of earth
Reason : This is in accordance with the principle of conservation of angular momentum
(a) A (b) B (c) C (d) D
5. Assertion : The time Period of pendulum, on a satlellite orbiting the earth is infinity
Reason : Time period of a pendulum is inversely proportional to g
(a) A (b) B (c) C (d) D
6. Assertion : The escape velocity on the surface of a planet of the same mass but
1
4
times the
radius of earth is 5.6 kms
-1
Reason : The escape velocity Ve = 2gR
(a) A (b) B (c) C (d) D
7. Assertion : The comet does not obey keplers law of planetaty motion
Reason : The comet does not have ellitical orbit
(a) A (b) B (c) C (d) D
8. Assertion : The square of the period of revolution of a planet is proportional to the cube of
its distance from the sun.
Reason : Suns gravitational field is inversely proportional to the square of its distance from
the planet
(a) A (b) B (c) C (d) D
9. Assertion : Space ship while entering the earths atmoshere is likely to catch fire
187
Reason : Temperature of upper atomosphere is very high
(a) A (b) B (c) C (d) D
10 Assertion : The earth is slowing down and as a result the moon is coming nearer to it
Reason : The angular momentum of earth - moon system is not conserved
(a) A (b) B (c) C (d) D
188
Assertion - Reason Type question :
1. A Both the asseration and reason are true and the latter is correct expalnation of the former.
2. A Both the asseration and reason are true and the latter is correct expalnation of the former.
3. A As the satellite is to be stationary over a particular place, its time period of revolution = 24
hours= time peroid of revolution of earth about its axis.
4. A
g
2 T

=
on a satellite, there is a weightless -ness, so g= 0 hence
= T
Thus both the asseration and reason are correct
5. A As no torque is acting on the planet, its angular momentum must stay constant in a magnitude
as well as direction there for planet of rotation must pass throught the center of earth
6. D
R
1
Ve i.e.
R
GM 2
Ve =
1
R
Ve' = Ve 2 Ve 2 11.2 22.4 kms
R
4

= = =
thus assertion is wrong but Reason is correct
7. A Both the asseration and reason are correct and the Reason is correct explanation of Asseration
8. A A Both the asseration and reason are correct and the Reason is correct explanation of Asseration
9. C Here Asseration is correct but Reason is wrong because the space ship while entering the earths
atmosphere may catch fire due to atmoshperic air friction
10. D Here Both the asseration and reason are wrong because the angular momentum of earth
moon system is conserved in the absence of extermal touque.
189
Comperehensions Type Questions
(1) If a smooth tunnel is dug across a diameter of earth and a particle is related from the surface
of earth, the particle oscillates simple harmonically along it.
(1) Time period of the particle is not equal to
(A)
g
R
2
(B)
2
3
R
GM
2
(C) 84.0 min (D) Nome of these
(2) Maximum speed of the these
(A)
R
GM 2
(B)
R
GM
(C)
R 2
GM 3
(D)
R 2
GM
2. When a paricle is projected from the surface of earth, it mechanincal energy and angular momentum
about center of earth at all time is constant
(i) A particle of mass m is projected from the surface of earth with velocity v
o
at angle u
with horizontal suppose h be the maximum height of particle from surface of earth and v
its speed at that point them v is
(A) v
0
coso (B) >v
0
coso
(C) <v
0
coso (D) zero
(ii) Maximum height h of the paritcle is
(A)
2g
sin Vo
2 2
=
(C)
2g
sin Vo
2 2
<
(B)
2g
sin Vo
2 2
>
(D) can be greater than or less than
2g
sin Vo
2 2
3. A solid sphere of mass M and radius R is surrounding by a spherical
shell of same mass M and raius 2R as shown. A small particle
of mass m is relased from rest from a height (h <<R) aobove the
shell there is a hole in the shell.
(i) in what time will it enter the hole at A
(A)
GM
hR 2
2
(C)
GM
hR
2
(B)
GM
2hR
2
(D) None of these
(ii) what time will it take to move from A to B ?
(A)
2
R
GMh
=
(C)
2
R
GMh
<
(B)
2
R
GMh
>
(D) None of these
190
(iii) with what apporximate speed will it colide at B ?
(A)
2 GM
R
(C)
2R
GM
(B)
2R
GM 3
(D)
R
GM
4. A planet is revolving round the sun in elliptical orbit. Velocity at perigee position (nearest) is
v
1
| and at apogee position (farthest) is v
2
Both these velocities are perpendicular to the joinging
center of sun and planet r is the minimum distance and r
2
the maximum distance.
(i) when the planet is at perigee position, it wants to revolve in a circular orbit by itself. For
this value of G
(A) Should increase (B) Should decrease
(C) data is in sufficeint (D) will not depend on the value of G
(ii) At apogee position suppose speed of planer is slightly decreased from v
2
, then what will
happen to minimum distance r
1
in the subsequent motion
(A) r
1
and r
2
bothe will dicreases
(B) r
1
and r
2
bothe will increases
(C) r
2
will remain as it is while r
1
will increase
(D) r
2
will remain as it is while r
1
will decrease
5. Garvitational potential at any point inside a spherical shall is uniform
and is given by
GM
R

where M is the mass of shell and R its


radius. At the center solid sphere, potential is
3
2

GM
R
(i) There is a concentric hole of radius R in a solid sphere of radius 2R Mass of the remaining
portion is M What is the gravitation potential at center?
(A)
7R
3GM
(B)
5GM
7R

(C)
14
GM 7
(D)
R 14
GM 9
191
Solution
(1) (i) have we
R
GM
g putting
g
R
2 T
2
= =
3
2
2
T R
Gm
=
(ii)maximum speed is at centre from conservation mechanicall energy(from cerface to center)
increase in K.E. = decrease in P.E.
2
i f i f
1
m U U m(V - V )
2
n = =
i f
2(v v ) =
R
GM
12
GM 3
R
GM
2 =
(

+ =
(2) (i) From conservation of angular momentum at A and B
( ) mVocos m R h = v +
0
R
V cos
R h
| |
v =
|
+
\ .
o
V cos v <
(ii) From consercation of mechanical energy, Decrease in K.E. = increase in P.E.
( )
R
h
1
mgh
V V m
2
1
2 2
0
+
=
cos V V here but
R
h
1
gh 2
V V
0
2 2
0
<
+
=
x V V let so sin V V V
2 2
0
2 2
0
2 2
0
= =
sin V x here
2 2
0
>
R
h
1
2gh
x
+
=
R
x
2g
x
h

=
i.e. h >
x
2g
i.e. h >
2 2
O
sin
2g
v u
192
(3) (i) Acceleration due to gravity near the surface of shell can be assumed to be uiform
2 2
2R
GM
(2R)
G(2M)
g = =
From
2
1
h
2
gt =
t =
GM
hR
g
h
2
2
2
=
(ii)
2 2
2 2
2
A
GM GMh
V gh h
R R
| |
= = =
|
\ .
From A to B field due to shell is zero, but field due to sphere is not-zero
hence t
AB
2
A
R R
V GMh
< <
(iii) K
A
=0 polential between A and B due to shell is
From energy conservation
K
A
+ U
A
= K
B
+ U
B
K
B
= (U
A
= U
B
) = m(V
A
V
B
)
1
2
mV
B
2
= m(V
A
VV
B
)
B A B
V 2(V V ) = =
GM
R
(4) At perigee position v
1
> v
0
where v
0
is the orbital velocity for circular motion
0
GM
V G
r
=
so value of G should incease , so that v
0
will increase for this position and which
will become equal to v
1
(ii) path will becomemore elliptical , keeping r
2
constant and r
1
to decrease
(5) Density of given material
3 3 3
4
3
M 3M
[(2R) R ] 28R
= =

Vwhole = Vhole + Vremaing


Vremaing = Vwhole Vhole
1 2
3 GM GM
2 2R R
(
= +
(

193
here
3
1
4 8
M (2R) M
3 7
= =
3
2
4 M
M ( ) R
3 7
= t =
14R
9GM
Vremaining =
Match the Column
(1) on the surface of earth acceleration due to gravity is g and gravitational potential is V match
the following
Table - 1 Table -2
(A) At height h = R value of g (P) decreases by a factor
1
4
(B) At height h = R/2 value of g (Q) decreases by a factor
1
2
(C) At height h = R value of v (R) increases by a factor 11/8
(D) At depth h = R/2 value of V (S) increases by a factor 2
(T) None
(2) Density of planet is two times the density of earth Radius of this planet is half (As compared
to earth)
Match the following
Table-1 Table-2
(A) Acceleration due to gravity on (P) Half
this planets surface
(B) Gravitational potential (Q) same
on the surface
(C) Gravitational potential (R) Two times
at centre
(D) Gravitational field strength (S) four times
at centre
(3) let V and E denote the gravitational potential and gravitational field at a point. Then the match
the follwing
Table-1 Table-2
(A) E = 0, V = 0 (P) At center of Spherical shell
(B) E
=
0, V= o (Q) At centre of solid sphere
(C) V
=
0, E -0 (R) At centre of circular ring
(D) V
=
0, E
=
0 (S) At centre of two point masses of equal magnitude
(T) None
194
(4) Match the following
Table-1 Table -2
(A) time period of an earth satellite in circular orbit (P) Independent of mas of satellite
(B) Orbital velocity of satellite (Q) independent of radius of orbit
(C) Mechnical energy of stellite (R) independent of mass of earth
(S) none
(5) Match the following (for a satellite in circular orbit)
Table-1 Table-2
(A) kinetic energy (P)
2
GMm
r
(B) potential energy (Q)
GM
r
(C) Total energy (R)
GMm
r

(D) orbital velocity (S)


2
GMm
r
solution :
(1) A-p, B-Q, C-s D-t
(2) A-Q, B-p, C-p D-o
(3) A-t, B-t, C-p,q,r,s, D-t
(4) A-q, B-t, C-r, D-s
(5) A-s, s-B, c-p, D-q

195
Unit - 7
Proporties of
Liquid a Solid
196
SUMMARY
Surface tension of a liquid is measured by the force
acting per unit length on either side of an imaginary line
drawn on the free surface of liquid, the direction of this
force being perpendicular to the line and tangential to
the free surface of liquid. So if F is the force acting on
one side of imaginary line of length l. then T = Fl.
1. It depends only on the nature of liquid and is independent of the area of surface or length of line
considered.
2. It is a scalar as it has a unique direction which is not to be specified.
3. Dimension
2 0 1
T L M

4. Unit :
( ) SI
m
N
5. It is a molecular phenomenon and its root cause is the electromagnetic forces.
Force Due to Surface Tension
If a body of weight W is placed on the liquid surface, whose surface tension is T, and if F is the
minimum force required to pull it away from the water then value of F for different bodies can be
calculated by the following table.
Body Force
1. Needle (length l) w T 2 F + =
2. Hollow disc (inner radius r
1
Outer radius r
2
) ( ) w T r r 2 F
2 1
+ + t =
3. Thin ring (radius r) w rT 4 F + t =
4. Circular plate (or disc) (radius r) w rT 2 F + t =
5. Square frame (side l) w T 8 F + =
6. Square plate w T 4 F + =
Examples of Surface Tension :
1. When mercury is split on a clean glass plate, it forms
globules. Tiny globules are spherical on the account of
surface tension because force of gravity is negligible. The
bigger globules because force of gravity is negiligible. The
bigger globules get plattened from the middle but have
round shape near the edges.
197
2. When a molten metal is poured into water from a
suitable height, the falling stream of metal breaks up
and the detached portion of the liquid in small quantity
acquire the spherical shape.
3. Rain drops are spherical in shape because each drop tends to acquire minimum surface area due to
surface tension, and for a given volume, the surface area of sphere is minimum.
4. Oil drop spreads on cold water. Whereas it may remain as a drop on hot water. This is due to the
fact that the surface tension of oil is less than that of cold water and is more than that of hot water.
5. If a small irregular piece of camphor is floated on the surface of pure water, it does not remain steady
but dances about on the surface. This is because, irregular shaped camphor dissolves unequally and
decreases the surface tension of the water locally. The unbalanced forces make it to move haphazardly
in different directions.
6. Take a frame of wire and dip it in soap solution
and take it out, a soap film will be formed in the
frame. Place a loop of wet thread gently on the
film. It will remain in the form, we place it on the
film according to figure.
Now, piercing the film with a pin at any point
inside the loop, It immediately takes the circular
from as shown in figure.
7. When a greased iron needle is placed genetly on the surface of
water at rest, so that it does not prick the water surface, the needle
floats on the surface of water despite it being heavier because the
weight of needle is balanced by the vertical components of the forces
of surface tension. If the water surface is pricked by one end of the
needle, the needle sinks down.
8. Hair of shaving brush / painting brush when dipped in
water spread out, but as soon as it is taken out, its hair
stick together.
Factors affecting surface tension
1. Temperature :
The surface tension of liquid decreases with rise of temperature. The surface tension of liquid is zero
at its boiling point and it vanishes at critical temperature. At critical temperature, intermolecular
forces for liquid and gases becomes equal and liquid can expand without any restriction. For small
temperature differences, the variation in surface tenstion with temperature is linear and is given by
the relation.
198
( ) t 1 T T
0 t
o =
Where T
t
and T
O
are the surface tensions at t
O
c and 0
O
C respectively and o is the temperature
coefficient of surface tension.
Examples :
(i) Hot soup tastes better than the cold soup.
(ii) Machinery parts get jammed in winter.
2. Impurities :
The presence of impurities either on the liquid surface or dissolved in it, considerably affect the
surface tension, depending upon the degree of contamination. A highly soluble substance like sodium
chloride when dissolved in water, increases the surface tension of water. But the sparingly soluble
substances like phenol when dissolved in water, decreases the surface tension of water.
Applications of Surface Tension
1. The oil and grease spots on cloths cannot be removed be pure water. On the other hand, when
detergents (like soap) are added in water, the surface tension of water decreases. As a result of this,
wetting power of soap solution increases. Also the force of adhesion between soap solution and oil
or grease on the clothes increases. Thus, Oil, grease and dirt particles get mixed with soap solution
easily. Hence, clothes are washed easily.
2. Surface tension of all lubricating oils and paints is kept low so that they spread over a large area.
3. A rough sea can be calmed by pouring oil on its surface.
4. In soldering, addition of 'flux' reduces the surface tension of molten tin, hence, it spreads.
Molecular Theory of Surface Tension
The maximum distance upto which the force of attraction between
two molecules is appriciable is called molecular range ( ) m 10
9
= .
A sphere with a molecule as centre and radius equal to molecular
range is called the sphere of influence. The liquid enclosed between
free surface (PQ) of the liquid and an imaginary plane (RS) at a
distance r (equal to molecular range) from the free surface of the
liquid form a liquid film.
To understand the concept of tension acting on the free surface of a liquid, let us consider four liquid
molecules like A, B, C and D. Their sphere of influence are shown in the figure.
1. Molecule A is well within the liquid, so it is attracted equally in all directions. Hence the net force on
this molecule is zero and it moves freely inside the liquid.
2 Molecule B is little below the free surface of the liquid and it is also attracted equally in all directions.
Hence the resultant force acting on it is also zero.
3. Molecule C is just below the upper surface of the liquid film and the part of its sphere of influence is
outside the free liquid surface. So the number of molecules in the upper half (attracting the molecules
upward) is less than the number of molecule in the lower half (attracting the molecule downward).
Thus the molecule C experiences a net downward force.
4. Molecule D is just on the free surface of the liquid. The upper half of the sphere of influence has no
liquid molecule. Hence the molecule D experiences a maximum downward force.
199
Thus all molecules lying on surface film experiences a net downward force. Therefore, free surface
of the liquid behaves like a stretched membrane.
Surface Energy :
The potential energy of surface molecules per unit area of the surface is called surface energy.
Unit :
2
m
J
Dimension :
2
ML

Suppose that the sliding wire LM is moved through a


small distance x, so as to take me position . . M L ' ' In
this process, area of the film increases by x 2 (on
the two sides) and to do so, the work done is given by,
( ) ( ) A T x 2 T x 2 T fx W A = = = =
A
W
T
A
=
i.e. Surface tension may be defined as the amount of work done in increasing the area of the liquid
surface by unity against the force of surface tension at constant temperature.
Work done in Blowing a liquid drop or soap-bubble
1. If the intial radius of liquid drop is r
1
and the final radius of liquid drop is r
2
then,
Area in increment T W =
( )
2
1
2
2
r r 4 T t =
2. In case of soap-bubble,
( )
2
1
2
2
r r 8 T W t = (Bubble has two free surfaces)
Splitting of Bigger Drop :
When a drop of radius R splits into n smaller drops, (each of radius r) then surface area of liquid
increases. Hence the work is to be done against surface tension.
Since the volume of liquid remains constant therefore
3 3
r
3
4
n R
3
4
t = t
3 3
nr R =
Work done A T A = ( )
2 2
R 4 r n 4 T t t =
Formation of Bigger drop :
Amount of surface energy released = Initial surface
energy - final surface energy
( ) T R 4 n T r 4 E
2 2
t t =
Excess Pressure :
Due to the property of surface tension a drop or bubble tends to contract and so compresses the
matter enclosed. This in turn increases the internal pressure which prevents further contraction and
200
equilibrium is achieved. So in equilibrium the pressure inside a bubble or drop is greater than outside
and the difference of pressure between two sides of the liquid surface is called excess pressure.
Excess pressure in different cases is given in the following table :
Plane surface Concave surface
AP = 0 AP =
2T
R
Convex surface Drop
AP =
2T
R
AP =
2T
R
Bubble in air Bubble in liquid
AP =
4T
R
AP =
2T
R
Cylindrical liquid surface Liquid film of unequal radii
R
I
P = A
(

+ = A
2 1
R
1
R
1
T 2 P
Shape of liquid meniscus :
When a capillary tube is dipped in a liquid, the liquid surface becomes curved near the point of
contact. This curved surface is due to the resultant of two force i.e. the force of cohesion and the
force of adhesion. The curved surface of the liquid is called meniscus of the liquid.
If liquid molecule A is in the contact with solid. (i.e. wall of capillary tube) then forces acting on
molecule A are
(i) Force of adhesion F
a
(ii) Force of cohesion F
C
(iii) Resultant force F
N
makes an angle o with Fa.
C a
C
o
C a
o
C
F F 2
F
135 cos F F
135 sin F
tan

=
+
= o
Example :
Pure water in silver coated capillary tube.
201
Angle of contact :
Angle of contact between a liquid and a solid is defined as the angle enclosed between the tangents
to the liquid surface and the solid surface inside the liquid, both the tangents being drawn at the point
of contact of the liquid with the solid.
Capillarity :
If a tube of very narrow bore is dipped in a liquid, it is found that the liquid in the capillary either
ascends or descends relative to the surrounding liquid. This phenomenon is called capillarity.
The root cause of capillarity is the difference in pressure on two sides of (concave and convex)
curved surface of liquid.
Examples :
(i) Ink rises in the fine pores of bloting paper leaving the paper dry.
(ii) A towel soaks water.
(iii) Oil rises in the long narrow spaces between the threads of wick.
Ascent Formula :
2T
hg
R
=

rhg Rhg
T
2cos 2
= =
Useful Facts and Formulae :
(1) Formation of double bubble :
Radius of the interface
( ) 1 ...
r r
r r
r
1 2
2 1

=
and ( ) 2 ...
r
1
r
1
T 4 P
2 1
|
|
.
|

\
|
= A
(2) Formation of a single bubble
Under isothermal condition two soap bubble of radii "a"
and "b" coalesce to from a single bubble of radius C.
Now
C
T 4
P P ,
b
T 4
P P ,
a
T 4
P P
0 C 0 b 0 a
+ = + = + =
3
a
a
3
4
V t =
Now as mass is conserved,
3
c
c
3
4
V t =
c b a
t = t + t
3
c
c
3
4
V t =
c
c c
b
b b
a
a a
RT
V P
RT
V P
RT
V P
= +
202
Temp is constant,
c b a
T T T = =
|
.
|

\
|
t |
.
|

\
|
+ = |
.
|

\
|
+ + |
.
|

\
|
t |
.
|

\
|
+
3
0 0
3
0
c
3
4
C
T 4
P
b
T 4
P a
3
4
a
T 4
P
( )
( )
2 2 2
3 3 3
0
c b a 4
b a c P
T
+

=
Hooke's Law and Modulus of Elasticity :
According to this Law, within the elastic limit, stress is proportional to the strain.
i.e. stress o strain
Stress
Strain

= E = constant
The constant E is called the modulus of elasticity.
There are three modulii of elasticity namely (i) Young's
modulus (y), (ii) Bulk modulus (B), and modulus of rigidity
(q), responding to three types of the strain.
(1) Young's modulus (Y)
It is defined as the ratio of normal stress to longitudinal strain within limit of proportionality

A
FL
L
A
F
strain al longitudin
stress Normal
Y = = =

2
r
mgL
Y
t
=
Force constant of wire,
L
YA F
K = =

Bulk Modulus :
When a solid or fluid (liquid or gas) is subjected to a uniform pressure all over the surface, such that
the shape remains the same, then there is a change in volume.
Then the ratio of normal stress to the volumetric strain within the elastic limits is called as Bulk
modulus. This is denoted by K.
strain Volumetric
stress Normal
K =
v
pV
V
v
A
F
A

=
A
=
203
Modulus of Rigidity :
Within limits of proportionality, the ratio of tangential stress to the shearing strain is called modulus of
rigidity of the material of the body and is denoted by q,
strain Shearing
stress Tangential
= q
In this case the shape of a body changes but its volume remains unchanged.
|
=
|
= q
A
F
A
F
Pascal's Law :
It states that if gravity effect is neglected, the pressure at every point of liquid in equilibrium of rest is
same.
Working of hydraulic lift, hydraulic press and hydroulic breaks :
It is used to lift the heavy loads. If a small force f is
applied on pistion of C then the pressure exerted on
the liquid
a
f
P =
[a = Area of cross section of the pistion in C]
This pressure is transmitted equally to piston of cylinder D.
|
.
|

\
|
o
= = =
A
f A
a
f
PA F
As A >> a, F << f
Thermal Expansion :
When matter is heated without any change in its state it usually expands. According to atomic theory
of matter, a symmetry in potential energy curve is responsible for thermal expansion. As energy of
atoms increases, hence the average distance between the atoms increases. So the matter as a whole
expands.
(1) Thermal expansion is minimum in case of solids but maximum in case of gases because intermolecular
force is maximum in solids but minimum in gases.
(2) Solids can expand in one dimension (linear expansion) two dimensions (superficial expansion) and
three dimensions (volume expansion) While liquids and gases usually suffer change in volume only.
Heat :
(1) The form of energy which is exchanged among various bodies or system on account of temperature
difference is defined as heat.
(2) We can change the temperature of a body by giving heat (temperature rises) or by removing heat
(temperature falls) from body.
(3) Heat is a scalar quantity. It's units are joule, erg, cal, kcal etc.
(4) 1 kcal = 1000 cal = 4186 J and 1 cal = 4.18 J
204
Specific Heat :
(1) The amount of heat energy required to raise the temperature of unit mass of a body through (or K)
is called specific heat of the material of the body.
If Q heat changes the temperature of mass m by u A then specific heat
u A
u
=
m
C
( )
1 2 2
T L Unit

u
(2) Molar specific Heat :
Molar specific heat of a substance is defined as the amount of heat required to raise the temperature
of one gram mole of the substance through a unit degree, it is represented by C
u A t
=
u A
=
u A
=
Q 1 Q
m
M
M
MQ
C
Unit :
1 2 2 1
T L M

u
(3) Latent Heat :
The amount of heat required to change the state of the mass m of the substance while its temperature
remaining constant is written as : Q = mL, where L is the latent heat Latent heat is also called as
Heat of Transformation, It's unit is
gm
cal
or
kg
J
and dimension :
2 2
T L

Elastic behaviour :
The propertory of matter by virture of which a body
tends to regain its original shape and size after the removal of
deforming force is called elasticity.
In solids, atoms and molecules are arranged in such a
way that each molecule is acted upon by the forces due to
heighbouring molecules. These forces are known as
intermolecular forces.
For simplicity, the two molecules in their equilibrium positions (at inter-molecular distance
r = r
0
) are shown by connecting them with a spring.
In fact, the spring connecting the two molecules represents the inter-molecular force between
them on applying the deforming forces, the molecules either come closer or go far apart from each
other and restoring forces are developed. When the deforming force is removed, these restoring
forces bring the molecules of the solid to their respective equilibrium position (r = r
0
) and hence the
body regains its original form.
Thermal Capacity :
It is defined as the amount of heat required to raise the remperature of the whole body (mass m)
through 1
0
C or 1K.
Thermal capacity
u A
= t = =
Q
c mc
The value of thermal capacity of a body depends upon the nature of the body and its mass.
Dimensions
1 2 2 1
T L M

u Units :
k
J
,
C
Cal
o
205
Principle of Calorimetry :
Calorimetry means 'measuring heat'.
When two bodies (one being solid and other liquid or both being liquid) at different temperatures are
mixed, heat will be transfered from body at higher temperature to a body at lowe temperature till
both acquire same temeprature. The body at higher temperature releases heat while body at lower
temeprature absorbs it, so that
Heat lost = Heat gained
i.e., Principle of calorimetry represents the law of conservation of heat energy.
Thermometry :
A branch of science which deals with the measurement of temperature of a substance is known as
thermometry.
Phase change :
We use the term phase to describe a specific state of matter, such as solid, liquid or gas. A
transtion from one phase to another is called a phase change.
For any given pressure a phase change takes place at a definite temperature, usually
accompanied by absorption or emission of heat and a change of volume and density.
In phase change 1
0
ce at 0
0
C melts into water at 0
0
C. Water at 100
0
C boils to form steam at
100
0
C.
Streamline flow :
Stream line flow of a liquid is that flow in which each element of the liquid passing through a
point travels along the same path and with the same velocity as the preceding element passes through
that point.
A streamline may be defined as the path, straight or
curved, the tangent to which at any point gives the direction
of the flow of liquid at that point.
The two streamlines cannot cross each other and
the greater is the crowding of streamlines at a place, the
greater is the velocity of liquid particles at that place.
Laminar Flow :
If a liquid is flowing over a horizontal surface with a steady flow and moves in the form of
layers of different velocities which do not mix with each other, then the flow of liquid is called laminar
flow.
In this flow, the velocity of liquid flow is always less than the critical velocity of the liquid. The
laminar flow is generally used synonymously with streamlined flow.
Turbulent flow :
When a liquid, moves with a velocity greater than
its critical velocity, the motion of the particles of liquid
become disordered or irregular. Such a flow is called a
turbulent flow.
In a turbulent flow, the path and the velocity of the particles of the liquid change continuously
and haphazardly with time from point to point. In a turbulent flow, most of the external energy
206
maintaining the flow is spent in producint eddies in the liquid and only a small fraction of energy is
available for forward flow.
Critical velocity :
The critical velocity is that velocity of liquid flow upto which its flow is streamlined and above
which its flow becomes turbulent.
Reynold's number :
Reynold's number is a pure number whichdetermines the nature of flow of liquid through a
pipe.
area unit per force Viscous
area unit per force Inprtial
N
R
=
3 AV
dt
dm
Now =
Inertial force per unit area
A
dt
dm
V
A
dt
d
|
.
|

\
|
=

=
=

=
2
V
A
AV V
Viscous force per unit area
r
nv
A
F
= =

q

=
Vr
N
R
If , 2000 N 0
R
s s If 3000 N 2000
R
s s If 3000 N
R
>
laminar or stream line Unstable flow definitely turbulent flow
Equation of continuity :
The equation of continuity is derived from the
principle of conservation of mass.
A non-viscous liquid in streamline flow passes
through a tube AB of varying cross section. Let the cross
sectional area of the pipe at points A and B be a
1
and a
2
respectively.
Mass of the liquid entering per second at A = Mass
of the liquid leaving per second at B.
2 2 2 1 1 1
v a v a =
2 2 1 1
v a v a =
If the liquid is incompressible

av = constant
which is the equation of continuity.
207
MCQ
For the answer of the following questions choose the correct alternative from among
the given ones.
1. Two wires are made of the same material and have the same volume. However, wire 1 has cross-
sectional area A and wire 2 has cross-sectional Area 3A. If the length of wire 1 increases by on
appling force F. How much force is needed to stretch wire 2 by same amount.
(A) F (B) 4F (C) 6F (D) 9F
2. The increases in length in l of a wire of length L by longitudinal stress. Then the stress is propotional
to..............
(A)

L
(B) L (C) (D) L
2

3. The dimensions of four wires of the same material are given below, in which wire the increase in
length will be maximum when the same strain is applied.
(A) Length 100 cm, Diameter 1 mm (B) Length 200 cm, Diameter 2 mm
(C) Length 300 cm, Diameter 3 mm (D) Length 50 cm, Diameter 0.5 mm
4. The young's modulus of a wire of length L and radius r is Y
2
m
N
. If the length and radius are
reduced to
2
L
and
2
r
. Then what will be its young's modulus ?
(A)
2
Y
(B) Y (C) 2Y (D) 4Y
5. A beam of metal supported at the two ends is loaded at the centre. The depression at the centre is
propotional to..............
(A) Y
2
(B) Y (C)
Y
1
(D)
2
Y
1
6. A wire is loaded by 6 kg at its one end, the increase in length is 12 mm. If the radius of the wire is
doubled and all other magnitudes are unchanged, then increase in length will be.............
(A) 6 mm (B) 3 mm (C) 24 mm (D) 48 mm
7. On increasing the length by 0.5 mm in a steel wire of length 2 m and area of cross-section 2 mm
2
the
force required is................
[Y for steel
m
N
10 2 . 2
11
=
]
(A) N 10 1 . 1
5
(B) N 10 1 . 1
4
(C) N 10 1 . 1
3
(D) N 10 1 . 1
2

8. A stress of
2 8
Nm 10 18 . 3 is applied to steel rod of length 1 m along its length. Its young's
modulus is
2
11
m
N
10 2
. Then what is the elongation produced in the rod in mm ?
(A) 3.18 (B) 6.36 (C) 5.18 (D) 1.59
208
9. Two springs P & Q of force constant K
P
& K
Q

P
Q
K
K
2
| |
=
|
\ .
are stretched by applying force equal
magnitude. If the energy stored in Q is E. Then what is the energy stored in P ?
(A) E (B) 2E (C)
2
E
(D)
4
E
10. A force F is needed to break a copper wire having radius R, The force needed to break a copper
wire of radius 2R will be........
(A)
2
F
(B) 2F (C) 4F (D)
4
F
11. A rubber cord 10m long is suspended vertically. How much does it stretch under its own weight.
( Density of rubber is
2 2
8
3
s
m
10 g ,
m
N
10 5 Y ,
m
kg
1500 = = )
(A) m 10 15
4
(B) m 10 5 . 7
4
(C) m 10 12
4
(D) m 10 25
4

12. The young's modulus of the material of a wire is equal to the ...........
(A) stress required to increase its length four times
(B) strtess required to prdouce unit strain
(C) strain produced in it (D) stress acting on it
13. If x, longitudinal strain is produced in a wire of young's modulus y then energy stroed in the material
of the wire per unit volume is..........
(A) yx
2
(B) 2yx
2
(C)
x y
2
1
2
(D)
2
yx
2
1
14. A steel wire of cross-sectional area
2 6
m 10 3

can with stand a maximum strain of 10
-3
Young's
modulus of steel is
.
m
N
10 2
2
11

The maximum mass the wire can hold is.............


|
.
|

\
|
=
2
s
m
10 g
(A) 40 kg (B) 60 kg (C) 80 kg (D) 100 kg
15. The young's modulus of a rubber string 8 cm long and density
3
m
kg
5 . 1
is
.
m
N
10 5
2
8

What will
be the length increase due to its own weight ?
(A) m 10 6 . 9
5
(B) m 10 6 . 9
11

(C) m 10 6 . 9
3
(D) m 6 . 9
16. A and B are two wires. The radius of A is twice that of B. They are stretched by the same load. Then
what is the stress on B ?
(A) Equal to that on A (B) Four times that on A
(C) Two times that on A (D) Half that on A
17. If the length of wire is reduced to half then it can hold the .............load.
(A) Half (B) Same (C) Double (D) One fourth
209
18. There are two wires of same material and same length. While the diameter of second wire is 2 times,
the diameter of first wire. Then what will be the ratio of extension produced in the wire by applying
same load ?
(A) 1 : 1 (B) 2 : 1 (C) 1 : 2 (D) 4 : 1
19. When the length of a wire having cross-section area
2 6
m 10

is stretched by 0.1% then tension in


it is 100 N. Young's modulus of material of wire is...........
(A)
2
12
m
N
10
(B)
2
2
m
N
10
(C)
2
10
m
N
10
(D)
2
11
m
N
10
20. Two wires of equal lengths are made of the same material wire A has a diameter that is twice as that
of wire B. If identical weights are suspended from the ends of these wires the increase in length
is............
(A) Four times for wire A as for wire B. (B) Twice for wire A as for wire B.
(C) Half for wire A as for wire B. (D) One-fourth for wire A as for wire B.
21. Steel and copper wires of same length are stretched by the same weight one after the other. Young's
modulus of steel the ratio increase in length ?
(A)
5
2
(B)
5
3
(C)
4
5
(D)
2
5
22. An area of a cross-section of rubber string is 2 cm
3
. Its length is doubled when stretched with a
linear force of
5
10 2 dynes. What will be young's modulus of the rubber in dynes ?
(A)
5
10 4 (B)
5
10 1 (C)
5
10 2 (D)
4
10 1
23. A substance breaks down by a stress of If the density of the material of the wire is then the length of
wire of the substance which will break under its own weight when suspended vertically is.............
(A) 66.6 m (B) 60.0 m (C) 33.3 m (D) 30.0 m
24. The temperature of a wire of length 1 meter and area of cross-sectional section 1 cm
2
is increased
from to If the rod is not allowed to increase in length. What will be the force required ?
|
.
|

\
|
= = o

2
11 o 5
m
N
10 Y , C / 10
(A) 10
3
N (B) 10
4
N (C) 10
5
N (D) 10
9
N
25. If longitudinal strain for a wire is 0.03 and its poisson's ratio is 0.5, then what is its lateral strain ?
(A) 0.003 (B) 0.0075 (C) 0.015 (D) 0.4
26. An aluminium rod (Young's modulus
2
9
m
N
10 7 =
) has a breaking strain of 0.2 % what is the
minimum cross-sectional area of the rod in order to support a load of 10
4
Newtons ?
(A)
2 2
m 10 1

(B)
2 3
m 10 4 . 1

(C)
2 3
m 10 5 . 3

(D)
2 4
m 10 1 . 7

210
27. Two wires of copper having the length in the ratio 4 : 1 and their are as 1 : 4 are stretched by the
same force. What will be the ratio of longitudinal strain in the two wires ?
(A) 1 : 16 (B) 16 : 1 (C) 1 : 64 (D) 64 : 1
28. A wire elongates by 1 mm when a load W is hanged from it. If the wire goes ever a pulley and two
weight W each are hang at the two ends. What will be the elongation of the wire ? (in mm)
(A) 2 (B) zero (C)
2

(D)

29. 200 kg weight hanged at a free edge of vertical wire of length 600.5 cm when removed the weight,
length reduced by 0.5 cm and it gets original status then what is the young moduluse of wire ?
(A) 2.35 10
12
N/m
2
(B) 1.35 10
10
N/m
2
(C) 13.5 10
11
N/m
2
(D) 23.5 10
9
N/m
2
30. The ratio of diameter of two wires of same material is n : 1 the length of wires are 4 m each. On
applying the same load. What will be the increase in length of their wire ?
(A) n
2
times (B) n times (C) 2n times (D) None of the above
31. Longitudinal stress of
2
mm
N
1
is applied on a wire what is the % increase in length ?
|
.
|

\
|
=
2
4
m
N
10 Y
(A) 0.002 (B) 0.001 (C) 0.003 (D) 0.01
32. A steel wire is stretched with a definate load of If the young's modulus of the wire is Y. For decreasing
the value of Y............
(A) Radius is to be decreased (B) Radius is to be increased
(C) Length is to be increased (D) None of the above
33. The area of cross-section of a steel wire is
|
.
|

\
|
=
2
11
m
N
10 2 Y
is 0.1 cm
2
, what will be the force
required to double its length ?
(A) N 10 2
12
(B) N 10 2
11
(C) N 10 2
10
(D) N 10 2
6

34. Two wires A & B are of same materials. Their lengths in the ratio 1 : 2 and diameters are in the ratio
2 : 1 when stretched by force
B A
F and F respectively, they get equal increase in lengths Then the
ratio
FB
FA
should be...........
(A) 1 : 2 (B) 1 : 1 (C) 2 : 1 (D) 8 : 1
35. The mean distance between the atoms of iron is
10
10 3

and interatomic force constant for iron is
m
N
7
. What is the young's modulus of elasticity for iron ?
(A)
2
5
m
N
10 33 . 2
(B)
2
10
m
N
10 3 . 23
(C)
2
10
m
N
10 233
(D)
2
10
m
N
10 33 . 2
211
36. A force of 200 N is applied at one end of a wire of length 2 m and having area of cross-section
2 2
cm 10

, the other end of the wire is rigidly fixed. If of linear expansion of the wire
C / 10 8
o 6
= o and young's modulus
2
11
m
N
10 2 . 2 Y =
and its temperature is increased by
5
0
C then the increase in the tension of the wire will be..........
(A) 4.2 N (B) 4.4 N (C) 2.4 N (D) 8.8 N
37. A uniform plank of young's modulus Y is moved over a smooth horizontal surface by a constant
horizontal force F, The area of cross-section of the plank is A. What is the compressive strain on its
plank in the direction by the force ?
(A)
AY
F
(B)
AY
F 2
(C)
|
.
|

\
|
AY
F
2
1
(D)
AY
F 3
38. The length of a wire is 1.0 m and the area of cross-section is
2 2
cm 10 0 . 1

. If the work done for
increase in length by 0.2 cm is 0.4 joule. Then what is the young's modulus ? Of material of the wire
?
(A)
2
10
m
N
10 0 . 2
(B)
2
10
m
N
10 0 . 4
(C)
2
11
m
N
10 0 . 2
(D)
2
11
m
N
10 0 . 4
39. A rubber cord catapult has cross-sectional area 25 mm
2
and initial length of cord is 10 cm. It is
stretched to 5 cm and then released to project a missile of mass 5 gm. Taking
2
8
rubber
m
N
10 5 Y =
velocity of projected missile is............
(A)
s
m
20
(B)
1
s
m
100

(C)
1
s
m
250

(D)
1
s
m
200

40. A wire of cross-section 4 mm
2
is stretched by 0.1 mm by a certain weight. How far (length) will be
wire of same material and length but of area 8 mm
2
stretched under the action of same force.
(A) 0.05 mm (B) 0.10 mm (C) 0.15 mm (D) 0.20 mm
41. According to Hooke's law of elasticity stress is increased the ratio of stress toi strain.........
(A) increases (B) Decreases
(C) becomes zero (D) Remains constant
42. Young's modulus of rubber is
2
4
m
N
10
and area of cross section is 2 cm
2
if force of
5
10 2 dyne is
applied along its length then its initial length L becomes .............
(A) 3L (B) 4L
(C) 2L (D) None of the above
212
43. A copper wire of length 4 m and area of cross-section 1.2 cm
2
is stretched with a force of
3
10 8 . 4 N.
If young's modulus for copper is
2
11
m
N
10 2 . 1
What will be the length increase of the wire ?
(A) 1.33 mm (B) 1.33 cm (C) 2.66 mm (D) 2.66 cm
44. If the interatomic spacing in a steel wire is & A 3
o
.
m
N
10 20 Y
2
10
Steel
= the force constant =
.............
(A)
A
N
10 6
2

(B)
A
N
10 6
9

(C)
A
N
10 6
5

(D)
A
N
10 6
5

45. A wire of length 2 m is made from 10cm


3
of copper. A force F is applied so that its length increases
by 2mm. Another wire of length 8 m is made from the same volume of copper. If the force F is
applied to it, its length will increase by............
(A) 0.8 cm (B) 1.6 cm (C) 2.4 cm (D) 3.2 cm
46. A wire of length L and radius r is rigidly fixed at one end on stretching the other end of the wire a
force F the increase in its lengths is L. If another wire of same material but of length 2L and radius 2r
is stretched with a force of 2F, the increase in its length will be ..........
(A)

(B)
2
(C)
2

(D)
4

47. In steel the young's modulus and the strain at the breaking point are
2
11
m
N
10 2
and 0.15
respectively the stress at the breaking point for steel is therefore ...........
(A)
2
11
m
N
10 33 . 1
(B)
2
12
m
N
10 33 . 1
(C)
2
13
m
N
10 5 . 7

(D)
2
10
m
N
10 3
48. Which of the following statement is correct
(A) Hooke's law is applicable only within elastic limit.
(B) The adiabatic and isothermal elastic constants of a gas area equal.
(C) Young's modulus is dimensionsless.
(D) Stress multiplied by strain is equal to stored energy.
49. The force required to stretch a steel wire of
2
cm 1 cross-section to 1.1 times its length would be
|
.
|

\
|
=
2
11
m
N
10 2 Y
(A) N 10 2
6
(B) N 10 2
3
(C) N 10 2
6
(D) N 10 2
7

213
50. Which one of the following quantities does not have unit of force per unit area..........
(A) stress (B) strain
(C) Young's modulus of elasticity (D) pressure
51. A copper wire and a steel wire of same diameter and length are connected end to end a force is
applied, which stretches their combined length by 1 cm, the two wires will have.......
(A) different stresses and strains (B) the same stress and strain
(C) the same strain but different stresses (D) the same stress but different strains
52. A steel ring of radius r and cross-section area 'A' is fitted on to a wooden disc of radius R(R > r )
If young's modulus be E then what is force with which the steel ring is expanded ?
(A)
r
R
AE
(B)
|
.
|

\
|
r
r R
AE
(C)
E R r
A R
| |
|
\ .
(D)
AR
Er
53. A wire of diameter 1 mm breaks under a tension of 100 N. Another wire of same material as that of
the first one, but of diameter 2 mm breaks under a tension of.........
(A) 500 N (B) 1000 N (C) 10,000 N (D) 4000 N
54. A fixed volume of iron is drawn into a wire of length L. The extension x produced in this wire be a
constant force F is propotional to..........
(A)
2
L
1
(B)
L
1
(C) L
2
(D) L
55. On applying a stress of
2
8
m
N
10 20
the length of a perfect elastic wire is doubled. What will be its
Young's modulus ?
(A)
2
8
m
N
10 40
(B)
2
8
m
N
10 20
(C)
2
8
m
N
10 10
(D)
2
8
m
N
10 5
56. To keep constant time, watches are fitted with balance wheel made of........
(A) invar (B) stainless steel (C) Tungsten (D) platinum
57. A wire is stretched by 0.01 m by a certain force F. Another wire of same material whose diameter
and length are double to the original wire is stretched by the same force ? Then what will be its
elongation ?
(A) 0.005 m (B) 0.01 m (C) 0.02 m (D) 0.002 m
58. The Coefficient of linear expansion of brass & steel are
2 1
& o o If we take a brass rod of length
1

& steel rod of length


2
at
C 0
o
, their difference in length ( )
1 2
will remain the same at a
temperature if.................
(A)
1 2 2 1
o = o (B)
1 2 1 2
o = o
(C)
2
2
2 1
2
2
o = o (D)
2 2 1 1
o = o
214
59. A rod is fixed between two points at C 20
o
The Coefficient of linear expansion of material of rad is
C / 10 1 . 1
o 5
and Young's modulus is
2
11
m
N
10 2 . 1
. Find the stress developed in the rod if
temperature of rod becomes C 10
o
.
(A)
2
7
m
N
10 32 . 1
(B)
2
15
m
N
10 10 . 1
(C)
2
8
m
N
10 32 . 1
(D)
2
6
m
N
10 10 . 1
60. How much force is required to produce an increase of 0.2% in the length of a bross wire of diameter
0.6 mm (Young's modulus for brass
2
11
m
N
10 9 . 0 =
).
(A) Nearly 17 N (B) Nearly 34 N (C) Nearly 51 N (D) Nearly 68 N
61. A 5m long aluminium wire
|
.
|

\
|
=
2
10
m
N
10 7 Y
of diameter 3mm supports a 40 kg mass. In order
to have the same elongation in a copper wire
2
10
m
N
10 12 Y =
of the same length under the same
weight, the diameter should now be in mm............
(A) 1.75 (B) 1.5 (C) 2.5 (D) 5.0
62. Two similar wires under the same load yield elongation of 0.1 mm and 0.05 mm respectively. If the
area of Cross - section of the first wire is 4mm
2
. Then what is the area of cross - section of the
second wire ?
(A)
2
mm 6 (B)
2
mm 8 (C)
2
mm 10 (D)
2
mm 12
63. An iron rod of length 2m and cross-section area of
2
mm 50 stretched by 0.5 mm, when a mass of
250 kg is hung from its lower end. What is young's modulus of the iron rod ?
(A)
2
10
m
N
10 6 . 19
(B)
2
15
m
N
10 6 . 19
(C)
2
18
m
N
10 6 . 19
(D)
2
20
m
N
10 6 . 19
64. A load W produces an extension of 1mm in a thread of radius r. Now if the load is made 4 w and
radius is made 2r all other things remaining same the extension will becomes..............
(A) 4 mm (B) 16 mm (C) 1 mm (D) 0.25 mm
65. A steel wire of 1m long and
2
mm 1 cross sectional area is hung from rigid end when weight of 1 kg
is hung from it then change in length will be............
|
.
|

\
|
=
2
11
m
N
10 2 Y
(A) 0.5 mm (B) 0.25 mm (C) 0.05 mm (D) 5 mm
215
66. Calculate the work done, if wire is loaded by 'Mg' weight and the increase in length is 'l' ?
(A) m
g
l (B) Zero (C)
mg

(D) 2mgl
67. Two wires of same diameter of the same material having the length and 2 . If the force F is
applied on each, what will be the ratio of the work done in the two wires ?
(A) 1 : 2 (B) 1 : 4 (C) 2 : 1 (D) 1 : 1
68. A 5 meter long wire is fixed to the ceiling. A weight of 10 kg is hung at the lower end and is 1 meter
above the four. The wire was elongated by 1 mm. What is the stored in the wire due to stretching ?
(A) Zero (B) 0.05 Joule
(C) 100 Joule (D) 500 Joule
69. If the force constant of a wire is k. What is the work done in increasing the length of the wire by

?
(A)
2
k
(B)
k
(C)
2
k
2

(D)
2
k
70. Wire A and B are made from the same material. A has twice the diameter and three times the length
of B. If the elastic limits are not reached when each is stretched by the same tension, what is the ratio
of energy stored in A to that in B ?
(A) 2 : 3 (B) 3 : 4 (C) 3 : 2 (D) 6 : 1
71. A wire suspended vertically from one of its ends is stretched by attaching a weight of 200 N to the
lower and. The weight stretches the wire by 1 mm. Then what is the elastic energy stored in the wire
?
(A) 0.1 J (B) 0.2 J (C) 10 J (D) 20 J
72. A brass rod of cross sectional area 1 cm
2
and length 0.2 m is compressed length wise by a weight of
5 kg. If young's modulus of elasticity of brass is
2
11
m
N
10 1
and
2
s
m
10 g =
Then what will be
increase in the energy of rod ?
(A) J 10
5
(B) J 10 5 . 2
5
(C) J 10 5
5
(D) J 10 5 . 2
4

73. Young's modulus of the material of a wire is Y. On pulling the wire by a force F the increase in its
length is x, what is the potential energy of the stretched wire ?
(A)
Fx
2
1
(B)
Yx
2
1
(C)
2
Fx
2
1
(D) None of these
74. The work per unit volume to stretch the length by 1% of a wire with cross - sectional area 1 mm
2
will
be............
|
.
|

\
|
=
2
11
m
N
10 9 Y
(A) J 10 9
11
(B) J 10 5 . 4
7
(C) J 10 9
7
(D) J 10 5 . 4
11

216
75. A wire of length 50 cm and cross - sectional area of 1 mm
2
is extended by 1mm what will be the
required work ? ( )
2 11
Nm 10 2 Y

=
(A) J 10 6
2
(B) J 10 2
2
(C) J 10 4
2
(D) J 10 1
2

76. If a spring extends by x cm loading then what is the energy stored by the spring ? (If T is tension in
the spring & K is spring constant)
(A)
x 2
T
2
(B)
k 2
T
2
(C)
2
T
x 2
(D)
k
T 2
2
77. On stretching a wire what is the elastic energy stored per unit volume ?
(A)
F
2AL
(B)
L 2
FA
(C)
A 2
FL
(D)
2
FL
78. When a force is applied on a wire of uniform cross-sectional area
2 6
m 10 3

and length 4m, the
increase in length is 1 mm. what will be energy stored in it ?
|
.
|

\
|
=
2
11
m
N
10 2 Y
(A) 62. 50 J (B) 0.177 J (C) 0.075 J (D) 0.150 J
79. k is the force constant of a spring what will be the work done in increasing its extension form
1
to
2
be ?
(A) ( )
1 2
k (B)
( )
1 2
2
k
+
(C)
( )
2
1
2
2
k +
(D)
( )
2
1
2
2
2
k
+
80. When a 4 kg mass is hung vertically on a light spring that obeys Hook's law, the spring stretches by
2 cm what will be the work required to be done by an external agent in stretching this spring by
5 cm?
(A) 4.900 Joule (B) 2.450 Joule (C) 0.495 Joule (D) 0.245 Joule
81. The isothermal elasticity of a gas is equal to....................
(A) Density (B) Volume (C) Pressure (D) Specific heat
82. If the volume of a block of aluminium is decreased, by the pressure (stress) on its surface is increased
by.................(Bulk modulus of
2 10
Nm 10 5 . 7 A

= )
(A)
2
10
m
N
10 5 . 7
(B)
2
8
m
N
10 5 . 7
(C)
2
6
m
N
10 5 . 7
(D)
2
4
m
N
10 5 . 7
83. The specific heat at constant pressure and at constant volume for an ideal gas are
v p
C and C and
isothermal elasticities are
u |
E and E respectively. What is the ratio of
u |
E and E .
(A)
p
v
C
C
(B)
v
p
C
C
(C)
v p
C C (D)
v p
C C
1
217
84. What is the ratio of the adiabatic to isothermal elasticities of a triatomic gas ?
(A)
4
3
(B)
3
4
(C) 1 (D)
3
5
85. To what depth below the surface of sea should a rubber ball be taken as to decrease its volume by
0.1% (Take : density of sea water
3
m
kg
1000 =
Bulk modulus of rubber
2
8
m
N
10 9 =
,acceleration
due to gravity
2
s
m
10 =
)
(A) 9 m (B) 18 m (C) 180 m (D) 90 m
86. The compressibility of water
5
10 4

per unit atmospheric pressure. The decrease in volume of
100 cubic centimeter of water under a pressure of 100 atmosphere will be................
(A) CC 10 4
5
(B) CC 10 4
5
(C) 0.025 CC (D) 0.004 CC
87. If a rubber ball is taken at the depth of 200m in a pool, Its volume decreases by 0.1% . If the density
of the water is
2 3
3
s
m
10 g &
m
kg
10 1 =
. Then waht will be the volume elasticity in
?
m
N
2
(A)
8
10
(B)
8
10 2 (C)
9
10
(D)
9
10 2
88. For a constant hydraulic stress on an object, the fractional change in the object volume
|
.
|

\
|
v
v A
and
its bulk modulus (B) are related as...............
(A) 0.01 (B) 0.06 (C) 0.02 (D) 0.03
89. When a pressure of 100 atmosphere is applied on a spherical ball then its volume reduces to 0.01%
What is the bulk moduls of the material of the rubber in
2
cm
dyne
.
(A)
12
10 10 (B)
12
10 1 (C)
12
10 100 (D)
12
10 20
90. The pressure applied from all directions on a cube is p. How much its temperature should be raised
to maintain the orginal volume ? The volume elasticity. of the cube is B and the coefficient of volume
expansion is o.
(A)
o|
P
(B)
|
o P
(C)
o
|p
(D)
p
o|
91. A uniform cube is subjected to volume compression. If each side is decreased by 1% Then what is
bulk strain ?
(A) 0.01 (B) 0.06 (C) 0.02 (D) 0.03
218
92. The ratio of two specific heats of gas
v
p
C
C
for Argon is 1.6 and for hydrogen is 1.4. Adiabatic
elasticity of Argon at pressure p is E. Adiabatic elasticity of hydrogen will also be equal to E at the
pressure.
(A) p (B)
P
8
7
(C)
P
7
8
(D) 1.4 P
93. What is the isothermal bulk modulus of a gas at atmospheric pressure ?
(A) 1 mm of Hg (B) 13.6 mm of Hg
(C)
2
5
m
N
10 013 . 1
(D)
2
5
m
N
10 026 . 2
94. The bulk modulus of an ideal gas at constant temperature.........
(A) is equal to its volume V (B) is equal to P/2
(C) is equal to its pressure P (D) cannot be determined
95. A material has poisson's ratio 0.50. If uniform rod of it suffers a longitudinal strain of
3
10 2

. Then
what is percentage change in volume ?
(A) 0.6 (B) 0.4 (C) 0.2 (D) 0
96. There is no change in the volume of a wire due to change in its length on stretching. What is the
possion's ratio of the material of the wire....
(A) +0.5 (B) -0.50 (C) 0.25 (D) -0.25
97. Which statement is true for a metal......
(A) q < Y (B) q = Y (C) q > Y (D)
1
Y

<
98. Which of the following relation is true
(A) ( ) 6 1 k Y 3 = (B)
n y
gny
k
+
=
(C) ( ) Y n k 6 6 + = (D)
n
n Y 5 . 0
6

=
99. Two wires A & B of same length and of the same material have the respective radius r, & r
2
their one
end is fixed with a rigid support and at the other end equal twisting couple is applied. Then what will
we be the ratio of the angle of twist at the end of A and the angle of twist at the end of B.
(A)
2
2
2
1
r
r
(B)
2
1
2
2
r
r
(C)
4
1
4
2
r
r
(D)
4
2
4
1
r
r
100. The poisson's ratio can not have the value...........
(A) 0.7 (B) 0.2 (C) 0.1 (D) 0.5
219
101. The value of poisson's ratio lies between........
(A)
2
1
to 1
(B)
2
1
to
4
3

(C)
1 to
2
1

(D) 1 to 2
102. If the young's modulus of the material is 3 times its modulus of rigidity. Then what will be its volume
elasticity ?
(A) zero (B) infinity (C)
2
10
m
N
10 2
(D)
2
10
m
N
10 3
103. For a given material the Young's modulus is 2.4 times that of rigidity modulus. What is its poisson's
ratio ?
(A) 2.4 (B) 1.2 (C) 0.4 (D) 0.2
104. The lower surface of a cube is fixed. On its upper surface is applied at an angle of 30
0
from its
surface. What will be the change of the type ?
(A) shape (B) size (C) none (D) shape & size
105. The upper end of a wire of radius 4 mm and length 100 cm is clamped and its other end is twisted
through an angle of 30
0
. Then what is the angle of shear ?
(A) 12
0
(B) 0.12
0
(C) 1.2
0
(D) 0.012
0
106. Mark the wrong statement.
(A) Sliding of moleculawr layer is much easier than compression or expansion.
(B) Receiprocal of bulk modulus is called compressibility.
(C) Twist is difficult in big rod as compared to small rod.
(D) Which is more strong out of hollow and solid cylinder having equal length and mass ?
107. A 2m long rod of radius 1 cm which is fixed from one end is gien a twist of 0.8 radians. What will be
the shear strain developed ?
(A) 0.002 (B) 0.004 (C) 0.008 (D) 0.016
108. Shearing stress causes change in
(A) length (B) breadth (C) shape (D) volume
109. What is the relationship between Young's modulus Y, Bulk modulus k and modulus of rigidity q ?
(A)
k 3
k 9
Y
+ q
q
=
(B)
k 3 y
k y 9
Y
+
=
(C)
k 3
k 9
Y
+
q
=
(D)
k 9
k 3
Y
+ q
q
=
110. What is the possible value of posson's ratio ?
(A) 1 (B) 0.9 (C) 0.8 (D) 0.4
111. The graph shown was obtained from experimental
measurements of the period of oscillations T for different
masses M placed in the scale pan on the lower end of the
passing through the origin is that the......
(A) Spring did not obey Hooke's law
(B) Amplitude of the oscillations was large
(C) Clock used needed regulating
(D) Mass of the pan was neglected
220
112. A graph is shown between stress and strain
for metals. The part in which Hooke's law
holds good is
(A) OA (B) AB
(C) BC (D) CD
113. The strain-stress curves of three wires of different materials
are shown in the figure. P, Q and R the elastic limits of the
wires the figure shows that
(A) Elasticity of wire P is maximum.
(B) Elasticity of wire Q is maximum.
(C) Tensile strength of R is maximum
(D) none of above
114. The diagram shows a force extension graph for a Rubber
band consider the following statements.
(I) It will be easier to compress these
rubber than expand it.
(II) Rubber does not return to its original
length after it is stretched.
(III) The rubber band will get heated if it
is stretched and relased.
Which of these can be deduced from the graph.
(A) III only (B) II and III (C) I and III (D) I only
115. The stress versus strain graph for wires of two material A
& B are as shown in the figure. If Y
A
& Y
B
are the young's
modulus of the materials then
(A) Y
B
= 2Y
A
(B) Y
A
= Y
B
(C) Y
B
= 3Y
A
(D) Y
A
= 3Y
B
116. The load versus elongation graph for four wires of the same
material is shown in the figure. The thickest wire is
represented by the line.
(A) OD (B) OC
(C) OB (D) OA
221
117. The adjacent graph shows the extension of a wire of length
1m suspended from the top of a roof at one end with the
load W connected to the other end. If the cross sectional
area of the wire is 10
-6
m
2
, calculate the young's modulus
of the material of the wire.
(A)
2
11
m
N
10 2
(B)
2
11
m
N
10 2

(C)
2
12
m
N
10 3

(D)
2
12
m
N
10 2

118. The graph is drawn between the applied force F and the
strain (x) for a thin uniform wire the wire behaves as a
liquid in the part.
(A) ab (B) bc
(C) cd (D) oa
119. The graph shows the behaviour of a length of wire in the
region for which the substance obey's Hooke's law. P &
Q represent.
(A) p = applied force, Q = extension
(B) p = extension, Q = applied force
(C) p = extension, Q = stored elastic energy
(D) p = stored elastic energy, Q = extension
120. The potential energy U between two nmolecules as a
function of the distance x between them has been shown
in the figure. The two molecules are.
(A) Attracted when x lies between A & B
and are repelled when x lies between B & C.
(B) Attracted when x lies between B and
C and are repelled when x lies between A and B.
(C) Attracted when they reach B.
(D) Repelled when they reach B.
121. The value of force constant between the applied elastic
force F and displacement will be
(A)
3
(B)
3
1
(C)
2
1
(D)
2
3
222
122. The diagram shows stress v/s strain curve
for the materials A and B. From the curves
we infer that
(A) A is brittle but B is ductile
(B) A is ductile and B is brittle
(C) Both A & B are ductile
(D) Both A & B are brittle
123. Which are of the following is the young's
modulus (in N/m
2
) for the wire having the
stress strain curve in the figure.
(A)
11
10 24
(B)
11
10 0 . 8
(C)
11
10 10
(D)
11
10 0 . 2
124. The diagram shows the change x in the
length of a thin uniform wire caused by the
application of stress F at two different
temperature T
1
& T
2
. The variation
(A)
2 1
T T >
(B)
2 1
T T <
(C)
2 1
T T =
(D) None of these
125. The point of maximum and minimum
attraction in the curve between potential
energy (U) and distawnce (r) of a diatomic
molecules are respectively.
(A) S and R (B) T and S
(C) R and S (D) S and T
Assertion and Reason :
Read the assertion and reason carefully to mark the correct option out of the option given below
(A) If both assertion and reason are true and reason is the correct explanation of the assertion.
(B) If both assertion and reason are true but reason is not the correct explanation of the assertion.
(C) If assertion is true but reason is false.
(D) If assertion and reason both are false.
223
126. Assertion : The stretching of a coil is determined by its shear modulus.
Reason : Shear modulus change only shape of a body keeping its dimensions unchanged.
(A) a (B) b (C) c (D) d
127. Assertion : The bridges are declared unsafe after a long use.
Reason : Elastic strength of bridges decrease with
(A) a (B) b (C) c (D) d
128. Assertion : Two identical solid balls, one of ivory and the other of wet clay are dropped from the
same height on the floor. Both the balls will rise to same height after.
Reason : Ivory and wet-clay have same elasticity.
(A) a (B) b (C) c (D) d
129. Assertion : Young's modulus for a perfectly plastic body is zero.
Reason : For a perfectly plastic body is zero.
(A) a (B) b (C) c (D) d
130. Assertion : Identical spring of steel and copper are equally stretched more will be done on the steel
spring.
Reason : Steel is more elastic than copper.
(A) a (B) b (C) c (D) d
131. A wire is stretched to double the length which of the following is false in this context ?
(A) Its volume increased (B) Its longitudinal strain is I
(C) Stress = Young's modlus (D) Stress = 2x Young's modulus
132. Which is the dimensional formula for modulus of rigidity ?
(A)
2 1 1
T L M

(B)
2 1 1
T L M

(C)
1 2 1
T L M

(D)
2 2 1
T L M

133. When more than 20 kg mass is tied to the end of wire it breaks what is maximum mass that can be
tied to the end of a wire of same material with half the radius ?
(A) 20 kg (B) 5 kg (C) 80 kg (D) 160 kg
134. When 100 N tensile force is applied to a rod of 10
-6
m
2
cross-sectional area, its length increases by
1% so young's modulus of material is..........
(A) 10
12
Pa (B) 10
11
Pa (C) 10
10
Pa (D) 10
2
Pa
135. A composite wire is made by joining ends of two wires of equal dimensions, one of copper and the
other of steel. When a weight is sttached to its end the ratio of increase in their length is .........
Copper Steel
Y
7
20
Y =
(A) 20.7 (B) 10.7 (C) 7:20 (D) 1:7
136. A rubber ball when taken to the bottom of a 100 m deep take decrease in volume by 1% Hence, the
bulk modulus of rubber is............
|
.
|

\
|
=
2
s
m
10 g
(A) 10
6
Pa (B) 10
8
Pa (C) 10
7
Pa (D) 10
9
Pa
224
137. Young's modulus of a rigid body is
(A) 0 (B) 1 (C)

(D) 0.5
138. Pressure on an object increases from . Pa 10 165 . 1 to Pa 10 01 . 1
5 5
He volume decrease by
10% at constant temperature. Bulk modulus of material is........
(A) Pa 10 55 . 1
5
(B) Pa 10 2 . 51
5

(C) Pa 10 4 . 102
5
(D) Pa 10 8 . 204
5

139. Cross-sectional area oif wire of length L is A. Young's modulus of material is Y. If this wire acts as a
spring what is the value of force constant ?
(A)
L
YA
(B)
L 2
YA
(C)
L
YA 2
(D)
A
YL
Surface Tension
140. Writing on black board with a pieace of chalk is possible by the property of
(A) Adhesive force (B) Cohesive force (C) Surface force (D) Viscosity
141. When there is no external force, the shape of liquid drop is determined by
(A) Surface tension of liquid (B) Density of Liquid
(C) Viscosity of liquid (D) Tempreture of air only
142. Soap helps in cleaning because
(A) chemicals of soap change
(B) It increase the surface tension of the soluiton.
(C) It absorbs the dirt.
(D) It lowers the surface tension of the solution
143. A beaker of radius 15 cm is filled with liquid of surface tension 0.075 N/m. Force across an imaginary
diameter on the surface ofliquid is
(A) 0.075 N (B) N 10 5 . 1
2

(C) 0.225 N (D) N 10 25 . 2


2

144. A square frame of side L is dipped in a liquid on taking out a membrance is fomed if the surface
tension of the liquid is T, the force acting on the frame will be.
(A) 2 TL (B) 4 TL (C) 8 TL (D) 10 TL
145. The force required to separate two glass plates of area 10
-2
m
2
with a film of water 0.05 mm thick
between them is (surface tension of water is
m
N
10 70
3

)
(A) 28 N (B) 14 N (C) 50 N (D) 38 N
225
146. Surface tension of a liquid is found to be infuenced by
(A) It increases with the increase of temprature.
(B) Nature of the liquid in contact.
(C) Presense of soap that increaase it.
(D) Its variation with the concentration of the liquid.
147. A thm metal disc of radius r floats on water surface B and bends the surface down wards along the
perimeter making an angle Q with vertical edge of the disc. If the disc displaces a weight of water W
and surface tension of water is T, then the weight at metal dis
(A) w rT 2 + t (B) w cos rT 2 u t
(C) w cos rT 2 + u t (D) u t = cos rT 2 w
148. A thin liquid film formed between a u-shaped wire and
a light slider supports a weight of N 10 5 . 1
2
(see
figure). The length of the slider is 30 cm and its weight
negligible. The surface tension of the liquid film is.
(A)
1
m N 0125 . 0

(B)
1
m N 1 . 0

(C)
1
m N 05 . 0

(D)
1
m N 025 . 0

Surface Energy
149. Radius of a soap bubble is 'r', surface tension of soap solution is T. Then without incresing the
temprature how much energy will be needed to double its radius.
(A) T r 4
2
t (B) T r 2
2
t
(C) T r 12
2
t (D) T r 24
2
t
150. The amount of work done in blowing a soap bubble such that its diameter increases from d to D is
(T = Surface tension of solution)
(A) ( ) T d D 4
2 2
t (B) ( ) T d D 8
2 2
t
(C) ( ) T d D
2 2
t (D) ( ) T d D 2
2 2
t
151. A soap bubble of radius r is blown up to form a bubble of radius 2r under isothermal conditions if the
T is the surface tension of soap solution the energy spent in the slowing is.
(A)
2
Tr 3t
(B)
2
Tr 6t
(C)
2
Tr 12t
(D)
2
Tr 24t
152. The surface tension of a liquid is 5 N/m. If a thin film of the area 0.02 m
2
is formed on a loop, then
its surface energy will be
(A) J 10 5
2
(B) J 10 5 . 2
2
(C) J 10 2
1
(D) J 10 5
1

226
153. A frame made of a metalic wire enclosing O surface area A is covered with a soap film. If the area
of the frame metalic wire is reduced by 50% the energy of the soap film will be changed by
(A) 100% (B) 75% (C) 50% (D) 25%
154. Two small drops mercury, each of radius R, coaless the form a single large drop. The ratio of the
total surface energies before and after the change is.
(A) 2
1
2 : 1
(B)
1 : 2
2
1
(C) 2 : 1 (D) 1 : 2
155. The work done is blowing a soap bubble of 10 cm radius is
(surface tension od the soap solution is
m / N
100
3
)
(A) Joule 10 36 . 75
4
(B) Joule 10 68 . 37
4

(C) Joule 10 72 . 150


4
(D) Joule 36 . 75
156. The work done increasing the size of a soap film drom cm 11 cm 10 i s Joule 10 3
4
. The
surface tension of the film is
(A)
m
N
10 5 . 1
2

(B)
m
N
10 0 . 3
2

(C)
m
N
10 0 . 6
2

(D)
m
N
10 0 . 11
2

157. A big drop of radius R is formed by 1000 small droplets of coater then the radius of small drop is
(A)
2
R
(B)
5
R
(C)
6
R
(D)
10
R
158. 8000 identioal water drops are combined to form a bigdrop. Then the ration of the final surface
energy to the intilial surface energy of all the drops together is
(A) 1 : 10 (B) 1 : 15 (C) 1 : 20 (D) 1 : 25
159. The relation between surface tension T. Surface area A and surface energy E is given by.
(A)
A
E
T =
(B) EA T = (C)
A
T
E =
(D)
E
A
T =
Angle of Contact
160. If a coater drop is kept between two glasses plates then its shape is
(A) (B) (C)
(D) None of these
227
161. A liquid wets a solid completely. The menisions of the liquid in a surfficently long tube is
(A) Flat (B) Concave (C) Convex (D) Cylindrical
Pressure Difference
162. When two soap bubbles of radius r
1
and r
2
(r
2
> r
1
) coalesce, the radius of curvature of common
surface is...........
(A)
1 2
r r (B)
2 1
1 2
r r
r r
(C)
1 2
2 1
r r
r r

(D)
1 2
r r +
163. The excess of pressure inside a soap bubble than that of the other pressure is
(A)
r
T 2
(B)
r
T 4
(C)
r 2
T
(D)
r
T
164. The radill of two soap bubbles are r
1
and r
2
. In isothermal conditions two meet together is vacum
Then the radius of the resultant bubble is given by
(A) ( ) 2 / r r R
2 1
+ = (B) ( )
3 2 1 1
r r r r R + + =
(C)
2
2
2
1
2
r r R + =
(D)
2 1
r r R + =
165. A spherical drop of coater has radius 1 mm if surface tension of contex is m / N 10 70
3
difference
of pressures between inside and outside of the spherical drop is
(A)
2
m
N
35
(B)
2
m
N
70
(C)
2
m
N
140
(D) zero
166. In capilley pressure below the curved surface at water will be
(A) Equal to atomospheric (B) Equal to upper side pressure
(C) More than upper side pressure (D) Lesser than upper side pressure
167. Two bubbles A and B (A>B) are joined through a narrow tube than
(A) The size of a will increase
(B) The size of B will increase
(C) The size of B will increase untill thenpressure equals
(D) None of these
168. If the excess pressure inside a soap bubble is balanced by oil column of height 2 mm then the surface
tension of soap solution will be.
Cr = 1 and density d = 0.8 gm/cc)
(A)
m
N
9 . 3
(B)
m
N
10 9 . 3
2

(C)
m
N
10 9 . 3
3

(D)
m
N
10 9 . 3
1

Capillarity
169. A capillary tube at radius R is immersed in water and water rises in it to a height H. Mass of water
in the capillary tube is M. If the radius of the tube is doubled. Mass of water that will rise in the
capillary tube will now be
(A) M (B) 2M (C) (D) 4M
228
170. A vesel whose bottom has round holes with diametre of 0.1 mm is filled with water. The maximum
height to which the water can be filled without leakage is
(S.T. of water =
cm
dyne 75
=
2
s
m
1000 g =
)
(A) 100 cm (B) 75 cm (C) 50 cm (D) 30 cm
171. The correct relation is
(A)
hdg
cos T 2
r
u
=
(B)
u
=
cos T 2
hdg
r
(C)
u
=
cos
dgh T 2
r
(D)
hdg 2
cos T
r
u
=
172. In a capillary tube water rises by 1.2 mm. The height of water that will rise in another capillary tube
having half the radius of the first is
(A) 1.2 mm (B) 2.4 mm (C) 0.6 mm (D) 0.4 mm
173. Water rises in a vertical capillary tube upto a beight of 2.0 cm. If tube is inclined at an angle of 60
0
with the vertical then the what length the water will rise in the tube.
(A) 2.0 cm (B) 4.0 cm (C)
cm
3
4
(D)
cm 2 2
174. The lower end of a glass capillary tube is dipped in water rises to a height of 8 cm the tube is then
broken at a height of 6 cm. The height of water column and engled as contact will be
(A)
1
3
6 cm . sin
4

(B)
1
4
6 cm . sin
5

(C)
4
3
cos . cm 6
1
(D)
1
1
6 cm . sin
2

175. A large number of water drops each of rdius r combine to have a drop of radius R. If the surface
tension is T and the mechanical equllvalent at heat is J then the rise in tempreature will be
(A)
rJ
T 2
(B)
RJ
T 3
(C)
|
.
|

\
|

R
1
r
1
J
J 3
(D)
|
.
|

\
|

R
1
r
1
J
T 2
Caraphical Question
176. The correct curve between the height or depression h of liquid in a capillary tube and its radius is
229
177. A soap bubble is blown with the help of a mechanical pump at the mouth-of a tube the pump
produces a certion increase per minit in the volume of the bubble irrespective of its internal pressure
the graph between the pressure inside the soap bubble and time t will be
178. Which graph present the variation of surface tension with temperature over small temperature ranges
for coater.
Comprehension type question
Passage - I
When liquid medicine of desting S is to be put in the eye. It is done with the help of a dropper
as the bulb on the top of the dropper is pressed a drop forms at the opening od the dropper we wish
to estimate the size of the drop. We dirst assume that the drop formed at the opening is spherical
because the requires a minimum increase in its surface energy. To determine the size we calculate the
net vertical force due to surface tension T when the radius od the drop is R. When this force
becomes smaller than the weight of the drop the drop gets detched from the dropper.
179. If the radius od the opening od the dropper is r ; the vertical force due to the surface tension on the
drop of radius R. (cassuming r << R) is
(A) T r 2t (B) T R 2t (C)
R
T r 2
2
t
(D)
r
T R 2
2
t
180. If
2 3 3 4
ms 10 m kg 10 , m 10 5

= = = T = 0.11 N m
-1
the radius of the drop when it
detaches from the dropper is approximately
(A) m 10 4 . 1
3
(B) m 10 3 . 3
3
(C) m 10 0 . 2
3
(D) m 10 1 . 4
3

230
181. After the drop detaches its surface energy is
(A) J 10 4 . 1
6
(B) J 10 7 . 2
6
(C) J 10 4 . 5
6
(D) J 10 1 . 8
6

Assertion & Reason type questions


Read the assertion and reason carefully and mark the correct option given below.
(a) If both assertion and reason are true and the reason is the correct explanation of the assertion.
(b) If both assertion and reason are true but reason is not the correct explanation of the assertion.
(c) If assertion is true but reason is false.
(d) If the assertion and reason both are false.
182. Assertion : It is better to wash the clothes in cold soap solution.
Reason : The surface tension of cold solution is more then the surface tension of hot solution.
(A) a (B) b (C) c (D) d
183. Assertion : When height of a tube is less then liquid rise in the capillary tube the liquid does not
overtow.
Reason : Product of radius of meniscus and height of liquid incapilling tube always remains constant.
(A) a (B) b (C) c (D) d
184. Assertion : The impurities always decrease the surface tension of a liquid.
Reason : The change in surface tension of the liquid depends upon the degree of cont amination of
the impurity.
(A) a (B) b (C) c (D) d
185. Assertion : The concept of surface tension is help only for liquids.
Reason : Surface tension does not hold for gases.
(A) a (B) b (C) c (D) d
186. Assertion : The water rises higher in a capillary tube of small diametre than in the capillary tube of
large diamtre.
Reason : Height through which liquid rises in a capillay tube is inversely proportional to the diameter
of the capillary tube.
(A) a (B) b (C) c (D) d
187. Assertion : Tiny drops of liquid resist deforming forces better than bigger drops.
Reason : Excess pressure inside a drop is directly proportional to surface tension.
(A) a (B) b (C) c (D) d
Pressure and Density
188. When a large bubble rises drom the bottom of a lake to the surface. Its radius double S. It atmospheric
pressure in euqal to that of colurnn of colurnn of water height H then the depth of Lake is
(A) H (B) 2H (C) 7H (D) 8H
231
189. A trangular lamind of area A and, height h is immersed in a liquid of density S in a vertical plane with
its base on the surface of the liquid. The thrust on lamina is
(A)
gh A
2
1

(B)
gh A
3
1

(C)
gh A
6
1

(D)
gh A
3
2

190. The density S of coater of bulk modulus B at a depth y in the ocean is related to the density at
surface so by the relation.
(A)
(


=
B
gy
1
0
0 (B)
(


+ =
B
gy
1
0
0
(C)
(


+ =
B
gyh
1
0
0 (D) (

=
gy
B
1
0
0
191. By sucking through a straw, a student can reduce the pressure in his lungs to 750 mm of Hg (density
3
cm
gm
6 . 13 =
) using the straw, he can drink water from a glass up to a maximum depth of
(A) 10 cm (B) 75 cm (C) 13.6 cm (D) 1.36 cm
192. The pressure on a swimmer 20 m below the surface of coater at sea level is
(A) 1.0 atm (B) 2.0 atm (C) 2.5 atm (D) 3.0 atm
Pascal's Law and Archimedes Principal
193. A spherical solid ball of volume V is made of a material of density S. It is falling through a liquid of
density S
2
(S
2
< S
1
). Assume that the liquid opplies a viscous force on the ball that is proportional to
square of its speed V. i.e. Fviscous = - KV
2
(K > 0) The terminal speed of the ball is
(A)
vg
K
1

(B)
vg
K
1

(C)
( )
1 2
g v
K

(D)
( )
1 2
g v
K

194. The fraction of floating object of volume V
O
and density do above the surface of a Liquid as density
d will be
(A)
d
d
0
(B)
0
0
d d
d d
+
(C)
d
d d
0

(D)
0
0
d d
d d

195. A body floats in water with one-thired od its volume above the surface of water. It is placed in oil it
floats with half of : Its volume above the surface of the oil. The specific gravity od the oil is.
(A)
3
5
(B)
3
4
(C)
2
3
(D) 1
196. If there were no gravity which of the following will not be there for a fluid.
(A) Viscosity (B) Surface tension
(C) pressure (D) Archime des's upward thrust
232
197. A piece of solid weighs 120 g in air, 80 g in water and 60 g in liquid the relative density of the solid
and that of the solid and that of the liquid are respectively.
(A) 3, 2 (B)
4
3
, 2
(C)
2 ,
4
3
(D)
2
3
, 3
198. Ice pieces are floating in a beaker A containing watre and also in a beakre B containing miscible
liquid of specific gravity 1.2 Ice melts the level of
(A) water increases in A (B) water decreases in A
(C) Liquid in B decrease B (D) Liquid in B increase
Fluid Flow
199. An engine pumps water continuously through a hose water leares the hose with a velocity V and m
is the mass per unit length of the watre Jet what is the rate at which kinetic energy is imperted to
water.
(A)
3
mv
2
1
(B)
3
mv (C)
2
mv
2
1
(D)
2
v mv
2
1
o
200. The height of the dam in an hydro electric power station is 10 m. In order to generate 1 MW of
electric power, the mass of water (in kg) that must full per second on the brades of turbine is
(A) 10
6
(B) 10
5
(C) 10
3
(D) 10
4
201. Eight drops of a liquid of density 3 and each of radius a are fallingt through air with a constant
velocity 3.75 cm S
1
when the eight drops coalesce to form a single drop the terminal velocity of the
new drop will be
(A)
1 2
ms 10 15

(B) s / m 10 4 . 2
2
(C)
1 2
ms 10 75 . 0

(D) s / m 10 25
2

202. A cylinder of height 2.0 m is completely filled with water. The velocity of efflux of water cim m/s
through a small hole on the side will of the cylinder mear its bottom is
(A) 10 (B) 20 (C) 25.5 (D) 5
203. There is a hole in the bottom of tank having water. If total pressure at bottom 3 atm (1 atm ) then the
velocity of water flowig drom hole is
(A)
s
m
400
(B)
s
m
60
(C)
s
m
600
(D) None of these
204. Two drops of the same radius are falling through air with a steady velocity for 5 cm per sec. If the
two drops coakesce the terminal velocity would be
(A) 10 cm per sec (B) 2.5 cm per sec
(C)
( ) sec per cm 4 5 3
1

(D)
sec per cm 2 5
205. An application of Bernouli's equation for liquid flow is found in
(A) Dynamic lift of an aeroplane (B) Viscocity metere
(C) Capillary rise (D) tly dulic press
233
206. An L-S
p
aped tube with a small office is held in a water
stream as shwon in fig. The upper end of the tube is
10.6 cm above the surface of water. What will be the
height of the set of water coming from the office velocity
of water stream is 2.45 m/s.
(A) zero (B) 20.0 cm
(C) 10.6 cm (D) 40 cm
207. A tank is filled with water up to a height H. Water is
allowed to come out of a hole P in one of the walls at a
depth D below the surface of water express the
horizontal distunce x in terms of H and D.
(A) ( )
o
= D H D x (B)
( )
o

=
2
D H D
x
(C) ( )
2
D H D x = (D) ( ) D H D 4 x =
208. An incomepressible fluid flows steadily through a cylinderical pipe which has radius 2r at point A and
radius r at B further along the flow direction. It the velocity at point A is V, its velocity at point B.
(A) 2v (B) v (C)
2
v
(D) 4v
209. If the terminal speed of a sphere of gold
|
.
|

\
|
=
3
m
kg
5 . 19 density
is 2.0 m/s in a viscous liquid
|
.
|

\
|
=
3
m
kg
5 . 1 density
find the terminal speed of a sphere of siher
|
.
|

\
|
=
3
m
kg
5 . 10 density
of the
same size in the same liquid.
(A)
s
m
133 . 0
(B)
s
m
2 . 0
(C)
s
m
1 . 0
(D)
s
m
4 . 0
210. Two solid spheres of same metal but of mass M and 8M full simutineously on a viscous liquid and
their terminal velocity are V and 'nv' then value of 'n' is
(A) 16 (B) 8 (C) 4 (D) 2
211. Two metal Spheres are falling through a liquid of density
3
3
m
kg
10 2
with the same uniform speed
the meterical density at sphere 1 and sphere 2 are
3
3
m
kg
10 8
and
3
3
m
kg
10 11
respectively. The
ration of their radii is.
(A)
8
11
(B)
8
11
(C)
2
3
(D)
2
3
234
212. Water is flowing continuously dram a temp having an internal diameter m 10 8
3
. The water velocity
as it leaves the tap is 0.4 m/s. The diameter of the water stream at a distance m 10 2
1
below the
tap is close to
(A) m 10 0 . 5
3
(B) m 10 5 . 7
3
(C) m 10 6 . 9
3
(D) m 10 6 . 3
3

213. A large open tank has two holes in the wall one is a square hole of side L ata depth y froam the top
and the other is a circular hole of radius R at a depth ay from the top. When the tank is completely
filled with water the quantities of water following out per second from both the holes are the some
then R is equal to
(A) L 2t (B)
t 2
L
(C) L (D)
t 2
L
214. A block of ice floats on a liquid of density 1.2 in a beaker then level of liquid when ice completely
melt.
(A) Remains same (B) Rises (C) Lowers (D) (A) (B) or (C)
Assertion & Reason type questions
Read the assertion and reason carefully to mark the correct option out of the options given below
(a) If both assertion and reason are true and the reason is the correct explanation of the assertion.
(b) If both assertion and reason are true but reason is not the correct explanation of the assertion.
(c) If assertion is true but reason is false.
(d) IF the assertion and reason are false.
(e) If assertion is flase but reason is true.
215. Assertion : The blood pressure in humans is greater at the feet than at the brain.
Reason : Pressure of liquid at any point is proportional to height clensity of liquid.
(A) a (B) b (C) c (D) d
216. Assertion : To empty an oil tank two holes so it will made.
Reason : Oil will come out of two holes so it will be emptied faster.
(A) a (B) b (C) c (D) d
217. Assertion : A bubble comes from the bottom of a lake to the top.
Reason : Its radius increases.
(A) a (B) b (C) c (D) d
Ordinary Thinking
Thermometary
218. Oxygen boils at 183
0
C. This temperature is approximately.
(A) 215
0
F (B) -297
0
F (C) 329
0
F (D) 361
0
F
235
219. The resistance of a rasistance thermometer has values 2.71 and 3.70 ohm at 10
0
C and 100
0
C. The
temprature at which the resistance is 3.26 ohm is
(A) 40
0
C (B) 50
0
C (C) 60
0
C (D) 70
0
C
220. Maximum density of H
2
O is at the temprature.
(A) 32
0
F (B) 39.2
0
F (C) 42
0
F (D) 4
0
F
221. At what temprature the centigrade (celsius) and Fahrenheit readings at the same.
(A) -40
0
(B) +40
0
C (C) 36.6
0
(D) -37
0
C
222. Mercury thermometers can be used to measure tempratures up to
(A) 100
0
C (B) 212
0
C (C) 360
0
C (D) 500
0
C
223. If temperature of an object is 140
0
F then its temperature in centigratde is
(A) 105
0
C (B) 32
0
C (C) 140
0
C (D) 60
0
C
224. When the room temprature becomes equal to the dew point the relative humidity of the room is
(A) 100 % (B) 0 % (C) 70 % (D) 85 %
225. If the length of a cylinder on heating increases by 2% the area of its base will increase by.
(A) 0.5 % (B) 2 % (C) 1 % (D) 4 %
226. Density of substance at 0
0
C is 10 gm/cc and at 100
0
Cits density is 9.7 gm/CC. The coefficient of
linear expansion of the substance will be
(A) 10
2
(B) 10
-2
(C) 10
-3
(D) 10
-4
227. A beaker is completely filled with water at 4
0
C It will overflow if
(A) Heated above 4
0
C
(B) Cooled below 4
0
C
(C) Both heated and cooled above and below 4
0
C respectively
(D) None of these
228. An iron bar of length 10m is heated from 0
0
C to 100
0
C. If the coefficient of linear thermal expansion
of iron is
C
10 10
6

the increase in the length of bar is


(A) 0.5 cm (B) 1.0 cm (C) 1.5 cm (D) 2.0 cm
Calorimetry
229. Melting point of ice.....
(A) Increases with increasing pressure (B) Decreases with increasing pressure
(C) Is independent of pressure (D) is proportional of pressure
230. Amount of heat required to raise the temprature of a body through 1k is called it is
(A) Water equivalent (B) Thermal capacity
(C) Entropy (D) Specific heat
231. A vessel contains 110 g of water the heat capacity of the vessel is equal to 10 g of water The initical
temprature of water in vessel is 10
0
C If 220 g of hot water at 70
0
C is poured in the vessel the Final
temperature meglecting radiation loss will be
(A) 70
0
C (B) 80
0
C (C) 60
0
C (D) 50
0
C
236
232. In a water fall the water falls from a height of 100 cm. If the entire K.E. of water is converted in to
heat the rise in temperature of water will be
(A) 0.23
0
C (B) 0.46
0
C (C) 2.3
0
C (D) 0.023
0
C
233. The temprature at which the vapour pressure of a liquid becomes equals of the external pressure is
its.
(A) Melting point (B) sublimation point
(C) Critical temprature (D) Boiling point
234. 10 g of ice at 0
0
C is mixed with 100 g of water at 50
0
C what is the resultant temprature of mixture.
(A) 31.2
0
C (B) 32.8
0
C (C) 36.7
0
C (D) 38.2
0
C
Critical Thinking
235. Two metal strips that constituate a thermostant must necessarily in their
(A) Mass (B) Length
(C) Resistivity (D) Coefficient of liner expansion
236. 2 kg of Ice at -20
0
C is mixed with 5 kg of water at 20
0
C in an insulating vessel having a megligible
heat capacity calculate the final mass of water remaining in the container. It is given that the specific
heats of water and ice care 1 keal/kg per
0
C and 0.5 Keal/kg 1
0
C while the latent heat of fusion of
ice is 80 kcoil/kg.
(A) 7 kg (B) 6 kg (C) 4 kg (D) 2 kg
237. A lead bullet at 27
0
C just melts when stopped by an obstancle Assuming that 25% of heat is
obsorbed by the obstacle then the velocity of the bullet at the time of striking.
[M.P. of lead = 327
0
C, specific heat of lead = 0.03 cal/x latent heat of fusion of lead = 6 cal/g and
J = 4.2 Jute / cal]
(A) 410 m/s (B) 1230 m/s (C) 307.5 m/s (D) None of these
238. An electric kettle takes 4A current at 220V How much time will it take to boil 1 kg of water from
temprature 2
0
C ? The temprature of boiling water is 10
0
C.
(A) 12.6 min (B) 4.2 min
(C) 6.3 min (D) 8.4 min
Graphical options
239. Ablock of ice at -10
0
C is slowly heated and covered to steam at 100
0
C which of the following
curves represents the phenomenon qualitatively.
237
240. The variation of density of water with temprature is represented by the
Assertion & Reason
Read the assertion and reason carefully to mark the correct option out of the option given below.
(a) If both asseration and reason are true and the reason is the correct explanation of the reason.
(b) If both assertion and reason are true but reason is not the correct explanation of the assertion.
(c) If assertion is true but reason is false.
(d) If the assertion and reason both are false.
(e) If assertion is false but reason is true.
241. Assertion : The melting point of the ice decreases with increases pressure
Reason : Ice contracts on melting
(A) a (B) b (C) c (D) d
242. Assertion : Fahrenheit is the smallest unit measuring temprature.
Reason : Fahrenheit was the first temprature scale used for measuring temprature.
(A) a (B) b (C) c (D) d
243. Assertion : Melting of solid causes no change in internal energy.
Reason :Latent heat is the heat required melt a unit mass of solid.
(A) a (B) b (C) c (D) d
244. Assertion : Specific heat capacity is the cause of formation of land and sea breeze.
Reason : The specific heat of water is more then land.
(A) a (B) b (C) c (D) d
245. Assertion : The moleculs of 0
0
C ice and 0
0
C water will have same potential energy.
Reason : Potential energy depends only on temprature of the system.
(A) a (B) b (C) c (D) d
246. Assertion : A beaker is completely filled with water at 4
0
C. It will overflow both where heated or
cooled.
Reason : There is expansion of water below and above 4
0
C.
(A) a (B) b (C) c (D) d
238
Answers Key
1 D 42 C 83 B 124 A 165 C 206 B
2 B 43 A 84 B 125 D 166 D 207 C
3 D 44 B 85 D 126 A 167 A 208 D
4 B 45 D 86 A 127 A 168 B 209 B
5 C 46 A 87 D 128 D 169 B 210 C
6 B 47 D 88 B 129 A 170 D 211 D
7 D 48 A 89 C 130 A 171 A 212 D
8 D 49 A 90 A 131 D 172 B 213 B
9 C 50 B 91 D 132 B 173 B 214 B
10 C 51 D 92 B 133 B 174 C 215 A
11 A 52 B 93 C 134 C 175 C 216 C
12 B 53 D 94 C 135 A 176 B 217 A
13 D 54 C 95 B 136 B 177 A 218 B
14 B 55 B 96 A 137 C 178 B 219 B
15 B 56 A 97 C 138 A 179 C 220 B
16 B 57 A 98 D 139 A 180 A 221 A
17 B 58 D 99 C 140 A 181 B 222 C
18 D 59 A 100 A 141 A 182 E 223 D
19 D 60 C 101 A 142 D 183 A 224 A
20 D 61 C 102 B 143 D 184 C 225 D
21 B 62 B 103 D 144 C 185 B 226 D
22 B 63 A 104 D 145 A 186 A 227 C
23 C 64 C 105 B 146 D 187 B 228 B
24 B 65 C 106 C 147 C 188 C 229 B
25 C 66 C 107 B 148 D 189 B 230 B
26 D 67 A 108 C 149 D 190 B 231 D
27 B 68 B 109 A 150 D 191 C 232 A
28 D 69 C 110 D 151 D 192 B 233 D
29 A 70 B 111 D 152 C 193 D 234 D
30 A 71 A 112 A 153 C 194 C 235 D
31 B 72 B 113 D 154 B 195 B 236 B
32 D 73 A 114 A 155 A 196 D 237 A
33 D 74 B 115 D 156 B 197 D 238 C
34 D 75 C 116 A 157 D 198 D 239 A
35 D 76 B 117 A 158 C 199 A 240 A
36 D 77 A 118 B 159 A 200 D 241 A
37 A 78 C 119 C 160 C 201 A 242 C
38 C 79 D 120 B 161 B 202 B 243 E
39 C 80 B 121 B 162 C 203 A 244 A
40 A 81 C 122 B 163 B 204 C 245 D
41 D 82 B 123 D 164 C 205 A 246 A
239
Hint
1.
(

= = = =
A
V
L L A V
y A
FV
Ay
FL
2

2
2
A F
V
y A
F o =

As cross sectional area of 2
nd
wire is 3 times therefore 9F force is required for same elongation.
2. Stress
L
stress strain

o o
3.
2
d
L
A
L L
A
F
Y
t
o o =

2
d
L
o
(As F and Y are constant)
The ratio of
2
d
L
is maximum for case CD
4. Young modulus of wire does not vary with dimension of wire. It is the property of given material.
5. Depression in beam.
2
3 3
wL
4Y bd
o =
1
Y
oo
6.
2
1
L o
r
If radius of the wire is doubled then increament in length will become 1/4 times, i.e.
mm 3
4
12
=
7.
YA
F
L
=

8.

A
=
A
F
Y Given stress
2
8
m
N
10 18 . 3
F/A

Y
l
A
A =
9. Here
2
k
KQ
p
=
According to Hooke's law
p p p
x k F =
Q p
F F = (Given)
( ) 1 ....
kP
kQ
x
x
Q
p
=
240
Energy stored in a spring is
(

= = =
2
kp
kQ
2
1
p k
Q k
kQ
kp
Q x
P x
k
k
U
U
2
2
2
2
q
p
Q
p


2
E
2
UQ
U
P
= =
10. Breaking force o area of crossection of ( )
2
r t wire
If radius of wire is doubled then breaking force will become four times.
11.
2
2
L
Y
=
12. Young's modulus of material
strain al Longitudin
stress Linear
=
If longitudinal strain is equal to unity, then
Y = Linear stress produced
13. Energy stored per unit volume
strain stress
2
1
=
14.
V
stress . Max
strain Max
Strain
stress
Y = =

Y
A
mg
strain Max =
15.
( )
m 10 6 . 9
10 5 2
8 . 9 5 . 1 10 8
Y 2
dg L
11
8
2
2 2

=


= =
16. stress
2
r
1
stress
Area
force
t
o =

( )
A B
2
2
A
B
S 4 S 2
B r
rA
S
S
= = |
.
|

\
|
=
17. Breaking force o area of cross section of wire.
i.e. load hold by the wire does not depend upon the length of the wire.
18.
AY
FL
=
2
r
1
o
(F, L & Y are constant)
19.
2 6
m 10 A

=
I
A
Y
| |
|
\ .
=
A | |
|
\ .

20.
2
r
1
AY
FL
o =
(F, Land Y are same)
241
21.
Steel
1
1
Steel
Y
Ya
a AY
FL
= =

(F, Land Y are constant)


22. If length of the wire is doubled then strain = 1
Area
Force
stress Y = =
23.
dg
P
L =
24. F = force developed u o A YA
25.
lateral strain
longitudinal strain
o =
26.
strain Y
F
A
strain
A
F
Y
x
= =
27. Strain & stress
A
F
o
28. Elongation in the wire
AY
TL
=

Elongation in wire tension in the wirein first case =


1
T wand in second case
w
w w
w 2
T
2
=
+
=
As Tension in the wire in both the cases are equal.

Elongation in the wire will be equal.


29.
FA
Y
Al
=
30. 2 2
r
1
y r
F
o o
(F, Land Y are constant)
31. Longitudinal strain
stress
= =
L Y
l
32. It is the specific property of a particular material at a given temperature which can be changed only
by temperature variations.
33. When the length of wire is doubled then L = and strain = 1
A
F
strain Y = =
34.
L
r
F
L
A Y F
2
o =

(y and L are constant)
242
35. 10
0
10 3
7
r
r
Y

= =

36. Increase in tension of wire Q YA A o =
37.
AY
F
strain
strain
A
F
Y = =
38.
L
AL
Y
2
1
W
2
=
39. Potential energy stored in the rubber cord calapult will be covered into kinetic energy of mass
L
YAL
2
1
mv
2
1
2
2
=
40.
AY
FL
=
A
1
o
(F, L and Y are constant)
mm 05 . 0
2
1 . 0
2 2
1
8
4
A
A
1
2
2
1
1
2
= = = = = =

41.
t tan cons
strain
stress
Y = =
It depends only on nature of material.
42.
AY
FL
=

Final length = initial lengh + increament = 2L


43.
AY
FL
=
44.
Yro K =
45.
( ) VY
FL
Y AL
FL
AY
FL
2 2
= = =
2
L o If the volume of wire remains constant.
46.
2 2
r
F
Y r
FL
AY
FL
o
t
= =
(Y = constant)
47. Breaking stress = Strain

young's modulus
48. In accordance with Hook's law
49. Strain Y A F =
243
50. Because strain is a dimensionless and unitless quantity.
51.
area
force
stress =
In the present case,
force applied and area of cross-section of wires are same. Therefore stress has to be the same.
Strain = stress /Y
Since, the young's modulus of steel wire is greater than the copper wire, therefore strain in case of
steel wire is less than that of in case of copper wire
52. Knowledge base
53. Breaking force
2
r o
If diameter becomes double then breaking force will become four times
i.e. N 4000 4 1000 =
54.
( ) AY
FL
Y AL
FL
AY
FL
2 2
= = =
If volume is fixed then
2
L o
55. Young's modulus
strain
stress
=
As the length of wire get doubled therefore strain = 1
2
8
m
N
10 20 strain Y = =
56. Because dimension of invar does not vary with temperature.
57.
y r
FL
2
t
=
2
r
L
o
(Y & F are constant)
58. ( ) ( ) Q 1 L and Q 1
1 1 1 2 2 2
A o + = A o + =
( ) ( ) ( )
1 1 2 2 1 2 1 2
Q l l o o A + =
now ( ) ( ) 0 , So L L
1 1 2 2 1 2 1 2
= o = o =
59. Thermal stress = Q y A o
60.
L
YA
F =

61.
2
2
1
Y
FL
r
r y
= o
t
(F, Land l are constant)
244
62.
y r
FL
2
t
=
A
1
o
(F, Land Y are constant)
2 1
1 2
A
A
=

63.
Y
MgL
A
=

64.
AY
FL
=
2
r
F
o
65.
mgL
L
YA
=
66. Work done
2
Mg
F
2
1
= =
67.
( )
Volume
Y
stress
2
1
w
2
=
As F, A and Y are same - W o volume (area is same)
o w (V = A1)
2
1
2 w
w
1
2
1
2
1
= = =

68.
F
2
1
w =
69.
2
k
2
1
k
2
1
F
2
1
w and
F
K

= = = =
70.
2
2
r
L
U ;
AY 2
F
F
2
1
U o = =

(F and Y are constant)
71.
F
2
1
U =
72.
( )
Volume
Y
stress
2
1
U
2
=
73. When a wire is stretched through a length then work has to be done. This work is stored in the wire
in the form of elastic potential energy.
Potential energy of stretched wire is
strain stress
2
1
U =

Fx
2
1
U 6 F
2
1
U = =
245
74.
( )
2
strain YX
2
1
U =
75.
L 2
YA
w
2
=

76.
K 2
T
K 2
F
U
2 2
= =
77. Energy stored per unit volume
|
.
|

\
|
|
.
|

\
|
=
L A
F
2
1

AL 2
FL
=
78.
2
L
YA
2
1
U =

79. At extension l
1
the stored energy
2
1
k
2
1
=
At extension l
2
the stored energy
2
1
k
2
1
=
Work done in increasing its extension from l
1
to
( )
2
1
2
2 2
k
2
1
=
80.
X
F
K =
81. Isothermal elasticity ki = P
82.
100
1
% 1
V
V
given
V
B V
P
Y
PV
B = =
A A
=
A

=
2
8
10
m
N
10 5 . 1
100
10 5 . 1
P =

=
83. Ratio of adiabatic & isothermal elasticities
V
P
P
C
C
V
P
V
= = =
c|
c|
84. For triatomic gas
3
4
V =
85.
v
v
P
k
A
=
86.
P
v
v
C A
A
=
CC 4 . 0 =
246
87.
v
v
hsg
v
v
P
k
A
=
A
A
=
88. If side of cube is L then
L
dL 3
v
dv
L V
3
= =
89.
2
12
2
11 6
cm
dynes
10
m
N
10 atm 10
100
01 . 0
100
k = = =
90. If the coefficient of volume expansion is o and raise in temprature is
u A
then
u A o = A v v
u A o =
A

v
v
volume elasticity
u A o
=
A
= |
P
v
v
P
91. If side of the cube is L then V = L
2
L
dL 3
v
dv
=
% change in voulume = 3 (% change in length) % 3 % 1 3 = =

Bulk modulus
03 . 0
v
v
=
A
=
92. Adiabatic elasticity vp = c
For Argon EA
r
= 1.6 p
For hydrogon EH
2
= 1.4 pl
As elasticity of hydrogen & Argon are equal
Pl 4 . 1 P 6 . 1 =
P
7
8
' P =
93. Isothermal elasticity
2
5
m
N
10 013 . 1 013 . 1 atm 1 Ki P = = = =
94. Isothermal bulk modulus = pressure of gas
95. ( )
dv dL
1 2
v L
= + o

(

= = =

2
1
5 . 0 6 10 4 10 2 2
v
dv
3 3

96. We know that


( )
L
dL
2 1
v
dv
o =
If
0
v
dv
then
2
1
= = o
i.e. there is no change in volume
247
97. Y = 2N ( 1 + o )
98.
( )
n
n y 5 . 0
1 n 2 Y

= o o + =
99. Twisting coulple u
t
=
2
r n
C
4
If material and length of the wires A and B equal twisting coulple are applied then
4
r
1
o u
4
1
2
2
1
r
r
|
|
.
|

\
|
=
u
u

100. Value of possion's ratio die in range of 1 to 1/2.


101. ( ) ( ) o + = o = 1 n 2 y ; 2 1 k 3 y
For Y=0 we get 0 2 1 = o also 0 1 = o +
o lies between
102. ( ) ( ) o + = o + = 1 n 2 n 3 1 n 2 y
2
1
1
2
3
= = o
Now, substituting the value of o in the following expression
( )
=
o
=
2 1 3
y
k
103. ( ) o + = 1 n 2 Y
104. There will be both shear stress and normal stress.
105. Angle of shear
' 12 . 0
' 30 100
10 4
L
r
1
=

=
u
= |

106. For twisting, angle of shear
L
1
o |
i.e. if L is more then | will be small
107. 004 . 0 2 8 . 0 10 L r
2
= | = | = u

108. ( ) ( ) o + = o = 1 n 2 Y and 2 1 K 3 Y Eliminating o we got
k 3 n
nk 9
Y
+
=
109. Poisson's ratio varies between - 1 and 0.5
110.
M T
k
n
2 T
2
o t =
If we draw a graph of between T
2
& M then it will be straight line and for 0 T ; 0 M
2
= =
i.e. graph should pass through the origin but from the graph it is not reflected it means the mass of
pan was neglected.
248
111. In the region OA, stress o strain . i.e.Hooke's las hold good.
112. As stress is shown on X axis and strain on Y axis so we can say that
slope
1
tan
1
cot Y =
u
= u =
So elasticity of wire P is minimum and of wire R is maximum.
113. Area of hysterisis loop gives the energy loss in the process of stretching and understretching of
rubber band and this loss will appear in the form of heating.
114.
tan
tan
Y
Y
B
A
B
A
u
u
=
115. i.e.for the same load thickest wire will show minimum elongation so graph D present the thickest
wire.
116. From the graph N 20 F , M 10
4
= =

M 1 , M 10 A
2 6
= =

4 6
10 10
1 20
Al
FL
Y

= =
2
11 10
m
N
10 2 10 20 = =
117. At point b yeilding of material starts.
118. Graph between applied force and extension will be straight line because in elastic range.
Applied force extension. but the graph between extension and stored elastic energy will be parabolic
in nature.
As
2 2
x U or kx
2
1
U o =
119.
|
.
|

\
|
=
dx
dv
F
In the region BC slope of the graph is positive. = F negative i.e. force is attractive in nature.
In the region AB slope of the graph is negative. = F positive i.e. force is repulsive in nature.
120. Force constant K = tan 30
0

3
1
=
121. In ductile materials, yeild point exist which in brittle material failure would occur without yeilding.
122. Young's modulus is defined
strain
stress
Y =
123. Elasticity of wire decreases at high temperatare i.e. at higher temprature slope of graph will be less,
So T
1
> T
2
.
124. Attraction will be minimum when the distance b/w the molecule is maximum. Altraction will be
maximum at that point where the positive slope is maximum b'sc
du
F
dx
=
249
125. B'se stretching of coil simply chnages its shape without any change in the length of the wire used in
coil Due to which shear of elasticity is prevolved.
126. A bridge during its use undergoes alternating strains for a large number of times each day depending
upon the movement of vehicle on it. When a bridge is used for long time. It losses its strength Due to
which the amount of strain in the bridge for a given stress will become large and ultimately the bridge
may collapse. This may not happen if the fridges are declared unsafe.
127. Ivory is more elastic than wet-clay. Hence, the ball of ivory will rise to a greater height. Infact the ball
of wet day will not rise at all it will be same, what flattended permanently.
128. Young's modulus of a material.
strain
stress
Y =
Here, stress force force
Area
staring Re
= As restoring force is zero. 0 Y =
129. Work done
( )
2
strain Y
2
1
strain stress
2
1
= =
Since, elasticity of steel is more than copper, ,
hence, more work has to be done in order to stretch the steel.
130.
y
x
A
Ft
=
133.

A
A
W
Y =
139.
L
L
A
F
Y
A
=
250
Hint
143. Soap helps to lower the surface tension of solution thus soap get stick to the dist partcles and grease
and these are removed by action of water.
145. Force required to separate the plates
t
TA 2
F =
147.
| | r 2 cos T coater displaced of weight t u + =
u t + = cos rT 2 w
148. mg TL 2 =
L 2
mg
T =
149.
( )
2
1
2
2
R R T 8 w t =

( ) ( ) ( )
2 2
r r 2 T 8 t = T r 24
2
t =
150.
( )
2
1
2
2
r r 8 T w t =

|
|
.
|

\
|
t
4
d
4
D
8 T
2 2
( )T d D 2
2 2
t =
151. Energy spent = T

increase in surface ared


( ) ( )
2 2
r 4 r 2 4 2 T t t = ( ) Joule r T 24
2
t =
152. A T w A =
153. Surface energy = Surface tension

surface ared
A 2 T E =
New surface energy
|
.
|

\
|
=
2
A
2 T F
1
% decrase in surface energy
100
E
E E
i

=
154. The ration of the total surface energies before and after the change
1 : n
2
1
=
1 : 2
3
1
=
155. T R 8 w
2
t =
156. A T w A =
A
w
T
A
=
157.
3
R
3
4
t
251
158. As volume remains constant
3 3
r 8000 R = r 20 R =
1 r 4 8000
T R 4
drop small 8000 of energy Surface
drop big one of energy Surface
2
2
t
t
=
159. Tension
Area
energy surface
=
A
E
T or =
160. Angle of contact is a 0
0
165.
R
T 2
p = A
167.
B A B A
P P So
r
1
p and r r < o >
So air will flow from B to A i.e. size of A will increase
168.
hdg
R
T 4
=
4
Rhdg
T =
m
N
10 9 . 3
2
=
169. Mass of liquid in capillary tube
t =
2
R M |
.
|

\
|
o
R
1
R M
2
R M o If radius become double then mass will become twice.
172.
rdg
T 2
h =
r
1
h o
2 2 1 1
h r h r =
173.
cm 0 . 4
60 cos cos
h
=
o
=
u
=
174. When a capillary tube is broken at a height of 6 cm the height of water column will be 6 cm.
As t tan cons
cos
h
or
rg
cos 25
h =
u
u
=
4
3
8
0 cos 6
cos or
cos
6
0 cos
8
o
o
= = u
u
= |
.
|

\
|
= u

4
3
cos
1
175. Rise in tempreture
|
.
|

\
|
= u A
R
1
r
1
Jsd
T 3
|
.
|

\
|
= u A
R
1
r
1
J
T 3
(For water S = 1 and d = 1)
176.
rdg
cos T 2
h
u
=
r
1
h o
so the graph between h and r will be rectangular hyperbold
r
T 4
p = A
r
1
p o
As radius of soap bubble increases with time
t
1
P o A
252
178. ( ) t I To T
C
o =
i.e. Surface tension decreases with increase in temperature
179. Due to surface tension vertical force on drop
u t = Sm r T Fv
2
R
r 2
T
R
r
r T
2
2
t
= t =
180. 9 . R
3
4
T
R
r 2
3
2
t =
t
181. ( )
2
3
10 4 . 1 4 11 . 0 A T U

t = =
183.
dg
T 2
hR
Rdg
T 2
h = =
constant hR =
Hence when the tube is of insufficient length radius of curvature of the liquid meniscus increasses so
as to maintain the product hR a finite constant.
i.e. as h decreases R increases and the liquid meniscus becomes more and more flat but the liquid
does not overflow.
184. The presence of impurities either on the liquid surface or dissolved in it considerably affect the force
of surface tension depending upon the degree of surface tension depending upon the degree of
contamination. A highly soluable substance like sodium chioride when dissolved in water increase
the surface tension. But the sparing soluable or substance like phenol when dissolved in water
reduces the surface tension of water.
185. We know that the intermolecular distance between the gas moleculas is Large as compaired to that
of liquid Due to it the forces of cohesion in the gas moleculas are very small and these are quite
Large for liquids. Therefor the concept of surface tension is applicable to Liquid but not to gasses.
186. The height of capillary rise is inversly propostional to radius (or diametre) of capillary tube
i.e.
r
1
h o
so, for smaller r the value of his higher
187. When a drop of Liquid is poured on a glass, plate, the shape of the drop also is governed the force
of gravity for every small drops the potential energy due to gravity is insignification. Compared to
that due to surface tention. Hence, in this case the shape of the drop is determined by sufrace tension
alone and drope becomes spherical.
188.
2 2 1 1
v p v p =
( ) ( )
3
0
3
r 2
3
4
p r
3
4
g h p = t +
253
189. Thrust on lamina = pressure at centroid

area
1
A
3 3
h g
A gh

= =
190. Bulk modulas
B
p
v v
v
p
V B
0 0
A
= A
A
A
=
(

A
=
B
p
1 v v
0
1
0
B
p
1 Density

(

A
=

(

A
+ =
B
p
1
0
191. = 760 - 750 = mnat Hg = 4g
192. Here, h = 20 m
Density of water
3
3
m
kg
10 =
Atmospheric pressure
pa 10 01 . 1 Pa
5
=
gh pa p + =
193. Weight of the ball = Buyoant force + viscous force
194. For the floatation Vo dog = Vin dg
d
do
V Vin
0
=
d
do
V V Vin V Vout
0 0 0
=
195. Weight of body
Weight of water displaced Weight of oil displaced
3
4
oil of grvity Specitic
w
0
=

=
196. Aschemedies principal explains buoyant force and bouant force depends on acceleration due to
gravity
204. If two drops of same radius r coalesce then radius of new drop is given by R
3 3 3 3 3
r 2 R r
3
4
r
3
4
R
3
4
= t + t = t
( ) r 2 R 3
1
=
Is drop of radius r is falling in viscous medium then it acquire a critical velocity V and
2
r Vo
2
3
1
2
1
2
r
r 2
r
R
v
v
|
|
|
.
|

\
|
= |
.
|

\
|
=
1
3
2
2
v 2 v = 5 2
3
2
=
( )
s
m
4 5 3
1
=
254
206. According to Bernouli's theoram
cm 30 . 0
10 2
45 . 2
h
9
v
h
2
2
=
|
|
.
|

\
|

= =
cm 0 . 30 =
207. Time taken by outer to reach the bottom =
( ) 2 H D
t
g

=
and velocity of water coming out of hole
gD 2 v =
Horizontal distance covered
t v x =
209. Terminal speed ( ) o =
n
g r
9
2
v
2
Where, = Density of the substance
= o Density of the liquid
If n and r are constant then
( ) o o v
210. Mass = volume

Density
t =
3
r
3
4
M
As the density remains constant
3
r M o
211. The terminal velocity of the spherical body of radius R density S falling through a liquid of density o
is given by
( )
2
2
9
t
R
V g
n
o
=
where n is the coefficient of vescosity of the liquid.
( )
1
2
1 1
T
2
9
R
V g
n
o
=
( )
2
2
2 2
t
2
and
9
R
V g
n
o
=
According to the given problem
255
212. Diameter m 10 8
3
=
s
m
4 . 0 V
1
=
v
2

gh 2 v
2
1
+ =
A
1
v
1
= A
2
v
2
215. The volume of liquid displaced by floating ice
o
=
M
V
D
Volume of water formed by melting ice,
w
M
V
F
o
=
F D
V V e . i
w
M
L
M
<
o
<
o
- Height of the blood column in the human body is more at feet than at the brain as P = hpg there face
the blood exeists more pressure at the feet than at the brain.
- When to holes are made in the Hn air keeps on externing through the other hole Due to this the
pressure inside the tin does not become less than at mosphere pleassure which happen only when
hole is made.
218.
F 297 F
9
32 F
5
183
9
32 F
5
C
o
=

=
219. Change in resistonce O = 99 . 0 71 . 2 70 . 3 to interval of temprature 90
0
C so change in resistance
O = 55 . 0 71 . 2 26 . 3 corresponds to change in temprature.
C 50 55 . 0
99 . 0
90
o
= =
220. Maxmimum density of water is at 4
0
C also
9
32 F
5
C
=
221.
9
32 F
5
C
=
222. The boiling point of mercury is 400
0
C. Therefore the mercury thermeter can be used to measure the
range upto 360
0
C.
223.
9
32 F
5
C
=
225.
L
L
. 2
A
A
L A
2
A
=
A
o
256
226. Coffcient of volume expansion
( )
2 1
T .
r
u A

=
A
A
=
Hence, cofficent of linear expansion
227. Water has maximum density at 4
0
C so if the water is heated above 4
0
C or Density cooled below
4
0
C density decreases. In other words it expands so it overflows in both cases.
228. Increase in length u A o = A Lo L
Calorimetry
229. Melting point of ice decreases with increase in pressuire.
230. capacity Thermal mc then k 1 if ; c . m = = u = u A u A = u
231. Let final temprature of water u bc heat taken = Heat given
( ) ( ) ( ) u = u + u 70 1 220 10 10 10 1 110
C 50 C 8 . 48
o o
~ = u
232. 100 0023 . 0 h 0023 . 0 = = u A C 23 . 0
o
=
233. At boiling point vapour pressure becomes equal to the external pressure.
234.
mw mi
cw
miLi
w mw
mix
+
u
= u
Critical Thinking
235. Thermostat is used in electric opporatas like refrigerator iron etc for automatic cut off Therefore for
metallic strips to bend on heating their coefficient of linear expansion should be different.
236. Initially ice will absorb heat to rise its temprature to 0
0
C then its melting takes place. If mi = Initial
mass of ice mi
-1
= Mass of ice that melts and mw = Initial mass of water By low of mixture Heat
gained by ice = Heat lost by water ( ) ( ) 20 mwC L mi 20 c mi
w
= + +
( ) kg 1 mi 20 1 5 80 mi 20 5 . 0 2
7
= = +

So final mass water Initial mass of water + mass of ice that melts = 5 + 1 = 6 kg
237. If mass of the bullet is m gm then total heat required for bullet to just melt down.
mL mc
1
+ u A = u
Now when bullet is stopped by the obstacle the loss in its mechanical energy
9 v ) 10 cm
2
1
2 3
=
(As kg 10 m mg
3
= )
As 25% of this energy is absorbed by the
257
= u2
10 mv
8
3
10 mv
2
1
100
75
3 2 3 2
=
J
Now the bullet melt if
1 2
u > u
238. u A = mc t P
VI
m 4200
p
m 4200
p
mc
t
u A
=
u A
=
u A
=
)
`

= C
kg
1
4200 water
o
( )
min 3 . 6 sec 381
4 220
20 100 1 4200
t ~ =


=
Graphical Optaions
239. Initially on heating temprature rises from -10
0
C to 0
0
C Then ice melt and temprature does not rise
After the whole ice has melted temprature begins to rise until reaches 100
0
C Then it becomes
constant as at the boiling point will not rise.
240. Density of water is maximum at 4
0
C and is less on either side of this temprature.
Assertion & Reason
241. With rise in pressure melting point of ice decreases also ice contracts on melting.
242. Celsius scale was the first temprature scale and fahrenheit is the smallest unit measuring temprature.
243. Melting is associated with increasing of internal energy without change in temprature in view of the
reason being correct the amount of heat absorbed or given out during change of state is expressed
where m is the mass of the substances and L is the latent heat of the substance.
244. If bath assertion and reason are true and the reason is the correct explanation of the assertion.
245. The potential energy of water molecules is more. The heat given to melt the ice at 0
0
C used up in
increasing the potentical energy of water molecules formed at 0
0
C
246. Water has maximum density at 4
0
C on heating above 4
0
C or cooling below 4
0
C density of water
decreases and its volume increases. Therefore water overflows in both the cases.
259
SUMMARY

Thermal equilibrium, teroth law of thermodynamics, Concept of temperature.

Heat, work and internal energy. First law of thermodynamics.

Second law of thermodynamics, reversible and irrevessible processes.


Isothermal and adiabatic process.

Carnot engine and its efficency Refrigerators.

0 0 0 0
C 0 F 32 C F
100 180 100 180
A A
= =
w P V = A (Isothesmal process)
RT n
1
2
v
v
(Isothesmal process)
PV = Const (Isothesmal process)
P v

=

Const (adiabasic process)
P
1-
T

= Const
TV
-1
= Constant
r =
cv
cp
Monoatomic gases
v p
3 5
C = R, = C = R,
2 2

= 1.67
Diatomic gases
v p
5 7

C = R, C = R, = 1.4
2 2
Polgatomic gases
v p
7R 9R
C = , C = ,
2 2

= 1.4
1 1 2 2
1
W = (P V P V )

(adiabafic Process)
1 2
R (T T )
=

1
m

Q u W A = A + A (First law of thermodynamics)


Q U Cv t A =A = A (V = Const)
Q Cp t A = A ( P = Const)
u 0 A =
(isothesmal process cyelic process)
260

Coefricent of Performance of refrigesatos


1
Q
W
n =
1 2
1
Q Q
Q

=

1 2
1
T T
T

Coefcient of performance of refrigetors.


2
Q
w
o=
2
1 2
Q
Q Q
=

2
1 2
T
T T
=

I deal gas Cp Cv R =
1. A difference of temperature of 25
0
C is equivalent to a difference of
(A) 72
0
F (B) 45
0
F (C) 32
0
F (D) 25
0
F
2. What is the value of absolute temperature on the Celsius Scale ?
(A) -273.15
0
C (B) 100
0
C (C) -32
0
C (D) 0
0
C
3. The temperature of a substance increases by 27
0
C What is the value of this increase of Kelvin
scale?
(A) 300K (B) 2-46K (C) 7 K (D) 27 K
4. At Which temperature the density of water is maximum?
(A) 4
0
F (B) 42
0
F (C) 32
0
F (D) 39.2
0
F
5. The graph AB Shown in figure is a plot of a temperature of a body in degree Fahrenhit than slope
of line AB is
(A)
9
5
(B)
5
9
(C)
9
1
(D)
9
3
261
6. The temperature on celsius scale is 25
0
C What is the corresponding temperature on the Fahrenheit
Scale?
(A) 40
0
F (B) 45
0
F (C) 50
0
F (D) 77
0
F
7. The temperature of a body on Kelvin Scale is found to be x.K.when it is measured by Fahrenhit
thesmometes. it is found to be x
0
F, then the value of x is .
(A) 313 (B) 301.24 (C) 574-25 (D) 40
8. A Centigrade and a Fahrenhit thesmometes are dipped in boiling wates-The wates temperature is
lowered until the Farenhit thesmometes registered 140
0
what is the fall in thrmometers
(A) 80
0
(B) 60
0
(C) 40
0
(D) 30
0
9. A uniform metal rod is used as a bas pendulum. If the room temperature rises by 10
0
C

and the
efficient of line as expansion of the metal of the rod is,
6 1
C
2 10 0

what will have percentage increase
in the period of the pendulum ?
(A) -2

10
-3
(B) 1

10
-3
(C) -1

10
-3
(D) 2

10
-3
10. A gas expands from 1 litre to 3 litre at atmospheric pressure. The work done by the gas is about
(A) 200 J (B) 2 J (C) 300 J (D) 2

10
5
J
11. Each molecule of a gas has f degrees of freedom. The radio
P
V
C

=
C
for the gas is
(A)
1
2
f
+
(B)
1
1
f
+
(C)
2
1
f
+
(D)
( ) 1
1
3
f
+
12. Is the cyclic Process Shown on the V P diagram, the magnitude of the work done is

(A)
2
1 2
2
P P
|
.
|

\
|
t
(B)
2
1 2
2
V V
|
.
|

\
|
t
(C) ( )
2 2 1 1
P V PV (D)
( )( )
1 2 1 2
V V P P
4

t
13. If the ratio of specific heat of a gas at Consgant pressure to that at constant volume is , the Change
in internal energy of the mass of gas, when the volume changes from V to 2V at Constant Pressure
p, is
(A)

1
PV

(B)

1
R

(C) PV (D)

PV

1
262
14. The change in internal energy, when a gas is cooled from 927
0
C

to 27
0
C
(A) 200% (B) 100% (C) 300% (D) 400%
15. For hydrogen gas Cp - Cv = a and for oxygen gas Cp - Cv = b, The relation between a and b is
given by
(A) a = 4b (B) a = b (C) a = 16b (D) a = 8b
16. In a thermodynamic process, pressure of a fixed mass of a gas is changed in such a manner that the
gas release 20J of heat and 8J of work has done on the gas- If the inifial internal energy of the gas
was 30j, then the final internal energy will be
(A) 58 J (B) 2 J (C) 42 J (D) 18 J
17. If for a gas
cv
cp
= 1.67, this gas is made up to molecules which are
(A) diatomic (B) Polytomic
(C) monoatomic (D) mixnese of diatomic and polytomic molecules
18. An ideal monoatomic gas is taken around the cycle ABCDA as Shown in the P V diagram. The
work done during the cycle is given by
(A) PV (B)
2
1
PV (C) 2 PV (D) 4 PV
19. A given mass of a gas expands from state A to B by three different paths 1, 2 and 3 as shown in the
figure. If W
1
,W
2
and W
3
respectively be the work done by the gas along the three paths, then
(A) W
1
>W
2
>W
3
(B) W
1
<W
2
<W
3
(C) W
1
=W
2
=W
3
(D) W
1
<W
2
, W
1
<W
3
20. One mole of a monoatomic gas
5

3
| |
=
|
\ .
is mixed with one mole of A diatomic gas
7

5
| |
=
|
\ .
what
will be value of for mixture ?
(A) 1-454 (B) 1-4 (C) 1-54 (D) 1-5
21. If du represents the increase in internal energy of a thesmodynamic system and dw the work done
by the system, which of the following statement is true ?
(A) du = dw in isothermal process (C) du = - dw in an aidabadic process
(B) du = dw in aidabadic process (D) da = - dw in an isothermal process
263
22. One mole of a monoatomic ideal gas is mixed with one mole of a diatomic ideal gas The molas
specific heat of the micture at constant volume is ..........
(A) 4 R (B) 3 R (C) R (D) 2R
23. One mole of a monoatomic gas is heate at a constant pressure of 1 atmosphere from 0k to 100 k.
If the gas constant R = 8.32 J/mol k the change in internal energy of the gas is approximate ?
(A) 23 J (B) 1.25

10
3
J (C) 8.67

10
3
J (D) 46 J
24. A gas mixture consists of 2 mde of oxygen and 4 mole of argon at tempressure T.Neglecting all
vibrational modes, the total internal energy of the system is
(A) 11 RT (B) 9 RT (C) 15 RT (D) 4 RT
25. A monoatomic ideal gas, initially at temperature T
1
is enclosed in a cylindes fitted with a frictionless
piston. The gas is allowed to expand adiabatically to a temperature T
2
by releasing the piston suddenly
If L
1
and L
2
the lengths of the gas colum be fore and afters expansion respectively, then then
2
1
T
T
is
given by
(A)
2
3
1
2
L
L
| |
|
\ .
(B)
2
3
2
1
L
L
| |
|
\ .
(C)
1
2
L
L
(D)
2
1
L
L
26. Starting with the same intial Conditions, an ideal gas expands from Volume V
1
to V
2
in three different
ways. The Work done by the gas is W
1
if the process is purely isothermal, W
2
if purely isobasic and
W
3
if purely adiabatic Then
(A) W
2
>W
1
>W
3
(B) W
2
>W
3
>W
1
(C) W
1
>W
2
>W
3
(D)W
1
>W
3
>W
2
27. In a given process on an ideal gas dw = 0 and dQ < 0. Then for the gas
(A) the volume will increase (B) the pressure will semain constant
(C) the temperature will decrease (D) the temperature will increase
28. Wafer of volume 2 filter in a containes is heated with a coil of 1kw at 27
0
C. The lid of the containes
is open and energy dissipates at the late of
J
160
S
. In how much time tempreture will rise from
27
0
C to 77
0
C.

Specific heat of wafers is 4.2
Kg
KJ
(A) 7 min (B) 6 min 2s (C) 14 min (D) 8 min 20 S
264
29. 70 calorie of heat are required to raise the temperature of 2 mole of an ideal gas at constant pressure
from 30
0
C to 35
0
C
The amount of heat required to raise the temperature of the same gas through the same range at
constant volume is .................. calorie.
(A) 50 (B) 30 (C) 70 (D) 90
30. When an ideal diatomic gas is heated at constant pressure, the Section of the heat energy supplied
which increases the infernal energy of the gas is..
(A)
7
3
(B)
5
3
(C)
5
2
(D)
7
5
31. Two cylinders A and B fitted with piston contain equal amounts of an ideal diatomic gas at 300 k.
The piston of A is free to move, While that of B is held fixed. The same amount of heat is given to the
gas in each cylindes. If the rise in temperature of the gas in A is 30K, then the rise in temperature of
the gas in B is.
(A) 30 K (B) 42 K (C) 18 K (D) 50 K
32. An insulated containes containing monoatomic gas of molas mass Mo is moving with a velocity, V.If
the container is suddenly stopped, find the change in temperature.
(A)
2
Mov
5R
(B)
2
Mov
4R
(C)
2
Mov
3R
(D)
2
Mov
2R
33. A Small spherical body of radius r is falling under gravity in a viscous medium. Due to friction the
medium gets heated. How does the late of heating depend on radius of body when it attains terminal
velocity!
(A) r
2
(B) r
3
(C) r
4
(D) r
5
34. The first law of thermodynamics is concerned with the conservation of
(A) momentum (B) energy (C) mass (D) temperature
35. If heat given to a system is 6 k cal and work done is 6kj. The change in internal energy is .......... KJ.
(A) 12.4 (B) 25 (C) 19.1 (D) 0
36. The internal energy change in a system that has absorbed 2 Kcal of heat and done 500J of work is
(A) 7900 J (B) 4400 J (C) 6400 J (D ) 8900 J
37. Which of the following is not a thermodynamical function.
(A) Enthalpy (B) Work done
(C) Gibb's energy (D) Internal energy
38. Which of the following is not a thermodynamic co-ordinate.
(A) R (B) P (C) T (D) V
39. The work of 62-25 KJ is performed in order to compress one kilo mole of gas adiabatically and in
this process the temperature of the gas increases by 5
0
C The gas is _______ R = 8-3
J
molk
(A) triatomic (C) monoatomic
(B) diatomic (D) a mixture of monoatomic and diatomic
265
40. Cp and Cv denote the specific heat of oxygen per unit mass at constant Pressure and volume
respectively, then
(A) cp - cv =
16
R
(B) Cp - Cv = R
(C) Cp - Cv = 32 R (D) Cp - Cv =
32
R
41. When a System is taken from State i to State f along the path iaf, it is found that Q = 70 cal and w
= 30 cal, along the path ibf. Q=52cal. W atoug the path ibf is
(A) 6 cal (B) 12 cal (C) 24 cal (D) 8 cal
42. One kg of adiatomic gas is at a pressure of
5
2
N
5 10
m

The density of the gas is


3
5kg
m
what is the
energy of the gas due to its thermal motion ?
(A) 2.5

10
5
J (B) 3.5

10
5
J (C) 4.5

10
5
J (D) 1.5

10
5
J
43. 200g of water is heated from 25
0
C
0
45
0
C Ignoring the slight expansion of the water the change in its
internal energy is (Specific heat of wafer
C 9
cal
1
0
)
(A) 33.4 KJ (B) 11.33 KJ (C) 5.57 KJ (D) 16.7 KJ
44. During an adiabatic process, the pressure of a gas ifound to be propostional to the fifth power of its
absolute temperature. The radio
cv
cp
for the gas is
(A)
5
4
(B)
4
3
(C)
4
5
(D) 4
45. One mole of oxygen is heated at constant pressure stasting at 0
0
C. How much heat energy in cal
must be added to the gas to double its volume ? Take R = 2
cal
molk
(A) 1938 (B) 1920 (C) 1911 (D) 1957
46. moles of a gas filled in a containes at temperature T is in equilibrium inidially - If the gas is
compressed slowly and is thesmally to half its initial volume the work done by the atmosphere on the
piston is
(A)
RT
-
2
(B)
2
RT
(C) RT ln(2 -
2
1
) (D) RTln
2
266
47. Heat capacity of a body depends on the .......... as well as on ..........
(A) material of the body, its mass (C) mass of the body, itd temperature
(B) material of the body, its temperature (D) Volume of the body, its mass
48. In thesmodynamics, the work done by the system is considered ......... and the work done on the
system is Considered ...........
(A) Positive, zero (B) nagative, Positive
(C) zero, negative (D) Positive, negative
49. A thesmodynamic system goes from States
(i) P,V to 2P, V (ii) P,V to P, 2V. Then what is work done in the two Cases.
(A) Zero, PV (B) Zero, Zero (C) PV, Zero (D) PV,PV
50. For free expansion of the gas which of the following is true ?
(A) Q = 0, W > 0 and

Eint = -W (B) W = 0, Q >0 and A Eint = Q


(C) W > 0, Q < 0 and A Eint = 0 (D) Q = W = 0 and
A
Eint = 0
51. For an adiabatic process involving an ideal gas
(A) P
1
= T
1
= constant (B) P
1
= T

= constant
(C) PT
1
= constant (D) P
1
T

= constant
52. Figure shows four P V diagrams. which of these curves represent.
isothermal and adiabatic processes ?
(A) A and C (B) A and B (C) C and D (D) B and D
53. An ideal gas is taken through cyclic process as shown in the figure. The net work done by the gas is
(A) PV (B) 2 PV (C) 3 PV (D) zero
54. moles of gas expands from volume V V
1
to V
2
at constant temperature T. The work done by the gas
is
(A) RT
|
|
.
|

\
|
1
2
V
V
(B) RT ln
|
|
.
|

\
|
1
2
V
V
n
(C) RT
|
|
.
|

\
|
1
V
V
1
2
(D) RT ln
|
|
.
|

\
|
+1
V
V
n
1
2
267
55. A Cyclic Process ABCD is Shown in the P V diagam. which of the following curves represent
the same Process ?
(A) (B) (C) (D)
56. A Cyclic process is Shown in the P T diagram.
Which of the curve show the same process on a VT diagram ?
(A) (B)
(C) (D)
57. One mole of an ideal gas
p
v
C

C
=
at absolute temperature T
1
is adiabatically compressed from an
initial pressure P
1
to a final pressure P
2
The resulting temperature T
2
of the gas is given by.
(A)

1
2
2 1
1
p
T = T
p
| |
|
\ .
(B)
1

2
2 1
1
p
T = T
p

| |
|
\ .
(C)

2
2 1
1
p
T = T
p
| |
|
\ .
(D)
1
2
2 1
1
p
T = T
p

| |
|
\ .
268
58. An ideal gas is taken through the cycle ABCA as shown in the figure. If the net heat
supplied to the gas in the cycle is 5J, the work done by the gas in the process CA is
(A) - 5 J (B) -10 J (C) -15 J (D) - 20 J
59. In anisothermal reversible expansion, if the volume of 96J of oxygen at 27
0
C is increased from 70
liter to 140 liter, then the work done by the gas will be
(A) 300 R log
e
(2)
(B) 81 R log
e
(2)
(C)
2
10
200R og (D) 100 R log
10
(2)
60. For an iso thermal expansion of a Perfect gas, the value of
P
P
A
is equal to
(A)
1
2
V
-
V
(B)
V

(C)
2
V
-
V
(D)
V

61. For an aidabatic expansion of a perfect gas, the value of


P
P A
is equal to
(A)
r
-
v
(B)
v
v A

(C)
2
v
-
v
(D)
v

-
v
62. If r denotes the ratio of adiabatic of two specific heats of a gas. Then what is the ratio of slope of an
adiabatic and isothermal P V curves at their point of intersection ?
(A)
1

(B) -1 (C) (D) + 1


63. Work done permol in an isothermal ? change is
(A)
2
1
log
v
RT e
v
(B)
1
2
v
v
10 log RT
(C)
2
1
v
v
10 log RT
(D)
2
1
v
v
e log RT
64. The isothermal Bulk modulus of an ideal gas at pressure P is
(A) vP (B) P (C)
2
p
(D)
p
v
65. The isothermal bulk modulus of a perfect gas at a normal Pressure is
(A) 1.013

10
6
2
m
N
(B)
2
11
m
N
10 013 . 1

(C)
2
5
m
N
10 013 . 1
(D)
2
11
m
N
10 013 . 1
269
66. An adiabatic Bulk modulus of an ideal gas at Pressure P is
(A) P (B)
p

(C)
P
(D)
2
P
67. What is an adiabatic Bulk mudulus of hydrogen gas at NTP? r = 1.4
(A) 1.4
2
M
N
(B) 1.4

10
5
2
M
N
(C) 1

10
-8
2
M
N
(D) 1

10
5
2
M
N
68. If a quantity of heat 1163.4 J is supplied to one mole of nitrogen gas, at room temprature at constant
pressure, then the rise intemperature is R = 8.31
J
m.l.k
(A) 28 K (B) 65 K (C) 54 K (D) 40 K
69. One mole of O
2
gas having a Volume equal to 22.4 liter at
O c

and 1 atmosiheric Pressure is


Compressed isothermally so that its volume reduces to 11.2 lites. The work done in this Process is
(A) 1672.4 J (B) -1728J (C) 1728J (D) -1572.4J
70. The Specific heat of a gas in an isothermal Process is
(A) zero (B) Negative (C) Infinite (D) Remairs
71. A Containes that suits the occurrence of an isothermal process should be made of
(A) Wood (B) Coppes (C) glass (D) Cloth
72. A thermodynamic Process in which temprature T of the system remains constant through out Variable
P and V may Change is called
(A) Isothermal Process (B) Isochoric Process
(C) Isobasic Process (D) None of this
73. When 1g of wates
O c

and
5
2
N
10
m
Pressure is Converted into ice of Volume 1.091 cm
3
the external
work done be .......... J.
(A) 0.0182 (B) -0.0091 (C) -0.0182 (D) 0.0091
74. The letent heat of Vaporisation of water is 2240
g
J
If the work done in the Process of expansion
of 1g is 168J. then increase in internal energy is .......... J
(A) 2072 (B) 2408 (C) 2240 (D) 1904
75. The Volume of an ideal gas is 1 liter column and its Pressure is equal to 72 cm of Hg. The Volume
of gas is made 900 cm
3
by compressing it isothermally. The stress of the gas will be ............. Hg
column.
(A) 4 cm (B) 6 cm (C) 7 cm (D) 8 cm
270
76. In adiabatic expansion
(A)
A
u=0 (B)
A
u = Positive (C) Ju = Nagative (D)
A
w = 0
77. 1 mm
3
Of a gas is compressed at 1 atmospheric pressure and temperature
27 C

to
627 C

What
is the final pressure under adiabatic condition. r = 1.5
(A)
2
5
m
N
10 80
(B)
2
5
m
N
10 36
(C)
2
5
m
N
10 56
(D)
2
5
m
N
10 27
78. A monoatomic gas for it =
3
5
is suddenly Compressed to
1
8
of its original volume adiabatically
then the final Pressure of gas is ............. times its intial Pressure.
(A) 8 (B) 32 (C)
5
24
(D)
3
40
79. The Pressure and density of a diatomic gas

=
7
5
Change adiabatically from (P,d) to (P
1
,d
1
) If
d'
d
=32 then
' p
p
Should be
(A) 128 (B)
128
1
(C) 32 (D) None of this
80. An ideal gas at
27 C

is Compressed adiabatically, to
27
8
of its original Volume. If v =
5
3
, then the
rise in temperaure is
(A) 225 k (B) 450 K (C) 375 K (D) 405 K
81. A diatomic gas intially at
18 C

is Compressed adiabatically to one eight of its original volume. The


temperature after Compression will be
(A)
10 C

(B) 668

K (C)
887 C

(D)
144 C

82. Work done by 0.1 mole of a gas at


27 C

to double its volume at constant Pressure is


Cal. R = 2
K
o
Cal
mol
(A) 600 (B) 546 (C) 60 (D) 54
83. A gas expands
3
0.25m
at Constant Pressure 10
3
2
m
N

the work done is
(A) 250 J (B) 2.5 erg (C) 250 W (D) 250 N
84. In an isochoric Process
1
T 27 C =

and
2
T 127 C =

then
2
1
P
P
will be equal to
(A)
59
9
(B)
3
2
(C)
3
4
(D)
4
3
85. If the temperature of 1 mole of ideal gas is changed from
O C

to
100 C

at constant pressure, then


work done in the process is ............ J. R = 8-3
molk
J
(A) 8-3

10
-3
(B) 8-3

10
2
(C) 8-3

10
-2
(D) 8-3

10
3
271
86. A mono atomic gas is supplied the heat Q very slowly keeping the pressure constant The work done
by the gas.
(A)
Q
5
2
(B)
Q
3
2
(C) Q
5
3
(D)
Q
5
1
87. The Volume of air increases by 5% in an adiabatic expansion. The percentange lecrease in its
Pressure will br.
(A) 5% (B) 6% (C) 7% (D) 8%
88. In PV diagram given below, the isochoric, isothermal and isobaric path respectively are
(A) BA,AD,DC (B) DC,CB,BA (C) AB,BC,CD (D) CA,DA,AB
89. In the following indicatos diagram, the net amount of work done will be
(A) Negative (B) zero (C) Positive (D) Infinity
90. Work done in the given PV diagram in the cyclic process is
(A) 4 PV (B) 3 PV (C) 2 PV (D)
2
PV
91. In the Cyclic Process shown is the figure, the work done by the gas in one cycle
(A)
1 1
V P 40 (B)
1 1
V P 20 (C)
1 1
V P 10 (D)
1 1
V P 5
272
92. An ideal gas is taken V path ACBA as Shown in figure, The net work done in the whole cycle is
(A)
1 1
V P 3 (B) Zero (C)
1 1
V P 5 (D)
1 1
V P 3
93. A thermodynamic system is taken from state A to B along ACB and is brought back to A along BDA
as Shown in the PV diagram. The net work done during the complete cycle is given by the area.
(A)
1 2 1
P P ACB P (B)
ACBDA
(C)
1 1
ACBB A A
(D) A A ADBB
1 1
94. Two identical samples of a gas are allowed to expand (i) isothermally (ii) adiabatically work done is
(A) more inanisothermal process (B) more is an adiabatic process
(C) equal in both process. (D) neithes of them
95. What is the relationship Pressure and temperature for an ideal gas under going adiabatic Change.
(A) PT


= Const (B) PT
-1 +
= Const (C) P
1 -
T

= Const (D) P
- 1
T

= Const
96. For adiabatic Process which relation is true mentioned below ?
p
v
C

=
C
(A) p

V = Const (B) T

V = Const (C) TV

=Const (D) TV
1
= Const
97. For adiabatic Process which one is wrong statement?
(A) dQ = 0 (B) entropy is not constant
(C) du = - dw (D) Q = constant
98. Air is filled in a motor tube at
27 C

and at a Pressure of a atmosphere. The tube suddenly bursts.


Then what is the temperature of air. given r of air = 1.5
(A) 150 K (B)
150 C

(C) 75 K (D)
27.5 C

99. If v is the radio of Specigic heats and R is the universal gas constant, then the molar Specific heat at
constant Volume Cv is given by ............
(A)
R
1
v
v
(B) vR (C)
R
1 v
(D)
( )
1 R
r
v
273
100. A Car not engine operating between temperature T
1
and T
2
has efficiency 0.4, when T
2
lowered by
50K, its efficiency uncreases to 0.5. Then T
1
and T
2
are respectively.
(A) 300 K and 100 K (B) 400 K and 200 K
(C) 600 K and 400 K (D) 400 K and 300 K
101. A monoatomic gas is used in a car not engine as the working substance, If during the adiabatic
expansion part of the cycle the volume of the gas increases from V to 8V
1
the efficiency of the engine
is ..
(A) 60% (B) 50% (C) 75% (D) 25%
102. A System under goes a Cyclic Process in which it absorbs Q
1
heat and gives out Q
2
heat. The
efficiency of the Process is n and the work done is W. Which formula is wrong ?
(A)
2 1
Q Q W = (B)
1
2
Q
Q
n =
(C)
1
Q
W
n=
(D)
1
2
Q
Q
1 n =
103. A car not's engine whose sink is at a temperature of 300K has an efficiency of 40% By space
should the temperature of the source be increase the efficiency to 60%
(A) 275 K (B) 325 K (C) 300 K (D) 250 K
104. An ideal gas heat engine is operating between
227 C

and
127 C

. It absorks
4
10 J
Of heat at the
higher temperature. The amount of heat Converted into. work is .......... J.
(A) 2000 (B) 4000 (C) 5600 (D) 8000
105. Efficiency of a car not engine is 50%, when temperature of outlet is 500K. in order to increase
efficiency up to 60% keeping temperature of intake the same what is temperature of out let.
(A) 200 K (B) 400 K (C) 600 K (D) 800 K
106. A car not engine takes
6
3 10 cal of heat from a reservoir at
627 C

, and gives to a sink at


27 C

.
The work done by the engine is
(A)
6
4.2 10 J (B)
6
16.8 10 J (C)
6
8.4 10 J (D) Zero
107. For which combination of working temperatures the efficiency of Car not's engine is highest.
(A) 80 K, 60 K (B) 100 K, 80 K (C) 60 K, 40 K (D) 40 K, 20 K
108. An ideal heat engine working between temperature T
1
and T
2
has an efficiency n. The new efficiency
if both the source and sink temperature are doubled, will be
(A) n (B) 2n (C) 3n (D)
2
n
109. An ideal refrigerator has a freetes at a temperature of
13 C

, The coefficent of perfomance of the
engine is 5. The temperature of the air to which heat is rejected will be.
(A)
325 C

(B)
39 C

(C) 325 K (D)


320 C

110. An engine is supposed to operate between two reservoirs at temperature


727 C

and
227 C

. The
maximum possible efficiency of such an engine is
(A)
4
3
(B)
4
1
(C)
2
1
(D) 1
274
111. A car not engine Convertsm one sixth of the heat input into work. When the temperature of the sink
is reduces by
62 C

the efficiency of the engine is doubled. The temperature of the source and sink
are
(A) 80
0
C, 37
0
C (B) 95
0
C, 28
0
C (C) 90
0
C, 37
0
C (D) 99
0
C, 37
0
C
112. Car not engine working between 300 K and 600 K has work output of 800J per cycle. What is
amount of heat energy supplied to the engine fromsource per cycle
(A)
J
1600
cycle
(B)
J
cycle
2000 (C)
J
1000
cycle
(D)
J
1800
cycle
113. What is the value of sink temperature when efficiency of engine is 100%
(A) 300 K (B) 273 K (C) 0 K (D) 400 K
114. A car not engine having a efficiency of n =
10
1
as heat engine is used as a refrigerators. if the work
done on the system is 10J. What is the amount of energy absorbed from the reservoir at lowes
temperature !
(A) 1 J (B) 90 J (C) 99 J (D) 100 J
115. The temperature of sink of car not engine is
27 C

. Efficiency of engine is 25% Then find the


temperature of source.
(A) 227
0
C (B) 327
0
C (C) 27
0
C (D) 127
0
C
116. The efficiency of car not's engine operating between reservoirs, maintained at temperature
27 C

and
123 C

is ............
(A) 0.5 (B) 0.4 (C) 0.6 (D) 0.25
117. If a heat engine absorbs 50KJ heat from a heat source and has efficiency of 40%, then the heat
released by it in heat sink is ...........
(A) 40 KJ (B) 30 KJ (C) 20 J (D) 20 KJ
118. The efficiency of heat engine is 30% If it gives 30KJ heat to the heat sink, than it should have
absorbed ........... KJ heat from heat source.
(A) 42.8 (B) 39 (C) 29 (D) 9
119. If a heat engine absorbs 2KJ heat from a heat source and release 1.5KJ heat into cold reservoir,
then its efficiency is ...........
(A) 0.5% (B) 75% (C) 25% (D) 50%
120. If the doors of a refrigerators is kept open, then which of the following is true?
(A) Room is cooled (B) Room is eithers cooled or heated
(C) Room is neither cooled nor heated (D) Room is heated
275
Assertion-Reason
Instructions :-
Read the assertion and reason carefully to mask the correct option out of the options given
below.
(A) If both assertion and reason are true and the reason is the correct explanation of the assertion.
(B) If both assertion and reason are true but reason is not be correct explanation of assertion.
(C) If assertion is true but reason is false.
(D) If the assertion and reason both are false.
121. Assertion : The melting point of ice decreases with increase of Pressure
Reason : Ice contracts on melting.
(A) C (B) B (C) A (D) D
122. Assertion : Fahrenhit is the smallest unit measuring temperature.
Reason : Fahrenhit was the first temperature scale used for measuring temperature.
(A) A (B) C (C) B (D) D
123. Assertion : A beakes is completely, filled with water at
4 C

. It will overlow, both when heated or


cooled.
Reason : These is expansion of water below 4
0
C
(A) A (B) B (C) C (D) D
124. Assertion : The total translation kinetic energy of all the molecules of a given mass of an ideal gas is
1-5 times the product of its Pressure and its volume.
Reason : The molecules of a gas collide with each other and velocities of the molecules change due
to the collision
(A) D (B) C (C) A (D) B
125. Assertion : The car not is useful in understanding the perfomance of heat engine
Reason : The car not cycle provides a way of determining the maximum possible efficiency achivable
with reservoirs of given temperatures.
(A) A (B) B (C) C (D) D
276
Match column
126. Heat given to process is positive, match the following column I with the corresponding option of
column I
1
Colum-i Colum-ii
(A) JK (p)
A
W >0
(B) KL (q)
A
Q <0
(C) LM (r)
A
W <0
(D) MJ (s)
A
Q >0
(A) A-p, B-q, C-r, D-s (C) A-r, B-s, C-p, D-q
(B) A-q, B-p, C-s, D-r (D) A-s, B-r, C-q, D-p
127. In Column I different Process is given match corresponding option of column I
1
Column - I Column - II
(A) adiabatic process (p)
A
p = 0
(B) Isobaric process (a)
A
u = 0
(C) Isochroic process (r)
A
Q = 0
(D) Isothermal process (s)
A
W = 0
(A) A-p, B-s, C-r, D-q (C) A-r, B-p, C-s, D-q
(B) A-s, B-q, C-p, D-r (D) A-q, B-r, C-q, D-p
Comprehehsion Type
In a containes of negligible heat capacity, 200g ice at
0 C

and 100g steam at


100 C

are added to
200g of water that has temperature
55 C

.Assume no heat is lost to the surroundings and the pressure


in the container is constant 1 atm.
128. What is the final temperature the System ?
(A)
72 C

(B)
48 C

(C)
100 C

(D)
94 C

129. At the final temperature, mass of the toal water present in the system is
(A) 493.6g (B) 483.3g (C) 472.6 g (D) 500 g
130. Amount of the Sm left in the system, is equal to
(A) 16-7 g (B) 8-4 g (C) 12 g (D) 0 g
277
KEY NOTE
1 B 26 A 51 B 76 C 101 C
2 A 27 C 52 C 77 D 102 B
3 D 28 D 53 A 78 B 103 D
4 D 29 A 54 B 79 A 104 A
5 A 30 D 55 C 80 C 105 B
6 D 31 B 56 D 81 B 106 C
7 C 32 C 57 B 82 C 107 D
8 C 33 D 58 A 83 A 108 A
9 B 34 B 59 C 84 D 109 B
10 A 35 C 60 D 85 B 110 C
11 C 36 A 61 D 86 A 111 D
12 D 37 B 62 C 87 C 112 A
13 A 38 A 63 A 88 D 113 C
14 C 39 C 64 B 89 C 114 B
15 B 40 D 65 C 90 A 115 D
16 D 41 B 66 A 91 B 116 A
17 C 42 A 67 B 92 D 117 B
18 A 43 D 68 D 93 B 118 A
19 B 44 C 69 D 94 A 119 C
20 D 45 C 70 C 95 C 120 D
21 C 46 B 71 B 96 D 121 C
22 D 47 A 72 A 97 B 122 B
23 D 48 D 73 D 98 A 123 A
24 A 49 A 74 A 99 C 124 D
25 B 50 D 75 D 100 D 125 A
126 B
127 C
128 C
129 B
130 A
278
HINT
1.
180
F
100
25
180
F
100
C
0 0
A
=
A
=
A
2. T = 273.15 +

0
C
0 = 273.15 +

0
C
3. equal
4. 4
0
C
9
32 F
5
c
=
5.
9
32 F
5
c
=
y = m x + c with comparision m =
9
5
6.
9
32 f
5
c
=
7.
5
273 K
9
32 F
=

8.
Tc TF
=
100 180
Tc
100
C 40 TC
0
= A
9.
6 5
1 1
2 0 2 10 10 10
2 2
T
T

A
= A = =
% in increase =
% 10 1 100 10 100
T
T
3 5
= =
A
10. W = pV
11.
p
C 1 R, C
2 2
v
f f
R
| |
= + =
|
\ .
p
v
1
1
C
2 2 2
2
= 1
C
2 2
f
f
R
f
f f
f f
R
| |
+
+ |
+
\ .
= = = +
279
12. W = area inside the closed curve
( )
1 2 1 2
V V
2
1
) p p (
2
1
t =
[Treat Circle as an cllipse]
13.
2 1
P(V -V ) PV
u = =
- 1 -1
14.
1 1 1 2 1 2
2 2 2 2
u T
u T u u T T
u T u T

= =
15. For all gases C
p
- C
v
= R
16. w u Q A + A = A
17.
p
v
C
2
= +
C f
18. W = area enclose by PV graph
PV V) (2V P) (2P BC X AB = = =
19. W = area under the PV curve
Smallest for curve 1, largest for curve 3
20. monoatomic gas
, R
2
3
cv=
diatomic gas
, R
2
5
cv=
one mole of each gas is mixed C
v
(mix) =
R 2 R
2
5
R
2
3
2
1
=
(

+
C
p
(mix) = R + C
v
= R + 2R = 3R
r =
p
v
C mix
3R
= = 1.5
C mix 2R
21. dQ = du + dw
22. C
v
mix =
2 1
2 2 1 1
n n
Cv n Cv n
+
+
23.
v

U = C T
24. u =
f
2
n Rt
n = number of mole of the gas
f = number of degree of freedom
Total 02 AB
U = U U +
280
25. TV = [an adiabatic process]
1 1
T V
2 2
= T T
1 2
2 1
T V
=
T V
| |
|
\ .
26. W = area under the curve and volume axis on the P V diagram.
27. dQ = du + dw
28. energy required in heating water = ms
A
from coil,
energyavilable
Power of coil Power lost
S
=
For 840 J time required = 1S

4.2

10
4
J = ?
5
4.2 10
500 8.33min 8min 205
840
t J

= = =
29. C
p
=
( ) Q P
r T
A
A
R cp cv =
= 7-2 = 5
K mol
cal
30.
1 5
7
u
Q
A
= =
A
A
u = n C
v

A
T
A
Q = n C
p

A
T
1 5
7
v
p
C u
Q C r
A
= = =
A
31. For A (
A
Q) P = r C
p
(
A
T) A (Isobaric process)
For B (
A
Q) V = C
v
(
A
T) B (Isochroic process)
(
A
Q )
P
= (
A
Q)
V
32. Decrease in K.E = increase in I.E
2
v
1
mv = C T
2
33. Rate of heat produced = (Viscous force F)

(Velocity V)
2
dQ
nr
dt
v = 6
2
2
o
( ) g 2

g
r
n
(

(
(

281
34. AQ = AU + AW
36. AQ = AU + AW
39. AQ = AU + AW
A
u =
A
u - 62.25 (adiabatic process)
u = 62.25 x 10
3
J
C
v
=
u
n t
A
A
for monoatomic gas C
v
=
K mol
J
45 . 12 3 8
2
3
R
2
3
= =
40. If CP
1
and CV
1
is a molar specific heat CP
1
- CV
1
= R ------------ (i)
1 1
Cp Cv
Cp , Cv
32 32
= =
[molar mass of O
2
= 32]
From equation (i) 32Cp - 32Cv = R
Cp - Cv =
32
R
41. (i) Q = w +
u A
(ii) Q W u = + A u (A equal)
42.
5
2
5
2
u RT
PV RT
u PV
=
=
=


[diatomic gas]
3
mass 1
V m
density 5
= =
43. u mc T A = A
44. given
5
P T
an adiabatic process
1
P T

45.
5
1

282
46. W = Patm V
= Patm
V
2
(i)
Intially Patm V = rRT
RT
V
pat

=
49. (i) Volume Constant 0 V P W = A =
(ii) Pressure Constant
2 V
V
W P dv PV = =
}
50. For free expansion Q = W = O
T = const Eint o =
51. PV v = Const (an adiabatic process)
PV = m Rt
RT
V
P

52. For both solpe is negative & Slope of adiabatic curve is more
53. W area of P Vdiagarm =
=
PV ) V 2 ( P
2
1
=
54. AB Constant P, increasing V, increasing T T
BC Constant T, increasing V, decreasing P
CD Constant V, decreasing P, decreasing T T
DA Constant T, decreasing V, increasing P
Also BC is at highes temperature than AD
57. PV

= const [adiabatic process]
ideal gas PV = RT (m = 1)
P
RT
V =
RT
P
P
| |

|
\ .

= const
( ) -1
T
P
r
r
= const
58.
AB BC CA
CA
Q = W +W +W
5 = 10+0+W
59.
2 2
10
1 1
V M V
W = RT oge 2-3 RT og
V Mo V
| |
=
|
\ .

283
60. PV = constant (isothermal Process)
P -V
PV - VP = 0 =
P V

61. PV

= Constant (an adiabatic Process)
Pr v
-1
A
V + V

A
P = 0
62.
( )
( )
P
P
P
P
adiabatic
isothermal
A
A
=
V
V
V
V
-
-
A
A
= g
63. W = mRT ln
1
2
V
V
= R T log
e

1
2
V
V
64. isothermal bulk modulus B = P
64. isothermal bulk modulus B = P = 1.013

10
5
2
m
N
66. an adiabatic bulk modulus B = vP
67. an adiabatic bulk modulus B = 1.4 1 10
5
= 1.4 10
5
2
m
N
68. dQ = Cp dT
69. W = - RT log e (isothermally compressed)
70. C
iso
=
T m
Q
A
A

= [ A T = 0, isothermal process]
71. For isothermal process base is conductory
73. W = PAV (isothemal process)
74.
A
u =
A
Q -
A
w
74. P
1
V
2
= P
2
V
2
(isothermal process)
76.
A
Q =
A
u+
A
W
0 =
A
u +
A
W
77.
-1
T
P

= Constant [adiabatic change]


2
3
2
1
1
2
T
T
1 P
2 P
|
|
.
|

\
|
= |
.
|

\
|
1 3
2 2
5
P2 900
=
300
10
| | | |
| |
\ . \ .
284
78. PV

= Constant (adiabatic compressed)
8
2 1
1 2
P V
=
P V
| |
|
\ .
79. PV

= Constant (adiabatic process
80.
-1
2 1
1 2
T V
=
T V
| |
|
\ .

81. TV
- 1
= Constant
T
2
= T
1
-1
1
2
V
V
| |
|
\ .

82.
K 600 T
T
300
V 2
V
T
T
V
V
2
2 2
1
2
1
= = =
W = PA V
83. W = PAV = 10
3

0.25 = 250 J
84. P

T (constant volume)
2
1
2
1
T
T
P
P
=
85. W = PA V
86. u ) Q ( W
P
A A = A
V P
) Q ( ) Q ( A A =
( )
( )
(

A
A
A =
P Q
V Q
1 ) Q (
P
(

A =
CP
CV
1 ) Q (
P
87.
dp dv
100 = - 100
p V
(adiabatic expansion)
89. cyclic process 1 nagative work
2 net positive
90. W area of closed PV diagram
= (3V-V) (3P-P) = 4PV
91. W = area under curve
285
92. W = area under curve
94.

(Area)
iso
>(Area)
adi
Wiso > w
ad
:
95. PV

= Con
96. PV

=

Con
97. Q A = O, Q = const, du = - dw (aidabatic process)
98.
1
2 2
1 1
T P
=
T P

| |
|
\ .

99.
CP
= CP = CV
CV

but CP - CV = R
100. n = 1-

1
2
T
T
100.
101.
-1 -1
1 1 2 2
T V = T V


(qdiabafic process)
-1
2
1 2
1
V
T =T
V
| |
|
\ .

103.
1
2
T
T
1 n =
104.
2
1
T 1-400 1
n = 1 - = =
T 500 5
1
W = n Q
105.
1
2
1
2
T
T
,
T
T
1 n =

Should be minimum
106.
1
2
T
T
1 n ) i ( =
n
T
T
1
T 2
T 2
1 n ) ii (
1
2
1
2 1
= = =
109.
2
1 2

T
T T
=

286
110.
2
1
1000
500
1
T
T
1 n
1
2
= = =
111.
2
1 1
T W 1 1
(i) n = 1 - = n = - (1)
T Q 6 6
=
1
2 1
T
62 T
1 n ) ii (

=
1 1
2
T
62
T
T
1 + =
) 2 (
T
62
n
1
+ =
Now, n
1
= 2n
112.
2 1
1 1 1 2
T T W
n = 1 - = Q = w
T Q T -T
| |

|
\ .
113.
2
1
1
T
n
T
=
114.
2
1
1
T
n
T
=
1
2
2
T
W = Q -1
T
| |
|
\ .
115.
2
1
T
n = 1 -
T
116.
2
1
T
n = 1 -
T
117.
2
1
Q
n = 1 -
Q
118.
2
1
Q
n = 1 -
Q
287
119.
2
1
Q
n = 1 -
Q
121. with rise in pressure melting point of ice decreases. Also ice contracts on melting.
122. celcius scale was the first temperature scale and Fahrenhit is the smallest unit measuring.
123. Water has maximum density at
4 C

on heating above
4 C

or cooling below
4 C

density of
water decreases and its volume increases, therefore, water overflows in the both cases.
124.
2
1 3
m ( ) = RT
2 2

125. car not cycle has maximum efficiency


126. (a) isochoric process u Q 0 w A = A = A
P

T P decrease, T also decrease


u A
negative
0 Q < A
(b) isobasic process, volume increase
0 W > A
(c) isochoric process u Q 0 W A = A = A
P

T, P increase

T increase

0 Q > A
(d) Volumedecrease
0 W < A
127. adiabatic process 0 Q = A
Isobasic process P = const
0 P = A
Isochroic process V = const
0 W = A
Isothermal process T = const
0 u = A
128 to 130.
head rewuired by ice and water to go up
to 100
0
C = m
1
L + m
1
sw A T + mw sw
A
T
= 200

80+200

100+200

45
= 16,000+20,000+9,000
= 45,000 cal
= give by m
s
mass of steam
= ms L
ms =
g 3 . 83
540
000 , 45
=
convert into waters of 100
0
C
Total water = 200 + 200 + 83.3
= 483.3 g
steam left = 100 - 83.3 = 16.79
289
SUMMARY
* Gas Laws:
1. Boyle's law: For a given mass of an ideal gas at constant temperature, the volume of a gas is
inversely proportional to its pressure, i.e.
P
1
V
or PV = constant

P
1
V
1
= P
2
V
2
|
|
.
|

\
|

=
m
P PV ) i (
= constant

P
= constant or
2
2
1
1
P P

Where density,
m
V
= , and m = constant
(ii) As number of molecules per unit volume
N
n
V
=
N
V
n
=
also N = const.
(iii)
|
.
|

\
|
=
n
N
P PV
= constant


n
P
= constant or
2
2
1
1
n
P
n
P
=
(iv) Graphical representation: (If m and T are constant)
2. Charle's law: At constant pressure, the volume of the given mass of a gas is directly proportional
to its absolute temperature.
i.e.
V
V T
T
= constant =
2
2
1
1
T
V
T
V
=
(i)
T
m
T
V

=
= constant V

m | |
=
|
\ .

or T = constant =
2 2 1 1
T T =
290
(ii) Graphical representation: (If m and P are constant)
3. Gay-Lussac`s Law or pressure law:
According to it for a given mass of an ideal gas at constant volume, pressure of a gas is
directly proportional to its absolute temperature.
i.e.
P oT or
T
P
= constant
1 2
1 2
P P
T T
=
(i) Graphical representation: (If m and V are constant)
* Avogadro's law: Equal Volume of all the gases under similar conditions of temperature and
pressure contain equal number of molecules.
i.e. N
1
= N
2
Avogadro Number: The number of particles (atoms or molecules) in one mole of substance
(gas) is called Avogadro number (N
A
) which has a magnitude
N
A
= 6.023

10
23
mol
-1
291
* Equation of State OR Ideal Gas Equation
The equation which relates the pressure (P), volame (V) and temperature (T) of the given
state of an of an ideal gas is known as ideal gas equation or equation of state.
Ideal gas equation is PV = RT
where = number of mole
R = universal gas constant
= 8.314 J.mol
-1
K
-1
* Different forms of Ideal gas-state-equation
(i) RT PV =
B B
A A
N R
RT Nk T k
N N
| | | |
= = =
| |
\ . \ .

where k
B
= Boltzmann`s constant
= 1.38

10
-23
JK
-1
(ii)
B B
N
P k T nk T
V
= =
where
V
N
n = = number density of gas
= number of molecules per unit volume
(iii)
o o
M M
PV RT
M M
| |
= =
|
\ .

where M
o
= molar mass of the gas

o o
M RT RT
P
V M M
= = = (
m

V
= density of the gas)
* The work done during the change in volume of the gas:
It can be obtained from the graph of P V,
f
i
V
V
W P d = }
* Assumption of Ideal gases (or kinetic theory of gases)
Kinetic theory of gases relates the macroscopic properties of gases (such as pressure, temperature
etc.) to the microscopic properties of the gas molecules (such as momentum, speed, kinetic
enengy of molecule etc.)
Assumptions:
1. Gas is made up of tiny particles, These particles are called molecules of the gas.
2. The molecules of a gas are identical, spherical, rigid and perfectly elastic point masses.
3. The molecules of a gas perform incessant random motion.
4. The molecules of a gas follow Newton's laws of motion.
5. The number of molecules in a gas is very large.
6. The volume of molecules is negligible in comparision with the volume of gas.
292
7. Inter molecular forces act only when two molecules come close to each other or collide.
8. The time spent in a collision between two moleules is negligible in comparision to time
between two successive collisions. The collisions between the molecules and between a
molecule and the wall of a container are elastic.
* Pressure of an Ideal gas
2 2
1 1
3 3
rms P = < > =
where = density of the gas
v
2
rms
= <v
2
> = mean squre velocity of molecule
* Relation between pressure and kinetic energy
2
rms
1
P (i)
3
=
K.E. per unit volume is
E =
1 M
2 V
v
2
rms
=
1
2
v
2
rms
( ) ii
From equation (i) and (ii)
2
3
P E =
* Root mean square speed (v
rms
) :
It is defined as the square root of mean of squares of the speed of the speed of different
molecules.
i.e. v
rms
=
2 2 2 2
1 2 3 N
........
N
+ + + +
From the expression of pressure
P =
1
3
v
2
rms
v
rms
=
3P

=
3PV
M
=
3RT
M
=
3
B
k T
m
where
M

V
=
= density of the gas
= =
A
N
M
m mass of each molecule
-
rms
T
- with increase in molecular weight rms speed of gas molecule decreases as
rms
1
M
v
293
- rms speed of gas molecules does not depend on the pressure of gas (If temperature remains
constant)
- At T= 0 K , v
rms
= 0, i.e. the rms speed of molecules of a gas is zero at O K. This temperature
is called absolute zero.
* Kinetic interpretation of temperature
Kinetic energy of of 1 mole ideal gas
2
1 1 3 3
E
2 2 2
rms
RT
M M RT
M
| |
= = =
|
\ .
- For 1 molecule
B
3
E k T
2
= , k
B
= Boltzmann`s constant
- For N molecule
B
3
E N k T
2
=
- Kinetic energy per nolecule of gas does not depend upon the mass of the molecule but only
depends on the temperature.
* Mean free path:-
The distance travelled by a gas molecule between two successive collisions is known as free path.
The average of such free paths travelled by a molecule is called mean free path.
mean free path,

=
Total distance travelled by a gas molecule between two successive collisions
Total number of collisions
-
2
d n 2
1
t
=
-
B
B
2 2
B
1 k T P
P nk T n
k T 2 n d 2 Pd
| |
= = = =
|
\ .

- Collision frequency = number of collisions per second.
rms

* Degrees of Freedom :
The term degrees of freedom of a molecule or gas are the number of independent motions that
a molecule or gas can have.
The independent motion of a system can be translational, rotational or vibrational or any combination
of these.
Degress of freedom,
f = 3A - B; where A = Number of independent particles,
B = Number of independent restrictions
- monoatomic gas 3 degrees of freedom
( All translational)
294
- Diatomic gas 5 degrees of freedom
( 3 translational + 2 rotational)
- triatomic gas 6 degrees of freedom (at room temperature)
(Non-linear) ( 3 translational + 3 rotational)
8 degrees of freedom (at very high temperature)
(3 translational + 3 rotational + 2 vibrational)
* Law of equipartition of energy (Boltzmann law)
According to this law, for any system in equilibrium, the total energy is equally distributed
among its various degrees of freedom and each degrees of freedom is associated with energy
B
1
k T
2
where k
B
= Boltzmann`s constant.
- At a given temperataue T, all ideal gas molecules no matter what their mass have the same average
translational kinetic energy =
B
3
k T
2
- At same temperature gases with different degrees of freedom (i.e. H
2
and He) will have different
average energy
B
k T
2
f
=
( f = degress of freedom different for different gases.)
- The total energy associated with each modlec
B
k T
2
f
=
* Specific heat of a gas :
- Specific heat at constant volume (C
v
)
- The amount of heat required to change the temperature of l mole of gas by 1 K, keeping its
volume constant, is called specific heat of the gas at constant volume.
- Specific heat at constant volume (C
p
)
The amount of heat required to change the temperature of l mole of gas by 1 K, keeping its
pressure constant, is called specific heat of the gas at constant pressure.
Molar specific heat:
- The quantity of heat required to change the temperature of 1 mole of gas by 1 K (or 1
o
C) is
called molar specific heat of the gas.
Ratio of C
P
and C
V
is .
P
V P
V
1 R
C 2 1 2
1 C R , C 1 R
C 2 2
R
2
f
f
f
f
f
| |
+
|
| | | |
\ .
= = = + = = +
| |
\ . \ .

295
MCQ
Choose the correct alternative from given options.
1. Volume, pressure and temperature of an ideal gas are V, P and T respectively. If mass of molecule
is m, then its density is [ k
B
= Boltzmann`s constant]
(A)
B
P
k T
(B)
B
Pm
k T
(C)
B
.P
k TV
(D)
B
mk T
2. The temperature of an ideal gas at atmospheric pressure is 300 K and volume 1 m
3
. If temperature
and volume become double, then pressure will be
(A) 4 10
5
Nm
2
(B) 2 10
5
Nm
2
(C) 1 10
5
Nm
2
(D) 0.5 10
5
Nm
2
3. At 100 K and 0.1 atmospheric pressure, the volume of helium gas is 10 litres. If volume and
pressure are doubled, its temperature will change to
(A) 127 K (B) 400 K (C) 25 K (D) 200 K
4. What is the mass of 2 litres of nitrogen at 22.4 atmospheric pressure and 273 K.
(R = 8.314 Jmol k
-1
)
(A) 14 22.4 g (B) 56 g (C) 28 g (D) None of these.
5. An electron tube was sealed off during manufacture at a pressure of 1.2

10
-7
mm of mercury at
27
0
C. Its volume is 100 cm
3
. The number of molecules that remain in the tube is _______
(density of mercury is 13.6 gcm
3
)
(A) 3.9 10
11
(B) 3 10
16
(C) 2 10
14
(D) 7 10
11
6. A vessel contains 1 mole of O
2
gas (relative molar mass 32) at a temperature T. The pressure of
the gas is P. An identical vessel containing 1 mole of He gas (relative molar mass 4) at a
temperature 2T has pressure of ..........
(A) 8 P (B)
p
8
(C) 2 P (D) P
7. The equation of state for 5 g of oxygen at a pressure P and temperature T, when occupying a
volume V, will be
(A)
RT
32
5
PV |
.
|

\
|
=
(B)
RT
16
5
PV |
.
|

\
|
=
(C)
RT
2
5
PV |
.
|

\
|
=
(D) PV = 5 RT
8. A gas at 1 atmosphere and having volume 100 ml is mixed with another gas of equal moles at
0.5 atm and having volume 50 ml in flask of one litre, what is the final pressure?
(A) 0.125 atm (B) 0.75 atm (C) 1 atm (D) 0.5 atm
9. The quantity
B
PV
k T
represents
(A) mass of gas (B) number of moles of gas
(C) number of molecules in gas (D) K. E. of gas
10. Equation of gas in terms of pressure (P), absolute temperature (T) and density () is
(A)
2
2 2
1
1 1

T P

T P
=
(B)
2 2
2
1 1
1
T
P
T
P
=
(C)
2
1 2
1
2 1
T
P
T
P
=
(D)
2
2 2
1
1 1
T
P
T
P
=
296
11. 0
2
gas is filled in a vessel. If pressure is doubled, temperature becomes four times, how many times
its density will become.
(A) 4 (B)
4
1
(C) 2 (D)
2
1
12. At a given volume and temperature the pressure of a gas
(A) Varies inversely as the square of its mass (B) Varies inversely as its mass
(C) is independent of its mass (D) Varies linearly as its mass
13. If pressure of a gas contained in a closed vessel is increased by 0.4% when heated by 1
0
C the
initial temperature must be.
(A) 250
0
C (B) 25
0
C (C) 250 K (D) 2500 K
14. To decrease the volume of a gas by 5% at constant temperature the pressure should be
(A) Incseased by 5.26% (B) Decreased by 5.26%
(C) Decreased by 11% (D) Increased by 11%
15. A gas at the temperature 250 K is contained in a closed vessel. If the gas is heated through
1 K, then the percentage increase in its pressurse will be
(A) 0.4% (B) 0.1 % (C) 0.8% (D) 0.2%
16. The product of the pressure and volume of an ideal gas is
(A) A constant
(B) Directly proportional to its temperature.
(C) Inversely proportional to its temperature.
(D) Approx. equal to the universal gas constant.
17. At O
o
C the density of a fixed mass of a gas divided by pressure is x. At 100
o
C, the ratio will
be
(A) x (B)
x
373
273
|
.
|

\
|
(C)
x
273
373
|
.
|

\
|
(D)
x
273
100
|
.
|

\
|
18. Air is pumped into an automobile tube upto a pressure of 200 kPa in the morning when the air
temperature is 22
o
C. During the day, temperature rises to 42
o
C and the tube expands by 2%
The pressure of the air in the tube at this temperature will be approximately.
(A) 209 kPa (B) 206 kPa (C) 200 kPa (D) 212 kPa
19. The volume of a gas at
20 C

is 200 ml. If the temperature is reduced to 20


o
C at constant
pressure, its volume will be.
(A) 172.6 ml (B) 17.26 ml (C) 19.27 ml (D) 192.7 ml
20. 2g of O
2
gas is taken at 27
o
C and pressure 76 cm Hg. Find out volume of gas (ln litre)
(A) 3.08 (B) 44.2 (C) 1.53 (D) 2.44
21. 1 mole of gas occupies a volume of 100 ml at 50 mm pressure. What is the volume occupied by
two moles of gas at 100 mm pressure and at same temperature
(A) 50 ml (B) 200 ml (C) 100 ml (D) 500 ml
297
22. A partition divides a container, having insulated walls, into two
compartments, I and II. The same gas is filled the compartments.
The ratio of number of molecules in compartments I and II is.
(A) 6:1 (B) 1:6 (C) 4:1 (D) 1:4
23. A cylinder contains 10 kg of gas at pressure of 10
7
N/m
2
. The quantity of gas taken out of the
cylinder, if final pressure is
2 6
Nm 10 5 . 2

, will be ______ (temperature of gas is constant)
(A) 5.2 kg (B) 3.7 kg (C) 7.5 kg (D) 1 kg
24. The volume of a gas at pressure 21 10
4
Nm
2
and temperature 27
o
C is 83 Litres.
If R = 8.3 J mol
1
K
1
. Then the quantity of gas in g-mole will be
(A) 42 (B) 7 (C) 14 (D) 15
25. The pressure and temperature of an ideal gas in a closed vessel are 720 kPa and 40
o
C respectively.
If
th
1
4
of the gas is released from the vessel and the temperature of the remaning gas is raised
to 353
o
C, final pressure of the gas is
(A) 1440 kPa (B) 540 kPa (C) 1080 kPa (D) 720 kPa
26. Suppose ideal gas equation follows
3
VP
= constant, Initial temperature and volume of the gas are
T and V respectively. If gas expand to 27 V, then temperature will become
(A) 9 T (B) 27 T (C)
T
9
(D) T
27. The temperature of a gas at pressure P and volume V is 27
0
C Keeping its volume constant if its
temperature is raised to 927
0
C, then its pressure will be
(A) 3 P (B) 2 P (C) 4 P (D) 6 P
28. Air is filled in a bottle at atmospheric pressure and it is corked at 35
o
C, If the cork can come
out at 3 atmospheric pressure then upto what temperature should the bottle be heated in order to
remove the cork.
(A) 325.5
o
C (B) 651
o
C (C) 851
o
C (D)None of these
29. At what temperature volume of an ideal gas becomes triple
(A) 819
o

C (B) 182
o

C (C) 646
o

C (D) 546
o

C
30. To double the volume of a given mass at an ideal gas at 27
0
C keeping the pressure constant one
must raise the temperature in degree centigrade
(A) 54
o
(B) 600
o
(C) 327
o
(D) 270
o
31. At constant temperature on incerasing the pressure of a gas 5% its volume will decrease by
(A) 5% (B) 5.26% (C) 4.76% (D) 4.26%
32. At on 0
o

C pressure measured by barometer is 760 mm. what will be pressure at 100
o

C
(A) 780 mm (B) 760 mm (C) 730 mm (D) None of these.
33. Hydrogen gas is filled in a ballon at 20
0
C. If temperature is made 40
0
C, pressure remaining the
same what fraction of haydrogen will come out
(A) 0.75 (B) 0.07 (C) 0.25 (D) 0.5
P, V, T
I
2P, 2V, T
II
298
34. When the pressure on 1200 ml of a gas is increased from 70 cm to 120 cm of mercury at constant
temperature, the new volume of the gas will be
(A) 400 ml (B) 600 ml (C) 700 ml (D) 500 ml
35. A gas at 27
0
C temperature and 30 atmospheric pressure is allowed to expand to the atmospheric
pressure if the volume becomes two times its initial volume, then the final temperature becomes
(A) 273
0
C (B) -173
0
C (C) 173
0
C (D) 100
0
C
36. A gas at 27
0
C has a volume V and pressure P. On heating its pressure is doubled and volume
becomes three times. The resulting temperature of the gas will be
(A) 1527
0
C (B) 600
0
C (C) 162
0
C (D) 1800
0
C
37. A perfect gas at 27
0
C is heated at constant pressure to 327
0
C. If original volume of gas at 27
o
C
is V then volume at 327
o
C is
(A) 2 V (B) V (C)
V
2
(D) 3 V
38. A vessel contains 1 mole of
2
O gas (molar mass 32) at a temperature T. The pressure of the gas
is P. An identical vessel containing one mole of He gas (molar mass 4) at temperature 2T has a
pressure of
(A) 2 P (B) P (C) 8 P (D)
P
8
39. The pressure and temperature of two different gases P and T having the volumes V for each. They
are mixed keeping the same volume and temperature, the pressure of the mixture will be,
(A) P (B)
P
2
(C) 4 P (D) 2 P
40. Air is filled at 60
0
C in a vessel of open mouth. The vessel is heated to a temperature T so that
th
1
4
part of air escapes. Assuming the volume of the vessel remaining constant the value of T is.
(A) 333
0
C (B) 171
0
C (C) 444
0
C (D) 80
0
C
41. A gas is filled in a cylinder, its temperature is incresecd by 20% on kelvin scale and volume is
reduced by 10%. How much percentage of the gas will leak out
(A) 15% (B) 25% (C) 40% (D) 30%
42. The pressure is exerted by the gas on the walls of the container because
(A) It sticks with the walls
(B) It is accelerated towards the walls
(C) It loses kinetic energy
(D) On collision with the walls there is a change in momentum
43. The relation between the gas pressure P and average kinetic energy per unit volume E is
(A)
E
3
2
P =
(B)
E
2
3
P =
(C) E P= (D)
E
P=
2
44. The root mean square speed of hydrogen molecules of an ideal hydrogen kept in a gas chamber
at 0
0
C is 3180 ms
1
. The pressure on the hydrogen gas is (Density of hydrogen gas is
2 3 5 2
8.99 10 kg m , 1atm 1.01 10 Nm

= )
(A) 1.0 atm (B) 3.0 atm (C) 2.0 atm (D) 1.5 atm
299
45. Gas at a pressure P
o
is contained in a vessel. If the masses of all the molecules are halved and
their speeds are doubled, the resulting pressure will be equal to
(A) 2 P
0
(B) 4 P
0
(C)
2
P
0
(D) P
0
46. A cylinder of capacity 20 litres is filled with
2
H gas. The total average kinetic energy of translatory
motion of its molecules is
5
1.5 10
J. The pressure of hydrogen in the cylinder is
(A) 4 10
6
Nm
2
(B) 3 10
6
Nm
2
(C) 5 10
6
Nm
2
(D) 2 10
6
Nm
2
47. The average kinetic energy per molecule of a gas at -23
0
C and 75 cm pressure is erg 10 5
14
for
2
H . The mean kinetic energy per molecule of the
2
O at 227 C

and 150 cm pressure will be


(A) erg 10 80
14
(B) erg 10 10
14

(C) erg 10 20
14
(D) erg 10 40
14

48. The ratio of mean kinetic energy of hydrogen and oxygen at a given temperature is
(A) 1:8 (B) 1:4 (C) 1:16 (D) 1:1
49. The ratio of mean kinetic energy of hydrogen and nitrogen at temperature 300 K and 450 K
respectively is
(A) 2:3 (B) 3:2 (C) 4:9 (D) 2:2
50. Pressure of an ideal gas is increased by keeping temperature constant what is the effect on kinetic
energy of molecules.
(A) Decrease (B) Increase
(C) No change (D) Can`t be determined
51. A sealed container with negligible co-efficient of volumetric expansion contains helium (a monoatomic
gas) when it is heated from 200 K to 600 K, the averagy K.E. of helium atom is
(A) Halved (B) Doubled (C) Unchanged (D) Increased by factor 2
52. The mean kinetic energy of a gas at 300 K is 100J. mean energy of the gas at 450 K is equal to
(A) 100 J (B) 150 J (C) 3000 J (D) 450 J
53. At what temperature is the kinetic energy of a gas molecule double that of its value at 27 C

(A) 54
0
C (B) 108
0
C (C) 327
0
C (D) 300
0
C
54. The average kinetic energy of a gas molecule at 27
0
C is 6.21 10
21
J. Its average kinetic energy
at 227
0
C will be
(A) J 10 22 . 5
21
(B) J 10 35 . 11
21
(C) J 10 2 . 52
21
(D) J 10 42 . 12
21

55. The average translational energy and rms speed of molecules in sample of oxygen gas at 300 K
are
21
6.21 10

J and 484 m s respectively. The corresponding values at 600 K are nearly


(assuming ideal gas behaviour)
(A)
21
6.21 10 J, 968 m s

(B)
21
12.42 10 J , 684 m s

(C)
21
12.42 10 J , 968 m s

(D)
21
8.78 10 J , 684 m s

300
56. The average translational kinetic energy of O
2
(molar mass 32) molecules at a particular temperature
is 0.068 eV. The translational kinetic energy of N
2
(molar mass 28) molecules in eV at the same
temperature is
(A) 0.003 eV (B) 0.068 eV (C) 0.056 eV (D) 0.678 eV
57. At O K which of the follwing properties of a gas will be zero
(A) Kinetic energy (B) Density (C) Potential energy (D) Vibrational energy
58. The kinetic energy of one mole gas at 300 K temperatue is E. At 400 K temperature kinetic enrgy
is E'.

The value of
E'
E
is
(A) 2 (B)
9
16
(C) 1.33 (D)
3
4
59. The average kinetic energy of hydrogen molecules at 300 K is E. At the same temperature the
average kinetic energy of oxygen molecules will be
(A)
E
16
(B) E (C) 4 E (D)
E
4
60. The temperature at which the average translational kinetic energy of a molecule is equal to the
energy gained by an electron accelerating from rest through a potential differencc of 1 volt is
(A) K 10 6 . 4
3
(B) K 10 7 . 7
3
(C) K 10 6 . 11
3
(D) K 10 2 . 23
3

61. At a given temperature the rms velocity of molecules of the gas is


(A) Proportional to molecular weight
(B) Inversely proportional to molecular weight
(C) Inversely proportional to square root of molecular weight
(D) Proportional to square of molecular weight
62. According to the kinetic theroy of gases the r.m.s velocity of gas molecules is directly proportional
to
(A)
2
T
(B)
T
(C) T (D)
T
1
63. The speeds of 5 molecules of a gas (in arbitrary units) are as follws: 2, 3, 4, 5, 6, The root mean
squre speed for these molecules is
(A) 4.24 (B) 2.91 (C) 4.0 (D) 3.52
64. To what temperature should the hydrogen at room temperature (27
o

C) be heated at constant
pressuse so that the rms velocity of its molecule becomes double of its previous value
(A)
927 C

(B)
600 C

(C)
108 C

(D)
1200 C

65. Root mean square velocity of a molecule is v at pressure P. If pressure is increased two times,
then the rms velocity becomes
(A)
3v
(B)
2v
(C)
0.5v
(D) v
66. The rms speed of gas molecules is given by
(A)
o
M
2.5
RT
(B)
o
RT
2.5
M
(C)
o
RT
1.73
M
(D)
o
M
1.73
RT
301
67. A sample of gas is at O
o

C. To what temperature it must be raised in order to double the rms
speed of molecule.
(A) 270
o

C (B) 819
o

C (C) 100
o

C (D) 1090
o

C
68. If the ratio of vapour density for hydrogen and oxygen is
1
16
, then under constant pressure the
ratio of their rms velocities will be
(A) 4:1 (B) 1:16 (C) 16:1 (D) 1:4
69. The molecules of a given mass of a gas have a rms velocity of 200m s at
27 C

and
5 2
1.0 10 Nm

pressure when the temperature is


127 C

and pressure is
5 2
0.5 10 Nm

, the
rms velocity in m s will be
(A)
2 100
(B)
3
2 100
(C)
3
400
(D) None of these
70. If the molecular weight of two gases are
1
M and
2
M , then at a given temperature the ratio of
root mean square velocity
1
v and
2
v will be
(A)
2
1
M
M
(B)
1
2
M
M
(C)
2 1
2 1
M M
M M
+

(D)
2 1
2 1
M M
M M

+
71. To what temperature should the hydrogen at
327 C

cooled at constant pressure, so that the root


mean square velocity of its molcules become half of its previous value
(A) 100 C

(B) 123 C

(C)
0 C

(D) 123 C

72. At what temperature is the root mean square velocity of gaseous hydrogen molecules equal to that
of oxygen molecules at
47 C

?
(A) -73 K (B) 80 K (C) 20 K (D) 3 K
73. The root mean square velocity of the molecules in a sample of helium is
th
5
7
that of the molecules
in a sample of hydrogen. If the temperature of hydorgen sample is 0
0
C, then the temperature of
the helium sample is about
(A)
273 C

(B)
0 C

(C) O K (D)
100 C

74. At room temperature ( 27 C

), the rms speed of the molecules of certain diatomic gas is found to


be 1930 m/s. The gas is
(A) O
2
(B)
2
C (C) H
2
(D) F
2
75. If three molecules have velocities 0.5, 1 and 2 the ratio of rms speed and average speed is (The
velocities are in km/s)
(A) 0.134 (B) 1.34 (C) 1.134 (D) 13.4
302
76. At what temperature pressure remaining constant will the rms speed of a gas molecules increase
by 10% of the rms speed at NTP?
(A) 57.3 K (B) 57.3
0
C (C) 557.3 K (D) -57.3
0
C
77. When temperature of an ideal gas is increased from 27
o
C to 227
o
C, its rms speed changed from
400 ms
1
to V
s
. The V
s
is
(A) 516 ms
1
(B) 746 ms
1
(C) 310 ms
1
(D) 450 ms
1
78. At what temperature the molecules of nitrogen will have the same rms. velocity as the molecules
of Oxygen at 127
o
C.
(A) 273
0
C (B) 350
0
C (C) 77
0
C (D) 457
0
C
79. The temperature of an ideal gas is increased from 27
0
C to 127
0
C, then percentage increase in
v
rms
is
(A) 33% (B) 11% (C) 15.5% (D) 37%
80. Let A and B the two gases and given :
B
A
B
A
M
M
4
T
T
=
. Where T is the temperature and M is
molecular mass. If v
A
and v
B
are the r.m.s speed, then the ratio
A
B
v
v
will be equal to __________
(A) 2 (B) 4 (C) 0.5 (D) 1
81. The rms. speed of the molecules of a gas in a vessel is 400 ms
-1
. If half of the gas leaks out,
at constant temperature, the r.m.s speed of the remaining molecules will be
(A) 800 ms
-1
(B) 200 ms
-1
(C)
2 400
ms
-1
(D) 400 ms
-1
82. At which temperature the velocity of 0
2
molecules will be equal to the rms velocity of N
2
molecules
at 0
0
C
(A) 93
0
C (B) 40
0
C
(C) 39
0
C (D) can not be calculated.
83. The rms speed of the molecules of a gas at a pressure 10
5
Pa and temperature 0
0
C is
0.5 km/s. If the pressure is kept constant but temperature is raised to 819
0
C, the rms speed
becomes
(A) 1.5 kms
-1
(B) 2 kms
-1
(C) 1 kms
-1
(D) 5 kms
-1
84. The root mean square velocity of a gas molecule of mass m at a given temperature is proportional
to
(A) m
0
(B) m
-1/2
(C) m
1/2
(D) m
85. The ratio of the vapour densities of two gases at a given temperature is 9:8, The ratio of the rms
velocities of their molecule is
(A)
2 2 : 3
(B)
3 : 2 2
(C) 9:8 (D) 8:9
86. At what temperature, pressure remaining unchanged, will the rms velocity of a gas be half its value
at O
o
C ?
(A) 204.75 K (B) 204.75
0
C (C) -204.75 K (D) -204.75
0
C
87. The rms velocity of gas molecules is 300 ms
-1
. The rms velocity of molecules of gas with twice
the molecular weight and half the absolute temperature is
(A) 300 ms
-1
(B) 150 ms
-1
(C) 600 ms
-1
(D) 75 ms
-1
303
88. Calculate the temperature at which rms velocity of S0
2
molecules is the same as that of O
2
molecules at 27
0
C. Molecular weights of Oxygen and SO
2
are 32 g and 64 g respectively
(A) 327
0
C (B) 327 K (C) 127
0
C (D) 227
0
C
89. For a gas, the rms speed at 800 K is
(A) Four times the value at 200 K
(B) Twice the value at 200 K
(C) Half the value at 200 K
(D) same as at 200 K
90. A mixture of 2 moles of helium gas (atomic mass = 4 amu), and 1 mole of argon gas (atomic mass
= 40 amu) is kept at 300 K in a container. The ratio of the rms speeds
rms
rms
(helium)
(argon)
v
v
is
(A) 0.45 (B) 2.24 (C) 3.16 (D) 0.32
91. The temperature of an ideal gas is increased from 27
0
C to 927
0
C. The root mean square speed
of its molecules becomes
(A) Four times (B) One-fourth (C) Half (D) Twice
92. At a given temperature the root mean square velocities of Oxygen and hydrogen molecules are in
the ratio
(A) 1:4 (B) 1:16 (C) 16:1 (D) 4:1
93. If mass of He atom is 4 times that of hydrogen atom, then rms speed of the is
(A) Two times of H v
rms
speed.
(B) Four times of H v
rms
speed.
(C) Same as of H v
rms
speed.
(D) half of H v
rms
speed.
94. At temperature T, the rms speed of helium molecules is the same as rms speed of hydrogen
mdecules at normal temperature and pressure. The value of T is
(A) 546
0
C (B) 0
0
C (C) 273
0
C (D) 136.5
0
C
95. The root mean square speed of hydrogen molecules at 300 K is 1930 m/s. Then the root mean
square speed of Oxygen molecules at 900 K will be
(A) 836 m/s (B) 643 m/s (C)
1930 3 m/s
(D)
1930
m/s
3
96. If rms speed of a gas is v
rms
= 1840 m/s and its density = 8.99

10
-2
kg/m
3
, the pressure of
the gas will be
(A) 1.01 10
3
Nm
2
(B) 1.01 10
5
Nm
2
(C) 1.01 10
7
Nm
2
(D) 1.01 Nm
2
97. When the temperature of a gas is raised from 27
0
C to 90
0
C , the percentage increase in the
rms velocity of the molecules will be
(A) 15% (B) 17.5% (C) 10% (D) 20%
98. The rms speed of a gas at a certain temperature is
2
times than that of the Oxygen molecule
at that temperature, the gas is_____
(A) SO
2
(B) CH
4
(C) H
2
(D) He
304
99. The temperature at which the rms speed of hydrogen molecules is equal to escape velocity on earth
surface will be
(A) 5030 K (B) 10063 K (C) 1060 K (D) 8270 K
100. What is the meanfree path and collision frequency of a nitrogen molecule in a cylinder containing
nitrogen at 2 atm and temperature 17
o
C ? Take the radius of nitrogen molecule to be 1A

.
Molecular mass of nitrogen = 28 , k
B
= 1.38 10
23
JK
1
, 1 atm = 1.013 10
5
Nm
2
(A)
9 7
10 58 . 2 , m 10 2 . 2

(B)
8 7
10 58 . 4 , m 10 1 . 1

(C)
9 7
10 58 . 4 , m 10 1 . 1

(D)
9 7
10 58 . 3 , m 10 2 . 2

101. The radius of a molecule of Argon gas is 1.78
0
A.

Find the mean free path of molecules of Argon
at 0
0
C temperature and 1 atm pressure.
23 1
B
k 1.38 10 JK

=
(A) m 10 65 . 6
8
(B) m 10 65 . 3
7
(C) m 10 65 . 6
7
(D) m 10 65 . 3
8

102. A monoatomic gas molecule has


(A) Three degrees of freedom (B) Five degrees of freedom
(C) Six degrees of freedom (D) Four degrees of freedom
103. A diatomic molecule has how many degrees of freedom (For rigid rotator)
(A) 4 (B) 3 (C) 6 (D) 5
104. The degrees of freedom for triatomic gas is ______ (At room temperature)
(A) 8 (B) 6 (C) 4 (D) 2
105. If the degrees of freedom of a gas are f, then the ratio of two specific heats
p
v
C
C
is given by
(A)
2
1
f
(B)
1
1
f
+ (C)
2
1
f
+
(D)
3
1
f
+
106. A diatomic gas molecule has translational, rotational and vibrational degrees of freedom. The
p
v
C
C
is
(A) 1.29 (B) 1.33 (C) 1.4 (D) 1.67
107. The value of C
v
for one mole of neon gas is
(A)
R
2
3
(B)
R
2
7
(C)
R
2
1
(D)
R
2
5
108. The relation between two specific heats of a gas is
(A)
V P
R
C C
J
= (B)
P V
C C J =
(C)
P V
R
C C
J
= (D)
V P
C C J =
305
109. The molar specific heat at constant pressure for a monoatomic gas is
(A)
R
2
3
(B)
R
2
5
(C) 4 R (D)
R
2
7
110. For a gas 67 . 0
C
R
V
= . This gas is made up of molecules which are
(A) Diatomic (B) monoatomic
(C) polyatomic (D) mixture of diatomic and polyatomic molecules
111. The specific heat of an ideal gas is
(A) Proportional to T
2
(B) Proportional to T
3
(C) Proportional to T (D) Independent of T
112. The specific heats at constant pressure is greater than that of the same gas at constant volume
because (A) At constant volume work is done in expanding the gas.
(B) At constant pressure work is done in expanding the gas.
(C) The molecular vibration increases more at constant pressure.
(D) The molecular attraction increases more at constant pressure.
113. One mole of ideal monoatomic gas
5
3
| |
=
|
\ .
is mixed with one mole of diatomic gas
7
3
| |
=
|
\ .
.
What is for the mixture? denotes the ratio of specific heat at constant pressure to that at
constant volume.
(A)
35
23
(B)
23
15
(C)
3
2
(D)
4
3
114. For a gas if ratio of specific heats at constant pressure and volume is , then value of degrees
of freedom is
(A)
2
1
(B) ( )
25
1
2

(C)
3 1
2 1


(D) ( )
9
1
2

115. The molar specific heat at constant pressure of an ideal gas is
R
2
7
. The ratio of specific heat
at constant pressure to that ratio at constant volume is
(A)
5
7
(B)
9
7
(C)
8
7
(D)
7
5
116. For a gas
7
5
=
, the gas may probably be
(A) Neon (B) Argon (C) Helium (D) Hydrogen
117. From the following P - T graph, what inference can be drawn
(A) V
2
< V
1
(B) V
2
= V
1
(C) V
2
>V
1
(D) none of these
306
118. The figure shows the volume V versus temperature T graphs for
a certain mass of a perfect gas at two constant pressure of
P
1
and P
2
. What inference can you draw from the graphs.
(A) P
1
< P
2
(B) P
1
>P
2
(C) P
1
= P
2
(D) No inference can be drawn due to insufficient information.
119. Which one the following graphs represents the behaviour of an ideal gas
(A) (B) (C) (D)
120. Under constant temperature, graph between p and
V
1
is
(A) Hyperbola (B) Circle (C) Parabola (D) Straight line
Direction:-
(a) If both Assertion and Reason are true and the Reason is correct explanation of the Assertion.
(b) If both Assertion and Reason are true, but Reason is not correct explanation of the Assertion.
(c) If Assertion is true; the Reason is false.
(d) If Assertion is false, but the reason is true.
121. Assertion : 300 cc of a gas at 27
0
C is cooled at -3
0
C at constant pressure. The final volume
of the gas would be 270 cc
Reason : This is as per charle's law
1
2
1
2
T
T
V
V
=
(A) a (B) b (C) c (D) d
122. Assertion : The time of collision of molecules is of the order of 10
-8
s, which is very very small
compared to the time between two successive collisions.
Reason : This is an experimental fact.
(A) a (B) b (C) c (D) d
123. Assertion : Mean free path of gas varies inversly as density of the gas.
Reason : Mean free path varies inversely as pressure of the gas.
(A) a (B) b (C) c (D) d
307
KEY NOTE
1 B 26 A 51 B 76 B 101 A
2 C 27 C 52 B 77 A 102 A
3 B 28 B 53 C 78 C 103 D
4 C 29 D 54 D 79 C 104 B
5 A 30 C 55 B 80 A 105 D
6 C 31 C 56 B 81 D 106 B
7 A 32 D 57 A 82 C 107 A
8 A 33 B 58 C 83 C 108 C
9 C 34 C 59 B 84 B 109 B
10 B 35 B 60 B 85 B 110 B
11 D 36 A 61 C 86 D 111 D
12 D 37 A 62 B 87 B 112
13 C 38 A 63 A 88 A 113 C
14 A 39 D 64 A 89 B 114 A
15 A 40 B 65 D 90 C 115 D
16 B 41 B 66 C 91 D 116 D
17 B 42 D 67 B 92 A 117 C
18 A 43 A 68 A 93 D 118 B
19 A 44 B 69 C 94 C 119 C
20 C 45 C 70 B 95 A 120 D
21 C 46 B 71 D 96 B 121 A
22 D 47 D 72 C 97 C 122 A
23 C 48 A 73 B 98 B 123 B
24 B 49 A 74 C 99 B
25 C 50 C 75 C 100 C
1 B 26 A 51 B 76 B 101 A
2 C 27 C 52 B 77 A 102 A
3 B 28 B 53 C 78 C 103 D
4 C 29 D 54 D 79 C 104 B
5 A 30 C 55 B 80 A 105
6 C 31 C 56 B 81 D 106 B
7 A 32 D 57 A 82 C 107 A
8 A 33 B 58 C 83 C 108 C
9 C 34 C 59 B 84 B 109 B
10 B 35 B 60 B 85 B 110 B
11 D 36 A 61 C 86 D 111 D
12 D 37 A 62 B 87 B 112
13 C 38 A 63 A 88 A 113 C
14 A 39 D 64 A 89 B 114 A
15 A 40 B 65 D 90 C 115 D
16 B 41 B 66 C 91 D 116 D
17 B 42 D 67 B 92 A 117 C
18 A 43 A 68 A 93 D 118 B
19 A 44 B 69 C 94 C 119 C
20 C 45 70 B 95 A 120 D
21 C 46 71 D 96 B 121 A
22 D 47 72 C 97 C 122 A
23 C 48 73 B 98 B 123 B
24 B 49 A 74 C 99 B
25 C 50 C 75 C 100 C
C
B
A
C
B
D
308
Hint / Solution
1. (B) PV = RT
O
M
PV RT
M
=
o
PM M
RT V
= =
o A
B
A
PM P m N Pm Pm
RT RT k T R
T
N

= = = =
| |
|
\ .
2. (C)
V
T
P RT PV o =
If T and V both doubled, then pressure remains same
3. (B) PV RT PV T =
If P and V doubled, then T becomes four times,
4. (C)
PV
PV RT
RT
= =
Mass of litre nitrogen =
o
M
5. (A) Gas equation for N molecules, PV = Nk
B
T
B
PV
N
k T
=
6. (C)
V
RT
P RT PV = =
For same R , and V, P T
7. (A) molecular weight of oxygen = 32 (g)
number of moles in 5 g of oxygen =
32
5

Equation of state is PV = RT


RT
32
5
PV=
8. (A) Total number of moles is conserved,
1 1 2 2
PV PV PV
RT RT RT
+ =
1 1 0 0 0 . 5 5 0 P 1 0 0 0
+ =
R T R T R T

(1 L = 100 m )
atm 125 . 0 P=
309
9. (C) The ideal gas equation is
RT PV =
( ) PV RT 1 = =
B A B
A
R
PV k N T k
N
| |
= =
|
\ .

A
B
PV
N
k T
= =
Avogadro's number. .
10. (B). RT PV =
o
M
PV RT
M
| |
=
|
\ .
o
M RT
V M
| |
=
|
\ .
o
RT M
M V
| |
=
|
\ .

o
P R
T M
=

= const.
11. (D) RT PV =
M
P RT
| |
=
|

\ .
o
M M P
P RT
M T
| | | |
=
| |

\ . \ .
12. (D) RT PV =
o
M
PV RT P M
M
| |
=
|
\ .
( V, T T Constant )
13. (C). RT PV =
P T ( Closed vessel i.e. volume is constant)
1 1
2 2
P T
P T
=
14. (A). RT PV = = Constant ( temp. is const.)
2 2 1 1
V P V P =
( )
1 1 2 1 2 1
95
95%
100
PV P V V V
| |
= =
|
\ .

310
2 1
1.0526 P P =
1 1
P 0526 . 0 P + =
1 1
P % 26 . 5 P + =

Pressure 5.26% increases.


15. (A) Closed Vessel . i.e. volume remains constant.
From, RT PV =
2 2
1 1
P T
P T
P T
=
1
1 2
1
1 2
T
T T
P
P P
=

16. (B) PV T ( ,R Constant)


17. (B)
o
M
PV RT RT
M
| |
= =
|
\ .
o
M M
PV RT
=
density
P

=
o
M
RT
At 0 C
density
P

| |

|
\ .
=
( )
M
x...........................(i)
R 273
=
At 100 C
density
P

| |
|
\ .
=
( )( )
M
.........................(ii)
R 373

At 100 C
density
P

| |
|
\ .
273
x
373
| |
=
|
\ .
18. (A)
PV
=R
T
= constant
1 1 2 2
1 2
P V P V
T T
=
19. (A) PV=RT
since P is const. V T
1 1
2 2
V T
V T
=
20. (C) PV= RT
o o
M MRT
= RT V =
M M P
| |

|
\ .
311
21. (C) PV= RT
1 1 1
2 2 2
P V
P V
=
(

T const.)
22. (D)
B
B
PV
PV Nk T N
k T
= =
Now,
( )( )
B B
2P 2V PV
N' 4 4 N
k T k T
| |
= = =
|
\ .
N 1
N' 4
=
23. (C) PV=RT
o
M
PV = RT P M
M
| |

|
\ .
(

V, R, T - Constant)
7
1 1
6
2 2 2
P M 10 10
P M 2.5 10 M
= =

M
2
= 2.5 kg

Hence mass of the gas taken out of the cylinder = 10 2.5 = 7.5 kg
24. (B)
PV
PV=RT =
RT

25. (C)
o
M
PV = RT PV = RT P MT
M
| |

|
\ .
2 2 2
1 1 1
P M T
P M T
| | | |
=
| |
\ . \ .
26. (A) VP
3
= constant = k
3
1
3
1 1
P P
V
V

1
3
k
P
V
=
1
3
k
PV= RT V RT
V
=
2
3
k V RT = =
2
3
RT
V
k
= =
Hence
2 2
3 3
1 1
2 2 2
V T V T
V T 27V T
| | | |
= =
| |
\ . \ .
2
2
1 T
T 9T
9 T
= =
312
27. (C) Using Gay-lussac's law
1 1
2 2
P T
P T
=
28. (B) At constant volume
1 2 2
2 1
1 2 1
P P P
T T
T T P
| |
= =
|
\ .
29. (D) At constant Pressure
V T
2 2 2
2 1
1 1 1
V T V
T T
V T V
| |
= =
|
\ .
30. (C)
1
1
2 2
T
V
V T
V T
=
31. (C)
2 1
2 1 1
1 2
1 V P 100 100
P V V 0.9524V
V V P 105 105
= = =
( )
2 1
V 1 0.0476 V =
1 1
V 0.0476V =
1 1
V 4.76% V =
32. (D)
2 2 2
2 1
1 1 1
P T T
P T P P
P T T
= =
33. (B)
2 2 2 1 2 1
1 1 1 1
V T V V T T
V T
V T V T

= =
( ) ( ) 273 40 273 20
V
V 273 20
+ +
A
=
+

313 293
0.07
293

= =
34. (C) At constant Pressure PV = constant
1 1 2 2
PV P V =
1 2
2 1
P V
P V
=
35. (B)
1 1 2 2 2 2
2 1
1 2 1 1
PV P V P V
T T
T T PV
| |
= =
|
\ .
36. (A)
1 1 2 2
1 2
PV P V
T T
=

2 2
2 1
l 1
P V
T T
PV
=
37. (A)
1 1
2 2
V T
V T
V T
=
313
38. (A) PV=RT

P=T ( V and R = constant )


2 2 2
1 1 1
P T
P T
=
39. (D)
1
PV
PV=RT
RT
= and
2
PV

RT
=
( )
1 2
RT
2 PV RT
P' 2P
V RT V
+
= = =
40. (B) For open mouth vessel, pressure is constant.
volume is also given constant.
Hence from PV RT =
o
M 1
PV RT T
M M
= o
1 2
2 1
T M
T M
=

1
4
th
part escapes, so remaining mass in the vessel is

2 1
3
M M
4
=

1
1
3
M
273 60
4
T M
+
=

0
T 444 K 171 C = =
41. (B) Let initial conditions = V, T
final conditions = V', T'
By Charle's law,
V T
( P remains constant )
V V' V V'
V' 1.2 V
T T' T 1.2 T
= = =
But as per question, volume is reduced by 10%
means

V'

= 0.9 V
so percentage of volume leaked out
( )
%
1.2 0.9 V
100 25%
1.2 V

= =
42. (D) Pressure
F 1 P
P
A A t
A
= =
A
(
P A
= change in momentum)
43. (A)
2
P
3
= ( Energy per unit volume )

2 E 2
E
3 V 3
= =
44. (B)
2
rms
rms
3P
P
3
v
v = =

314
45. (A)
1 1 2
rms rms
2 2 1
P M 3P 3PV P

M M P M
v
v = = v =
v
46. (C)
3
E PV
2
=

2E
P
3V
=
47. (B) The average kinetic energy
1 1
B
2 2
3 E T
E k T
2 E T
= =
48. (D) Kinetic energy is a function of temperature.
49. (A)
1 1
2 2
E T
E T
E T
=
50. (C) Kinetic energy of ideal gas depends only on its temperature. Hence, it remains constant
whether pressure is increased or decreased.
51. (B) Kinetic energy is directly proportional to temperature. Hence if temperature is doubled,
kinetic energy will also be doubled.
52. (B) Average kinetic energy Temperature
1 1
2 2
E T
E T
=
53. (C)
1 1
2 2
E T
E T
E T
=
54. (D)
1 1
2 2
E T
E T
E T
=
55. (B) Average translational K.E. of a molecule is =
B
3
k T
2
At 300 K, average K.E. =
21
6.21 10

J
At 600 K average K.E. =
21
2 6.21 10


=
21
12.42 10 J

We know that v
rms
=
B
3k T
m
At 300 K, v
rms
= 484 ms
1
At 600 K, v
rms
=
1
2 484 684 ms

=
56. (B) Average translational K.E. of a molecules
B
3
k T
2
=
(Where, k
B
= Boltzmann's constant )
This is same, for all gases at same temperature.
57. (A) At 0 K Kinetic energy is zero.
315
58. (C)
3
E RT E T
2
=

E' T'
E T
=
59. (B) E T
60. (B)
B
3
k T 1 eV
2
=

19
3
23
B
2 eV 2 1.6 10
T 7.7 10 K
3 k 3 1.38 10

= = =

61. (C) v
rms
=
o
3RT
M
v
rms


o
1
M
62. (B) v
rms

T
63. (C) v
rms
=
2 2 2 2 2
1 2 3 4 5
5
v + v + v + v + v
= 4.24
64. (A) v
rms

T

( )
( )
rms
2 2
rms 1
1
T
T
v
=
v
65. (D) rms velocity does not depend on pressure.
66. (C) v
rms
=
o o o
3RT RT RT
3 1.73
M M M
= =
67. (B) v
rms

T
, To double the rms speed temperature should be made four times i.e.
2 1
T 4T =
68. (A)
1 2
2 1
3P 16
4:1
1
rms
v
v
= = = =
v
69. (C) rms velocity doesn't depend on pressure, it depends upon temperature only.
2
rms rms rms
3RT
= T T
Mo
v v v

1 1
2 2
T
T
v
=
v
70. (B)
rms 1
o 1
3RT 1
=
M M
v v and
2
2
1
M
v
1 2
2 1
M
M
v
=
v
71. (D)
2
rms rms rms
3RT
= T T
Mo
v v v
72. (C)
rms o rms
o
3RT
T M ( , R constant)
M
v = v
316
73. (B) rms rms
o o
3RT T
M M
v = v
2
2 2
He H
He
H H He
T M
5
7 T M

v
= =
v

He
25 4
T 273
49 2
=
0
273 K 0 C ~ ~
74. (C) rms
o
3RT
M
v =
( )
o 2 2
rms
3 RT 3 8.3 300
M
1920

= =
v

3
2 10 kg 2g

= =
Gas is hydrogen.
75. (C) rms speed, v
rms
=
2 2 2
1 2 3
3
v + v + v
average speed,
1 2 3
3
v + v + v
v =
76. (B) As
( )
( )
rms
t
rms 0
O
T
T
v
=
v
and rms speed increases by 10%
273 t
1.1
273
+
=
or ( )
2 273 t
1.1 1.21
273
+
= =
( )
0
t 273 1.21 1 57.3 C = =
77. (A)
rms
T v
2 2
1 1
T
T
v
=
v
78. (C)
rms o
o
3RT
T M
M
v = (
rms
, R v constant)
( )
( )
2 2
2
2
O
N N
O O
O
M
T
T M
=
79. (C)
rms
o
3RT
M
v =
% increase in
2 1
o o
rms
1
o
3RT 3 RT
M M 20 17.32
100% 100% 15.5%
17.32 3RT
M

v = = =
317
80. (A)
A B A B
A B A B
T T T T
4 2
M M M M
= =
A B
A B
3 RT 3 RT
2
M M
=
A
A B
B
2 2
v
v = v =
v
81. (D) Since temperature is constant.
so v
rms
remains same.
82. (C)
rms o
o
3 RT
= T M
M
v
(
rms
, R v Const. )
( )
( )
2 2
2
2
o
O O
N o
N
M
T
T M
=
83. (C)
rms
T v
( )
( )
rms
1 1
rms 2
2
T
T
v
=
v
84. (B)
B
rms
3k T
m
v =

-1
2
rms
m v
85. (B) At a given temperature
rms
1
v

86. (D)
rms
T v and ( ) ( )
rms rms
2 1
1
2
v = v

( )
( )
rms
2
rms 0
1
T 1
T 2
v
= =
v

273 t 1
273 0 2
+
=
+
273 t 1 273
t 273
273 4 4
+
= =
= 68.25-273 = -204.75
0
C
87. (B)
rms
3 RT
Mo
v =
( ) ( )
( )
T
2
rms rms
1 2
o o
3R 3 RT
;
M 2M
v = v =

o
3RT
4M
=

o
1 3RT
2 4M
=

( )
1
2
rms
v
= = 150 ms
1
318
88. (A)
rms
o
3 RT
M
v =
( )( )( )
1
rms -3
3 8.314 290 3RT
= = 508.24 ms
Mo 2810

| |
| v =
|
\ .

here ( ) ( )
rms rms
1 2
v = v
( ) ( )
2 2 2
2 2
O SO SO
o o
O SO
T T T
300
M M 32 64
= =
2
Tso 600 K =
0
327 C =
89. (B)
rms
T v
1 1
2 2
T
T
v
v
=
90. (C)
( )
( )
rms
He
rms
Ar
v
v

( )
( )
o
He
o
Ar
3RT
M
40
10 3.16
4 3RT
M
= = = ~
91. (D)
rms
T v
2 2
1 1
T
T
v
=
v
92. (A) rms
o
1
M
v
So
( )
( )
( )
( )
2 2
2 2
rms o
O H
rms o
H O
M
=
M
v
v
93. (D)
He H
rms
H He
1 m

m m
v
v =
v
94. (C) v
rms
=
o
0
3RT
T M
M

( )
( )
o
He He
H o
H
M
T
=
T M

95. (A) v
rms
=
( )
( )
( )
( )
2
2 2
2
2 2
rms O o
O H
o rms H o
H O
T M
3 RT
=
M T M
v

v
96. (B)
rms
3P
= v

or
2
rms
P =
3
v
319
97. (C)
2 2
rms
o 1 1
3RT T 273+90
= = = =1.1
M T 273+27
v
v
v
% increase =
2
1
1 100% = 0.1100% =10%
| | v

|
v
\ .
98. (B)
( )
( )
o
1 2
rms
o 2 o
1
M
1
=
M M
v
v
v

( )
( )
o
2
o
2
M
1
= M = 16
32 2

Hence the gas is


4
CH
99. (B) Escape velocity from the earth's surface is 11.2 kms
1
So, v
rms
= V
escape

( )
2
escape o
o
M
3RT
T
M 3R
v
= =
100. (C)
B
2
k T
=
2Pd

( )( )
( )
( )( )( )
-23
2
-5 -10
1.3810 290
=
2 3.14 2.02610 210

-7
= 1.11 10 m
Collision Frequency = no. of collision per second
rms
v
=


9
-7
508.24
= =4.5810
1.1110
( )( )( )
1
rms -3
o
3 8.314 290 3RT
= = 508.24 ms
M 2810

| |
| v =
|
\ .

101. (A)
B
2 2
1 k T
=
2nd 2 Pd
=
8
6.65 10 m

=
102. (A) A monoatomic gas molecule has only three translational degrees of freedom.
103. (D) A diatomic molecule has three translational and two rotational degrees of freedom. Hence
total degrees of freedom, f = 3+2 = 5
104. (B) For a triatomic gas f = 6 ( 3 translation + 3 rotational )
105. (C)
2
1
p
v
C
C f
= = +
106. (B) Degrees of freedom = 3 ( translatory ) + 2 ( rotatory ) + 1 ( vibratory ) = 6
2
1
p
v
C
C f
= = +
=
2
1
6
+
=
1 4
1 1.33
3 3
+ = =
107. (A) Neon gas is mono atomic and for mono atomic gases
v
3
C R
2
=
108. (C) When C
p
and C
v
are given with caloric and R with Joule then C
p
C
v
=
R
J
320
109. (B)
p v p v
C C R C R C = = + ( )
3
R = R+ R =3
2 2
f
R f +

5
R
2
=
110. (B)
v
R 3
C = = 1.5 R = R
0.67 2
This is the value for mono atomic gases
111. According to the equilibrium theorem, the molar heat capacities should be independent of temperature
How ever, variations in C
v
and C
p
are observed as the temperature changes. At very high
temperatures, vibrations are also inportant and that affects the values of C
v
and C
p
for diatomic
and poly atomic gases. Here in the question according to given information (D) may be correct
answer.
112. (B)
113. (C)
( )
( ) ( )
5 7
3 5
1 1 2 2
5 7
3 5 1 2
mix
1 2
5 7
1 2 3 5
1 1

1 1 3 1 1
1.5
1 1
2
1 1 1 1


+
+

= = = =
+ +

114. (A)
2 2 1 2
1
2 1 1
f
f
f f
=1+ = = =

115. (D) molar specific heat at constant pressure,
p
7
C R
2
=
Since
p v v p
C C R C C R = =
7
R R
2
=

5
R
2
=
116. (D)
7
5
=
for a diatomic gas.
117. (C)
2 1 2 1
As tan > tan u > u u u
2 1
T T
P P
| | | |
>
| |
\ . \ .
Also from PV RT = ,
T
V
P

2 1
V V >
118. (B)
1 2 1 2
As tan < tan u < u u u
1 2
V V
T T
| | | |
<
| |
\ . \ .
from PV=RT;
V 1
T P
Hence 1 2
1 2
1 1
P P
P P
| | | |
< >
| |
\ . \ .
119. (C) For an ideal gas PV = constant
i.e PV doesn't vary with V
120. (D) At constant temperature, PV = constant
1
P
V

321
Unit - 10
Ocsillations And
Waves
322
SUMMARY
1. Waves : The motion of the disturbance in the medium (or in free space) is called wave pulse or
generally a wave.
2. Amplitude of a wave : Amplitude of oscillation of particles of the medium is called the amplitude of
a wave.
3. Wavelength and frequency : The linear distance between any two points or particles having phase
difference of 2t rad is called the wavelength () of the wave.
Frequency of wave is just the frequency of oscillation of particles of the medium. Relation between
wavelength and frequency :
v = f, =
k
e

where, v is the speed of wave in the medium.


4. Mechanical waves : The waves which require elastic medium for their transmission are called
mechanical waves, e.g. sound waves.
5. Transverse and longitudinal waves : Waves in which the oscillations are in a direction perpendicular
to the direction of wave propagation are called the transverse wave.
Waves in which the oscillations of the particles of medium are a!cng the direction of wave propagation
are called longitudinal waves.
6. Wave Equation : The equation which describe the displacement for any particle of medium at a required
time is called wave equation. Various forms of wave equations are as follows :
(i) y A sin ( t - kx) = e (ii)
t x
y A sin
T
| |
=
|

\ .
(iii)
x
y A sin 2 t
v
| |
= t
|
\ .
(iv) ( )
2
y A sin vt x
t
=

The above equations are for the wave travelling in the direction of increasing value of x. If the wave
is travelling in the direction of decreasing value of x then put '+' instead of '' in above equations.
7. The elasticity and inertia of the medium are necessary for the propagation of the mechanical waves.
8. The speed of the transverse waves in a medium like string kept under tension,
T
v =

where, T = Tension in the string and (I = mass per unit length of the string -- y
9. Speed of sound waves in elastic medium,
E
v =

where, E = Elastic constant of a medium, = Density of the medium.


Speed of longitudinal waves in a fluid,
B P
v

= =

323
where, B Bulk modulus of a medium
p
V
C
y
C
=
= 1.41 (for air)
Speed of longitudinal waves in a linear medium like a rod,
v

where, = Young modulus, = Density of a medium


At constant pressure and constant humidity, speed of sound waves in gas is directly proportional to
the square root of its absolute temperature.
RT
v v T
M

=
The speed of sound in a gas does not depend on the pressure variation.
10. Principle of Superposition : When a particle of medium comes under the influence of two or more
waves simultaneously, its net displacement is the vector sum of displacement that could occur under
the influence of the individual waves.
11. Stationary Waves : When two waves having same amplitude and frequency and travelling in mutually
opposite directions are superposed the resultant wave formed loses the property of propagation. Such
a wave is .called a stationary wave.
Equation of stationary wave : y = 2 A sinkx cos et
Amplitude of stationary wave : 2 A sin kx
Position of nodes in stationary wave x
n
=
n
2

where, n = 1,2, 3.....At all these points the amplitude is zero.


Position of antinodes in stationary wave's,
x
n
= (2n 1)
4

where , n 1, 2, 3,....
The amplitude of all these points is 2A.
12. Frequencies corresponding to different normal modes of vibration in a stretched string of length L
fixed at both the ends are given by,
n
nv n T
f
2L 2L
= =

where n 1, 2, 3......
13. In a closed pipe the values of possible wavelengths required for stationary wave pattern are given by.
4L
n
(2n - 1)
=
and possible frequencies,
n 1
v
f (2n 1) (2n 1) f
4L
= =
where, n = 1, 2, 3,..... and L = length of pipe.
In a closed pipe only odd harmonics f
1
, 3f
1
, 5f
1
.... are possible.
324
14. In an open pipe the values of possible wavelength required for stationary waves are given by,
2L
n
n
=
and possible frequencies,
n 1
nv
f nf
2L
= =
where, n 1, 2, 3,......and
L - length of pipe.
In open pipe of the harmonics like f
1
, 2f
1
, 3f
1
..... are possible.
15. Beat: The phenomenon of the loudness of sound becoming maximum periodically due to superposition
of two sound waves of equal amplitude and slightly different frequencies is called the 'beats'.
Number of beats produced in unit time = f
1
f
2
.
16. Doppler Effect : Whenever there is a relative motion between a source of sound and a listener with
respect to the medium in which the waves are propagating the frequency of sound experienced by the
listener is different from that which is emitted by the source. This phenomenon is called Doppler
effect.
Frequency listened by the listener,
L
L
S
S
v v
f f
v v

Where, v = velocity of sound, v


L
= velocity of a listener,
v
S
= velocity of a source, f
S
= frequency of sound emitted by the source.
17. If a body repeats its motion along a certain path, about a fixed point, at a definite interval of time, it is
said to have periodic motion.
18. If a body moves to and fro, back and forth, or up and down about a fixed point in a fixed interval of
time, such a motion is called an oscillatory motion.
19. When a body moves to and fro repeatedly about an equilibrium position under a restoring force, which
is always directed towards equilibrium position and whose magnitude at any instant is directly
proportional to the displacement of the body from the equilibrium position of that instant then such a
motion is known as simple harmonic motion.
20. The maximum displacement of the oscillator on cither side of mean position is called amplitude of the
oscillator.
21. The time taken by the oscillator to complete one oscillation is known as periodic time or time period or
period (T) of the oscillator.
22. The number of oscillation completed by the simple harmonic oscillator in one second is known as its
frcquency(f).
23. 2 t times the frequency of oscillator is the angular frequency CO of that oscillator..
24.
1 2 1 2
T or f or =
f T T
t t
= = = e
e
25. For simple harmonic motion, the displacement y(t} of a particle from its equilibrium position is represented
by sine, cosine or its linear combination like
y( t ) = A sin ( t + )
y(t) = B cos ( t + )
y( t ) A' sin t + B' cos t
whereA' Acos and B' = Bsin
e |
e |
= e e
= | |
325
26. The velocity of SHO is given by
2 2
v A y = e
27. The acceleration of SHO is given by a = -e
2
y
28. A particle of mass m oscillating under the influence of Hook's Law exhibits simple harmonic motion
with
k
;
m
e=
m
T 2
k
= t
29. Differential equation for SHM is
2
2
d y
+ y = 0
dth
e
30. For scries combination of n spring of spring constants k
1
, k
2
, k
3
,..., k
n
, the equivalent spring constant is
1 2 n
1 1 1 1
= = ...
k k k k
+
the periodic time
m
T 2
k
= t
31. For parallel combination of n springs of spring constants k ky k ...., kn, the equivalent spring
constant is
k = k
1
+ k
2
+ k
3
+ .... + k
n
and period
m
T 2
k
= t
32. The kinetic energy of the SHO is K =
2
1
m
2
e
(A
2
- y
2
)
33. The potential energy of the SHO is U =
2
1
ky
2
34. The total mechanical energy of SHO is E = K + U =
2
1
m
2
e A
2
=
2
1
kA
2
35. For SHO, at y 0, the potential energy is minimum (U = 0) and the kinetic energy is maximum
2
1
( K kA E )
2
= =
36. For SHO, at y = A, the potential energy is maximum
2
1
(U kA E )
2
= =
and the kinetic energy gy
is minimum (K = 0)
37. Simple harmonic motion is the projection of uniform circular motion on a diameter of the reference
circle.
326
38. For simple pendulum, for small angular displacement
1
T = 2 and
g
t
2 g
2 f =
T l
t
e= t =
39. For simple pendulum, T is independent of the mass of the bob as well as the amplitude of the oscillaions.
40. The differential equaiton for damped harmonic oscillation is
with the displacement
2
2
d y dy
m b = + ky = 0
dt
dt
=
and angular frequency
2
2
k b
'
m 4m
e =
41.
2 etlm
1
E( t ) kA e
2
=
gives the mechanical energy of damped oscillation at time t.
42. A system oscillates under the influence of external periodic force are forced oscillations.
43. The differential equation for forced oscillations is
0
2
2
d y b dy k F
+ y = sin t
m dt m m
dt
= e
44. The amplitude for forced oscillation is
0
1
2 2 2 2 2
2
0
F
A
[m ( ) + b ]
=
e e e
327
MCQ
For the answer of the following questions choose the correct alternative from among the given ones.
SECTION I:
1. If the equation for a particle performing S.H.M. is given by y = Sin2t +
3
Cos2t, its periodic time
will be ..s.
(A) 21 ( B ) p (C) 2p (D) 4p.
2. The distance travelled by a particle performing S.H.M. during time interval equal to its periodic time
is ..
(A) A (B) 2A (C) 4A (D) Zero.
3. A person standing in a stationary lift measures the periodic time of a simple pendulum inside the lift
to be equal to T. Now, if the lift moves along the vertically upward direction with an acceleration of
3
g
, then the periodic time of the lift will now be
(A)
T 3
(B) T
2
3
(C)
3
T
(D)
3
T
4. If the equation for displacement of two particles executing S.H.M. is given by y
1
= 2Sin(10t+)
and y
2
= 3Cos10t respectively, then the phase difference between the velocity of two particles
will be ..
(A) (B) (C)
2


(D)
2

+
.
5. When a body having mass m is suspended from the free end of two
springs suspended from a rigid support, as shown in figure, its periodic
time of oscillation is T. If only one of the two springs are used, then
the periodic time would be
(A)
2
T
(B)
2
T
(C)
T 2
(D) 2T
6. If the maximum velocity of two springs ( both has same mass ) executing S.H.M. and having force
constants k
1
and k
2
respectively are same, then the ratio of their amplitudes will be .
(A)
2
1
k
k
(B)
1
2
k
k
(C)
2
1
k
k
(D)
1
2
k
k
328
7. As shown in figure, two masses of 3.0 kg and 1.0 kg are attached at the two ends of a spring having
force constant 300 N m
- 1
. The natural frequency of oscillation for the system will be hz.
( Ignore friction )
(A) (B) 1/3
(C) 4 (D) 3
8. The bob of a simple pendulum having length l is displaced from its equilibrium position by an angle
of and released. If the velocity of the bob, while passing through its equilibrium position is v, then
v = ..
(A) ) 1 ( 2 Cos gl (B) ) 1 ( 2 Sin gl +
(C) ) 1 ( 2 Sin gl (D) ) 1 ( 2 Cos gl +
9. If
4
1
of a spring having length l is cutoff, then what will be the spring constant of remaining part?
(A) k (B) 4k (C)
3
4k
(D)
4
3k
10. The amplitude for a S.H.M. given by the equation x = 3Sin3pt + 4Cos3pt is m.
(A) 5 (B) 7 (C) 4 (D) 3.
11. When an elastic spring is given a displacement of 10mm, it gains an potential energy equal to U. If
this spring is given an additional displacement of 10 mm, then its potential energy will be ..
(A) U (B) 2U (C) 4U (D) U/4.
12. The increase in periodic time of a simple pendulum executing S.H.M. is .when its length
is increased by 21%.
(A) 42 % (B) 10% (C) 11% (D) 21%.
13. A particle executing S.H.M. has an amplitude A and periodic time T. The minimum time required by
the particle to get displaced by
2
A
from its equilibrium position is .. s.
(A) T (B) T/4` (C) T/8 (D) T/16.
14. If a body having mass M is suspended from the free ends of two springs A and B, their periodic time
are found to be T
1
and T
2
respectively. If both these springs are now connected in series and
if the same mass is suspended from the free end, then the periodic time is found to be T.
Therefore ..
(A) T = T
1
+ T
2
(B)
2 1
1 1 1
T T T
+ =
(C) T
2
= T
1
2
+ T
2
2
(D) 2
2
2
1
2
1 1 1
T T T
+ =
.
3kg 1kg
329
15. The displacement of a S.H.O. is given by the equation x = A Cos ( t +
8

). At what time will it attain


maximum velocity?
(A)

8
3
(B)

3
8
(C)

16
3
(D)

16
.
16. At what position will the potential energy of a S.H.O. become equal to one third its kinetic energy?
(A)
2
A

(B)
2
A

(C)
3
A

(D)
A 3
.
17. Three identical springs are shown in figure. When a 4 kg
mass is suspended from spring A, its length increases by
1cm. Now if a 6 kg mass is suspended from the free end
of spring C, then increase in its length is cm.
(A) 1.5 (B) 3.0
(C) 4.5 (D) 6.0.
18. For particles A and B executing S.H.M., the equation for displacement is given by y
1
=
0.1Sin(100t+p/3) and y
2
= 0.1Cospt respectively. The phase difference between velocity of
particle A with respect to that of B is
(A)
3

(B)
6

(C)
6

(D)
3

19. The periodic time of a simple pendulum is T


1
. Now if the point of suspension of this pendulum
starts moving along the vertical direction according to the equation y = kt
2
, the periodic time
of the pendulum becomes T
2
. Therefore, 2
2
2
1
T
T
= ( k = 1 m/s
2
& g= 10 m/s
2
)
(A) 6/5 (B) 5/6 (C) 4/5 (D) 1
20. A hollow sphere is filled with water. There is a hole at the bottom of this sphere. This sphere is
suspended with a string from a rigid support and given an oscillation. During oscillation, the
hole is opened up and the periodic time of this oscillating system is measured. The periodic
time of the system.
(A) will remain constant
(B) Will increase upto a certain time
(C) Increases initially and then decreases to attain its initial periodic time
(D) Initially decreases and then will attain the initial periodic time value.
330
21. The periodic time of a S.H.O. oscillating about a fixed point is 2 s. After what time will the kinetic
energy of the oscillator become 25% of its total energy?
(A) 1/12 s (B) 1/6 s (C) s (D) 1/3 s.
22. A body having mass 5g is executing S.H.M. with an amplitude of 0.3 m. If the periodic time of the
system is
10

s, then the maximum force acting on body is .


(A) 0.6 N (B) 0.3 N (C) 6 N (D) 3 N
23. As shown in figure, a body having mass m is attached with two springs having spring constants k
1
and k
2
. The frequency of oscillation is f. Now, if the springs constants of both the springs are
increased 4 times, then the frequency of oscillation will be equal to .
(A) 2f (B) f/2
(C) f/4 (D) 4f
24. The figure shows a graph of displacement versus time for a particle executing S.H.M. The acceleration
of the S.H.O. at the end of time t =
3
4
second is ..cm.s
2
(A)
2
32
3
(B)
32
2

(C)
32
2

(D)
2
32
3

25. As shown in figure, the object having mass M is executing S.H.M. with an amplitude A. The
amplitude of point P shown in figure will be .
(A)
2
1
k
A k
(B)
1
2
k
A k
(C)
2 1
1
k k
A k
+
(D)
2 1
2
k k
A k
+
26. A particle is executing S.H.M. between x = - A and x = +A. If the time taken by the particle to travel
from x = 0 to A/2 is T
1
and that taken to travel from x = A/2 to x = A is T
2
, then .
(A) T
1
< T
2
( B ) T
1
> T
2
(C) T
1
= 2T
2
(D) T
1
= T
2
27. For a particle executing S.H.M., when the potential energy of the oscillator becomes 1/8 the maximum
potential energy, the displacement of the oscillator in terms of amplitude A will be .
(A)
2
A
(B)
2 2
A
(C)
2
A
(D)
2 3
A
.
331
28. The average values of potential energy and kinetic energy over a cycle for a S.H.O. will be
.. respectively.
(A) 0 ,
2 2
2
1
A m
(B)
2 2
2
1
A m
, 0
(C)
2 2
2
1
A m
,
2 2
2
1
A m
(D)
2 2
4
1
A m
,
2 2
4
1
A m
.
29. The ratio of force constants of two springs is 1:5. The equal mass suspended at the free ends of both
springs are performing S.H.M. If the maximum acceleration for both springs are equal, the ratio of
amplitudes for both springs is
(A)
5
1
(B)
5
1
(C)
1
5
(D)
1
5
30. When a mass M is suspended from the free end of a spring, its periodic time is found to be T. Now,
if the spring is divided into two equal parts and the same mass M is suspended and oscillated, the
periodic time of oscillation is found to be T. Then ..
(A) T < T (B) T = T (C) T > T (D) Nothing can be said.
31. The periodic time of two oscillators are T and
4
5T
respectively. Both oscillators starts their oscillation
simultaneously from the mid point of their path of motion. When the oscillator having periodic time T
completes one oscillation, the phase difference between the two oscillators will be
(A) 90
0
(B) 112
0
(C) 72
0
(D) 45
0
32. A rectangular block having mass m and cross sectional area A is floating in a liquid having density r.
If this block in its equilibrium position is given a small vertical displacement, its starts oscillating with
periodic time T. Then in this case..
(A)
m
T
1

(B) T (C)
A
T
1

(D)

1
T
33. As shown in figure, a spring attached to the ground vertically has a horizontal
massless plate with a 2 kg mass in it. When the spring ( massless ) is pressed
slightly and released, the 2 kg mass, starts executing S.H.M. The force constant
of the spring is 200 N m
- 1
. For what minimum value of amplitude, will the mass
loose contact with the plate? ( Take g = 10 ms
2
)
(A) 10.0 cm (B) 8.0 cm
(C) 4.0 cm (D) For any value less than 12.0 cm.
332
34. Which of the equation given below represents a S.H.M.?
(A) acceleration = - k ( x + a ) (B) acceleration = k ( x + a )
(C) acceleration = kx (D) acceleration = - k
0
x + k
1
x
2
{ Here k, k
0
and k
1
are force constants and units of x and a is meter }
35. The displacement for a particle performing S.H.M. is given by x = A Cos( t + ). If the initial
position of the particle is 1 cm and its initial velocity is p cm s
- 1
, then what will be its initial phase? The
angular frequency of the particle is p s
-1
.
(A)
4
2
(B)
4
7
(C)
4
5
(D)
4
3
36. Two simple pendulums having lengths 144 cm and 121 cm starts executing oscillations. At some
time, both bobs of the pendulum are at the equilibrium positions and in same phase. After how many
oscillations of the shorter pendulum will both the bobs pass through the equilibrium position and
will have same phase?
(A) 11 (B) 12 (C) 21 (D) 20
37. The maximum velocity and maximum acceleration of a particle executing S.H.M. are 1 m/s and 3.14
m/s
2
respectively. The frequency of oscillation for this particle is ..
(A) 0.5 s
- 1
(B) 3.14 s
- 1
(C) 0.25 s
- 1
(D) 2 s
- 1
38. A particle having mass 1 kg is executing S.H.M. with an amplitude of 0.01 m and a frequency of 60
hz. The maximum force acting on this particle is .. N
(A) 144p
2
(B) 288p
2
(C) 188p
2
(D) None of these.
39. A simple pendulum having length l is given a small angular displacement at time t = 0 and released.
After time t, the linear displacement of the bob of the pendulum is given by
(A) x = aSin2p
t
g
l
(B) x = aCos2p
t
l
g
(C) x = aSin
t
l
g
(D) x = aCos
t
l
g
40. Two masses m
1
and m
2
are attached to the two ends of a massless spring having force constant k.
When the system is in equilibrium, if the mass m
1
is detached, then the angular frequency of mass m
2
will be .
(A)
1
m
k
(B)
2
m
k
(C) 1
2
m
m
k
+
(D)
2 1
m m
k
+
41. When the displacement of a S.H.O. is equal to A/2, what fraction of total energy will be equal to
kinetic energy? { A is amplitude }
(A) 2/7 (B) (C) 2/9 (D) 5/7
333
42. The speed of a particle executing motion changes with time according to the equation y = aSint +
bCost, then ..
(A) Motion is periodic but not a S.H.M.
(B) It is a S.H.M. with amplitude equal to a+b
(C) It is a S.H.M. with amplitude equal to a
2
+ b
2
(D) Motion is a S.H.M. with amplitude equal to
2 2
b a +
.
43. A body is placed on a horizontal plank executing S.H.M. along vertical direction. Its amplitude of
oscillation is 3.92 x 10
3
m. What should be the minimum periodic time so that the body does not
loose contact with the plank?
(A) 0.1256 s (B) 0.1356 s (C) 0.1456 s (D) 0.1556 s
44. If the kinetic energy of a particle executing S.H.M. is given by K = K
0
Cos
2
t, then the displacement
of the particle is given by .
(A)
t Sin
m
K

2 / 1
2
0
|
.
|

\
|
(B)
t Sin
m
K

2 / 1
2
0
2
|
.
|

\
|
(C)
t Sin
mK

2 / 1
0
2
2
|
|
.
|

\
|
(D)
t Sin
m
K

2 / 1
0
2
|
.
|

\
|
45. The equation for displacement of two identical particles performing S.H.M. is given by
x
1
=4Sin(20t+p/6) and x
2
=10Sint. For what value of will both particles have same energy?
(A) 4 units (B) 8 units (C) 16 units (D) 20 units
46. A spring having length l and spring constant k is divided into two parts having lengths l
1
and l
2
. If l
1
= nl
2
, the force constant of the spring having length l
2
is .
(A) k(1+n) (B) k
|
.
|

\
| +
n
n 1
(C) k (D)
) 1 ( + n
k
47. When a mass m is suspended from the free end of a massless spring having force constant k, its
oscillates with frequency f. Now if the spring is divided into two equal parts and a mass 2m is
suspended from the end of anyone of them, it will oscillate with a frequency equal to .
(A) f (B) 2f (C)
2
f
(D)
f 2
48. A mass m on an inclined smooth surface is attached to two springs as shown in figure. The other
ends of both springs are attached to rigid surface. If the force constant of both spring is k, then the
periodic time of oscillation for the system is
(A)
2 / 1
2
2 |
.
|

\
|
k
M

(B)
2 / 1
2
2 |
.
|

\
|
k
M

(C)
2 / 1
2
2 |
.
|

\
|
k
MgSin

(D)
2 / 1
2
2 |
.
|

\
|
k
Mg

334
49. A body of mass 1 kg suspended from the free end of a spring having force constant 400 Nm
-1
is
executing S.H.M. When the total energy of the system is 2 joule, the maximum acceleration is
ms
2
.
(A) 8 ms
2
(B) 10 ms
2
(C) 40 ms
2
(D) 40 cms
2
50. When a block of mass m is suspended from the free end of a massless spring having force constant
k, its length increases by y. Now when the block is slightly pulled downwards and released, it starts
executing S.H.M with amplitude A and angular frequency . The total energy of the system comprising
of the block and spring is .
(A)
2 2
2
1
A m
(B)
2 2 2
2
1
2
1
ky A m +
(C)
2
2
1
ky
(D)
2 2 2
2
1
2
1
ky A m
.
51. A spring is attached to the center of a frictionless horizontal turn table and at the other end a body of
mass 2 kg is attached. The length of the spring is 35 cm. Now when the turn table is rotated with an
angular speed of 10 rad s
1
, the length of the spring becomes 40 cm then the force constant of the
spring is .. N/m.
(A) 1.2 x 10
3
(B) 1.6 x 10
3
(C) 2.2 x 10
3
(D) 2.6 x 10
3
52. As shown in figure (a) and (b), a body of mass m is attached
at the ends of the spring system. All springs have the same
spring constant k. Now when both systems oscillates along
vertical direction, the ratio of their periodic time is ..
(A) (B)
(C) 2 (D)4
53. A simple pendulum is executing S.H.M. around point O between the end points B and C with a
periodic time of 6 s. If the distance between B and C is 20 cm then in what time will the bob move
from C to D? Point D is at the mid-point of C and O.
(A) 1 s (B) 2 s (C) 3 s (D) 4 s
54. A small spherical steel ball is placed at a distance slightly away from the center of a concave mirror
having radius of curvature 250 cm. If the ball is released, it will now move on the curved surface.
What will be the periodic time of this motion? Ignore frictional force and take g = 10 m/s
2
.
(A)
s
4

(B) p s (c)
s
2

(D) 2p s
55. Two identical springs are attached at the opposite ends of a rod having length l and mass m. The
rod could rotate about its mid-point O as shown in figure. Now, if the point A of the rod is
pressed slightly and released, the rod starts executing oscillatory motion. The periodic time of this
motion is
(A)
k
m
2
2
(B)
k
m 2
2
(C)
k
m
3
2

(D)
k
m
2
3

l
2
l
2
k
k
335
56. A simple pendulum having length l is suspended at the roof of a train moving with constant acceleration
a along horizontal direction. The periodic time of this pendulum is
(A)
g
l
T 2 =
(B)
a g
l
T
+
= 2
(C)
a g
l
T

= 2
(D) 2 2
2
a g
l
T
+
=
.
57. A trolley is sliding down a frictionless slope having inclination . If a simple pendulum is suspended
on top of this trolley, its periodic time is given by
eff
g
l
T 2 =
, where g
efff
= ..
(A) g (B) g sin

(C) g cos

(D) g tan

58. One end of a massless spring having force constant k and length 50 cm is attached at the upper end
of a plane inclined at an angle = 30
0
.When a body of mass m = 1.5 kg is attached at the lower end
of the spring, the length of the spring increases by 2.5 cm. Now, if the mass is displaced by a small
amount and released, t he amplitude of the resultant oscillation is ..
(A)
7

(B)
7
2
(C)
5

(D)
5
2
59. Two blocks A and B are attached to the two ends of a
spring having length L and force constant k on a
horizontal surface. Initially the system is in equilibrium.
Now a third block having same mass m, moving with
velocity v collides with block A. In this situation
(A) During maximum contraction of the spring, the kinetic energy of the system A-B will be zero.
(B) During maximum contraction of the spring, the kinetic energy of the system A-B will be mv
2
/ 4
(C) Maximum contraction of the spring is
k
m
v
(D) Maximum contraction of the spring is
k
m
v
2
60. The displacement of a particle executing S.H.M. is given by y = 4Cos
2
(t/2)Sin1000t. This displacement
is due to superposition of S.H.M.s.
(A) 2 (B) 3 (C) 4 (D) 5
61. The displacement of a particle is given by x = A
Coset. Which of the following graph represents
variation in potential energy as a function of time
t and displacement x.
(A) I, III (B) II, IV
(C) II, III (D) I, IV
PE
O
t
v
336
62. A system is executing S.H.M. The potential energy of the system for displacement x is E
1
and for a
displacement of y, the potential energy of the system is E
2
. The potential energy for a displacement
of (x+y) is
(A) E
1
+ E
2
(B)
2
2
2
1
E E +
(C) E
1
+ E
2
+ 2
2 1
E E (D)
2 1
E E
63. A system is executing S.H.M. with a periodic time of 4/5 s under the influence of force F
1
. When a
force F
2
is applied, the periodic time is (2/5) s. Now if F
1
and F
2
are applied simultaneously along
the same direction, the periodic time will be
(A)
5 5
4
(B)
5 4
5
(C)
5 4
8
(D)
5 5
8
64. The periodic time of a simple pendulum is 3.3 s. Now if the point of support of the pendulum starts
moving along the vertically upward direction with a velocity v = kt ( where k = 2.1 m/s
2
), then the
new periodic time is s. { Take g = 10 m/s
2
}
(A) 3 (B) 2.5 (C) 3.33 (D) 2.33
65. A block is placed on a horizontal table. The table executes S.H.M. along the horizontal plane with a
period T. The coefficient of static friction between the table and block is . The maximum amplitude
of oscillation should be so that the block does not slide off the table.
(A)

5
gT
(B)
2
2
4
gT
(C)

2
gT
(D) gT
66. As shown in figure, a block A having mass M is attached to one end of a massless spring. The block
is on a frictionless horizontal surface and the free end of the spring is attached to a wall. Another
block B having mass m is placed on top of block A. Now on displacing this system horizontally
and released, it executes S.H.M. What should be the maximum amplitude of oscillation so that B
does not slide off A? Coefficient of static friction between the surfaces of the blocks is .
(A) A
max
=
k
mg
(B) A
max
=
k
g M m ) ( +
( C) A
max
=
k
g m M ) (
(D) A A
max
=
k
g m M ) ( 2 +
337
U(x)
U(x)
U(x)
U(x)
67. A particle is executing S.H.M. about the origin at x = 0. Which of the following graph shows
variation in potential energy with displacement?
(A) (B)
(C) (D)
68. A horizontal plank is executing SHM along the vertical direction with angular frequency . A coin is
placed on top of this plank. If the amplitude of oscillation is increased gradually, for what maximum
amplitude will the coin be on the verge of loosing contact with the plank?
(A) When is plank is at its maximum height (B) When the plank is at the midpoint.
(C) When the amplitude is
2

g
(D) When the amplitude is
2
2

g
SECTION : II
Assertion Reason type questions :
Note:
For the following questions, statement as well as the reason(s) are given. Each questions has four options.
Select the correct option.
(a) Statement 1 is true, statement- 2 is true; statement-2 is the correct explanation of statement 1 .
(b) Statement 1 is true, statement- 2 is true but statement-2 is not the correct explanation of statement
1 .
(c) Statement 1 is true, statement- 2 is false
(d) Statement 1 is false, statement- 2 is true
(A) a (B) b (C) c (D) d
69. Statement 1 : If a spring having spring constant k is divided into equal parts, then the spring
constant of each part will be 2k.
338
Statement 2 : When the length of the elastic spring is increased ( stretched ) by x, then the amount
of work required to be done is
2
2
1
kx
(A) a (B) b (C) c (D) d
70. Statement 1 : The periodic time of a S.H.O. depends on its amplitude and force constant.
Statement 2 : The elasticity and inertia decides the frequency of S.H.O.
(A) a (B) b (C) c (D) d
71. Statement 1 : For small amplitude, the motion of a simple pendulum is a S.H.M. with
periodic time
g
l
T 2 =
. For large amplitudes, periodic time is greater than
g
l
2
.
Statement 2 : For large amplitude, the speed of the bob is more when it passes through the
mid-point ( equilibrium point ).
(A) a (B) b (C) c (D) d
72. Statement 1 : Periodic time of a simple pendulum is independent of the mass of the bob.
Statement 2 : The restoring force does not depend on the mass of the bob.
(A) a (B) b (C) c (D) d
73. Statement 1: The periodic time of a simple pendulum increases on the surface of moon.
Statement 2 : Moon is very small as compared to Earth.
(A) a (B) b (C) c (D) d
74. Statement 1: If the length of a simple pendulum is increased by 3%, then the periodic time
changes by 1.5%.
Statement 2 : Periodic time of a simple pendulum is proportional to its length.
(A) a (B) b (C) c (D) d
75. Statement 1: For a particle executing S.H.M. with an amplitude of 0.01 m and frequency
30 hz, the maximum acceleration is 36p
2
m/s
2
.
Statement 2 : The maximum acceleration for the above particle is +
2
A, where A is amplitude.
(A) a (B) b (C) c (D) d
76. Statement 1 : The periodic time of a stiff spring is less than that of a soft spring.
Statement 2 : The periodic time of a spring depends on its force constant value and for a stiff
spring, it is more.
(A) a (B) b (C) c (D) d
339
77. St at ement 1 : The amplitude of an oscillator decreases with time.
Statement 2 : The frequency of an oscillator decreases with time.
(A) a (B) b (C) c (D) d
78. Statement 1 : For a particle executing SHM, the amplitude and phase is decided by its initial
position and initial velocity.
Statement 2 : In a SHM, the amplitude and phase is dependent on the restoring force.
(A) a (B) b (C) c (D) d
SECTION III
COMPREHENSION BASED QUESTIONS
NOTE: Questions 79 to 81 are based on the following passage.
Passage 1:
As shown in figure, two light springs having force constants k
1
= 1.8 N m
1
and k
2
= 3.2 N m
1
and a
block having mass m = 200 g are placed on a frictionless horizontal surface. One end of both springs are
attached to rigid supports. The distance between the free ends of the spring is 60 cm and the block is
moving in this gap with a speed v = 120 cm s
1
.
v
79. When the block is moving towards spring k
2
, what will be the time taken for the spring to get
maximum compressed from point D?
(A) p s (B) (p/2) s (C) (p/3) s (D) (p/4) s
80. When the block is moving towards k
1
, what will be the time taken for it to get maximum compressed
from point C?
(A) p s (B) (2/3) s (C) (p/3) s (D) (p/4) s
81. What will be the periodic time of the block, between the two springs?
(A) 1+ (5p/6) s (B) 1+ (7p/6) s (C) 1+ (5p/12) s (D) 1+ (7p/12) s
NOTE: Questions 82 to 84 are based on the following passage.
Passage 2 :
A block having mass M is placed on a horizontal frictionless surface. This mass is attached to one end of
a spring having force constant k. The other end of the spring is attached to a rigid wall. This system
consisting of spring and mass M is executing SHM with amplitude A and frequency f. When the block is
passing through the mid-point of its path of motion, a body of mass m is placed on top of it, as a result of
which its amplitude and frequency changes to A

and f.
340
82. The ratio of frequencies
f
f
'
= ..
(A)
|
.
|

\
|
+ M m
M
(B)
|
.
|

\
|
+ M m
m
(C)
|
.
|

\
|
'
mA
MA
(D)
|
|
.
|

\
| +
mA
A m M
'
) (
83. If the velocity before putting the mass and after putting it is v and v
1
respectively, then
v
v
1
= .
(A)
|
.
|

\
|
+ M m
M
(B)
|
.
|

\
| +
M
m M
(C)
A
A
m M
m M
1
|
.
|

\
|

+
(D)
A
A
m M
m M
1
|
.
|

\
|
+

.
84. The ratio of amplitudes
A
A
1
= .
(A)
|
.
|

\
| +
m
m M
(B)
|
.
|

\
|
+ m M
m
(C)
|
.
|

\
|
+ m M
M
(D)
|
.
|

\
| +
M
m M
NOTE: Questions 85 to 90 are based on the following passage.
Passage 3:
The equation for displacement of a particle at time t is given by the equation y = 3Cos2t + 4Sin2t.

.
85. The motion of the particle is
(A) Damped motion (B) Periodic motion (C) Rotational motion (D) S.H.M.
86. The periodic time of oscillation is
(A) 2 s (B) p s (C) (/2) s (D) 2p s
87. The amplitude of oscillation is cm
(A) 1 (B) 3 (C) 5 (D) 7
88. The maximum acceleration of the particle is ..cm / s
2
.
(A) 4 (B) 12 (C) 20 (D) 28
89. If the mass of the particle is 5 gm, then the total energy of the particle is erg.
(A) 250 (B) 125 (C) 500 (D) 375
90. The frequency of the particle is s
- 1
.
(A) (1/p) (B) p (C) (1/2p) (D) (p/2)
341
Waves
SECTION I :
91. Equation for a harmonic progressive wave is given by y = A sin ( 15pt + 10px + p/3) where x is in
meter and t is in seconds. This wave is .
(A) Travelling along the positive x direction with a speed of 1.5 ms
1
.
(B) Travelling along the negative x direction with a speed of 1.5 ms
1
.
(C) Has a wavelength of 1.5 m along the x direction.
(D) Has a wavelength of 1.5 m along the positive x direction.
92. If the velocity of sound wave in humid air is v
m
and that in dry air is v
d
, then
(A) v
m
> v
d
( B ) v
m
< v
d
( C ) v
m
= v
d
( D ) v
m
>> v
d
93. The ratio of frequencies of two waves travelling through the same medium is 2:5. The ratio of their
wavelengths will be
(A) 2:5 (B) 5:2 (C) 3:5 (D) 5:3
94. If the maximum frequency of a sound wave at room temperature is 20,000 hz then its minimum
wavelength will be approximately ( v = 340 ms
1
)
(A) 0.2 A
0
( B ) 5 A
0
( C ) 5 cm to 2 m (D) 20 mm
95. If the equation of a wave in a string having linear mass density 0.04 kg m
1
is given by y = 0.02
Sin (

|
.
|

\
|

50 . 0 04 . 0
2
x t

, then the tension in the string is ..N. ( All values are in mks )
(A) 6.25 (B) 4.0 (C) 12.5 (D) 0.5
96. If the equation for a transverse wave is y = A Sin2p
|
.
|

\
|

x
T
t
, then for what wavelength will the
maximum velocity of the particle be double the wave velocity?
(A)
4
A
(B)
2
A
(C) pA (D) 2pA
97. Consider two points lying at a distance of 10 m and 15 m from an oscillating source. If the periodic
time of oscillation is 0.05 s and the velocity of wave produced is 300 m/s, then what will be the
phase difference the two points?
(A) p (B) p/6 (C) p/3 (D) 2p/3
342
98. A string is divided into three parts having lengths l
1
, l
2
and l
3
each. If the fundamental frequency
of these parts are f
1
, f
2
and f
3
respectively, then the fundamental frequency of the original
string f = .
(A)
3
2 1
f f f f + + = (B) f = f
1
+ f
2
+ f
3
(C)
3 2 1
1 1 1 1
f f f f
+ + =
(D)
3 2 1
1 1 1 1
f f f f
+ + =
99. Waves produced by two tuning forks are given by y
1
= 4Sin500pt and y
2
= 2Sin506pt. The number
of beats produced per minute is .
(A) 360 (B) 180 (c) 60 (D) 3
100. Equation for a progressive harmonic wave is given by y = 8Sin2p( 0.1x 2t), where x and y are in
cm and t is in seconds. What will be the phase difference between two particles of this wave
separated by a distance of 2 cm?
(A) 18
0
(B) 36
0
(C) 72
0
(D) 54
0
101. As shown in figure, two pulses in a string having center to center distance of 8 cm are travelling along
mutually opposite direction. If the speed of both the pulse is 2 cm/s, then after 2 s, the energy of
these pulses will be
(A) zero
(B) totally kinetic energy
(C) totally potential energy
(D) Partially potential energy and partially kinetic energy.
102. Two waves are represented by y
1
= ASint and y
2
= aCost. The phase of the first wave, w.r.t. to
the second wave is .
(A) more by radian (B) less by p radian (C) more by p/2 (D) less by p/2
103. If the resultant of two waves having amplitude b is b, then the phase difference between the two
waves is .
(A) 120
0
(B) 60
0
(C) 90
0
(D) 180
0
104. If two antinodes and three nodes are formed in a distance of 1.21 A
0
, then the wavelength of the
stationary wave is .
( A ) 2.42 A
0
(B) 6.05 A
0
(C) 3.63 A
0
(D) 1.21 A
0
105. The function Sin
2
(t ) represents
(A) A SHM with periodic time p/. (B) A SHM with a periodic time 2p/.
(C) A periodic motion with periodic time p/. (D) A periodic motion with period 2p/.
343
106. If two almost identical waves having frequencies n
1
and n
2
, produced one after the other superposes
then the time interval to obtain a beat of maximum intensity is ..
(A)
2 1
1
n n
(B)
2 1
1 1
n n

(C)
2 1
1 1
n n
+
(D)
2 1
1
n n +
107. When two sound waves having amplitude A , angular frequency and a phase difference of p/2
superposes, the maximum amplitude and angular frequency of the resultant wave is
(A)
A 2
, (B)
2
A
,
2

(C)
2
A
, (D)
, 2A
2

108. The amplitude of a wave in a string is 2 cm. This wave is propagating along the x-direction with a
speed of 128 m/s. Five such waves are accommodated in 4 m length of the string. The equation for
this wave is .
(A) y = 0.02Sin(15.7x 2010t ) m (B) y = 0.02Sin(15.7x + 2010t ) m
(C) y = 0.02Sin(7.85x 1005t ) m (D) y = 0.02Sin(7.85x + 1005t ) m
109. A string of length 70 cm is stretched between two rigid supports. The resonant frequency for this
string is found to be 420 hz and 315 hz. If there are no resonant frequencies between these two
values, then what would be the minimum resonant frequency of this string?
(A) 10.5 hz (B) 1.05 hz (C) 105 hz (D) 1050 hz
110. Sound waves propagates with a speed of 350 m/s through air and with a speed of 3500 m/s through
brass. If a sound wave having frequency 700 hz passes from air to brass, then its
wavelength .
(A) decreases by a fraction of 10 (B) increases 20 times
(C) increases 10 times (D) decreases by a fraction of 20
111. A transverse wave is represented by y = ASin (t-kx). For what value of its wavelength will the
wave velocity be equal to the maximum velocity of the particle taking part in the wave propagation?
(A) 2pA (B) A (C) pA (D) pA/2
112. Two monoatomic ideal gases 1 and 2 has molecular weights m
1
and m
2
. Both are kept in two
different containers at the same temperature. The ratio of velocity of sound wave in gas
1 and 2 is .
(A)
1
2
m
m
(B)
2
1
m
m
(C)
2
1
m
m
(D)
1
2
m
m
113. A wire having length L is kept under tension between x = 0 and x = L. In one experiment, the
equation of the wave and energy is given by y
1
= ASin
|
.
|

\
|
L
x
Sint and E
1
respectively. In another
experiment, it is y
2
= ASin
|
.
|

\
|
L
x 2
Sin2t and E
2
. Then
(A) E
2
= E
1
(B) E
2
= 2E
1
(C) E
2
= 4E
1
(D) E
2
= 16E
1
344
114. Twenty four tuning forks are arranged in such a way that each fork produces 6 beats/s with the
preceding fork. If the frequency of the last tuning fork is double than the first fork, then the frequency
of the second tuning fork is
(A) 132 (B) 138 (C) 276 (D) 144
115. If two SHMs are given by the equation y
1
= 0.1 Sin(100pt + p/3) and y
2
= 0.1 Cospt, then the
phase difference between the velocity of particle 1 and 2 is
(A) p/6 (B) - p/3 (C) p/3 (D) - p/6
116. The wave number for a wave having wavelength 0.005 m is .. m
1
.
(A) 5 (B) 50 (C) 100 (D) 200
117. An listener is moving towards a stationary source of sound with a speed times the speed of sound.
What will be the percentage increase in the frequency of sound heard by the listener?
(A) 20% (B) 25% (C) 2.5% (D) 5%
118. When the resonance tube experiment, to measure speed of sound is performed in winter, the first
harmonic is obtained for 16 cm length of air column. If the same experiment is performed in summer,
the second harmonic is obtained for x length of air column. Then .
(A) 32 > x > 16 (B) 16 > x (C) x > 48 (D) 48 > x > 32
119. What should be the speed of a source of sound moving towards a stationary listener, so that the
frequency of sound heard by the listener is double the frequency of sound produced by the source?
{ Speed of sound wave is v }
(A) v ( B ) 2v (C) v/2 (D) v /4
120. A metal wire having linear mass density 10 g/m is passed over two supports separated by a distance
of 1 m. The wire is kept in tension by suspending a 10 kg mass. The mid point of the wire passes
through a magnetic field provided by magnets and an a.c. supply having frequency n is passed
through the wire. If the wire starts vibrating with its resonant frequency, what is the frequency of a.c.
supply?
(A) 50 hz (B) 100 hz (C) 200 hz (D) 25 hz
121. If the listener and the source of sound moves along the same direction with the same speed, then..
(A)
s
L
f
f
< 1 (B)
s
L
f
f
=0 (C)
s
L
f
f
= 1 (D)
s
L
f
f
>1
122. A wire of length 10 m and mass 3 kg is suspended from a rigid support. The wire has uniform cross
sectional area. Now a block of mass 1 kg is suspended at the free end of the wire and a wave having
wavelength 0.05 m is produced at the lower end of the wire. What will be the wavelength of this
wave when it reached the upper end of the wire?
(A) 0.12 m (B) 0.18 m (C) 0.14 m (D) 0.10 m
123. If the mass of 1 mole of air is 29 x 10
3
kg, then the speed of sound in it at STP is .. ( =7/5).
{ T = 273 K, P = 1.01 x 10
5
Pa }
(A) 270 m/s (B) 290 m/s (C) 330 m/s (D) 350 m/s
345
124. A wave travelling along a string is described by y = 0.005Sin(40x 2t) in SI units. The wavelength
and frequency of the wave are
(A) (p/5) m ; 0.12 hz (B) (p/10) m ; 0.24 hz (C) (p/40) m ; 0.48 hz (D) (p/20) m ; 0.32 hz
125. Two sitar strings A and B playing the note Dha are slightly out of time and produce beats of
frequency 5 hz. The tension of the string B is slightly increased and the beat frequency is found to
decrease to 3 hz. What is the original frequency of B if the frequency of A is 427 hz?
(A) 432 (B) 422 (C) 437 (D) 417
126. A rocket is moving at a speed of 130 m/s towards a stationary target. While moving, it emits a wave
of frequency 800 hz. Calculate the frequency of the sound as detected by the target. ( Speed of
wave = 330 m/s)
(A) 1320 hz (B) 2540 hz (C) 1270 hz (D) 660 hz
127. Length of a steel wire is 11 m and its mass is 2.2 kg. What should be the tension in the wire so that
the speed of a transverse wave in it is equal to the speed of sound in dry air at 20
0
C temperature?
(A) 2.31 x 10
4
N (B) 2.25 x 10
4
N (C) 2.06 x 10
4
N (D) 2.56 x 10
4
N
128. A wire stretched between two rigid supports vibrates with a frequency of 45 hz. If the mass of the
wire is 3.5 x 10
2
kg and its linear mass density is 4.0 x 10
- 2
kg/m, what will be the tension in the
wire ?
(A) 212 N (B) 236 N (C) 248 N (D) 254 N
129. Tube A has both ends open while tube B has one end closed, otherwise they are identical. The ratio
of fundamental frequency of tube A and B is ..
(A) 1:2 (B) 1:4 (C) 2:1 (D) 4:1
130. A tuning fork arrangement produces 4 beats/second with one fork of frequency 288 hz. A little wax
is applied on the unknown fork and it then produces 2 beats/s. The frequency of the unknown fork
is .hz.
(A) 286 (B) 292 (C) 294 (D) 288
131. A wave y = aSin(t kx ) on a string meets with another wave producing a node at x = 0. Then
the equation of the unknown wave is
(A) y = a Sin(t + kx ) (B) y = - aSin( t+kx) (C) y = aSin(t kx ) (D) y = - aSin(t kx)
132. When temperature increases, the frequency of a tuning fork.
(A) Increases (B) Decreases
(C) remains same (D) Increases or decreases depending on the material.
133. A tuning fork of known frequency 256 hz makes 5 beats per second with the vibrating string of a
piano. The beats frequency decreases to 2 beats/s when the tension in the piano string is slightly
increased. The frequency of the piano string before increase in the tension was ..hz.
(A) 256 + 2 (B) 256 2 (C) 256 5 (D) 256 + 5.
346
134. An observer moves towards a stationary source of sound with a velocity one fifth the velocity of
sound. What is the percentage increase in the apparent frequency?
(A) 5% (B) 20% (C) zero (D) 0.5%
135. The speed of sound in Oxygen ( O
2
) at a certain temperature is 460 m/s. The speed of sound in
helium at the same temperature will be ..ms
- 1
. (Assume both gases to be ideal)
(A) 330 (B) 460 (C) 5002 (D) None of these
136. In a longitudinal wave, pressure variation and displacement variation are
(A) In phase (B) 90
0
out of phase (C) 45
0

out of phase (D) 180
0
out of phase
137. A tuning fork of frequency 480 hz produces 10 beats/s when sounded with a vibrating sonometer
string. What must have been the frequency of the string if a slight increase in tension produces fewer
beats per second than before?
(A) 480 (B) 490 hz (C) 460 hz (D) 470 hz
138. Which of the following functions represents a wave?
(A) ( x vt)
2
(B) ln( x + vt ) (C)
2 ) ( vt x
e
+
(D)
vt x +
1
139. Two sound waves are represented by y = aSin(t-kx) and y = aCos(t-kx). The phase difference
between the waves in water is ..
(A)
2

(B)
4

(C) (D)
4
3
140. A string of linear density 0.2 kg/m is stretched with a force of 500 N. A transverse wave of length
4.0 m and amplitude 1/l meter is travelling along the string. The speed of the wave is .. m/s.
(A) 50 (B) 62.5 (C) 2500 (D) 12.5
141. Two wires made up of same material are of equal lengths but their radii are in the ratio 1:2. On
stretching each of these two strings by the same tension, the ratio between their fundamental frequency
is ..
(A) 1:2 (B) 2:1 (C) 1:4 (D) 4:1
142. The tension in a wire is decreased by 19%, then the percentage decrease in frequency will be

(A) 19% (B) 10% (C) 0.19% (D) None of these


143. An open organ pipe has fundamental frequency 100 hz. What frequency will be produced if its one
end is closed?
(A) 100, 200, 300, . (B) 50, 150, 250.
(C) 50, 100, 200, 300 (D) 50, 100, 150, 200,..
144. A closed organ pipe has fundamental frequency 100 hz. What frequencies will be produced if its
other end is also opened?
(A) 200, 400, 600, 800,.. (B) 200, 300, 400, 500, ..
(C) 100, 300, 500, 700, (D) 100, 200, 300, 400, .
347
145. A column of air of length 50 cm resonates with a stretched string of length 40 cm. The length of the
same air column which will resonate with 60 cm of the same string at the same tension is ..
(A) 100 cm (B) 75 cm (C) 50 cm (D) 25 cm
146. Two forks A and B when sounded together produce 4 beats/s. The fork A is in unison with 30 cm
length of a sonometer wire and B is in unison with 25 cm length of the same wire at the same tension.
The frequencies of the fork are .
(A) 24 hz, 28 hz (B) 20 hz, 24 hz (C) 16 hz , 20 hz (D) 26 hz, 30 hz
147. A tuning fork of frequency 200 hz is in unison with a sonometer wire. The number of beats heard per
second when the tension is increased by 1 % is .
(A) 1 (B) 2 (C) 4 (D) 0.5
148. A bus is moving with a velocity of 5 m/s towards a huge wall. The driver sounds a horn of frequency
165 hz. If the speed of sound in air is 335 m/s, the number of beats heard per second by the
passengers in the bus will be .
(A) 3 (B) 4 (C) 5 (D) 6
149. A vehicle with a horn of frequency n is moving with a velocity of 30 m/s in a direction perpendicular
to the straight line joining the observer and the vehicle. The observer perceives the sound to have a
frequency ( n + n
1
). If the sound velocity in air is 300 m/s, then
(A) n
1
= 10n (B) n
1
= 0 (C) n
1
= 0.1 n (D) n
1
= - 0.1n
150. In a sine wave, position of different particles at time t = 0 is shown in figure. The equation for this
wave travelling along the positive x direction can be .
(A) y = Asin ( t kx )
(B) y = Acos (kx t)
(C) y = Acos( t kx )
(D) y = Asin(kx t)
151. Which of the following changes at an antinode in a stationary wave?
(A) Density only (B) Pressure only
(C) Both pressure and density (D) Neither pressure nor density
152. A sonometer wire supports a 4 kg load and vibrates in fundamental mode with a tuning fork of
frequency 416 hz. The length of the wire between the bridges is now doubled. In order to maintain
fundamental mode, the load should be changed to .
(A) 1 kg (B) 2 kg (C) 8 kg (D) 16 kg
153. In brass, the velocity of a longitudinal wave is 100 times the velocity of a transverse wave. If Y = 1
x 10
11
N/m
2
, then stress in the wire is
(A) 1 x 10
13
N/m
2
(B) 1 x 10
9
N/m
2
(C) 1 x 10
11
N/m
2
(D) 1 x 10
7
N/m
2
.
y
0
x
348
154. The frequency of tuning fork A is 2% more than the frequency of a standard fork. Frequency of
tuning fork B is 3% less than the frequency of the standard fork. If 6 beats per second are heard
when the two forks A and B are excited, then frequency of A is hz.
(A) 120 (B) 122.4 (C) 116.4 (D) 130
155. Fundamental frequency of a sonometer wire is n. If the length and diameter of the wire are doubled
keeping the tension same, the new fundamental frequency is
(A)
2
2n
(B)
2 2
n
(C)
n 2
(D)
4
n
156. A car blowing its horn at 480 hz moves towards a high wall at a speed of 20 m/s. If the speed of
sound is 340 m/s, the frequency of the reflected sound heard by the driver sitting in the car will be
closest to ..hz.
(A) 540 (B) 524 (C) 568 (D) 480
157. A cylindrical tube open at both ends has a fundamental frequency f in air. The tube is dipped vertically
in water so that half of it is in water. The fundamental frequency of the air column is now
(A) f/2 (B) f (C) 3f/4 (D) 2f
158. Three sound waves of equal amplitudes have frequencies (v-1 ), v, ( v+1). They superpose to give
beats. The number of beats produced per second will be .
(A) 3 (B) 2 (C) 1 (D) 4
159. A wave travelling along the x-axis is described by the equation y(x,t) = 0.005Cos(x - t). If the
wavelength and the time period of the wave are 0.08 m and 2.0 s respectively, then and in
appropriate units are
(A) = 12.50p , = p/2.0 (B) = 25p , = p
(C) = 0.08/p , = 2.0/p (D) = 0.04/p , = 1.0/p
160. A wave travelling along a string is described by the equation y = ASin(t-kx). The maximum particle
velocity is ..
(A) A (B) /k (C) d/dk (D) x
161. A string is stretched between fixed points seperated by 75 cm. It is observed to have a resonant
frequencies of 420 hz and 315 hz. There are other resonant frequencies between these two. Then
the lowest frequency for this string is ..hz.
(A) 1.05 (B) 1050 (C) 10.5 (D) 105
162. Two tuning forks P and Q when set vibrating gives 4 beats/second. If the prong of fork P is filed, the
beats are reduced to 2/s. What is the frequency of P, if that of Q is 250 hz.?
(A) 246 hz (B) 250 hz (C) 254 hz (D) 252 hz
163. The length of a string tied across two rigid supports is 40 cm. The maximum wavelength of a stationary
wave that can be produced in it is . cm.
(A) 20 (B) 40 (C) 80 (D) 120
349
164. A stationary wave of frequency 200 hz are formed in air. If the velocity of the wave is 360 m/s, the
shortest distance between two antinodes is .m
(A) 1.8 (B) 3.6 (C) 0.9 (D) 0.45
165. A tuning fork produces 8 beats per second with both 80 cm and 70 cm of stretched wire of a
sonometer. Frequency of the fork is .hz.
(A) 120 (B) 128 (C) 112 (D) 240
166. An open pipe is in resonance in 2
nd
harmonic with frequency f
1
. Now one end of the tube is closed
and frequency is increased to f
2
, such that the resonance again occurs in the nth harmonic. Choose
the correct option.
(A) n = 3, f
2
= (3/4)f
1
(B) n = 3, f
2
= (5/4)f
1
(C) n = 5, f
2
= (5/4)f
1
(D) n = 5, f
2
= (3/4)f
1
SECTION : II
Assertion Reason type questions :
Note:
For the following questions, statement as well as the reason(s) are given. Each questions has four options.
Select the correct option.
(a) Statement 1 is true, statement- 2 is true; statement-2 is the correct explanation of statement 1 .
(b) Statement 1 is true, statement- 2 is true but statement-2 is not the correct explanation of statement
1 .
(c) Statement 1 is true, statement- 2 is false
(d) Statement 1 is false, statement- 2 is true
167. Statement 1: Two waves moving in a uniform string having uniform tension cannot have different
velocities.
Statement 2 : Elastic and inertial properties of string are same for all waves in same string.
Moreover speed of wave in a string depends on its elastic and inertial properties only.
(A) a (B) b (C) c (D) d
168. Statement 1: When a sound source moves towards observer, then frequency of sound increases.
Statement 2 : Wavelength of sound in a medium moving towards the observer decreases.
(A) a (B) b (C) c (D) d
169. Statement 1: Newtons equation for speed of sound was found wrong because he assumed the
process to be isothermal.
Statement 2 : When sound propagates, the compressions and rarefactions happen so rapidly
that there is not enough time for heat to be distributed.
(A) a (B) b (C) c (D) d
350
170. Statement 1 : When pressure in a gas changes, velocity of sound in gas may change.
Statement 2 : Velocity of sound is directly proportional to square root of pressure.
(A) a (B) b (C) c (D) d
171. Statement 1 : If wave enters from one medium to another medium then sum of amplitudes of
reflected wave and transmitted wave is equal to the amplitude of incident wave.
Statement 2 : If wave enters from one medium to another medium some part of energy is
transmitted and rest of the energy is reflected back.
(A) a (B) b (C) c (D) d
SECTION III
COMPREHENSION BASED QUESTIONS
NOTE: Questions 172 to 174 are based on the following passage.
Passage 1
A string 25 cm long and having a mass of 2.5 g is under tension. A pipe closed at one end is 40 cm long.
When the string is set vibrating in its first overtone and the air in the pipe in its fundamental frequency, 8
beats per second is heard. It is observed that decreasing the tension in the string decreases the beat
frequency. The speed of sound in air is 320 ms
1
.
172. The frequency of the fundamental mode of the closed pipe is .. hz
(A) 100 (B) 200 (C) 300 (D) 400
173. The frequency of the string vibrating in its 1
st
overtone is hz
(A) 92 (B) 108 (C) 192 (D) 208.
174. The tension in the string is very nearly equal to
(A) 25 N (B) 27 N (C) 28 N (D) 30 N
NOTE: Questions 175 to 178 are based on the following passage.
Passage 2
Standing waves are produced by the superposition of two waves y
1
= 0.05Sin(3pt 2x) and y
2
=
0.05Sin(3pt + 2x) where x and y are in meters and t is in seconds.
175. The speed ( in ms
1
) of each wave is
(A) 1.5 (B) 3.0 (C) 3p/2 (D) 3p
176. The distance ( in meters ) between two consecutive nodes is .
(A) p/2 (B) p (C) 0.5 (D) 1.0
351
177. The amplitude of a particle at x = 0.5 m is
(A) 1.08 x 10
1
m (B) 5.4 x 10
2
m (C) (p/2) x 10
1
m (D) p x 10
1
m
178. The velocity ( in ms
1
) of a particle at x = 0.25 m at t = 0.5 s is
(A) 0.1p (B) 0.3p (C) zero (D) 0.3
NOTE: Questions 179 to 181 are based on the following passage.
Passage 3
When two sound waves travel in the same direction in a medium, the displacement of a particle located at
x at time t is given by y
1
= 0.05Cos(0.50px - 100pt ) & y
2
= 0.05Cos ( 0.46px - 92pt ), where y
1
, y
2
and
x are in meter and t is in seconds.
179. What is the speed of sound in the medium?
(A) 332 m/s (B) 100 m/s (C) 92 m/s (D) 200 m/s
180. How many times per second does an observer hear the sound of maximum intensity?
(A) 4 (B) 8 (C) 12 (D) 16
181. At x = 0, how many times between t = 0 and t = 1 s does the resultant displacement become zero?
(A) 46 (B) 50 (C) 92 (D) 100
NOTE: Questions 182 to 183 are based on the following passage.
Passage 4
The equation y = 10Sin
4
x
Cos10t represents a stationary wave where x and y are in centimeter and t is
in seconds.
182. The amplitude of each component wave is .
(A) 5 cm (B) 10 cm (C) 20 cm (D) between 5 cm and 10 cm.
183. The separation between two consecutive nodes is
(A) 2 cm (B) 4 cm (C) 5 cm (D) 8 cm
352
1 (B) 41 (B) 81 (D) 121 (C) 161 (D)
2 (C) 42 (D) 82 (A) 122 (D) 162 (A)
3 (B) 43 (A) 83 (A) 123 (C) 163 (C)
4 (C) 44 (B) 84 (C) 124 (D) 164 (C)
5 (A) 45 (B) 85 (D) 125 (B) 165 (A)
6 (D) 46 (A) 86 (B) 126 (A) 166 (C)
7 (D) 47 (A) 87 (C) 127 (A) 167 (D)
8 (A) 48 (A) 88 (C) 128 (C) 168 (A)
9 (C) 49 (C) 89 (A) 129 (C) 169 (A)
10 (A) 50 (B) 90 (A) 130 (B) 170 (B)
11 (C) 51 (B) 91 (B) 131 (B) 171 (D)
12 (B) 52 (B) 92 (A) 132 (B) 172 (B)
13 (C) 53 (A) 93 (B) 133 (C) 173 (D)
14 (C) 54 (B) 94 (D) 134 (B) 174 (B)
15 (A) 55 (C) 95 (A) 135 (D) 175 (C)
16 (A) 56 (D) 96 (C) 136 (D) 176 (A)
17 (B) 57 (C) 97 (D) 137 (D) 177 (B)
18 (C) 58 (A) 98 (C) 138 (C) 178 (C)
19 (A) 59 (B) 99 (B) 139 (A) 179 (D)
20 (C) 60 (B) 100 (C) 140 (A) 180 (A)
21 (D) 61 (A) 101 (B) 141 (B) 181 (D)
22 (A) 62 (C) 102 (D) 142 (B) 182 (A)
23 (A) 63 (A) 103 (A) 143 (B) 183 (B)
24 (D) 64 (A) 104 (D) 144 (A)
25 (D) 65 (B) 105 (A) 145 (B)
26 (A) 66 (B) 106 (A) 146 (B)
27 (B) 67 (D) 107 (A) 147 (A)
28 (D) 68 (C) 108 (C) 148 (C)
29 (C) 69 (B) 109 (C) 149 (B)
30 (C) 70 (D) 110 (C) 150 (D)
31 (C) 71 (B) 111 (A) 151 (D)
32 (C) 72 (C) 112 (A) 152 (D)
33 (A) 73 (B) 113 (C) 153 (D)
34 (A) 74 (C) 114 (B) 154 (A)
35 (D) 75 (B) 115 (B) 155 (D)
36 (B) 76 (A) 116 (D) 156 (A)
37 (A) 77 (C) 117 (B) 157 (B)
38 (A) 78 (C) 118 (C) 158 (C)
39 (D) 79 (D) 119 (C) 159 (B)
40 (B) 80 (C) 120 (A) 160 (A)
KEY NOTE
353
HINT
1.
y sin 2t 3 cos 2t = +
1 3
y 2 sin 2t cos 2t
2 2


= +
`

)
{ } 2 cos sin 2t sin cos 2t 2sin(2t ) = | + | = + |
Compaling than with y Asin(wt ), = + | we get
2
w 2 2 t s
T
t
= = = t
2.
3.
T 2
g
= t

When lift moves up with accleratim
g
3
the effective graritatianl acclenations
in
1 g 4g
g g
3 3
= + =

new peliodic time


'
'
l
T ' 2
g
= t
4.
1
1
dy
2 10cos(10t )
dt
v = = +u
2 2
3 10sin t 30cos(10t )
t
v = = +

Phase diffdence =( )
( )
10t 10t
2 2
t t
+ u + = u
5. For series combination,
1 2
S 1 2
1 2
K k k
K ( k k K)
K k 2
= = = =
+

now
'
' s
k 1 T
T = 2 T = 2 T
T k k
=
6. For maximum velocity,
1 1 2 2
A A e = e
1 2 2 2 2
1 2
2 1 1 1 1
A k / m k
( m m )
A k / m k
e
= = = =
e

354
7. Reduced mass of system
1 2
1 2
m m
0.75 kg
m m
= =
+

freq of oscillation
1 k 20
f 3Hz
2 m
= =
t t

8. KE at o = PE at A
{ }
2
1
m mgh mg 1 cos
2
v = = u
2g (1 cos ) v = u
9.
1 1 2 2
K K k = =
1 2
3
K K k
4 4
| | | |
= =
| |
\ . \ .

force constant of spring having lenght


3
4
l in
2
4
k k
3
=
10. Amplitnde of SHM given by x = a sin t+b cos t in e e
2 2
A a + b =
=
1
2 2
2
(3 4 ) 5m + =
11.
2
2 2 2
2
1 1
u y
u y u 4u
u y
| |
o = =
|
\ .
12.
1
T l o
because 2t and g are constants
2 2 1
1 1 1
1.21
1.1
T l l
T l l
= = =
2 1
1
T T
%increase 100 10%
T

= =
13. y Asin( t ) = e + |
{ }
A 1
Asin t 0 sin t
4
2 2
t
= e | = = e =
2
.t
T 4
t t
=

T
t
8
=
355
14.
1 2 2
1 1 2 2 2
1 1 2
m 4 .M 4 M
T 2 k and k
k T T
t t
= t = =
for Series connection;
1 2
1 2
k k M
T 2 where k
k k k
= t =
+
1
2 2
1 2
2
M
T 2 (T T )
4 M
= t +
t
2 2 2 2 2
1 2 1 2
T T T T T T = + = +
15.
cos( ) sin ( )
8
2
dx
x A t v A t
dt
t
t
= e + = = e e +
If sin( t ) 1,
8
t
e + = then velocity will be maximam
3 3
t t t
8 2 8 8
t t t t
e + = e = =
e
16. 3u = k
2 2 2
1 1 A
3 ky k(A y ) y
2 2 2
= =
17. For spring
b
A restaning face F=kx

displacement
F
x
k
=
figure (b) if the resultant spring contant
1
m k ,

then
1 1 2
1 2
k k
k
k k
=
+
(Series)
k
2
=

in figure (b) if on applying a force F`, if


displacement in x`, then
F'
x '
k '
=
x ' F' k 6
2 3
x k ' F 4
= = =
x ' 3x 3 1cm 3cm = = =
18.
1
1
dy
dt
v =
2
2
dy
and v
dt
=
356
19. Here
2
y kt =
2
2
2
dy d y
2kt 2k 2 ms
dt dt

= = =

the point of support in moving upwards with an accelaration of 2


m
2
s

effective
n
accl .
2
g' g a 12 m/ s = + =
Now 1 2
T 2 and T 2
g g'
= t = t

20.
'
T 2
g
= t

as water leaks, the center of gravity moves down and hence

increases.

T increases initially
When all the water has leaked, the center of gravity moves up and hence

decreases and
hence T decreases Finally the centre of gravity steady at the center of sphde and so T will remain
constant.
21. Kinetic energy = 25 % E
1
K E
4
=
22.
2
2
max max
2
4
F ma mA mA 0.6 N.
T
t
= = e = =
23. For parallel combination
p 1 2
k k k = +

freqrency
l 2
k k 1 k 1
f (1)
2 m 2 m
+
= =
t t
Now when
1 2
k and k in increased 4 times,
1 1 2
4(K K ) 1
f 2f .(from(1))
2 m
+
= =
t
24. From graph A=1 cm T=8s
2 3
y Asin t Asin t y cm
T 2
t
= e = =
2 1 1
2 2
2 2 2
-4 3 3 cm
a w y .
T 2 3 s
t
= = = t
357
25. Now
1
k k =

contant
1 1 2 2 2 1 1
k k k = =
Now, A = l
1
1 2 2
1 =
2 2 2
k +k k
l A
k k k
| | | |

| |
+
\ . \ .
26.
2 2
w A x , v =
the velocity for moving form x=o to
A
2
x = will ge more them for
A
2
x tox A = =
1 2.
T T <
27.
max
1
U u
8
=
2
2 2 2
1 1 1 A
ky kA y
2 8 2 8
| |
= =
|
\ .
28. In the expression for both Kinetic and potential energy, We have the square of the halmonic
functions (sine or cisine).
The average of which over a cycle is
1
2
2 2
E 1
u K m A
2 4
< >= =< >= e
29. Angnlar freqvency
k
m
e =
Since m is constant,
1
K eo
Now,
2 max
max
a
a A
A
= e e =
max max
a a
k A
A K
= =
1
A
k
o
30. For a spring,
m 1
T 2 T ( m
k k
= t o is comtant)
358
31. Phase of
st
1
oscillater 1 1
1
2
t t
T
t
u = e + | = + |
For
nd
2
oscillater,
2 2
2
2
t t+
T
t
u = e + |= |
phase diff
1 2
-
32. Restoring force F = Ayg = (Ag)y = ky
1
m 1
k A g T 2 T
k
A
= = t o
33. To loose comtact, the condition in ;
2
m A mg e =
2
g
A =
e

2
mg
( k mw )
k
= =
34. In SHM, accelelation and displacement are opposite in direction Also a o y. .
35. Here t = 0, x = 1 cm and
1 1
cm s , w s

v = t = t
Now, x Acos( t ) = e + | ---- (1)
Velocity
dx
A sin( t )
dt
v = = e e + |
----- (2)
Solved the equation (1) and (2)
36.
1 2
144 121
T 2 and T 2
g g
= t = t
1 2
T T >

When the shorter pendulum completes n oscillations, the longer one completes (n-1)
oscilla tions (when in same phase).
2 1
nT (n 1)T =
37.
2
r
3.14
r
e
e = =
e

1
3.14
2 f 3.14 f 0.5s
2

t = = =
t
38. Maximum force
2 2 2
m A m4 f A = e = t
39. Periodic time
l 2 g
T 2 and
g T l
t
= t e= e =
Linear displacement x = a coset
359
40.
1 2
k
m m
e =
+
or removing m, angular frequency
2
k
'
m
e =
41. Kinetic energy
2 2 2
1
K m (A y )
2
= e
Now total energy
2 2
1
E m A
2
= e
42. y = a sin et + b cos et,
Taking a = Acos and b = Asin,
y Acos sin t Asin cos t = u e + u e
Asin( t ) = e + u
Now
2 2 2 2 2.
a b A A a b + = = +
43. The body will not loose contact with the surface,
if
2
2
2
m4
mg m r r
T
t
= e = {where r is amplitude}
r
T 2
g
= t
44. Maximum kinetic energy
2 2
0
1
K m A
2
= e
1
2
0
2
2K
A
m
| |
=
|
e
\ .

Equation for displacement is ;


1
2
o
2
2K
y Asin t sin t
m
| |
= e = e
|
e
\ .
45.
2 2 2 2
1
E m A E A
2
= e oe
2 2 2
1 1 2 2
E (A ) ( A ) ( A ) o e e = e
46.
2
k be the spring constant of the spring having lenght
2.

Now,
1 2.
+ =
2 2.
n + =
47.
1 k 1 2k
f and f ' { k' 2k}
2 m 2 2m
= = =
t t

f ' f =
360
48. Here both springs are in parallel. The restoring force on the system in only due to spring and not due
to gravitational force

We can ignore the slope.


Equivalent spring cantant =k+k=2k

Periodic time
1
M
T 2
2k
= t
49. Enelgy stoved =Work done
2
1
E kA
2
=
Now maximum accelaration
2
max
a A = e
50. Potential energy gainad by the spring on suspending mass m
2
1
is ky .
2
When system executes SHM, the energy gained by the system
2 2
1
m A
2
= e

total final energy of the system


2 2 2
1 1
m A ky .
2 2
= e +
51. Radius of the rotational motion r =0.4 m
When the turn table rotates, the restoring force
developed in the spring = centrifugal force
2 2
restore
F m r 2(10) 0.4 80N = e = =
Now increase in lenght of spring = 40-35 = 5 cm

Force constant
3
F 80
k 1.6 10 N/ m.
x 0.05
= =
52. In case-I, springs are connected in parallel.

equivalent force constant


p 1 2
k k k 2k. = + =

Peliodic time p
m m
T 2 2
kp 2k
= t = t
In case-II, spring are connected in series.

Equivalent force constant


1 2
s
1 2
k k k
k
k k 2
= =
+

periodic time s
s
m 2m
T 2 2
k k
= t = t

p
s
T
1
T 2
=
.
361
53. Here T=6 s
Amplitude
1
OB OC BC 10cm
2
= = =

OD =5 cm
Now driplacement x Asin(wt ) (1) = + |
where A=10 gm,
2
3
T
t
t
e = =
rad
Now if at t=0, oscillator is at C.
i.e. at t=0, x=A
A Asin( 0 ) = e + | (OR) A=A sin |
sin 1 | =
2
t
| =
Putting this in eqn. (1)
x Asin( t ) Acos t
2
t
= e + = e = 10 cos et

for x = 5 cm,
5=10cos et
1
cos t
2
e =
t
3
t
e =
t 1 S =
54. Force responsible for oscillation in
F mgsin mg { is small} = u = u u
x
mg
R
=
Comparing this with
F= - kx ;
mg
k
R
=
55. Let the rod be pressed down by x at point
A and released.

both spring gets displaced by x

Restoring torgue produced


t =
kx kx
2 2
| | | |
+
| |
\ . \ .

=kx

Now tan
2x
x
2
u = =


B
O
D
C
362
If
u
in small; tan
2x
u ~ u =

x
2
u
=

2
k
torque k
2 2
u u | |
t = =
|
\ .

Now moment of inertia of rod with reference to O is if I, then


2 2
2
Id k
dt 2
| | u
= u
|
\ .


2 2
2
d k
dt 2I
| | u
= u
|
\ .

Comparing with
2
2
2
d
;
dt
u
= e u
2 2
k 2 m
where and I
2I T 12
t
e= e= =

2m
T
3k
= t
56. Here 2 acceleration vectors g. and a are acting along mutually prependicular direction .
n 2 2
eff
effective acceleratioin l g g a = +
eff
T 2
g
= t

57.
2 2 2
off
x
g a (g ay) = +
here, a
x
2
gsin cos, ay = gsin
2 2 2
x y y
a g a 2ga = + +
=
2 2 2 2 2 2 2 2
a sin cos g g sin 2g sin u u+ + u u
2 2
g (1 sin ) = u
2 2
g cos = u
eff
g g cos = u
58. When the length of spring increases by x=2.5 cm
force F mgsin = u

force constant
F mgsin
k
x x
u
= =
m x
T 2 2 s.
k gsin 7
t
= t = t =
u
363
59. Since the spring is massless, when C
collides with A, both A and B will gain equal momentum. Also, since A and B have equal mass,
both will have same velocity. Let this velocity be u.

Acc. to the law of conservation of momentum,


m mu mu 2mu v = + =
2
v
u =
Now if the compression produced in the spring is x, then acc. to law of conservation of energy,
2 2 2 2
1 1 1 1
m mu mu kx
2 2 2 2
v = + +
2 2 2
2 2
2 2
4
kx v kx
v u
m m
| |
= + = +
|
\ .
4
2 2
(1)
2 2
kx v m
x v
m k
= =
Now block A and B will have equal kinetic energy.
2 2 2 2
1 1 1
kx mu mu mu
2 2 2
= + =
During maximum contraction,
kinetic energy of the system A-B is
mu
2
=
2
.
4
m v
60. Displacement
2
t
y = 4 cos sin100t
2
| |
|
\ .
61. Here x = A cos e t
Now potential energy
2 2
1
m x {taking P.E. as a function of x}
2
= e

when x=0, potential energy=0


graph (b) III
Also, potential energy
2 2
1
m (Acos t)
2
= e e
{taking P. E. as a function of time}
At t=0 potential energy
2 2
1
m A
2
= e

graph -I
364
62.
2 2 2
1 1
1 1
(1)
2 2
E m x E x m = e = e
2 2 2
2 2
1 1
E m y E y m (2)
2 2
= e = e
2 2 2
1 1
E m (x y) E (x y) m (3)
2 2
= e + = + e
From (1), (2) {(3),
1 2
E E E = +
or
1 2 1 2
E E E 2 E E = + +
63.
2
1
1
k kx F
(1)
m mx mx
e = = =
Similarly,
2
2
2
F
(2)
mx
e =
If F
1
and F
2
acts simaltaneously, then angular frequency
1 2
2
F F
w (3)
mx
+
=
From (1), (2) and (3);
2 2 2
1 2
e = e + e
Now, use equ.
2
=
T
t
e
64. Initial periodic time 1
T 2 (1)
g
= t

When pendulum moves along vertical direction, effictive acceleration


eff
g g a = +
where a in accleration of pendulum.
Now,
2
d d (kt)
a k 2.1 m.s
dt dt

v
= = = =

New periodic time 2


eff
T 2 (2)
g
= t

1
2
1 eff
T g
T g
=
365
65. Block will not slide if mg ma >
g a >
To prevent the block from sliding the maximum acceleration
of table must be
max
a g =
Now maximum accleration
2
max
a A = e
2
max
A g e =
2
max
2 2
g gT
A
4

= =
e t
66. Angular frequency of system
1
2
k
(1)
(M m
(
e=
(
+

Now to prevent B from sliding off A, the maximum force acting on B should not be more than
the frictional force mg .
2
max max max
f ma m A (2) = = e
From (1) & (2)
max max
k
f m A
m M
| |
=
|
+
\ .
To prevent block from sliding,
max
max
mkA
f mg mg
m M
= =
+
67. Restoring force F=- kx
Now,
du
F
dx
=

du
kx
dx
=

du k.x.dx =
x 2
0
kx
U(x) k.x.dx C
2
= = +
}
Where C in contant of integration.
Now in a SHM, potential energy at the equilibrium position is zero.
u(x 0) 0 C 0 = = =
2
1
u(x) kx
2
=
in an equation for a parabola.
366
68. At the upper most end,
when
2
mg R m A, = + e coin will loose contact.
Taking R=0
2
m A mg e =
2
g
A =
e
69. Force required to increase the lenght by x in F=kx-----(1)
After spring is divided into 2 equal parts,
x
F k ' x ' where x'
2
= =
x
k' (2)
2
=
from (1) & (2);
k ' 2k =
70. Frequency of SHM depends on elasticity & inertia.
71. Restoring force F mgsin OR = u

e e
F mg where g g sin = = u
If
u
is small,
sin ~

Effective value of g is
e
g
For large oscillation , g sin g ( sin ) u < u u < u
1
T 2
g
> t

72. Restoring force F=- mg sin
u
which depends on m
73. g in less on moon

form the equation


1
T 2 ,
g
= t

T will increase
As compared to earth, moon in small
367
74. Periodic time
T
1
T
2
A = A

{ On differentiation}
T
1.5%
T
A
=
75.
2 2 2 2 2
max
a A 4 f A 4 (30) 0.01 = e = t = t
2
2
m
36
s
= t
Since the oscillator moves between + A & -A,
maximum acceleration
2
36 = t
77. Energy dissipates & so amplitnde decreases.
Statement -2 in false.
78. Statement -1 in true. statement -2 in false. In a SHM, amplitnde & phase does not depend on
restornig force.
79. Time taken by the spring
2
k to get maximum compressed from point D= half period of oscillation
of the block.
(if block in attached at the frce end of spring)
i.e.
2
2
2
T 1 m 1 0.2
t 2 2 s
2 2 k 2 0.3 4
t
= = t = t =
80. Similarly
1
1
T
t s
2 3
t
= =
81. Time period of Block T=Time taken by the block to move from C to D and D to C
82.
1 k 1 k
f and f '
2 M 2 m M
= =
t t +
83. According to the law of conservation of momentum,
MV (M m) ' = + v
84. According to the law of conservation of energy,
Kinetic Energy at mid point = potential Energy at the end points
2 2
1 1
2 2
Mv kA =
And
2 2
1 1
1 1
( )
2 2
M m v kA + =
368
85. y = 3 cos 2t + 4 sin 2t
Asin 3 And Acos 4 | = | =
y Asin cos 2t Acos sin 2t = | + |
y Asin ( t ) = e +|
Which shows that the motion is simple harmonic motion
86.
2 2
T s
2
t t
= = = t
e
87. Amplitude
2 2
A 3 4 5 cm = + =
88. Maximum Accelaration of
2 2 2
a particle A 5(2) 20 cms

= e = =
89. Mechanical Energy
2 2
1
m A 250 erg
2
= e =
90. frequency of the particle
1
1 1
f S
T

= =
t
91. On comparing y = A sin (15 10 )
3
t x
t
t + t +
with y = A sin ( ) t kx e + + u
92. At constant pressure density of water vapour is less than dry air.

with increase in humidity according to the equation


p
v

=

the
velocity of sound increases.
93.
1 1 2
2 1
f

f
f


o =

94. From the equation v = f, min


max
=
f
v
=
17 mm which is nearer to 20 mm
95. On comparing with the wave equation
y = A sin 2

t x
we get
T
| |
t
|

\ .
, T = 0.04 s,

=0.5 m
1
25
2
v ms

=
2
6.25 N T v = t =
369
96. Maximum velocity of particle = Ae

wave velocity = f

Maximum velocity of particle = 2 x wave velocity


A = 2f A e = t
97. Putting values in
vT =
if phase diff.. = in the interval Ax is Ao then
2 2 2
x (15 10)
15 3
t t t
Ao = A = =

98. Freq. of a wave in a string


1
fo

1
2 3
= + +
1 2 3
1 1 1 1
f f f f
= + +
99. On comparing
1
y 4sin 500 t = t with
1 1
y A sin t = e
we get
1 1 1
2 f = 500 f 250Hz e = t t =
Similarly
2
y 2sin506 t = t


2 2 2
2 f 506 f 253 Hz e = t = t =

Freq. of beats
2 1
f f 3 = =

No.of beats heard per minute 3 60 180 = =


100. y 8sin 2 (0.1x 2t) = t

y 8sin 2 (2t 0.1x) = t comparing with


t x
y Asin
T
| |
=
|

\ .
We get
1

=0.1 10cm =
Now path diffrence between 2 particles
2
.x kx
t
o = =

2 1 80 2
1 0

o =
= 72
o
101. Distance covered by the pulse = speed x time = 4 cm in 2 seconds both will cover 4 cm & the
centre of both will superpose & potential energy will be zero.

Total energy will be in the from of kinetic energy. .


370
102.
1
y asin t = e &
2

y a cos t a sin ( t
2
= e = e + )

1st wave is ilagging behind in phase by

2
103. Here A is the amplitude of resultant wave formed by 2 waves having amplitude A
1
and A
2
respectively.
2 2 2
1 2 1 2
A A A 2A A cos = + + u
Also u in the phase
1 2
A , A
Now puttng
1 2
A A b = = &
A b =
, We get
( )
2 2
b 2b 1 cos = + u
0
1
cos 120
2

u = u =
104. As seen from fig., distance between 3 nodes in


105. ( )
2
1 cos 2 t 1 1
y sin t cos 2 t ____________ 1
2 2 2
e
= e = = e
( )
1
= 2 sin 2 t sin 2 t
2
e e =e e
2
a 2 cos 2 t = e e
2
1
= 2 2 -Y
2
| |
e
|
\ .
{From eng. (1)}
= 4e
2

1
y
2
| |

|
\ .
a -y {

SHM}
Now,
2
2 T
T
= e =
e
106. No.of beats produced per second =
1 2
n n =

Time interval between 2 consecutive beats


1 2
1
n n
=

107. Since the phase difference between the 2 waves in

2
they are oscillating along mutually
perpendicalal direction.

Resultant ampltude
2 2
= A A 2 A + =

Angular freq. will remain same.


371
108.
1
A 0.02 m, = 128 ms
k

e
= =

4 2
5 =4 = m, k 2.5 7.85
5
= = =
128 k 128 7.85 1005 e = = ~
( ) y A sin kx t = e

y = 0.02 sin (7.85x - 1005 t)


109. Let the number of loops obtained for 315Hz and 420Hz n and (n+1) respectively.
n 1
f nf 315 = =
( )
n 1 1
f n 1 f 420
+
= + =
n 1 n 1
f f f 105 Hz
+
= =
110. When, sound waves travel fromone medium to another, its frequency does not change.

=
= consant
a b
a b


=
b
b a a
a

10

= =
111. Wave velocity = max. velo.of particle
A
k
e
= e
A =
1
k 2

=
t
= 2A
112. Speed of sound in an ideal gas
RT

=
1 2
2 1
m
m
=
1
m
| |
v
|
\ .
113.
2 2
1
E m A
2
= e

2 2 2
1
m 4 f A
2
=
2
2
2 1 1
2 2
E f f 1
E f
E f 2f 4
| |
| |
= = =
| |
\ .
\ .

2 1
E 4E =
372
114. Let hte freq. of 1st fork be f
1
frequency of 2nd fork = ( )
1 1
f 6 f 6 2 1 + = +
freq. of th 24th fork = ( )
1
f 6 24 1 +
1
f 138 = +
Now, freq. of 24th fork = 2 x freq. of 1st fork (given)

1 1
f 138 2 f + =
1
f 138 Hz =
115. Differentabing
1
y 0.1 sin =

100 t +
3
| |
|
\ .
w.r.t time,
1

(0.1) (100) cos 100 t +


3
| |
=
|
\ .
Similarly differentiating
2
y 0.1 cos t = e w.r.t. time,
2

0.1 sin t
2
| |
= +
|
\ .

phase difference between the 2 velocities is



t + t rad
3 2 6
| | | |
= + =
| |
\ . \ .
116. Wave number
1
1 1
200 m
0.005

= = =
117. Frequency heard by the listener
L
L s
+
f f

| |
=
|
\ .
( )
s
=0
L 2
s
f +
f
=

+
5
4
4
= =

% increase
L S
S
f f 5 4
100 100 25%
f 4
| |
= = =
|
\ .
118. From
RT
=
M

,
T
In summer, velocity increases & hence decreases and so L increases.
The length of 2nd halmonics
1
x = 3L 3 16 48 cm = =
In summer, velocity being more,
1
x 3L > x 48 >
373
119. In
L
L s
s
+
f f

| |
=
|
+
\ .
putting
L L s
0, f 2f , , = = =
s s
=
s s
s

2f f

| |
=
|

\ .


s
2 =
s

2
=
120. For resonance, the frequency of a.c. supply should be same as fundamnetal freq. of wire.
1 T
f
2L
=
= 50 Hz
121.
L L
S S
f +
f
=
+
or
L L
S S
f
f -

=
but,
L S
=
L
S
f
1
f
=
122. Since the rope is heavy, the tension at the lower end & top end of the rope will be different.
Mass of rope
2
m 3kg =
Mass of block
1
m 1 kg =

tension at the lower end


1 1
T m g 1 g N = = &
at the upper end in ( )
2 1 2
T m m g 4 g N = + =
Now speed of wave in rope = =

T T
f
= T ( f ,
are constants)

Wave length at lower and


1 1
= T & at
the upper end
2 2
= T
2 2
1 1
T
T
=

2
2
1
T

T
=
=
1
=
2
0.1m =
123. Speed of sound
P

=
mass of 1 mole air

Volume of 1 mole air


=

3
3 3
29 10 kg
1.3
22.4 10 m

= =

speed
5
1
7 1.01 10
330 ms
5 1.3

= =
374
124 From the phase angle (40-2t), we get k = 40 OR
20
40
2 t
= =

t
and e = 2 OR
Hz f f
1
2 2

t = = t
125 Increase in tension of string increases its frequency. If the original frequency of B(f
B
) were
greater than that of A(f
A
), further the increase in f
B
should have resulted in increase in the beat
frequency. But the beat frequency is found to decrease. This shows that f
A
- f
B
= 5 Hz and
f
A
=427 Hz, we get
f
B
= 422 Hz
126
L
L S
330 0
f f 800 1320 Hz
s 330 130
| | (
= = =
|
(

\ .
127
2 2 2
(340)
11
2.2

L
M
T

T
= = = =
4
T 2.31 10 N =
128
4l f T

T
2l
1
f
2 2
= =

2
2 2
2
M 4f M
T 4f 248 N

| |
= = =
|
\ .
129 In tube A, l
A
2 =
In tube B, l
B
4 =
2l

A
A
= =

1
2

4l

B
A
B
B
= = =
130 The was decreases the frequency of unknown fork. The possible unknown frequencies are, (288+4)
Hz and (288-4) Hz. Wax reduces 284 Hz and so beats should increases. It is not given in the
question. This frequency is ruled out. Wax reduces 292 Hz and so beats should decrease. It is
given that the beats decrease from 2 to 4. Hence the unknown fork has frequency 292 Hz.
consider option (a)
131 Stationary wave : Y = a sin (wt-kx) + a sin(wt+kx)
When x = 0, Y
=
0. The option is not acceptable
consider option (b)
stationary wave : Y = a sin (wt-kx) - a sin(wt+kx)
At x = 0, Y = 0. This option holds good.
Option (c) gives Y = 2a sin(wt - kx)
At x = 0, Y
=
0
Option (d) gives Y = 0.
Hence option (b) holds good.
375
132 When temperature increases, l increases. Hence frequency decreases.
133 The possible frequency of piano are (256 + 5)Hz and (256 - 5)Hz.
For a piano string

T
2l
1
=
When tension T increases v increases.
(i) If 261 Hz increases, beats / second increase. This is not given.
(ii) If 251 Hz increases due to tension, beats / second decrease. This is given.
134 By Dopplers effect,
|
|
.
|

\
|
+
+
=
S
L
S
L


f
f

L L
S

f 6
5
f 5
+
+
= = =
5
1
1
5
6
1
f
f
f
f - f
increase Fractional
S
L
S
S L
= = = =
% 20
5
100
increase percentage = =
135
M
RT P
=

= u
2 o
He
1
2

32
4

=
Hence option (d) is correct.
136 In a longitudinal wave, pressure is maximum where displacement is minimum. Therefore pres-
sure and displacement variations are

180
out of phase
137 Frequency of tuning fork f
1
= 480 Hz. Number of beats s
-1
, n = 10
Frequency of string f
2
= (480 + 10)Hz. A slight increase in tension increase f
2
f
2
= 480 - 10 = 470 Hz.
138 (c) is the correct choice because its value is finite at all times.
139 As u = u cos ) 90 sin(

The phase difference between the two waves is


2
t
140
1
50ms
0.2
500


= = =
141 Here, 2 1
2
1
2 1
T T ,
2
1
r
r
, = = =
1
1
1
1
2
1
f
t
=
T
r l
,
2
2
2
2
2
1
f
t
=
T
r l

1
2
r
r
f
f
2
1
2
1
= =
376
142
10
9
100
81
1
2
1
2
= = =
T
T
f
f

% 10 100
1
2 1
=

f
f f
143 When one end is closed
f
100
2
50H
1 z
= =
2 1 3 1
f =3f =150Hz,f =5f =250Hz and so on...
144 When other end of pipe is opened, its fundamental frequency becomes 200Hz. The overtone
have frequencies 400, 600, 800 Hz..
145
As
2
'
'
60
40
'
50
' 75cm
2 2
1
2
2

= = =
146
6
5
30
25
f
f
1
2
2
1
= = =

2 1 2
f -f =4. on solving, weget f =24 Hz
Hz 20 f
1
=
147
200
1
1
100
1
1
100
101
f
f
2
1
1
2
+ = |
.
|

\
|
+ = =
200
f
f f
1
1 2
+ = 1
200
f
f f s ab be of numbers
1
1 2
1
= = =

148
L L
S S
f +
= ,
f +
Here Hz 165 f , ms 5 , ms 5
S
1
S
1
L
= = u + = u

L
f =170 Hz 5 165 170 s ab be of Number
1
= =

149 As the source is moving perpendicular to straight line joining the observer and source, (as if
moving along a circle), apparent frequency is not affected n
1
= 0
150 As is clear from figure, at t = 0, x = 0, displacement-y = 0 Therefore option (A) OR (D) may be
correct. In case of (D) y = A sin (kx - wt)
dy
= - A cos(kx- t)
dt
e

dy
= kA cos(kx-t)
dx

dy
dy dy
dt
v - v
dy
dt dx
dx
= =
i.e. particle velocity = - (wave speed) xslope and slope at x = 0 and t = 0 is positive, in figure
Therefore, particle velocity is in negative y - direction
151 At a displacement antinode, a pressure node is present. Since pressure does not change at its
node, nor does density.
377
152 For a sonometer fundamental

=
T
22
1
f
To maintatin the fundamental mode, in doubling the length, tension must be quadrupled.
153 velocity of transverse waves T
2
T T
m r
u = =
t
velocity of longitudinal waves L
Y
u =

L
2
T
Y Y
T/ r stress
u
= =
u t
154 Let the frequency of standard fork = x
x
100
97
f , x
100
102
f
B A
= =
,
x
100
97
f
B
=
Now
x
x
f f
B A
100
97
100
102
=
Hz x 120 =
155 If the length of the wire between the two bridges is

, then the frequency of vibration is


d r
T
2l
1
m
T
2l
1
n
2
= =
If the length and diameter of the wire are doubled keeping the tension same, then new funda-
mental frequency will be n/4
156
S
L
S
L


f
f
+
+
=
using this equation the frequency of reflected sound heard by the girl,
L
L
S

f

S
f
+
=
157 f
open
pen

2
o
=

open
closed closed
losed open

f As
4 4 /2 2
c
(
= = =
(


open
open

f
2
= =

i.e. frequency remains unchanged.


158 If we assume that all the three waves are in same phase at t = 0, we shall hear only 1 beat s
-1
378
159 y (x, t) = 0.005 cos ) ( t x | o compare it with standard equation
y (x, t) = A cos (kx - wt) = A cos
|
.
|

\
| t
=

t
t
T
2 2

2 2
and =
T
t t
o = |

160 Given that the displacement of particle is y = A sin (et - kx) ..........(i)
The particle velocity
dy
p ................(ii)
dt
v = =
Now, on diffrentiating eqn.1 with respect to t
dy
A cos( t kx)
dt
= e e
From eqn.(2 mental mode of the colsed pipe is
Hz 200
0.40 4
320
4L

f
1
=

= =
173 Since the beat frequency is 8, the frequncy of the string vibrating in its first Overtone is 192 Hz
or 208 Hz.
Where for 1st Overtone frequency
m
T 1
' f
1

=
........(1)
It is given that the beat frequency decreases if the tension in the string is decreased.
1 1
f ' f > Hence ' f
1
= 208Hz and not 192Hz
174 substituting the values of
m .
and ' f
1
in equation 1 we get T = 27.04 N
175
2
2 m

= = t
1
2f 3
3 ms
2

= =
176 Distance between two consecutive nodes =
m
2 2
t
=

177 The resultant displacement is given by,


y = 0.1 cos 2x sin3tt Or y = A sin 3tt
Where A is the Amplitude of standing waves given by 0.1 cos 2x
At x = 0.5m, cos 2x = cos (1rad) =

cos cos57.3 0.054 m


3.14
| |
= =
|
\ .

Amplitude A at (x = 0.5m) = 0.1

0.54 = 0.54m
178 Particle velocity
dy d
(0.1 Cos 2x sin 3t) = 0.1 x 3 cos 2x sin 3 t
dt dt
= = t t
at x = 0.25m and t = 0.5 s, v = 0
379
179 The two displacements can be written as
1 1 1
y A cos (k x t) = e
and
2 2 2
y A cos (k x t) = e
compare this equation with given equation and get solution.
180 Beat frequncy =
2 1
f f =
1 2

2 2
e e
t t
181 The resultant displacement is given by
2 1
y y y + =
1 1 2 2
A cos ( k x t) A cos (k x t) = e + e For x = 0 we have
1 2
y=A cos t+A cos t
y 0.10 cos (96 t) cos (4 t) = t t
Between t = 0 and t = 1 s, Cos 96t becomes zero 96times and cos 4t becomes zero 4
times Hence the resultant displacement Y at x = 0 becomes zero 100 times
between t = 0 and t = 15.
182
1
y = A sin(kx+ t) e
r
y A sin (kx t) y yi yr = e = +
y 2A sin kx cos t = e Here 2A=10 5 A =
380
N O T E S

You might also like